Текст
                    ЗАДАЧИ ПО МАТЕМАТИКЕ для любознательных
Д. В. КЛИМЕНЧЕНКО
ЗАДАЧИ
ПО МАТЕМАТИКЕ
ДЛЯ ЛЮБОЗНАТЕЛЬНЫХ

Д. В. КЛИМЕНЧЕНКО ЗАДАЧИ ПО МАТЕМАТИКЕ ДЛЯ ЛЮБОЗНАТЕЛЬНЫХ КНИГА ДЛЯ УЧАЩИХСЯ 5-6 КЛАССОВ СРЕДНЕЙ ШКОЛЫ МОСКВА «ПРОСВЕЩЕНИЕ» 1992 Scan А
ББК 22.1 К49 Рецензенты: доцент, кандидат педагогических наук Н.А. Терешин (МПГУ им. Ленина), доцент, кандидат педагогических наук Н. П. Кострикина (Карагандинский государственный университет), учитель математики школы № 463 Москвы А.М. Гольдман Клименченко Д. В. К49 Задачи по математике для любознательных: Кн. для учащихся 5-6 кл. сред. шк.-М.: Просвещение, 1992-192 с.: ил-ISBN 5-09-002714-5. Задачи, представленные в книге, распределены по темам курса математики 5 -6 классов. Спектр трудности задач весьма широк: от самых простых до таких, которые могут заставить задуматься даже старшеклассников. В книге приводятся поиски решения задач, а также решения и ответы. К каждой теме предлагается небольшой теоретический материал, содержащий сведения из истории математики. Книга адресована интересующимся математикой. Она поможет выявить и развить математические способности школьников. 4306020000 345 103(03)92 КБ-25 95-1991 ББК 22.1 ISBN 5-09-002714-5 © Клименченко Д. В., 1992
ДОРОГИЕ ЮНЫЕ ДРУЗЬЯ! Каждый из вас изучает математику с первого дня пребывания в школе и будет ее изучать до окончания школы. Кому же придется продолжать учебу в старших классах с физико-математическим уклоном или в высших учебных заведениях после окончания школы, тот более основательно познакомится с этой удивительной наукой. Еще в древности одним из важнейших достоинств человека считали владение математическими знаниями. В Индии, например, только тот юноша считался подготовленным к жизни, кто овладел искусством решения задач, физических упражнений и стихосложения. Слово «математика» в переводе с греческого означает знание, наука. Не говорит ли уже это о ее месте среди наук? Непрерывно возрастают роль и значение математики в современной жизни. В условиях научно-технического прогресса труд приобретает все более творческий характер, и к этому надо себя готовить за школьной партой. Изучение математики осуществляется в основном в процессе решения задач. Решение задач выступает и как цель, и как средство обучения. Умение решать задачи является одним из основных критериев уровня математического развития обучающихся. Вот почему любая контрольная работа, любой экзамен по математике в качестве основной части содержит задачи. В ходе работы над задачами формируется творческое мышление. Что же нужно, чтобы научиться решать задачи? Прислушаемся к советам известного современного педагога-математика Д. Пойа. «Если вы хотите научиться плавать,-пишет он,-то смело входите в воду, а если хотите научиться решать задачи, то решайте их» (Пойа Д. Математи- ческое открытие-М.: Наука, 1970-С. 13). Хотелось бы надеяться, что предлагаемые в настоящей книге задачи окажут вам в этом некоторую помощь. Здесь вы найдете нестандартные занимательные задачи, тесно связанные с программным материалом по математике для 5-6 классов. Ни одна из задач не требует никаких дополнительных сведений к материалу основного курса. Но надо вдумчиво, творчески отнестись к уже имеющимся знаниям, применить их в новых, в какой-то мере оригинальных ситуациях. Внутри каждого параграфа задачи расположены в порядке возрастания трудности, однако нельзя поручиться, что принятая система удовлетворит всех. Ведь то, что одному дается легко, другому покажется трудным, и наоборот. При решении задач мы обращаемся нередко к формулам, представленным, как правило, в обобщенном буквенном виде. Но в математике, по словам и 3
замечательной русской женщины-математика С. В. Ковалевской, главное, первичное-это содержание, а затем для их выражения существует свой язык-формулы. Ряд задач представлен в занимательной форме. Пусть читателя заинтри- гует и увлечет необычность ситуации, жизненно-практическая ценность, неочевидность ответа на поставленный вопрос и появится желание совершить пусть даже нелегкий путь поиска решения задачи. Занимательность и строгость вполне совместимы. Здесь вы встретите также задачи, содержащие понятие стоимости (напри- мер, цена тетради, книги, ткани и др.). В связи с этим у вас могут возникнуть вопросы. Заранее отвечаем, что приведенные в задачах цены следует рассмат- ривать как условные и относиться к ним исключительно как к математическим величинам. После условий задач в конце каждого параграфа приводятся примерные указания по поиску решений задач. Это своеобразные вехи на пути к решению, советы, намеки. Обращайтесь к ним только в случае, если сами не видите пути преодоления препятствия и цель вам кажется недоступной. Кроме поисков решений, в заключительной части книги приводятся подробные решения и ответы. Но не спешите и к ним обращаться, так как известно, что одна самостоятельно решенная задача даст больше пользы для развития ума, чем двадцать других, решения которых вы узнали от друзей или прочитали в книге. Задачи в этой книге-для вдумчивых, смелых и настойчивых. Каждая самостоятельно решенная вами задача-это, возможно, небольшая, но все же победа. И если вы получите более красивое решение, чем приведенное в книге, то автор вас от души поздравляет. Ну а если вам не удалось решить задачу сразу-не огорчайтесь, а проявите терпение, выдержку, настойчивость. Это поможет вам преодолеть трудности, и вас непременно ожидает успех. Развивайте свои способности самостоятельно решать задачи, требующие глубоких размышлений. Успеха вам!
НУМЕРАЦИЯ ЧИСЕЛ Первые ваши встречи с математикой связаны со счетом. А чтобы научиться считать, надо уметь называть числа и обозначать их. Необходимо овладеть устной и письменной нумерацией. Знание нумерации-это необходимое условие для изучения арифметических действий, а значит, и всей арифметики - признанной царицы математики. Известный русский писатель и педагог Л. Н. Толстой говорил, что тот, кто поймет счисление (нумерацию), тот легко поймет всю арифметику. Та нумерация, которой сейчас пользуются, получила название десятичной позиционной. Десятичной она называется потому, что каждая единица более высокого разряда содержит 10 единиц низшего соседнего разряда, а позицион- ной-по той причине, что значение знака (цифры) зависит не только от его начертания, но и от места (позиции), которое занимает в записи числа этот знак. Потребовалось много тысячелетий, чтобы люди научились называть и записывать числа так, как это делаем мы с вами. Начало же десятичной (но непозиционной) нумерации было положено в Древнем Египте, а позиционная идея исходит из Вавилона. Создание же позиционной десятичной нумерации было в основном завершено индийскими математиками в V-VII вв. н. э. И сейчас уже представить трудно, как могло бы обойтись человечество без этого замечательного достижения. И действительно, имея всего 10 различных знаков (цифр), можно записать любое число. И правила выполнения действий над такими числами оказываются довольно простыми и удобными. Вы не пробовали складывать или вычитать числа, записанные в римской нумерации? А умножение и деление выполнять в римской нумерации куда сложнее. Французский математик Лаплас (1749-1827) такими словами оценил это «открытие»: «Мысль-выражать все числа немногими знаками, придавая им значения по форме, еще значение по месту, настолько проста, что именно из-за этой простоты трудно оценить, насколько она удивительна». 5
Почему же эту нумерацию часто называют арабской? А дело в том, что арабы, познакомившись с' этой нумерацией первыми, по достоинству ее оценили, усвоили и перенесли в Европу. В настоящее время в связи с разви- тием ЭВМ получила распространение двоичная нумерация. Для записи чисел в этой нумерации используются лишь две цифры 0 и 1. Чтобы хорошо усвоить нумерацию чисел, надо научиться глубокому и всестороннему их «видению», подмечать и умело использовать их особен- ности и скрытые свойства. В этом вам помогут задачи настоящего параграфа. Старайтесь решать их самостоятельно. Естественно, при решении этих задач придется обратиться к некоторым сведениям по математике, которые получены вами уже в начальных классах. ЗАДАЧИ 1. Запишите миллион: а) при помощи трех сотен и знаков действий; б) при помощи шести десятков и знаков действий. 2. Запишите и прочитайте все семизначные числа, сумма цифр в каждом из которых равна 2. Сколько таких чисел? 3. На листе бумаги записано число 686. Как, не выполняя никаких записей и вычислений, получить число, большее данного на 303? 4. Таня записала на листе бумаги некоторое двузначное число; сидевшей напротив Свете записанное число представилось другим, меньшим на 75. Какое число записала Таня? 5. Как получить сто, исключив из девяноста десять? 6. Во всех трех равенствах XIII = VII — VI, VII = V — I, XI + V = V, со- ставленных из палочек, допущены ошибки. Переложите в каждом из них по одной палочке так, чтобы равенства стали верными. 7. Сколько среди двузначных чисел таких, в записи которых: а) имеется хотя бы одна цифра 3; б) число десятков меньше числа единиц? 8. Сколько среди натуральных чисел, не превышающих 1000, таких, у которых каждая последующая цифра больше предыдущей? 9. Запишите подряд 20 пятерок. Поставьте между некоторыми цифрами знаки сложения так, чтобы получилась сумма 1000. 10. Напишите наименьшее натуральное число, составленное из всех возможных различных цифр. 11. Напишите наименьшее натуральное число, составленное из всех возможных .различных цифр, делящееся без остатка на 5. 12. Найдите четырехзначное число, две средние цифры которого образуют число, в 5 раз большее числа тысяч и в 3 раза большее числа единиц. 13. В числе 513 879 406 вычеркните 4 цифры так, чтобы оставшиеся цифры в том же порядке составили: а) наибольшее число; б) наименьшее число. 14. Записаны все числа подряд от 1 до 40. Не изменяя порядка записи, вычеркните 60 цифр так, чтобы оставшиеся цифры выражали: а) наименьшее число; б) наибольшее число. 15. Из книги выпал кусок, первая страница которого имеет номер 143, а номер последней состоит из тех же цифр, но записанных в другом порядке. Сколько страниц выпало из книги? 16. Сколько всего можно записать четырехзначных чисел, в каждом из которых цифры не повторяются, используя цифры 0, 1, 2, 3, и таких, чтобы цифры 0 и 2 не были в них соседними? 6
17. Ученики получили задание написать несколько различных трехзначных чисел, не содержащих в записи цифру 7. Сколько всего можно записать таких чисел? 18. Может ли двузначное число равняться сумме своих цифр? 19. Шифр замка-автомата-семизначное число, три первые цифры кото- рого одинаковы, остальные четыре цифры также одинаковы. Сумма всех цифр этого числа-число двузначное, первая цифра которого совпадает с первой цифрой шифра, а последняя-с последней. Найдите этот шифр. 20. Андрея попросили назвать номер квартиры, которую получила его семья в новом доме. Он ответил, что этот номер выражается числом, которое в 17 раз больше числа, стоящего в разряде единиц номера. Какой же номер этой квартиры? 21. Наблюдательный Юра заметил, что если в двузначном числе, выра- жающем расстояние в километрах, которое они сегодня проехали, вставить нуль между цифрами десятков и единиц, то получится число, в 9 раз большее исходного. Какое расстояние проехали? 22. Если к любому двузначному числу приписать справа число, записанное теми же цифрами, но в обратном порядке, то получим четырехзначное число, делящееся на 11 без остатка. Докажите. 23. Расстояние между двумя городами в километрах выражается таким двузначным числом, что левая его цифра равна разности между этим числом и числом, записанным теми же цифрами, но в обратном порядке. Чему равно это расстояние? 24. Найдите число, сумма цифр которого равна разности между 328 и искомым числом. 25. Существует ли трехзначное число, которое уменьшается втрое от перестановки начальной его цифры в конец числа? 26. Количество килограммов металлолома, собранного одним классом, выражается трехзначным числом. Если первую цифру 9 этого числа перенести в конец, то получим число килограммов металлолома, собранного другим классом. Известно, что один класс собрал на 216 кг больше другого. Сколько металлолома собрал каждый класс? 27. Найдите цифру, обладающую тем свойством, что если приписать ее в конце произвольного натурального числа, то получим число, равное сумме трех слагаемых, одно из которых-первоначальное число, второе-число, обозначенное искомой цифрой, и третье-произведение первых двух сла- гаемых. 28. Запишите четырехзначное число, у которого каждая последующая цифра на 1 больше предыдущей, затем запишите число теми же цифрами, но в обратном порядке и из большего числа вычтите меньшее. Повторите это несколько раз, беря иные числа, и сравните полученные результаты. Решите задачу в общем виде. Что вы заметили? 29. Все натуральные числа от 1 до 100 включительно разбиты на 2 группы-четные и нечетные. Определите, в какой из этих групп сумма всех цифр, использованных для написания чисел, больше и на сколько. 30. Количество учеников, обучающихся в V - VI классах школы, выражает- ся трехзначным числом. Из цифр этого числа (без повторений их) можно составить 6 различных двузначных чисел, сумма которых вдвое больше числа учеников V-VI классов. Сколько учеников в этих классах? 7
31. Количество тополей, которые посадили учащиеся, выражается трех- значным числом, а лип-двузначным. Всего они посадили 144 дерева. Если поменять местами крайние цифры в этих двух числах и сложить полученные числа, то получим 603. Сколько посадили тополей и сколько лип? 32. На доске было записано два одинаковых двузначных числа. К одному из них слева приписали 100, а к другому-справа 1, в результате чего первое число стало в 37 раз больше второго. Какие числа были записаны на доске? 33. Сколько всего шестизначных чисел, у которых все цифры нечетные? 34. Используя каждую из девяти цифр 1, 2, 3, ..., 9 по одному разу, запишите три такие числа, чтобы сумма их была наибольшей из всех возможных, а разность при этом между большим и меньшим числами была как можно меньше. Найдите эту сумму и разность. 35. Оля записала некоторое трехзначное число, затем нашла сумму его цифр и записала результат, дальше нашла сумму цифр последнего числа и записала результат. Все эти три числа можно записать так: АПА; ПО; □. (Одинаковые фигуры соответствуют одинаковым цифрам.) Восстановите запись чисел, которую выполнила Оля. 36. Количество учащихся V-VI классов одной из школ выражается трехзначным числом. Если найти произведение цифр этого числа, а затем произведение цифр полученного числа, то все эти три числа можно записать так: АОО; АП; □. (Одинаковые фигуры соответствуют одинаковым цифрам.) Сколько в школе учащихся V-VI классов? 37. В стозначном числе 12345678901234567890... 1234567890 вычеркнули все цифры, стоящие на нечетных местах. В полученном пятидесятизначном числе вновь вычеркнули цифры, стоящие на нечетных местах. Вычеркивание продолжалось до тех пор, пока ничего не осталась. Какая цифра была вычеркнута последней? 38. «Сколько учащихся в нашем городе?»-поинтересовались дети. «А вы узнаете сами, если известно, что число, выражающее количество учащихся, является наибольшим из всех таких чисел, у которых любые две соседние цифры образуют каждый раз число, делящееся без остатка на 23»,- ответил им учитель. Пользуясь этими данными, ребята решили задачу. Решите и вы ее. 39. Найдите все трехзначные числа х, в записи которых цифры не повторяются, такие, чтобы разность этого числа и числа, записанного теми же цифрами, но в обратном порядке, была также числом трехзначным, состоя- щим из тех же цифр, что и число х. 40. Запишите словами число 1980 и подсчитайте количество всех букв в этих словах. Полученное число, выражающее количество букв, снова запишите словами и подсчитайте количество букв в них, затем проделайте такую же операцию еще 10 раз и запишите подряд все 13 чисел. а) Установите закономерность в написании этого ряда начиная с пятого числа. б) Докажите, что каким бы числом ни начинался составленный таким образом ряд, некоторая группа чисел в нем начнет повторяться в определен- ном порядке начиная с некоторого номера. 8
Поиски решений 1. Для решения задачи выясняем, знаки какого действия следует исполь- зовать в записи. 2. Вы уже догадались, что записывать искомые числа следует начинать с крайней левой цифры. А какой она может быть? 3-4. Некоторые цифры (0, 8, 6, 9) обладают интересным свойством, используя которое и находим решение. 5. Не думайте, что над вами просто пошутили. Разве обязательно записывать только в десятичной позиционной нумерации? Вы знакомы и с иной нумерацией. 6. С помощью палочек составлены не только числа, но знаки действий и равенства, поэтому перекладывать можно любые из этих палочек, составляя новые равенства. 7-8. Было бы слишком обременительным решать эти задачи, записывая все числа подряд и отбирая непосредственно из них нужные. Подумать только-в задаче 8 надо было бы записать 990 чисел! Вот тут-то и нужен рациональный подход: исследовать ситуацию и установить закономерности. Рассматривая в задаче 8 числовые промежутки 123, ..., 129; 134, ..., 139 и т. д., устанавливаем количества нужных трехзначных чисел, у которых цифра сотен 1, а затем-2 и т. д., и все результаты суммируем. Это же касается и двузначных чисел. 9. Проверим, можно ли оставить подряд 4 пятерки, не поставив ни одного знака «4-». А если поставить знаки « + » между каждой парой пятерок, получим ли в сумме 1 000? В результате устанавливаем, каким числом должно быть одно из слагаемых. Дальше нетрудно решить вопрос относительно остальных слагаемых. (Эту задачу, как и другие, не следует решать наугад.) 10. Зная, что всего различных цифр 10, должно быть записано десяти- значное число. Какую цифру следует записать крайней левой, чтобы число было наименьшим? Какие цифры писать правее левой записанной? (Учтите особенности цифры 0.) 11. Пользуясь решением задачи 10, перенесем одну цифру в конец записи числа. Какую? 12. Четырехзначное число будет определено, если будут определены одна из крайних его цифр или две средние, образующие двузначное число, большее одного из однозначных чисел в 3 раза и другого-в 5 раз. Такое двузначное число единственное. Какое? Зная его, запишем искомое четырехзначное число. 13. Чтобы получить наибольшее возможное число, надо стремиться оставить левыми крайними цифрами наибольшие из возможных. Для получе- ния наименьшего числа, наоборот, оставить слева цифры, обозначающие наименьшие однозначные числа. 9
14. Чтобы решить вопрос, какие цифры оставить, надо знать прежде всего, сколько их оставить. Для этого узнаем, сколько всего цифр в записи чисел от 1 до 40. Используя идею решения задачи 13, находим решение данной задачи. 15. Для определения числа выпавших страниц необходимо знать номер первой выпавшей страницы и последней. Если, например, номер первой выпавшей страницы 25, а последней 52, выпало столько страниц, сколько чисел от 25 до 52, т. е. 28 (но не 27! Почему?). Так как выпасть может только целое число листов, то каким числом может выражаться последняя страница выпавшего куска? Учитывая это, находим номер последней страницы выпав- шего куска. 16. Количество всех чисел, удовлетворяющих условию задачи, можно было бы узнать, если бы знать, сколько их начинается с цифры 1, 2 и 3. Такие числа можно составить и непосредственно пересчитать. 17. А может, проще подсчитать, сколько таких трехзначных чисел, в записи которых имеется хотя бы одна семерка, а затем, зная общее количество трехзначных чисел, найти нужное? Ясно, что от 700 до 799 таких чисел 100, в пределах же каждой иной сотни их меньше. Достаточно подсчитать их в пределах одной такой сотни, например от 100 до 200. 18. Чтобы ответить на вопрос задачи, достаточно записать двузначное число, у которого а десятков и b единиц, и исследовать возможность справедливости равенства, записанного по условию задачи. 19. Решение аналогично решению задачи 18, но надо учесть, что цифра а записана 3 раза, а цифра 6-4 раза в семизначном числе. 20. В задаче не говорится о значности числа-номера квартиры, но его можно так же, как и в предыдущих задачах, записать в виде 10а + Ь, где а необязательно однозначное число (цифра). 21. Для решения задачи можно записать равенство согласно условию. Записав число 10а + Ь, надо записать число, полученное после того, как между цифрами десятков и единиц вставили 0. 22. Исследовать возможность делимости четырехзначного числа на 11 сможем в том случае, если образуем его из двузначного числа ab и запишем в виде суммы разрядных слагаемых. (Запись ab означает двузначное число, в котором а десятков и Ь единиц, т. е. 10а + Ь.) Запишем, например, число 2552 в виде суммы разрядных слагаемых и приведем к виду 1001 -2 + 110-5. Числа 1001 и 110 делятся на 11. А теперь решим задачу в общем виде. 23. Задача сводится к определению цифр десятков и единиц. Для этого запишем равенство согласно условию задачи. И если в результате получим 9Ь = 8а, то а = 9 и b = 8, так как а и b цифры. (Идея эта используется при решении задачи 19 и многих последующих задач.) 24- 26. Задачи эти являются несколько усложненным вариантом задач, в которых шла речь о двузначных числах. Для их решения надо уметь записать трехзначное число, у которого, например, а сотен, b десятков и с единиц; составить равенство и исследовать его. Получив равенство 11Л + 2с = 25 в задаче 24, нетрудно определить значения b и с. (Не забывайте, что это цифры.) Или в задаче 25 надо 97а установить: может ли при 1 а 9 быть целым числом? 27. Если к некоторому числу справа приписать цифру, то это равносильно выполнению двух арифметических действий. Каких? Если первоначальное ю
число а, а приписали к нему справа цифру х, то как записать полученное число в виде суммы разрядных слагаемых? Выяснили? Вот теперь можно записать равенство по условию задачи и исследовать его. 28. Решив задачу, используя конкретные числа, рассмотрите ее при условии, что наименьшая цифра, например, а. (Предложите вашим друзьям фокус на основе этой задачи. Подумайте, в чем он состоит.) 29. Не ошибитесь! Речь идет о сравнении суммы не чисел (четных и нечетных), а цифр, употребляемых для их записи. Задача сводится к сравнению суммы четных и нечетных однозначных чисел. Затем следует рассмотреть числа с цифрой десятков 1, 2 и т.д. Число 100 занимает здесь особое положение. 30. Трехзначное число будет записано, если имеются цифры: а сотен, b десятков и с единиц. Для их определения составляются все возможные двузначные числа при помощи этих цифр (6 чисел) и записывается равенство по условию задачи. 31. Прежде всего обращаем внимание на то, что сумма трехзначного числа и двузначного имеет в разряде сотен 1. Определив цифру сотен искомого трехзначного числа, определим тем самым цифру единиц обращенного числа (числа, записанного теми же цифрами, но в обратном порядке). Зная цифру единиц суммы обращенных чисел, можно найти цифру единиц числа, обращенного искомому двузначному. (Это цифра десятков искомого двузнач- ного числа.) Далее определяется цифра единиц искомого трехзначного числа, а затем и остальные цифры. 32. Важно заметить, что первоначально записанные числа оканчиваются цифрой 7, для чего надо условие просто перевести на язык символики, т.е. 100tz/> = аЫ -37. Так нередко открываются решения задач, если их условия записать в символах и знаках. 33. Количество всех шестизначных чисел определить можем: это, конечно, нетрудно. Легко определяется также количество чисел, оканчивающихся нечетной цифрой, а из них можно определить количество тех, у которых вторая цифра справа нечетная, и т. д. (Не забудьте только, что крайней цифрой не может быть 0.) 34. Принимая во внимание, что сумма должна быть наибольшей из возможных, выбираем цифры сотен, десятков и единиц (какие?) для трех чисел. Но, чтобы разность между большим и меньшим числами была наименьшей из возможных, надо из возможных цифр десятков 6, 5, 4 и единиц 3, 2, 1 выбрать для этих чисел нужные. Какие? 35. Заметив, что сумма цифр двузначного числа обозначена цифрой десятков искомого числа, определяем цифру единиц двузначного числа. (Вы, конечно, знаете, что, например, 5 + 0 = 5.) Но тогда А + □ + А = 10- □. откуда определяется □, после чего и А. 36. Как и в предыдущей задаче, анализ, очевидно, начнем из сопоставления второго и третьего числа. Что вы заметили? Верно, произведение цифр двузначного числа обозначено цифрой единиц этого числа. Какова же цифра десятков, то есть А? Дальше определяются цифры десятков и единиц трехзначного числа. (Они одинаковы.) 37. Наиболее «прозрачный» способ решения-записать все цифры сто- значного числа (сто знаков!) и выполнить последовательно требования задачи. Но это и наиболее нерациональный способ, будем искать более рациональный. 11
Обратим внимание на то, какие цифры стоят на нечетных местах и какие-на четных. Какие цифры остаются после первого вычеркивания 50 цифр? После вычеркивания еще 25 цифр пронаблюдаем на первом отрезке из нескольких цифр, в какой последовательности запишутся оставшиеся цифры. После вычеркивания 13 цифр запишем оставшиеся 12 (эта процедура уже не займет много места и времени) и доведем решение до конца. 38. Нелегкую задачу задал ребятам учитель. Поищем такие двузначные числа, каждое из которых делится на 23. Сколько их? Верно, 5 (учитывая и 00). Попробуем «выстроить» их в ряд так, чтобы последняя цифра первого была первой цифрой второго и т. д. Можно ли «продолжить» это число, приписывая цифры слева или справа так, чтобы выполнялось условие делимости на 23? (Это и будет доказательством, что число записано наибольшее.) 39. Подумаем, нельзя ли воспользоваться идеями решений задач 24, 25, 30. Да, выразив разность между искомым числом, цифры которого а, Ь и с, и ему обращенным, исследуем полученное выражение, определив все возможные наборы а, Ь, с, а затем непосредственным испытанием отберем искомые числа. 40. В написанном ряду чисел найдем группу цифр, которые повторяются. Для доказательства пункта б) надо, во-первых, показать, что всегда в записи ряда чисел появится однозначное число, после которого все последующие будут однозначными; во-вторых, найти наименьшее число, которое при этом может появиться; в-третьих, исследовать все случаи появления первого однозначного числа в записи ряда чисел.
АРИФМЕТИЧЕСКИЕ ДЕЙСТВИЯ НАД НАТУРАЛЬНЫМИ ЧИСЛАМИ И НУЛЕМ И ИХ СВОЙСТВА Необходимость выполнять арифметические действия (вычислять) так же, как и считать, диктуется практикой, самой жизнью. Умениями вычислять люди овладевали постепенно с очень давних времен. Представьте себе: земледельцы (а выращивать урожай люди научились в далекие времена) собрали с одного участка 25 мерок зерна, а с другого -15 и засыпали в одно хранилище. Им надо знать, сколько всего у них зерна. Можно, конечно, перемерить и посчитать, но это неудобно и долго. Как сделать проще, вы уже знаете: надо найти сумму двух чисел. Ну а если к тому же надо узнать, на сколько хватит этого зерна, если расходовать по 4 мерки в месяц, то необходимо уметь и делить. Вот такие жизненные задачи (а они встречались на каждом шагу) принуждали человека изобретать правила выполнения действий над числами. Самыми древними числами были натуральные (от латинского слова natur- природа). Понятие арифметических действий в разные времена у разных народов было различным. Древние египтяне к арифметическим действиям относили сложение, удвоение и деление пополам. Позже некоторые европейские ученые (XIII в.) насчитывали 9 арифметических действий, в том числе и нумерацию. В первом учебнике по математике для «российского юношества»-«Арифме- тике»-Л. Ф. Магницкого (1703) нумерация чисел тоже относилась к арифме- тическим действиям. Для обозначения арифметических действий сначала употреблялись слова, затем-буквы. Знаки « + », « —» и точка как знак умножения впервые употреблены в учебниках по арифметике в XV в., а знак деления (две точки)-в XVII в., но окончательно все эти знаки утвердились в работах выдающегося немецкого ученого Г. В. Лейбница (XVII в.). Не правда ли, что знаки эти удобны в употреблении и их совсем нетрудно запомнить и записать? Справедливы полушуточные стихи: 13
Как нет на свете без ножек столов, Как нет на свете без рожек козлов, Котов без усов и без панцирей раков, Так нет в математике действий без знаков. При решении задач настоящего параграфа надо не только уметь хорошо вычислять, используя знания об арифметических действиях и их свойствах, но и проявить смекалку. При выполнении громоздких вычислений в задачах этого и других параграфов обращайтесь к вашему помощнику-калькулятору. ЗАДАЧИ 1. Для подарка сыну отец купил 4 книги. Все книги без первой стоят 84 к., без второй -80 к., без третьей-76 к., без четвертой-72 к. Какова стоимость каждой книги? 2. Пять учеников купили 100 тетрадей. Коля и Вася купили 52 тетради. Вася и Юра-43, Юра и Саша-34, Саша и Сережа-30. Сколько тетрадей купил каждый из них? 3. В школе с I по VIII класс имеется только по одному классу. В I — IV классах всего 130 учеников. В V классе^на 7 учеников больше, чем во II, в VI-на 5 учеников меньше, чем в I, в VII-на 10 учеников больше, чем в IV, и в VIII-на 4 ученика меньше, чем в I. Сколько всего учеников в I —VIII классах? 4. Витя и Вова собрали вместе 27 кг макулатуры. Если бы число килограммов макулатуры, собранной Витей, увеличить в 5 раз, а собранной Вовой - в 3 раза, то у них вместе было бы 111 кг. Сколько килограммов собрал каждый мальчик? 5. В течение трех дней в буфете продавали сок. В первый день продали 1 большую и 4 средних банки, во второй-2 больших и 6 литровых банок и в третий -1 большую, 3 средних и 3 литровых банки. Сколько литров сока продано за 3 дня, если ежедневно продавали одинаковое количество сока? 6. Сумма уменьшаемого, вычитаемого и разности 1024. Найти умень- шаемое, вычитаемое и разность, если разность меньше вычитаемого на 88. 7. По тропинке вдоль кустов Это вместе шли куда-то Шло одиннадцать хвостов, Индюки и жеребята. Насчитать я также смог, А теперь вопрос таков: Что шагало тридцать ног. Сколько было индюков? Спросим также у ребят: Сколько было жеребят? 8. Сколько разных произведений, кратных десяти, можно образовать из чисел 2, 3, 5, 7, 9? 9. Магазину надо было получить со склада 185 кг конфет в закрытых ящиках. На складе имеются ящики с конфетами по 16 кг, 17 кг, 21 кг. Каких ящиков и сколько мог получить магазин? 10. Килограмм конфет одного сорта на 80 к. дороже, чем другого. Андрей и Юра купили по 150 г конфет, среди которых были конфеты того и другого сорта, причем у Андрея конфет первого сорта вдвое больше, чем второго, а у Юры тех и других поровну. Кто из мальчиков уплатил за покупку больше и на сколько?
11. Сколько ударов за сутки сделают часы, если они отбивают целое число часов да еще одним ударом отмечают середину каждого часа? 12. У Змея Горыныча 2000 голов. Сказочный богатырь может отрубить ему одним ударом меча 33, 21, 17 или 1 голову, но при этом у Змея вырастает в замену соответственно 48, 0, 14, 349 голов. Если отрублены все головы, то новые не вырастают. Может ли богатырь победить Змея? Как ему надо действовать? 13. В новом девятиэтажном доме, в котором первый этаж отведен под магазины, семья Сережи получила квартиру 211. На каком этаже и в каком подъезде находится эта квартира, если на третьем этаже одного из подъездов этого дома находятся квартиры от 55 до 60? (Все подъезды и этажи одинаковы.) 14. Расшифруйте запись ** + *** = ♦***, если известно, что оба слагаемых и сумма не изменятся, если прочитать их справа налево. 15. Чтобы открылись ворота в сказочный город Числоград, надо было на табло при въезде |в| । । । । г । is-т зажечь числа в свободных клетках так, чтобы сумма чисел, стоящих в любых трех соседних клетках, равнялась 20. Помогите ребятам попасть в этот город. 16. Магазин отпустил две партии тетрадей. Если бы во второй партии оказалось на 5 тетрадей больше, чем там было на самом деле, то произведение чисел, выражающих количества тетрадей в каждой партии, увеличилось бы на 1250, а если бы в первой партии оказалось на 7 тетрадей больше, то это произведение увеличилось бы на 3150. Сколько всего тетрадей отпустил магазин? 17. Найдите наибольшее число, которое при делении на 31 в частном дает 30. 18. В одном банковском документе было записано, что из имевшейся суммы ежедневно выдавалось денег поровну, причем имевшаяся первоначаль- но сумма денег в рублях, число дней, выдаваемая ежедневно сумма и оставшаяся после последней выдачи сумма оканчиваются соответственно на 7, 5, 3 и 1. Взглянув на такую запись, ревизор сразу же заметил, что она ошибочна. Какое он имел для этого основание? 19. Четверокласснику Диме было задано на дом выразить некоторое целое число часов в секундах. Он красиво написал ответ в тетради и тут же закрыл ее. Придя в школу, Дима обнаружил, что две цифры расползлись в кляксы 234, 2* О Ко всему он забыл, какое именно число часов нужно было выразить в секун- дах. Какие цифры нужно написать вместо клякс? 20. Журнал состоит из 16 вложенных друг в друга двойных листов. На каком двойном листе сумма чисел, обозначающих номера страниц, будет наибольшей? 21. Пять участников соревнования стали его призерами, набрав по 20, 19 и 18 очков и заняв соответственно первое, второе и третье места. Сколько участников завоевали каждое призовое место, если вместе они набрали 94 очка? 22. Членам кружка «Умелые руки» надо’ было разрезать кусок проволоки 15
длиной 102 см на части длиной 15 см и 12 см и так, чтобы не было обрезков. Дети нашли все возможные решения. Найдите и вы их. 23. Представьте число 231 в виде суммы нескольких натуральных чисел так, чтобы произведение этих слагаемых тоже равнялось 231. 24. Задача Л. Н. Толстого. Пять братьев разделили после отца наследство поровну. В наследстве было три дома. Так как три дома разделить нельзя было на 5 частей, то их взяли 3 старших брата, а меньшим за то выделили деньги. Каждый из трех братьев заплатил по 800 р., меньшие братья разделили эти деньги между собой, и тогда у всех стало поровну. Много ли стоит один дом? 25. В каждом числовом ряду а)-д) числа следуют в определенной. закономерности. Установите эти закономерности и запишите еще по 2 числа: а) 19, 20, 22, 25, 29, ... б) 5, 8, 14, 26, 50, ... в) 253, 238, 223, 208, 193, ... г) 12, 11, 16, 16, 20, 21, 24, 26, ... д) 15, 29, 56, 109, 214, ... 26. Вот задача не для робких! Вычитай, дели и множь, Плюсы ставь, а также скобки! Верим, к финишу придешь! 5555 = 3 5555 = 7 5555 = 55 5555 = 4 5555 = 26 5555 = 120 5555 = 5 5555 = 30 5555 = 130 5555 = 6 5555 = 50 5555 = 625 27. Употребляя цифру 7 по 4 раза, знаки действий и скобки, представьте все числа от 1 до 10 включительно. 28. Сколько потребуется стандартных листов бумаги, чтобы напечатать все числа от 1 до 1000000 включительно, если печатать на одной стороне листа? На одном листе 30 строк по 60 знаков в каждой, между числами-ин- тервалы, каждый из которых равнозначен двум знакам. 29. Найдите ошибку в каждом из этих примеров: а) 225:25 + 75= 100- 16 25(9:1 + 3) = 84 25-12 = 84 25 = 7(?!) б) 5005 - 2002 = 35 • 143 - 143 • 14 5005 - 35 143 = 2002 - 143 -14 5(1001 - 7-143) = 2(1001 -7-143) 5 = 2 (?!) 30. Задумайте однозначное число: удвойте его, прибавьте 3, умножьте на 5, прибавьте 7, пользуясь последней цифрой полученного результата, запишите однозначное число, прибавьте к нему 18 и разделите результат на 5. Какое число получили? Какое бы число ни задумали, всегда получите в конечном результате одно и то же число. Объясните почему. 31. Частное от деления одного числа на другое есть целое число, в два раза меньшее одного из них и в 6 раз большее другого. Найдите это частное. 16
32. Если каждому из своих детей мама даст по 13 тетрадей, то у нее останется 8 тетрадей; если же она им даст по 15 тетрадей, то все тетради будут розданы. Сколько тетрадей было у мамы? 33. Разделив некоторое целое число на 15, Сережа получил в остатке 8, а разделив его на 20, он получил в остатке 17. Покажите, что Сережа ошибся. 34. В книге записей целое число, выражающее количество метров продан- ной материи, было залито чернилами. Нельзя было также прочитать общую выручку, но было видно окончание этой записи: 7 р. 28 к. -и известно, что эта сумма не превышает 500 р. Цена 1 м материи 4 р. 36 к. Помогите ревизору восстановить эту запись. 35. Задумайте число, записанное в одном столбце: 10 23 16 29 32 27 15 28 31 9 14 32 30 8 26 36 24 12 25 13 23 16 24 17 30 Как отгадать задуманное число по сумме чисел (без этого числа) строки или столбца, в которых записано задуманное число? (Предложите этот фокус своим товарищам.) 36. Умножьте размер обуви на 2, прибавьте к произведению 39, умножьте полученную сумму на 50, прибавьте к произведению 40, вычтите из суммы год своего рождения. Вы получите четырехзначное число, первые две цифры которого-номер обуви, а две последние-возраст в конце календарного 1990 г. Объясните почему. (Предложите этот фокус своим друзьям.) 37. Задумайте трехзначное число: удвойте первую его цифру (число сотен), к полученному произведению прибавьте 3, умножьте полученную сумму на 5 и прибавьте к полученному результату вторую цифру задуманного числа (число десятков). Наконец, к полученной сумме справа припишите последнюю цифру задуманного числа. Докажите, что для получения задуманного числа достаточно из полученного результата вычесть 150. 38. На занятии математического кружка ребятам был предложен такой математический фокус: Напиши число любое, На него я погляжу И затем клочок бумаги При тебе в конверт вложу. Под своим числом любое Напиши число опять, И число одно позволю Сам себе я написать. Проведем черту и сложим Наших чисел стройный ряд. Нет ошибки? Осторожней?! Все в порядке? Очень рад! Сумма-я прошу, проверьте!- У меня давно в конверте! В чем тут фокус? В чем секрет? Сообщи мне свой ответ. Попробуйте раскрыть секрет этого фокуса. 17
39. В соревновании по составлению и решению математических задач каждый из трех классов должен был составить по 5 задач и предложить их для решения двум другим классам, а также решить задачи, составленные клас- сами-соперниками. За каждую правильно решенную задачу насчитывалось 5 баллов тому классу, который ее решил, и за каждую нерешенную задачу насчитывалось 4 балла тому классу, который является ее автором. При подведении итогов оказалось, что 5 А решил только одну из задач, составлен- ных 5 Б классом, и 4 задачи, составленные 5 В классом; 5 Б класс решил 3 задачи, составленные 5 В классом, и 2 -задачи, составленные 5 А классом; 5 В решил 4 задачи, составленные 5 А классом, но не решил столько же задач, составленных 5 Б классом. Сколько баллов набрал каждый класс и как распределились между ними места? 40. В футбольном турнире каждая из участвующих команд сыграла с каж- дой по одному разу. Команды набрали 16, 14, 10, 10, 8, 6, 5, 3 очков. Сколько команд участвовало в турнире и сколько очков потеряли команды, участво- вавшие в игре и занявшие первые 4 места? (За выигрыш команда получает 2 очка, за ничью-одно.) 41. Имеется 67 гирь массой I г, 2 г, 3 г, ..., 67 г. Можно ли разложить их на три равные по массе группы? 42. В обычном наборе домино 28 косточек. Сколько косточек содержало бы домино, у которого значения, указанные на косточках, изменялись бы не от 0 до 6, а от 0 до 12? 43. Можно ли в четырехсотзначном числе 86198619 ... 8619 вычеркнуть несколько цифр в начале и в конце так, чтобы сумма оставшихся цифр равнялась 1986? 44. Найти сумму всех возможных различных трехзначных чисел, все цифры которых нечетные. 45. Таня и Оля долго пытались расставить девять чисел от 1 до 9 по окружности так, чтобы сумма любых трех чисел, стоящих подряд, делилась на 3 и была больше 12, но безуспешно. Можно ли так расставить числа? 46. Найдите 4 тройки целых неотрицательных чисел, таких, чтобы каждое число от I до 81 включительно можно было представить в виде суммы четырех чисел-по одному из каждой тройки. 47. За последние пять лет было награждено 27 колхозников. Причем, в каждом последующем году награждалось больше, чем в предыдущем. В последний год было награждено в 3 раза больше, чем в первый. Сколько колхозников было награждено в третий год? 48. Сделав по 5 выстрелов по мишени, Коля и Петя выбили 10, 9, 9, 8, 8, 5, 4, 4, 3, 2 очков. Первыми тремя выстрелами они выбили по одинаковому числу очков, а тремя последними Коля выбил в 3 раза больше, чем Петя. Сколько очков выбил каждый из них третьим выстрелом? 49. Сережа записал некоторое пятизначное число и умножил его на 9. К своему удивлению, он получил в результате число, записанное теми же цифрами, но в обратном порядке. Какое число записал Сережа? 50. В примере 1985 **: 102 = **** (деление без остатка) восстановите цифры, обозначенные звездочками. 51. Если в некотором шестизначном числе переставить крайнюю слева цифру 7 в конец числа, то получим число в 5 раз меньше первоначального. Найдите первоначальное число. 18
52. Когда число ПОТОП взяли слагаемым 99999 раз, то получили число, три последние цифры которого 285. Какое число обозначено словом ПОТОП? (Одинаковыми буквами обозначены одинаковые цифры.) 53. Веселый клоун Нибумбум Сегодня мрачен и угрюм. Что огорчает Нибумбума? Пример решал он восемь раз, И каждый раз другая сумма! Печальный случай! (А у вас?) При решеньи не забудьте (В том-то вся и тонкость смысла!) Одинаковые буквы - одинаковые цифры! КОШКА + К О Ш К А КОШКА СОБА КА 54. Задача очень непроста- Найти не каждый сможет: Чему равняется звезда, Велосипед и ежик? ВЕЛОСИПЕД ЕЖИК 7 + ЗВЕЗДА ЕЖИК 4 6 ВЕЛОСИПЕД ЕЖИК 1 ВЕЛОСИПЕД О ЗВЕЗДА Расшифруйте ребусы: 55. ШЕПНУЛ ШЕПНУЛ + Ш Е П Н У Л ШЕПНУЛ ШЕПНУЛ 56. Д В Д в + о л л ч о л А А О ч и с л О К Р ИКНУЛ 57. о ДИН 58. Д В А + X о дин Д В А и ДВА * * * * _|_ ♦ ♦ ♦ в Е * * * Ч Е Т Ы Р Е (Одинаковым буквам соответствуют одинаковые цифры.) 19
59. Петя и Вася рассчитали, что если они будут идти пешком, проходя пс 4 км в час, в соседнюю деревню, находящуюся в четырех километрах от них то опоздают на 10 мин на футбольный матч, проводившийся там на первенство района. Как они должны поступить, чтобы прибыть на матч одновременно и получить наибольший выигрыш во времени, имея в своем распоряжении велосипед, на котором можно ехать только одному, но в три раза быстрее, чем идти пешком? За сколько минут до начала матча при этом они прибудут? 60. Винни-Пух и Пятачок одновременно отправились в гости друг к Другу. Но поскольку Винни-Пух всю дорогу сочинял очередную «шумелку», а Пята- чок считал пролетавших ворон, ойи не заметили друг друга при встрече. После встречи Пятачок подошел к дому Винни-Пуха через 4 мин, а Винни-Пух подошел к дому Пятачка через 1 мин. Сколько минут был в пути каждый из них? 61. Сто учеников, у каждого из которых на майке был номер, втали по окружности в такой последовательности своих номеров: 1, 2, 3, ..., 100. По команде стали выходить так: с номером 1 на майке остается, с номером 2 выходит, с номером 3 остается, с номером 4 выходит и т. д. через одного по окружности до тех пор, пока не остался один ученик. Какой номер на майке у этого ученика? 62. Впишите в свободные кружки натуральные чис- ла так, чтобы в них оказались все числа от 1 до 19 и во __ всех пяти рядах каждого из трех направлений (верти- кального и двух наклонных) сумма чисел, стоящих в одном ряду, была одной и той же. 63. Записываются сначала два числа 2 и 4, затем их сумма—число 6, затем сумма двух последних одно- значных чисел 4 4- 6 = 10, дальше 14-0=1, затем ) 04-1 = 1, 14-1=2 и т.д. В результате получается такой ряд: .zx ) 246101 1235813... Какие цифры не могут встретиться в этой записи, если как угодно далеко продолжить этот ряд? Какая цифра будет на сотом месте? 64. Записываются сначала два числа 2 и 3, затем они перемножаются и записывается их произведение-число 6, затем записывается произведение двух последних однозначных чисел 3-6 = 18, дальше 1*8 = 8, затем 8*8 = 64 и т.д. В результате получим такой ряд цифр: 2361886424... Какие цифры не могут встретиться в таком ряду? Какая цифра стоит на тысячном месте? 65. Сколько надо взять слагаемых суммы 1 4- 2 4- 3 4-..., чтобы в результате сложения получить трехзначное число, в записи которого все цифры одинаковы? 20
Поиски решений 1. Как можно сказать иначе, что «все книги без первой стоят 84 к.»? Верно, это стоимость второй, третьей и четвертой книг. Значит, стоимость каждой книги в отдельности нашли бы, если бы знали стоимость всех книг. Для этого можно найти утроенную их стоимость. 2. Число тетрадей, купленных каждым учеником в отдельности, можно было бы найти, если бы сумели найти, сколько тетрадей купили четверо из пяти учеников. Нельзя ли найти, сколько тетрадей купили Коля, Вася, Юра и Саша? 3. Чтобы найти число учеников I —VIII классов, надо знать, сколько учеников в V —VIII классах. Для этого надо найти, на сколько больше или меньше учеников в V-VIII классах, чем в I — IV. 4. Если утроить 27 и результат сравнить с числом 111, то какой вывод можно сделать? Что покажет разность между этими числами? 5. Для определения числа литров сока, проданного за 3 дня, надо знать емкости средней и большой банок. Для этого следует сравнить проданное за каждый день. (Здесь полезна схематическая запись условия.) 6. Так как по условию известна сумма уменьшаемого, вычитаемого и раз- ности, то, зная, что уменьшаемое равно сумме вычитаемого и разности, легко найти уменьшаемое. 7. Так как «шло одиннадцать хвостов», то сколько было всего индюков и жеребят? Число каждого вида в отдельности можно найти, сопоставляя число ног индюка и жеребенка и зная, что «шагало тридцать ног». (Учтите, что замена индюка на жеребенка не приведет к изменению общего количества хвостов (голов), но общее число ног увеличится на 2.) 8. Для подсчета нужных произведений можно их все составить исходя из того, что в каждом из них непременно должно быть 2 числа (множителя) из данных. Какие это числа? 9. Для решения задачи можно пытаться оперировать числами 16, 17 и 21 для получения числа 185. Однако это подобно плаванию корабля без компаса. Не лучше ли поступить наоборот: «анатомировать» число 185 для получения нужных комбинаций? Например, представить число 185 сначала в виде произведения, а затем один из множителей-в виде суммы двух слагаемых. 10. За покупку заплатил больше тот, у кого больше конфет первого сорта. Масса конфет каждого сорта у Андрея и Юры определяется легко. А значит, и нетрудно найти, на сколько больше конфет первого сорта у одного, чем у другого. Учитывая разницу в стоимости 1 кг конфет разных сортов, можно найти разницу в стоимости конфет любой массы. 11. Для решения задачи достаточно просуммировать числа ударов за 12 ч 21
(полсуток), а затем результат удвоить. (Для рационального подсчета суммы воспользуемся свойством сумм чисел, равноотстоящих от концов в записи I +2 + ... + II + 12.) 12. Надо, конечно, действовать так, чтобы победить с наименьшей затра- той сил и времени. Не советуйте богатырю рубить 1 голову, так как после этого сразу вырастет 349. А вот если срубить 21 голову, то не вырастет ни одной. Это соблазнительно, но будьте осторожны! Замахнувшись 95 раз рубить по 21 голове, останется после этого... Сколько же останется голов? То-то и оно! Значит, срубая по 21 голове, надо где-то, на каком-то шаге, изменить тактику. Подумаем: сколько же раз надо рубить по 21 голове? Затем: как же рубить оставшиеся 26 голов? 13. Для определения этажа и подъезда, где находится квартира указанного номера, надо знать число квартир на одном этаже подъезда. Такущ информа- цию условие задачи дает: на третьем этаже одного из подъездов квартиры от 55 до 60. Учитывая, что первый этаж занят под магазины, можно найти число квартир в одном подъезде, а дальше уже недалеко и до цели. 14. .Целесообразно начать поиски решений с установления вида первого слагаемого (двузначного числа), не изменяющегося при прочтении слева направо и наоборот. Какими должны быть его цифры? Учитывая; что сумма - число четырехзначное, испытаем возможные значения второго слагаемого (трехзначного числа). 15. Решать задачу, вписывая числа наугад,-дело утомительное и мало- перспективное. Попробуем исследовать и установить закономерность в рас- положении чисел. Для этого обозначим буквами по порядку числа, которые надо вписать в пустые клетки, и запишем равенства по условию задачи. 16. Для решения задачи необходимо использовать правило об изменении произведения от изменения одного из множителей. Вы его можете вывести сами, если обратитесь хотя бы к конкретным примерам. 17. Зная делитель, всегда можно указать наибольший из возможных остатков. Каков он по сравнению с делителем? 18. Чтобы обнаружить ошибку в записи, достаточно воспользоваться зависимостью между делимым, делителем, частным и остатком. Будем искать решение, установив, какие числа (какими цифрами оканчиваются) в данной ситуации выступают в роли делимого, делителя, частного и остатка. 19. Будем искать решение, приняв во внимание, что число, в котором расползлись две цифры, получено в результате умножения числа секунд в одном часе (какое это число?) на целое число часов. Первой определяется вторая цифра справа, а затем-вторая неизвестная цифра путем деления. 20. Прежде всего надо уяснить, как расположены страницы на двойных листах, вложенных один в другой; записать их и проверить сумму. 21. Замечаем, что некоторые участники набрали одинаковое число _очков. Зная, по сколько очков набрали три призера и сколько все пять, можно найти, сколько очков набрали остальные два, а тогда уже и сколько каждый. 22. Прежде всего выясняем, может ли число частей по 15 см бы гь не- четным. Исходя из этого рассматриваем все возможные случаи и устанавли- ваем возможное число частей по 15 см и по 12 см. 23. Значительно меньше способов представления числа 231 в виде про- изведения, нежели в виде суммы. Очевидно, с этого и следует начинать. Если окажется, что полученных множителей будет недостаточно уже в качестве 22
слагаемых для получения нужной суммы, то поискать такие числа, которыми можно пополнить. (Вспомните свойство числа 1 при умножении.) 24. Для определения стоимости одного дома надо знать стоимость всего наследства (трех домов). К определению стоимости наследства следует, очевидно, идти используя число-сумму денег, полученных двумя братьями. 25. Сравнивая сосеДние числа в рядах а)-в), устанавливаем, в какой закономерности следует возрастание или убывание чисел. Труднее дело обстоит с числами ряда г). Попробуем выделить здесь два ряда. Для получения каждого следующего числа из предыдущего ряда д) надо выпол- нить два действия. Попробуем их найти. 26- 27. Поищите несколько решений этих задач, отличных от приведенных в решениях и ответах. 28. В поисках, решения этой задачи-расчета потребуется найти количество чисел различной значности-от однозначных до семизначных и цифр для их написания. (Не ошибитесь в определении количества интервалов!) 29. В задаче-софизме создается внешняя видимость правильности пре- образований и вычислений, но допускаются ошибки - нарушения законов и правил математики. Чтобы их обнаружить, следует последовательно кри- тически оценить весь ход операций. 30. Для объяснения причины следует взять число х (или а) и выполнить последовательно все указанные операции. Используйте этот пример для составления фокуса, который проведите с друзьями. Его можно разнообра- зить, предлагая другие числа. 31. Частное можно найти и сразу, но, чтобы было понятнее и убедительнее, проделайте в общем виде то, о чем идет речь в условии. 32. Речь идет о нахождении такого числа, которое при делении на 15 и на 13 дает одинаковые частные, но в первом случае остатка нет (остаток 0), а в другом-остаток 8. Решить можно или подбором, или в общем виде, обозначив делитель (число детей) буквой. 33. Чтобы обнаружить ошибку Сережи, надо, обозначив частное, записать равенство выражений и показать, что оно не может быть верным. 34. Решение будем искать, сопоставляя общую стоимость и цену одного метра. При этом можно определить, какой цифрой может оканчиваться число Петров проданной материи. Если оно оканчивается цифрой 8. то, записав: ,3 4 8 8 + ... х ... 7 2 8 можно определить, какими могут быть х и у. 35. Секрет сразу раскрывается, если подсчитать и сравнить сумму чисел каждой строки и столбца. Можно составить и другие таблицы. 36. Для выяснения причины получения такого «интересного» четырех- значного числа размер обуви целесообразно записать в общем виде дву- значным числом аБ и последовательно проделать операции, данные в условии задачи. (Некоторые числа условия можно изменить.) 37. Попытка вести доказательство, представляя конкретные примеры, не может увенчаться успехом. В данном случае следовало бы рассмотреть 900 23
примеров. Представляете, что это за работа? Поэтому для доказательства надо, очевидно, брать трехзначное число в общем виде. Это мы умеем делать (см., например, задачу 24 «Нумерация»). Потребуется также перевести на «язык» арифметических действий приписывание к числу справа данной цифры. Например, имеем число 3, приписали 5, получили 35. Какие действия надо выполнить, чтобы, имея число 3, получить 35? 38. А можем ли мы сразу записать сумму двух слагаемых, если одно из них, например, 2538, а другое 9999? Найдите эту сумму и вы увидите, как ее можно было сразу записать, зная слагаемое 2538. Не правда ли слагаемое, записанное девятками, обладает интересным свойством? Значит, для получения такого числа (в записи которого толь- ко девятки) надо правильно выбрать третье записываемое число отга- дывающим. 39. Составление таблицы по условию задачи облегчит поиски решения. 40. Число всех набранных очкЬв нетрудно выразить при помощи числа сыгранных игр. Если бы удалось число сыгранных игр выразить при помощи числа всех команд п, то можно было бы записать равенство, в одной части которого было бы число очков, выраженное через п, а в другой-сумма данных в условии задачи набранных командами очков, и тогда можно было бы определить п (число команд). 41. Поиски решения сводятся к установлению возможности делимости на 3 суммарной массы всех гирь. Для определения суммы используем прием решения задачи 11. 42. Для подсчета всех косточек необходимо знать, сколько косточек с нулем, с единицей (но без нуля) и т. д. Суммирование выполняем по способу задачи 41. 43. Зная, что сумма цифр числа 1986 (или иного четырехзначного числа, составленного из этих цифр) равна 24, можно определить, сколько раз это число содержится в 1986 и каков при этом будет остаток. Рассматривая цифры четырехсотзначного числа слева направо, выясняем, какие цифры и сколько надо вычеркнуть, затем-сколько раз оставить группу цифр (1, 9, 8, 6) и сколько цифр вычеркнуть в конце четырехсотзначного числа. 44. Сумму можно найти, если записать слагаемые, например, в порядке их возрастания (разумеется, писать будем не все слагаемые), определить число слагаемых, запись которых начинается с цифр 1, 3, 5, 7, 9, а затем-их общее количество и воспользоваться рациональным способом вычисления этой суммы (см. задачи 41, 42). 45. Отыскать способ решения (если оно существует) или показать, что задача не имеет решения, можно лишь при условии, если будем рассматривать задачу в несколько обобщенном виде, обозначая, например, числа а2, ..., а9, где а может быть любым числом от 1 до 9. Тогда, рассматривая суммы этих чисел по 3 (как указано в условии), можно найти их общую сумму, а затем показать, что эти суммы чисел по 3 равны между собой. 46. Какими должны быть слагаемые, чтобы получить при сложении четырех целых чисел 1? (Не забыли о числе 0?) Для удобства запишем эти 4 числа в столбец. Если верхнее число этого столбца 1, то можно записать верхнюю строку из трех чисел. Далее находим последовательно остальные числа второй строки, третьей и четвертой, принимая во внимание, что должны получить в сумме 4, 5, 6 и т.д. (Придется немного потрудиться.) 24
47. Задача будет решена, если удастся найти такие 5 чисел, чтобы наимень- шее из них было в 3 раза меньше наибольшего, каждое последующее больше предыдущего и сумма всех чисел равнялась бы 27. Такие числа можно найти путем испытания значений для наименьшего числа награжденных, начиная с 1. 48. Так как надо обеспечить выполнение условия задачи, что тремя последними выстрелами один выбил в 3 раза больше очков, чем другой, то будем искать наименьшее возможное число очков, которые можно выбить тремя выстрелами. Определив при этом, по сколько очков выбили Петя и Коля тремя последними выстрелами, можно распределить между ними оставшиеся очки, что при анализе приведет к ответу на вопрос задачи. 49. Определяется крайняя левая цифра исходного пятизначного числа, так как при умножении этого числа на 9 получается число той же значности. Но так как при умножении на 9 получаем число, обращенное первому мно- жителю, то найденная цифра будет последней цифрой числа-произведения. Зная ее, определяем последнюю цифру первого множителя. Продолжая этот анализ, найдем остальные цифры. В поисках решений целесообразно вести последовательность записи. 50. Неизвестные цифры можно найти, выполняя деление «под углом» по известным правилам (говорят: «используя алгоритм деления»). 51. Так как при делении шестизначного числа, у которого крайняя левая цифра 7, на 5 получаем число шестизначное, то можно записать равенство: 1 ♦♦♦♦ 7 • 5 = 7**** 5. Для определения остальных цифр каждый раз сопостав- ляем числа левой и правой частей равенства, принимая во внимание, что число 1 7 получено путем перенесения в числе 7 ♦♦♦• 5 цифры 7 в его конец. 52. Поскольку известны три последние цифры результата, то для отыска- ния неизвестных цифр, зашифрованных буквами, весьма желательно свести задачу к выполнению действия сложения или вычитания. Возможно ли это? Взять слагаемым число несколько раз-это значит выполнить действие умножение. Но умножение на 99999 можно заменить двумя, однако, более удобными действиями. Какими? Выполнив действие умножение на число вида 100... 0 и вычтя первый множитель, можно записать равенство, в результате анализа которого можно найти значение букв, т. е. достичь цели. 53. Обратив внимание на то, что последние две буквы (цифры) слагаемых и суммы одинаковы, постараемся их расшифровать. Понятно, что одна из этих букв (или А, или К) означает 0, а другая-5. Может ли А = 5, чтобы К = 0? Остальные буквы, рассматриваемые справа налево, расшифровыва- ются в зависимости от этих двух. 54. Расшифровку ребуса попробуем начать с рассмотрения средней колон- ки слагаемых и их суммы. При сложении двух одинаковых чисел и третьего, отличного от них, при условии передачи единицы из низшего разряда получаем число, оканчивающееся цифрой 0. Какой же может быть сумма ЕЖИК + ЕЖИК + ВЕЛОСИПЕД? Из двух значений удовлетворяет лишь одно. Имея сумму трех слагаемых (ЕЖИК, ЕЖИК, ВЕЛОСИПЕД), устанавливаем, какие слагаемые удовлет- воряют условию задачи. Получив «ключ», легко откроем «замок». 55. Так как цифры-первая справа, вторая и третья слагаемых-такие же, как соответствующие цифры суммы, то расшифровку ребуса и следует начинать с них, последовательно продвигаясь влево. Из двух возможных 25
значений 0 и 5 удовлетворяет Л лишь одно (надо смотреть на соседнюю цифру слева). Остальные цифры выбираем из оставшихся методом проб. 56. При умножении на трехзначное число в промежуточных вычислениях получаем не три, а два слагаемых. Нельзя ли определить одну из цифр множителя? При умножении трехзначного числа ДВА на однозначное Д получаем трехзначное. Можем получить 2 значения для Д. (При дальнейшем анализе отбираем из двух одно.) Значения А и О получим из рассмотрения ДВА А = ОЛЛО. 57. С чего начать анализ и расшифровку ребуса? Верно, с буквы Д, так как она может принимать только два значения в зависимости от того, имеется передача единицы из соседнего низшего разряда или нет. Дальше исследуем значения И и в зависимости от него А и О. (Найдите все 13 решений.) 58. Рассматривая ДВА А, видим, что результат оканчивается на Е. Из этого можно сделать вывод, какие значения не может принимать А. Но так как ДВА-В оканчивается на В, то для А и В допустимы лишь определенные значения. Какие? В заключение определяем Д. 59. Важно заметить, что ребята отправляются одновременно и прибывают в конечный пункт одновременно. Кроме того, надо получить наибольший выигрыш во времени, т.е. быть все время в движении как одному, так и другому. Имеется в виду здесь, что велосипед можно оставлять на дороге без присмотра. Затраченное на дорогу время можно найти, если знать, сколько времени каждый из них шел и сколько ехал на велосипеде. 60. Полезно изобразить схематически условие задачи. Из сравнения по схеме пути движения каждого персонажа можно сравнить скорости и найти время движения. 61. После выхода пятидесяти учеников нетрудно себе представить, с ка- кими номерами ученики остались. Так как во второй раз выходят 25 учеников, то можно установить, в какой последовательности располагаются оставшиеся номера. Дальше выходят 12 учеников, а оставшиеся 13 номеров можно уже все записать и продолжить вычеркивание. 62. Поскольку в одном из рядов известны все числа (3, 17, 18), то имеется возможность найти сумму чисел ряда (постоянную ряда). В тех рядах, в которых по одному свободному кружку, легко определяются неизвестные числа. Так последовательно можно заполнить 4 верхних наклонных ряда и 2 крайних вертикальных. В следующих двух наклонных рядах будет по 2 неизвестных числа. Их можно отыскать, записав два равенства с двумя неизвестными. Оставшиеся 4 нижних кружка нетрудно заполнить, так как останется использовать 4 известных числа. 63. Для решения задачи попробуем выделить в ряду группу цифр, которая повторяется (период). Если это удастся, то можно проверить, будет ли в записи цифра 9, а также можно установить число цифр в периоде, после чего определить цифру, стоящую на сотом месте. 64. Если предыдущая задача решена, то нетрудно найти решение этой задачи. Главное здесь-уметь наблюдать, анализировать и делать выводы. 65. Число слагаемых п можно было бы определить, если бы сумму чисел выразить через это число и исследовать полученное выражение, пытаясь найти такое значение п, при котором значение выражения есть трехзначное число, в записи которого все цифры одинаковы. Найти же сумму чисел от 1 до п можно по тому способу, который был применен в задачах 11, 41, 42. (Трехзначное число полезно представить так: ааа = 11 \ - а = 37-3-а.)
КВАДРАТ и куб числа Вам уже известно, что все арифметические действия взаимосвязаны между собой. Вспомните, как вводили действие умножения во втором классе. Рассматривая сумму одинаковых слагаемых, например 5 4- 5 -F 5, говорили, что по 5 берем 3 раза и будем записывать 5-3, называя это умножением. Теперь в свою очередь произведение одинаковых множителей будем пред- ставлять новым действием. Если, например, имеем 5 * 5, то это записывается 52 и читается «пять в квадрате» (или «квадрат пяти»), или 5*5*5 = 53 и читается «пять в кубе». Вы, очевидно, догадываетесь, откуда пошли эти названия. Если длина стороны квадрата а, то его площадь аа = а2, а если а-длина ребра куба, то его объем а -ста = а\ Квадрат и куб числа-это отдельные примеры возведения в степень, которое к арифметическим действиям не относят. (В дальнейшем вы будете это изучать более основательно.) В записи а" а называют основанием степени, п - показателем, ап-степенью. Квадратами и кубами чисел пользовались еще в древности. Например, в Вавилоне для решения задач составлялись специальные таблицы квадратов и кубов чисел. В сочинении Архимеда (287-212 гг. до н. э.) «Псаммит» используется идея применения степеней для упрощения вычислений. Идея использования так называемых квадратных чисел 1, 4, 9, 16 и т.д. связана со строительной техникой. Основным строительным материалом в Ин- дии и Вавилоне были плиты. Если квадратные плиты условно обозначим точками, то количества плит (то- е * чек), необходимых для построения квадратной фигу- ры, будут выражаться квадратными числами, как это • • • видно из рисунка: 1 + 3 = 4 = 22 L2____9 Ь+3 + 5 = 9 = 32 1 + 3 + 5 + 7 = 16 = 42 и т. д. 27
Так решались задачи о количестве необходимых плит для строительства сооружений. Аналогично можно говорить о числах кубических 1, 8, 27, 64 и т.д. При решении задач этого параграфа используются квадраты и кубы. ЗАДАЧИ 1. Разность двух натуральных чисел, являющихся точными квадратами, оканчивается цифрой 2. Какими цифрами оканчиваются уменьшаемое и вы- читаемое, если последняя цифра уменьшаемого больше последней цифры вычитаемого? 2. Запишите в ряд первые 15 последовательных нечетных чисел. Разбейте числа этого ряда на группы так, чтобы в первой группе было одно число, во второй-последующие два, в третьей-три и т.д. Найдите сумму чисел каждой группы и укажите общую для всех групп закономерность. 3. Установите правило, по которому составлена таблица, и впишите недостающие числа: 9 81 2 16 256 2 11 11 6 216 3 5 3 4. Когда при возведении числа 4 в различные степени Саша получил три числа, у которых все три цифры единиц различные, Андрей, не вы- числяя результатов, заметил, что Саша ошибся. Какое он имел для этого основание? 5. Туристы одной из групп покупали различные сувениры, причем каждый из них брал набор сувениров одинаковой стоимости и столько, сколько рублей стоит один сувенир. Все туристы расплачивались десятирублевыми купюрами (одной или несколькими), каждый получил сдачу, которая не совпадала со сдачей ни одного из остальных туристов. Какую сдачу мог получить каждый турист и какое наибольшее число туристов могло быть в группе? 6. Витя нашел такое наименьшее из возможных натуральных чисел, при умножении которого на 2 получается точный квадрат, а при умножении на 3-точный куб. Какое число нашел Витя? 7. Покажите, что число 425102348541 не является точным квадратом натурального числа. 8. В записи некоторого числа имеется 123 единицы, а остальные цифры- нули. Может ли это число быть точным квадратом? 28
9. В примере (**)3 = ***9 вместо звездочек поставьте цифры, чтобы получилось верное равенство. 10. В примере (дП)Л = ПОП цифры зашифрованы геометрическими фигурами. (Одинаковое цифры соответствуют одинаковым фигурам, раз- ные-разным.) Расшифруйте эти цифры. 11. В примере (АД)° = АППЛ цифры зашифрованы геометрическими фигурами. Расшифруйте их. 12. Расшифруйте ребус (АР)М = МИР, если одинаковыми буквами за- шифрованы одинаковые цифры, а разными-разные. 13. Общий урожай кукурузы в центнерах, собранный с некоторого участка поля, выражается четырехзначным числом, записанным при помощи цифр 0, 2, 3 и 5. Когда подсчитали среднюю урожайность с 1 га, то оказалось, что она выражается таким же числом центнеров, как и площадь участка в гектарах. Определите общий урожай кукурузы. 14. Вытирая запись на классной доске, Ира случайно стерла по три цифры в записи вычислений кубов двух чисел, оставив только цифры единиц-8 и 9. Помогите ей восстановить все цифры и найдите числа, которые возводились в куб. 15. Найдите трехзначное число, равное кубу суммы его цифр. 16. Пешеход назвал автоинспектору номер машины, водитель которой грубо нарушил правила дорожного движения. Этол номер выражается четы- рехзначным числом, цифра единиц которого такая же, как цифра десятков, а цифра сотен такая же, как цифра тысяч. Кроме того, это число является точным квадратом. Какое это число? 17. Сумма в рублях, перечисленная одним из предприятий в Фонд мира, выражается таким наибольшим четным пятизначным числом, что первые три его цифры образуют число, являющееся точным квадратом, а последние три цифры-точным кубом. Какая сумма перечислена предприятием в Фонд мира? 18. Замените буквы цифрами так, чтобы в каждой строке (столбце) стояли трехзначные числа, являющиеся точными квадратами. (Одинаковым буквам должны соответствовать одинаковые цифры, разным-разные.) т и Р и в А р А К 19. Впишите в клеточки цифры так, что- бы во всех трех горизонтальных строках и в каждом из пяти вертикальных столбцов получились различные квадраты натураль- ных чисел. 20. - Папа, а у тебя интересный воз- раст,-обратился к отцу сын. - Чем же, сынок?-спросил отец. - А вот смотри, если прибавить к твоему 2»
возрасту, 38 годам, число, записанное теми же цифрами, но в обратном порядке, то получим точный квадрат. - Да, действительно, 38 + 83 = 121 = 111 2,-согласился отец. - А вот теперь давай проверим, не случится ли подобное еще через несколько лет, - предложил отец сыну. И они решили эту задачу. Решите и вы 21. На карточках написаны цифры Сложите с их помощью четырехзначное и двузначное числа, такие, чтобы квадрат одного из них равнялся другому. 22. Члены математического кружка «Ис- катель», заметив, что число 289 можно по- лучить, если возвести в квадрат сумму цифр 8 и 9, т. е. 289 = (8 + 9)2, решили найти такие трехзначные числа с цифрой сотен, меньшей четырех, каждое из которых равно сумме цифр его десятков и единиц, возведенной в степень, показатель которой есть цифра сотен. Помогите им решить эту задачу. 23. Найдите трехзначное число, квадрат которого есть такое шестизначное число, что каждая последующая его цифра, считая слева направо, больше предыдущей. 24. Возвратившись с колхозного огорода, ребята горячо обсуждали итоги работы на уборке ими моркови. «А каковы размеры участка, на котором мы работали?»-обратились они с вопросом к своему бригадиру Васе. «Записи измерений я где-то потерял,-ответил он,-но я помню, что участок квадрат- ный и площадь его в квадратных метрах выражается четырехзначным числом, в записи которого все цифры только четные, но различные». По этим данным ребята нашли длину стороны участка и его площадь. Найдите и вы их. Поиски решений 1. Для решения задачи необходимо знать, какими цифрами могут окан- чиваться квадраты натуральных чисел, что сделать нетрудно, и отобрать те из них, которые удовлетворяют условию задачи. 2. Чтобы установить закономерность для указанных групп чисел, доста- точно выполнить последовательно указанные в условии операции и про- анализировать полученные результаты, представляя их как кубы. 30
3. Поиски решения сводятся к выявлению правила, по которому находятся числа средней колонки по числам двух крайних. 4. Чтобы обнаружить ошибку Саши, достаточно установить, какими цифрами могут оканчиваться степени числа 4. 5. Для решения важно заметить, что число туристов такое же, как количество внесенных ими денежных сумм для оплаты за сувениры или количество полученных различных сдач. Но числами какого вида выражаются оплачиваемые суммы денег? Зная это, можно указать, какими различными цифрами они могут оканчиваться, а значит, и какие сдачи могли быть получены. 6. Прежде всего заметим, что искомое число должно содержать множители 23 и З2. 7. Достаточно установить, что данное число делится на некоторое число, но не делится на его квадрат. (Если, например, а = т • п, но а ф т2к, то число не может быть точным квадратом.) Проще это сделать для 3 и 9. 8. Если найдено решение задачи 7, то решение этой задачи трудности не представит. 9. Если в результате возведения в куб получаем число, оканчивающееся цифрой 9, то можно определить цифру, которой оканчивается возвышаемое в куб число. Но двузначных чисел, оканчивающихся на некоторую определен- ную цифру, девять. Для выявления нужного числа надо определить наиболь- шее возможное двузначное число, куб которого есть число четырехзначное. 10. Поиски решения целесообразно начать с определения значения Л. Учитывая дальше, что двузначное число, возвышаемое в степень, оканчивает- ся на такую же цифру, как и полученное в результате трехзначное, можно определить значение □. 11. Во-первых, заметим, что □ = 3 или □ = 2, во-вторых, крайние цифры результата четырехзначного числа такие же, как и цифры двузначного. Испытанием определяем двузначное число. 12. Расшифровку ребуса удобнее начать с определения значения М, после чего легко определяется значение А, а затем уже и Р. 13. Нельзя ли установить особенности четырехзначного числа, состав- ленного из цифр 0, 2, 3, 5, исходя из данных о площади и урожайности? Какой цифрой может оканчиваться это число? (Затем определяются и другие цифры: сначала-цифра десятков, затем-сотен и последняя-тысяч.) 14. Поиски решения начнем с выяснения значности чисел, полученных в результате возведения в куб, а также чисел, возводимых в куб. Зная последние цифры результатов, можно найти цифры, которыми оканчиваются числа, возводимые в куб, а затем и сами числа. 15. Определив, какими числами может выражаться сумма цифр трех- значного числа, являющегося точным кубом (а таких чисел будет немного), путем проб можно отобрать нужное число. 16. Из условия того, что четырехзначное число есть точный квадрат, можно найти, на какие цифры оно может оканчиваться. Установив затем вид чисел, возводимых в квадрат, методом проб можно отобрать нужное число. 17. Так как четное трехзначное число по условию задачи является точным кубом, то нетрудно найти, какие числа могли возводиться в куб, а также и сами кубы. Первая цифра этого трехзначного числа является последней цифрой другого трехзначного числа, являющегося точным квадратом. Ото- 31
брав из имеющихся вариантов нужную цифру, можно определить трехзначное число, являющееся точным квадратом. (Не забудьте, что это число должно быть наибольшим из всех возможных.) 18. Нужные числа будем отбирать из трехзначных чисел, являющихся точными квадратами, в записи которых цифры не повторяются. Всего таких чисел 13. Из них надо отобрать такие 3 числа, чтобы вторая цифра первого была такой, как первая цифра (слева) втброго, а третья первого, как первая третьего; кроме того, третья цифра второго такая же, как вторая третьего. 20. Отыскивая решение задачи в общем виде, следует двузначное число (а иным числом возраст отца, естественно, не выражается) взять в общем виде, выполнить указанные в условии задачи операции и исследовать полученное выражение, установив, в каких случаях его значение будет числом, являющим- ся точным квадратом. 19. Решение можно искать методом проб, используя данные об одно- значных и двузначных числах, являющихся точными квадратами (их срав- нительно немного). 21. Решение задачи будем начинать с отбора цифр, которыми могут оканчиваться квадраты чисел. При этом не забудем, что 6-цифра-«акробатка» и в наборе нет одинаковых цифр, кроме цифры 3. .Определив цифру единиц четырехзначного числа методом проб (с учетом наличных цифр), определяем двузначное число. 22. Первым шагом в решении задачи является отыскание возможных цифр сотен трехзначных чисел. Затем методом проб отбираем нужные числа. 23. Зная, какой цифрой может оканчиваться квадрат числа и что каждая последующая цифра искомого числа больше предыдущей, можно определить цифру единиц искомого шестизначного числа. Зная эту цифру, можно нахо- дить возможные цифры единиц трехзначного числа, возводимого в квадрат. На основании крайней правой цифры шестизначного числа определяются возможные варианты его крайней левой цифры. Зная крайние цифры шести- значного числа, определяем крайние цифры трехзначного. Сузив круг воз- можных вариантов трехзначных чисел, отбираем то из них, которое удов- летворяет условию. 24. Будем искать решения, не пользуясь таблицей квадратов. Определив наибольшее и наименьшее четырехзначные числа, являющиеся точными квад- ратами, определяем и те двузначные числа, которые возводились в квадрат. Из этих двузначных чисел надо отобрать те, квадраты которых оканчиваются четной цифрой. Сколько их? Из них надо исключить те, квадраты которых имеют нечетную крайнюю левую цифру. Сколько у вас осталось чисел для испытания? Верно, 10 чисел. Испытанием находим нужное число.
ДЕЛИМОСТЬ НАТУРАЛЬНЫХ ЧИСЕЛ Если при решении задачи надо выполнить действия сложения или умноже- ния, то мы не ставим под сомнение возможность их выполнения, главное- уметь складывать и умножать. Если же имеем дело с вычитанием, то это действие не всегда выполнимо, если рассматриваются только неотрицатель- ные числа (натуральные числа и нуль). Но с первого взгляда на уменьшаемое и вычитаемое можно сделать заключение о возможности или невозможности выполнения вычитания. Иначе дело обстоит с действием деления, оно далеко не всегда выполняется нацело. На практике возникает необходимость, не выполняя деления, пред- сказать-делится число нацело или нет. Вот почему в математике особое внимание уделяется делимости чисел, исследуются условия делимости, вы- водятся определенные правила и признаки. Признаки делимости на 2, 3 и 5 были известны с давних времен. Так, например, признак делимости на 2 знали древние египтяне за две тысячи лет до н. э., а признак делимости на 9 был известен грекам в третьем столетии до н. э. Впервые признаки делимости на 2, 3 и 5 были обстоятельно изложены итальянским математиком Леонардо Фибоначчи (ок. 1170-после 1228). Вы- дающийся французский математик и физик Блез Паскаль (1623-1662) еще в раннем возрасте вывел общий признак делимости чисел, из которого следуют все частные признаки. Признак Паскаля состоит в следующем: натуральное число а разделится на другое натуральное число b только в том случае, если сумма произведений цифр числа а на соответствующие остатки, получаемые при делении разрядных единиц на число 6, делится на это число. Например, число 2 814 делится на 7, так как 26 + 8- 2+ 1-3 + 4 = 35 делится на 7. (Здесь 6-остаток от деления 1000 на 7, 2-остаток от деления 100 на 7 и 3-остаток от деления 10 на 7.) В этом параграфе предлагаются задачи на делимость чисел. 2 Зак 2531 Д В Клименченко 33
ЗАДАЧИ 1. Докажите, что из трех любых натуральных чисел всегда можно выбрать такие два, сумма которых делится на 2. 2. Сколько чисел от 1 до 100, таких, каждое из которых делится на 3, но в своей записи не имеет ни одной тройки? 3. Докажите или опровергните утверждение: «Разность между трехзнач- ным числом и суммой его цифр всегда делится на 9». 4. Докажите, что слово ХАХАХА делится на 7, если в нем буквами X и А обозначены любые цифры. (Одинаковые буквы обозначают одинаковые цифры, разные буквы-разные цифры.) 5. Если из задуманного трехзначного числа вычесть 7, то полученная разность разделится на 7, если вычесть 8, то полученная разность разделится на 8, если вычесть 9, то полученная разность разделится на 9. Какое наименьшее из возможных чисел задумано? 6. На вопрос: «Сколько среди двузначных чисел таких, у каждого из которых сумма цифр равна 9?»-Степа Верхоглядкин стал перебирать все двузначные числа подряд, отбирая нужные ему. Покажите ему более краткий способ решения задачи. 7. Найдите наименьшее шестизначное число, делящееся на 3, 7 и 13 без остатка. 8. Докажите, что если к любому пятизначному числу приписать справа (или слева) это же число, то полученное десятизначное число делится на 11. 9. Докажите, что если к любому трехзначному числу приписать трех- значное число, записанное теми же цифрами, но в обратном порядке, то получится число, делящееся на 11. 10. Найдите наибольшее и наименьшее трехзначные числа, каждое из которых делится на 6 и имеет в своей записи цифру 7. 11. Если сумма первой и второй цифр трехзначного числа, у которого одинаковые цифры сотен и единиц, делится на 7, то и число делится на 7. Докажите. 12. Докажите, что среди восьми натуральных чисел найдутся хотя бы два числа, разность которых делится на 7. 13. Для школьной библиотеки приобрели 17 одинаковых книг. Сколько они стоят, если за 9 таких книг заплатили больше Пр. 30 к., но меньше Пр. 40 к.? 14. Сколько всего натуральных чисел, меньших 100, которые: а) делятся на 2, но не делятся на 3; б) делятся на 2 или на 3; в) не делятся ни на 2, ни на 3? 15. Расстояние в километрах, которое пролетел самолет, выражается четырехзначным числом, делящимся на 45, а две средние цифры его 39. Найдите это расстояние, если оно не превышает 5000 км. 16. В магазин привезли меньше 600, но больше 500 тарелок. Когда стали раскладывать их десятками, то не хватило трех тарелок до полного числа десятков, а когда стали раскладывать дюжинами (по 12 тарелок), то осталось 7 тарелок. Сколько было тарелок? 17. Найдите цифры а и b в числе 42а4Ь, если известно, что это число делится на 72. 18. Найдите цифры сотен и единиц числа 72 * 3 ♦, если это число делится без остатка на 45. 34
19. Андрей нашел произведение всех чисел от 1 до 11 включительно и записал результат на доске. Во время перерыва кто-то случайно вытер три цифры, и в записи осталось число 399*68**. Помогите восстановить цифры, не прибегая к повторному нахождению произведения. 20, Найдите наименьшее натуральное число, делящееся на 72, в записи которого встречаются все цифры от 1 до 9. 21. Найдите наименьшее натуральное число, кратное 36, в записи которого встречаются все 10 цифр по одному разу. 22. Делая покупки в магазине «Спорттовары», Юра с папой должны были получить сдачу 31 р. Когда кассир выдал им по 1 р., 3 р. и по 5 р.-всего 10 купюр, Юра, не подсчитывая сумму, сразу же заметил, что кассир ошибся. Поблагодарив мальчика, кассир сразу же исправил ошибку. Как рассуждал Юра? 23. Когда за альбом стоимостью 6 р., книгу стоимостью 12 р., 9 коробок цветных карандашей и 15 линеек кассир выбил чек на 22 р. 85 к., то покупатель хотя и не знал стоимости карандашей и линеек, но сразу же заметил, что кассир ошибся. Какое он имел на это основание? 24. Маугли попросил своих друзей-обезьян принести ему орехов. Обезьяны набрали поровну орехов и понесли Маугли. Но по дороге они поссорились, и каждая обезьяна бросила в каждую по ореху. В результате Маугли досталось лишь 35 орехов. По скольку орехов обезьяны собрали, если известно, что каждая из них принесла больше одного ореха? 25. Ира заплатила за книгу пятикопеечными монетами, а Марина за такую же книгу-трехкопеечными. Вмести они внесли в кассу больше 10, но меньше 20 монет. Сколько стоит книга? 26. Чтобы можно было пользоваться автоматами для покупки воды и открыток во время экскурсии по городу, туристы обратились к кассиру с просьбой разменять им деньги только монетами по 3 к. и 5 к. Каждый турист имел больше 8 к. Всегда ли мог удовлетворить их просьбу кассир, имея достаточное количество нужных монет? 27. Возьмите 2 любых трехзначных числа, не делящихся на 37, но так, чтобы сумма их делилась на 37. Приписав одно из таких чисел к другому, получите шестизначное число. Проверьте, делится ли оно на 37. Сформули- руйте, пользуясь рассмотренным примером, утверждение и докажите его. 28. Поле разделили на 9 участков, некоторые из полученных участков снова разделили на 9 участков, некоторые из полученных участков снова разделили на 9 участков и т.д. Может ли в результате получиться 1986 участков? 29. Когда из чисел от 1 до 333 Таня исключила все числа, делящиеся на 3, но не делящиеся на 7, и все числа, делящиеся на 7, но не делящиеся на 3, то получила 215 чисел. Верно ли она решила задачу? 30. Готовясь к занятию кружка, ребята нашли такие 2 натуральных последовательных числа, наименьшие из возможных, что сумма цифр каждого из них делится на 17. Какие числа нашли ребята? 31. Сколько всего натуральных чисел, ие превышающих 500 и не делящих- ся ни на 2, ни на 3, ни на 5? 32. Число и его последняя цифра не делятся на 3 без остатка. Докажите, что, приписав несколько раз к данному числу его последнюю цифру, можно получить число, делящееся на 3. Какое наименьшее число раз надо приписать эту цифру? 2*
33. Когда трехзначное число, две крайние цифры (левые) которого одина- ковы, а правая 5, разделили на однозначное число, то в остатке получили 8. Найдите делимое, делитель и частное. 34. Окончив читать книгу, Вася подсчитал, что для нумерации всех ее страниц потребовалась 301 цифра. Покажите, что он ошибся. 35. Записав 6 различных чисел, среди которых нет числа 1, в порядке их возрастания и перемножив, Оля получила в результате 135 135. Запишите числа, которые Оля перемножила. 36. Число п • (п 4- 2) оканчивается цифрой 4: Назовите предпоследнюю цифру этого числа. 37. Найдите шесть различных частных, полученных от деления чисел, кратных семи, на 7. Известно, что каждое из этих чисел составлено из пяти двоек и трех единиц. 38. Найдите наименьшее натуральное число, делящееся на 63, сумма цифр которого равна 63. Поиски решений 1. Чтобы сумма двух слагаемых была четной, подумаем, какими должны быть слагаемые. Могут ли все три числа быть различной четности? 2. Ответить на вопрос задачи можно в том случае, если знать количество чисел, делящихся на 3, и количество тех из них, которые имеют в своей записи хотя бы одну цифру 3. Как первое, так и второе определить легко. 3. Записав число в общем виде и выполнив указанные операции, исследуем полученное выражение, показав, что оно делится на 9. 4. Если бы удалось шестизначное число представить в виде произведения таких множителей, чтобы один из них делился на 7, то задача была бы решена. Если, например, один множитель ХА, то какой будет второй? (Если затруд- няетесь решить в общем виде, то рассмотрите числовой пример 252 525.) 5. Чтобы ответить на вопрос задачи, надо найти наименьшее из возмож- ных чисел, делящееся на каждое из названных в условии задачи чисел. 6. Возможные цифры двузначных чисел можно получить исходя из^состава числа 9 (суммы цифр). Нетрудно также найти количество двузначных чисел, делящихся на 9. 7. По примеру задачи 5 можно найти наименьшее трехзначное число. Если бы узнать наименьшее число, на которое надо умножить найденное трех- значное так, чтобы в результате получить наименьшее шестизначное, то задача была бы решена. Подбором найти такое число трудновато, поэтому попробуем решать обратную задачу: будем делить наименьшее шестизначное 36
число 100000 на найденное трехзначное и тогда найдем, какое наименьшее число надо к нему прибавить, чтобы полученное число разделилось на трехзначное. 8. Для решения задачи надо уметь записать результат после того, как к пятизначному числу а приписали такое же число. При затруднении разо- браться в задаче в общем виде можно сначала обратиться к числовому примеру, а затем решить в общем виде. Например, к числу 28 345 припишем такое же число и получим 2834528 345. А это можно записать так: 2834 500000 + 28 345 = 28345-100001. 9. В отличие от предыдущей задачи в этой придется записать трехзначное число в виде суммы разрядных слагаемых (взяв, например, цифры а, Ь, с). 10. Поиски других двух цифр для записи искомых трехзначных чисел основаны на использовании признаков делимости на 2 и на 3 с учетом того, что надо получить наибольшее и наименьшее числа. Прежде всего определим место для цифры 7. 11. Записывая трехзначное число в общем виде, будем иметь сумму. Для доказательства делимости суммы двух слагаемых достаточно показать де- лимость каждого слагаемого. Именно к такому виду и надо привести слагаемые, учитывая, что сумма первой и второй цифр делится на 7. 12. Чтобы показать, что разность двух чисел делится на третье число, достаточно убедиться, что остатки, полученные от деления каждого из этих чисел на третье число, равны между собой. Подумаем, какое наибольшее число различных остатков будет при делении на 7. Найдутся ли среди полученных восьми остатков одинаковые? Примечание. При решении этой задачи используйте так называемый принцип Дирихле, который в упрощенном виде можно изложить так: ес- ли имеется предметов больше, чем ящиков, и нужно разложить все их в ящи- ки, то найдется по крайней мере один ящик, в котором больше одного предмета. 13. Для определения стоимости 17 книг надо найти цену одной книги. Но цену одной книги определили бы, если бы знали стоимость 9 книг. Для этого надо найти число, меньшее 1140, но большее ИЗО, делящееся на 9. Для этого определим, какой цифрой надо заменить 0 в числе ИЗО. 14. а) Надо найти количество чисел, делящихся на 2, и из этого количества исключить делящиеся на 6 (на 2 и на 3). б) Определив количество чисел, делящихся на 2 (см. задачу а)), затем - делящихся на 3, из этого общего количества исключить числа, делящиеся на 6. в) Используем решение задачи б). 15. Решение задачи сводится к определению крайних цифр искомого числа, а они определяются из условия делимости искомого числа на 45, т. е. на 5 и на 9. 16. При делении на 12 искомого числа известен остаток (по условию задачи). Нельзя ли определить остаток при делении его на 10? Что вы при этом заметили? Значит, если бы нашли число, делящееся на 10 и на 12, то, прибавив к нему 7, нашли бы искомое число. (Не забывайте при этом, что оно больше, чем 500, но меньше, чем 600.) Чтобы число делилось на 10 и на 12, оно должно делиться на 60. Между числами 500 и 600 такое число одно. 37
17. Неизвестные цифры определяются из условия делимости на 72, т. е. из условия делимости на некоторые однозначные числа. Какие? Сначала, очевид- но, надо использовать условие делимости на 9, а затем-на 8, определив сначала а + Ь, а затем а и Ь. 18. Используются, как и в задаче 15, признаки делимости на 5 и на 9. 19. Из условия делимости искомого числа на каждое из чисел от 1 до 11 сразу определяются две крайние правые цифры. Для определения остав- шейся неизвестной цифры используем один из признаков делимости. Какой? (Не ошибитесь: признак делимости на 3 здесь не приведет к нужному результату.) 20. Для решения задачи необходимо выполнить условие: используя для записи 9 цифр по одному разу, получить наименьшее число, делящееся на 72. (Признаки делимости используются те же, что и в задаче 17.) Из 9 цифр надо выбрать такие три, которые записываются крайними справа, чтобы вы- полнялось два условия: искомое число делилось на 8 и эти цифры обозначали наибольшие из возможных однозначных чисел. 21. Поскольку искомое число делится на 36, то при решении задачи должны использоваться признаки делимости на два однозначных числа. Какие? (В дальнейших поисках решения используются идеи решения за- дачи 20.) 22. Ошибку обнаружить без подсчета суммы можно, исходя из нечетности числа 31, являющегося суммой трех слагаемых. Если три слагаемых (или одно из них) нечетные, то какими должны быть числа купюр? Может ли общее их число быть при этом четным? 23. Чтобы иметь основание для возражения, надо показать, что 9 коробок карандашей и 15 линеек не могут стоить: 22 р. 85 к,-(6 р. + 12 р.) = 4 р. 85 к. Для этого, обозначив буквами цены коробки карандашей и линейки и записав равенство, показать, что оно не может быть верным. 24. Число орехов, собранных каждой обезьяной, можно было бы найти, если бы знали число брошенных и принесенных каждой обезьяной орехов. Но число брошенных орехов определяется с помощью числа обезьян. Число обезьян и число принесенных каждой из них орехов определяются с помощью данного в условии числа 35. Как? (Придется рассмотреть 2 случая.) 25. Обратив внимание на то, что число копеек, выражающее стоимость книги, делится на 5 и на 3, можно прийти к выводу, что оно делится на 15. Теперь надо искать число, делящееся на 15, такое, чтобы сумма частных от деления его на 5 и на 3 была больше 10, но меньше 20. 26. Принимая во внимание, что при делении на 3 различных остатков будет меньше, чем при делении на 5, числа можно разделить на 3 группы, записываемые выражениями 3«, Зи + 1, Зп 4- 2. Остается путем преобразований лишь показать, что число каждого вида можно представить тройками или пятерками или тройками и пятерками. 27. Для доказательства трехзначные числа надо брать в общем виде (обозначив буквами, и для простоты не в виде суммы разрядных слагаемых). Записав шестизначное число по условию задачи, надо преобразовать 38
его к такому виду, чтобы выделилась сумма взятых первоначально чисел (возможно, а + Ь), и исследовать на делимость на число 37. 28. Если удастся выразить число всех участков через число п разделенных участков, то можно записать равенство, связывающее п и число 1986, и про- верить, существует ли такое натуральное п, при котором равенство будет верным.* 29. Для того чтобы проверить, допущена Таней ошибка или нет, надо Цодсчитать количество чисел, удовлетворяющих условию задачи. Можно нйити количество чисел, делящихся на 3, а затем из них-делящихся на 7 и исключить последние. Аналогично поступаем и с подсчетом чисел, делящихся на 7, но не делящихся на 3. 30. Подумаем: могут ли суммы цифр искомых чисел быть одинаковыми? Чтобы число было наименьшим с данной суммой цифр, оно должно быть как можно меньшей значности. Тогда в его записи какие должны быть цифры? Испытав наименьшее из чисел, сумма цифр которого 34, и следующее за ним число, приходим к необходимости рассмотрения числа, большего с такой же суммой цифр. Немного терпения в поисках-и вас ждет удача. 31. Количество чисел, не делящихся ни на 2, ни на 3, ни на 5, можно узнать, если найдем количество чисел, делящихся на 2, или на 3, или на 5, и тогда из 500 вычтем это число. Доследнюю задачу решить нетрудно. Рассуждаем так: на 2 делится половина всех чисел до 500, на 3-треть, а чтоб найти, сколько делится на 2 или на 3, надо сложить найденные числа и исключить количество чисел, делящихся на 6. (Так как эти числа были учтены дважды.) Так же поступаем и при рассмотрении делимости на 5. 32. В поисках решений будем руководствоваться тем, что, во-первых, остатки при делении числа и суммы его цифр на 3 одинаковы; во-вторых, сумма чисел (цифр) делится на 3, если сумма остатков при делении каждой из цифр на 3 делится на 3; в-третьих, при делении на 3 возможны только два вида остатков-1 или 2. В данном случае число может быть представлено в виде суммы последней цифры и числа, изображенного оставшимися цифрами с нулем в его конце. Какого вида могут быть остатки при делении этих слагаемых на 3? Могут ли они быть разными, поскольку данное число не делится на 3? Если они 1 и 1 или 2 и 2, то находим, сколько слагаемых недостает до числа, делящегося на 3, а значит, и сколько раз надо приписать последню цифру к другому числу. Но возможен и другой случай: число, изображенное оставшимися цифрами, делится на 3. В этом случае имеем только один остаток при делении последней цифры на 3. Сколько недостает слагаемых (остатков), чтобы их сумма делилась на 3? Это и есть второй ответ на вопрос задачи. 33. Поиски решения задачи начинаем с поиска делителя, так как известен остаток. (Не забыли, что делитель-число однозначное?) Дальше выясняем вид делимого, принимая во внимание, что его последняя цифра 5, а две первые одинаковы. Зная делитель и признак делимости на 9, можно найти неизвест- ные цифры делимого. 34. Чтобы обнаружить ошибку, надо показать, что не может быть столько сраниц, для нумерации которых потребовался бы 301 знак (цифра). Для этого подсчитаем, сколько цифр потребуется для нумерации страниц, обозначенных однозначными и двузначными числами. Оставшиеся цифры используются для 39
нумерации страниц, обозначенных трехзначными числами. Делится ли по- лученное число на 3? 35. Нетрудно сначала данное шестизначное число представить в виде произведения двух чисел, одно из которых трехзначное. Пользуясь призна- ками делимости, трехзначное число можно представить в виде произведения трех чисел. Второе число (множитель) 1001 (это так называемое число Шахерезады) делится на 7, 11 и 13. 36. Для определения предпоследней цифры числа надо установить, на какое число оно делится. Для этого исследуем четность чисел п и (л + 2). Остается использовать признак делимости на 4. (Задача имеет несколько решений.) 37. На основании задачи 35 имеем: число вида abcabc (необязательно все цифры а, b и с разные) делится на 7. В условии же данной задачи надо найти число восьмизначное. Какие же цифры (двузначное число) следует приписать к шестизначному числу слева или справа, чтобы сохранить делимость на 7 и выполнить требование задачи? (Задача имеет несколько решений.) 38. Прежде всего исследуем, в каком случае число будет наименьшим при данной сумме его цифр. Чем большее однозначное число обозначает цифра, тем меньше цифр будет в записи числа при данной сумме его цифр. Число, в записи которого 7 девяток, делится на 9, но на 7 не делится, так как на 7 делится число, записаное 6 девятками (см. задачу 37). Значит, последнюю девятку надо заменить наименьшим числом, делящимся на 7 и на 9. Какое это число? Но полученное число не наименьшее? А как получить наименьшее? Верно, переставить 63 из конца в начало числа.
ПРОСТЫЕ И СОСТАВНЫЕ ЧИСЛА. РАЗЛОЖЕНИЕ ЧИСЕЛ НА МНОЖИТЕЛИ Решая задачи и выполняя действия над натуральными числами, вам неоднократно приходилось иметь дело с такими числами, одни из которых делятся только на 1 и на само себя, а другие имеют больше, чем два делителя. Натуральные числа, имеющие два и только два делителя, назвали простыми, а числа, имеющие больше двух делителей,-составными. Как видим, 1 не относится ни к тем, ни к другим. Всякое составное число является произведением некоторых простых чисел. Выходит, что простые числа-это своеобразные кирпичики, из которых скла- дываются составные числа. Не случайно поэтому простыми числами начали интересоваться еще в древности. Была замечена неравномерность их рас- пределения в натуральном ряду-по мере продвижения от меньшего числа к большему простые числа встречаются все реже. Возникает вопрос: имеется ли наибольшее простое число? Ответ на него был найден древнегреческим ученым Евклидом (III в. до н. э.). Он дал простое и остроумное доказательство того факта, что не существует наибольшего простого числа. А нельзя ли отобрать простые числа на отрезке натурального ряда от 1 до некоторого числа? Для этого можно воспользоваться способом, указанным другим ученым Древней Греции-Эратосфеном. Способ этот получил название «решето Эратосфена». Это кажущееся странным название получено по той простой причине, что Эратосфен прокалывал нанесенные на восковое по- крытие составные числа. Оказывается, в первой сотне простых чисел 25. (Проверьте!) Во все последующие времена после Евклида и Эратосфена ученых увлекала идея более глубокого исследования природы простых чисел. Выдающихся результатов в этом достигли наши отечественные математики, и прежде всего П.Л. Чебышев (1821-1894) и И.М. Виноградов (1891-1985). 41
В настоящее время поиски больших простых чисел ведет ЭВМ. Многие проблемы простых чисел до сих пор ждут еще своего решения. Например, и сейчас неизвестно, существует ли бесконечное множество простых чисел-близнецов (таких, разность которых равна 2), например 5 и 7, 11 и 13. Один ученый высказал такую мысль, что простые числа остаются сущест- вами, всегда готовыми ускользнуть от исследователя. Решая задачи этого параграфа, вы углубите свои знания о простых числах и разложении на множители составных чисел. ЗАДАЧИ 1. В семье шестеро детей. Пятеро из них соответственно на 2, 6, 8, 12 и 14 лет старше самого младшего, причем возраст каждого ребенка в годах выражается простым числом. Сколько лет младшему? 2. Перемножив четыре числа, Нина получила в результате число, цифра единиц которого 0. Кйкие числа она перемножила и какой получила результат, если множители простые последовательные числа? 3. Как только Дима назвал число 17-сумму четырех простых чисел, так Андрей сразу же нашел их произведение,' хотя Дима слагаемых не называл. Как рассуждал Андрей? Чему равно найденное им произведение? 4. На какую цифру может оканчиваться произведение двух простых не- однозначных чисел? 5. Простым или составным является число 19861986 4- 199O1"0? 6. Простым или составным является каждое из таких чисел: 19911991 + 1, 19911"1 - 1? 7. Записав подряд цифры от 1 до 9, Олег получил девятизначное число. Простое это будет число или составное? А если как угодно менять порядок цифр в этом числе, то будет ли меняться ответ на такой вопрос? 8. В новогодний вечер Дед Мороз дал ребятам такое задание: используя каждый раз все девять цифр от 1 до 9 по одному разу, вставить между каждыми двумя соседними цифрами « + » или « —» так, чтобы после вы- полнения действий получить все возможные простые двузначные числа. Помогите ребятам выполнить это задание. Сколько всего таких чисел можно получить? 9. Найдите сумму трехзначных чисел, каждое из которых является произ- ведением четырех неравных между собой простых чисел. 10. Чтобы войти в замок Арифмос, надо набрать шифр: записать последо- вательно в возрастающем порядке по одному разу 10 простых первых чисел натурального ряда. В полученном многозначном числе, не переставляя цифры, вычеркнуть половину цифр так, чтобы оставшиеся выражали: а) наименьшее возможное число; б) наибольшее. Найдите эти числа. 11. Можно ли при сложении двух, трех, четырех или пяти первых последо- вательных нечетных чисел натурального ряда получить простое число? 12. Выполнив домашнее задание по математике, Лена подготовила и за- дачу на занятие математического кружка. Она нашла два двузначных простых числа, получаемые друг из друга перестановкой цифр, а их разность-точный квадрат. Какие числа нашла Лена? 13. «Номер моего телефона,-сказал руководитель похода ребятам,-пяти- 42
значньщ. Первая цифра обозначает простое число, а последние две цифры получаются из предыдущей пары, обозначающей простое число, переста- новкой и образуют точный квадрат. Число, обращенное этому номеру, четное». Какой же номер телефона у руководителя похода? .14. Сумма квадратов двух некоторых простых чисел оканчивается цифрой 9. Найдите все такие простые числа. 15. Когда одноклассники поинтересовались номером квартиры Коли, ко- торую получила его семья в новом доме, то Коля-любитель математики - ответил: «Номер нашей квартиры выражается трехзначным числом, которое является точным кубом. Если поменять в нем местами цифры сотен и единиц, то получим простое число». Какой же номер квартиры Коли? 16. Помогите Буратино разложить все 28 косточек домино в четыре кучки так, чтобы суммы очков в кучках были четырьмя последовательными простыми числами. 17. Какой нечетной цифрой может оканчиваться сумма двух простых чисел, если она не является однозначным числом? 18. Любители математики Коля и Толя долго и безуспешно пытались найти такие три различные цифры, чтобы все трехзначные числа, составленные из них без повторений, были простыми. Помогите им в этом. 19. Готовясь к математическому вечеру, ребята составили такую задачу: «Вписать в кружки (рис. 1) 12 последовательных простых чисел (от 3 до 41) так, чтобы суммы трех чисел в вершинах треугольников были равны. При этом суммы чисел на внутренней и внешней окружностях также должны быть одинаковы». Решите и вы эту задачу. 20. Найдите р9 если каждое из чисел р, р + 10 и р + 14 простое. Рис. 1 43
21. Найдите р, если каждое из чисел р, 2р 4- 1 и 4р 4- 1 простое. 22. Известно, что 3, 5 и 7-три последовательных простых нечетных числа. Имеются ли в натуральном ряду еще три последовательных нечетных простых числа? ' 23. Найдите все такие простые числа, чтобы каждое из них можно было представить в виде суммы и разности двух простых чисел. 24. Миша долго и безуспешно пытался найти такие два числа, чтобы их произведение и сумма делились на некоторое простое число, но хотя бы одно из них не делилось на это простое. Помогите Мише выйти из создавшегося положения. 25. Найдите три простых числа, произведение которых втрое больше их суммы. 26. Докажите, что если произведение 1 • 2 • 3... п при п > 3 не делится на п 4- 1, то 4и 4- 1 - число простое. 27. Мальчики двух шестых классов в школьных мастерских изготовили для детских садиков 123 стульчика. Сколько работало мальчиков и сколько стульчиков изготовил каждый из них, если они изготовили их поровну? 28. По всем вагонам пассажирского поезда поровну разместили 737 ту- ристов. Сколько было вагонов и сколько туристов в каждом вагоне? 29. Произведение некоторых простых чисел равно ЗООЗО. Каким числом является их сумма-простым или составным? 30. Барон Мюнхгаузен утверждал, что ему удалось найти такое натураль- ное число, произведение всех цифр которого равно 6552. Покажите, что он сказал неправду. 31. Докажите, что трехзначное число, в записи которого все цифры одинаковы, в разложении на простые множители содержит число 37. 32. В каждую клетку квадратной таблицы 4x4 клетки требуется вписать по простому числу так, чтобы в каждом столбце и каждой строке произведение чисел равнялось 1989. Если нельзя этого сделать, докажите, если можно- впишите. 33. Чтобы ответить на вопрос: «Делится ли разность 9198б-81980 на 5 и на 10?»-можно призвать на помощь ЭВМ. А может, вы посоревнуетесь с ней? Попробуйте. 34. Для вычисления произведения всех натуральных чисел от 1 до 50 включительно, конечно, лучше воспользоваться ЭВМ. А вот назвать число нулей, которыми оканчивается это произведение, вы можете без труда и не прибегая к помощи компьютера. Сколько же их? 35. Докажите, что если в трехзначном числе две последние 1щфры одина- ковы и сумма его цифр делится на 7, то в разложении этого числа на простые множители имеется число 7. 36. Пусть имеем два трехзначных числа, дающие одинаковые остатки при делении на 7. Приписав одно число к другому, получим шестизначное число. Покажите, что полученное число в разложении на простые множители содержит число 7. 37. Вычисляя сумму всех различных простых делителей некоторого шести- значного числа, в записи которого все цифры одинаковы, Степа Растеряйкин получил 70, а Петя Угадайкин-80. Покажите, что они оба ошиблись. 38. Члены математического кружка «Эврика» утверждают, что если в шес- тизначном числе суммы цифр первой и четвертой, второй и пятой, третьей 44
и шестой (считая слева направо) равны между собой, то это число в разложе- нии на простые множители содержит 37. А как вы думаете? 39. Можно ли, употребляя каждую цифру по одному разу, составить десятизначное число, делящееся без остатка на 1980? Примечание. Для решения этой задачи потребуется дополнительное сведение о делимости числа на 11. Если разность между суммами цифр числа, стоящих на четных и нечетных местах (знакопеременная сумма цифр), делится на L1, то число делится на 11. 40. Пароход имеет а труб, b винтов, на его борту с человек. Он отправился в рейс и-го числа, fc-ro месяца, (1900 + р)-го года. Произведение шести чисел а, Ь, с, л, /с, р, увеличенное на число, куб которого есть возраст капитана в годах, равно 4752862. Определите неизвестные. 41. После долгих настойчивых поисков Андрей нашел такое двузначное число, которое не оканчивается на 0 и обладает таким свойством, что, вставив нуль между цифрами десятков и единиц, получается трехзначное число, делящееся без остатка на исходное двузначное. Когда он показал свой результат учителю, то тот, похвалив мальчика, заметил, что таким свойством обладает не только то число, которое нашел Андрей. Найдите все такие двузначные числа. Сколько их? Поиски решений 1. Так как наименьшее искомое число однозначное (почему?), то его легко найти простым испытанием (методом проб). 2. Два из четырех простых множителя определяются из условия «в произ- ведении цифра единиц 0». Поиски остальных множителей не составляют особого труда. 3. Произведение можно найти, если известны множители, а они опреде- ляются из условия, что сумма четырех простых чисел равна 17. 4. Последнюю цифру произведения можно определить, если знать, какими цифрами могут оканчиваться множители. Этот вопрос разрешается исходя из условия, что множители-числа простые и неоднозначные. 5. Пусть вас не смущают большие числа да еще с такими показателями. По аналогии с квадратом и кубом числа запись 19861986 надо понимать так: число 1986 берется множителем 1986 раз. Трудно, конечно, представить себе такое число. Но ведь нам надо знать только, на какую цифру оно оканчивается. А зная последние цифры слагаемых, можно определить последнюю цифру суммы, и все станет понятным. 6. Если вы нашли решение задачи 5, то здесь затруднений в поисках решения не возникнет. 45
7. Для ответа на вопрос задачи, пользуясь признаками делимости, про- верим, имеет ли это число хотя бы один делитель, отличный от 1 и самого этого числа. Подумаем, какой признак здесь достаточно использовать. 8. Чтобы поиски решения были целенаправленными, прежде всего опреде- лим, сколько чисел можно получить и какие именно. Если между цифрами ставить только знаки « +», то получим наибольшее возможное число. Какое? Оно непростое, но дает возможность отобрать все простые числа, которые надо получить, вставляя знаки между цифрами. Таких простых двузначных чисел меньших 45, всего 10. Зная их, нетрудно расставить знаки. 9. Чтобы найти сумму, надо сначала найти числа, которые следует сложить. (Не ошибитесь, наберитесь терпения в поисках требуемых трех- значных чисел.) Можно начать записывать произведения с наименьших возможных множителей, например 2-3-5-7 = 210. Всего таких чисел 16. 10. Получив в записи 16 цифр, поступаем примерно так, как при решении задач 13, 14 (с. 117). 11. Выполняем указанное действие и исследуем полученные результаты. 12. Отобрать нужные числа можно испытанием, но прежде надо выяснить, при помощи каких цифр могут быть записаны такие двузначные числа. (Таких цифр 4.) 13. Обратив внимание на то, что первая цифра данного числа является последней обращенного ему числа и что она четная и обозначает простое число, получаем начальную цифру номера. Для определения других цифр отбираем двузначные числа, являющиеся точными квадратами, и путем проб выявляем то, которое удовлетворяет условию задачи. 14. В таком кратком условии задачи выражается богатый смысл, и путем последовательного анализа его надо раскрыть. Порассуждаем: во-первых, выясняем, какими должны быть слагаемые, если их сумма нечетная; во- вторых, четным или нечетным будет числом если квадрат его число четное; в-третьих, если квадрат числа оканчивается'цифрой 5, то какой цифрой оканчивается это число, и, наконец, определяем, простые числа, если одно из них четное, а второе оканчивается цифрой 5. 15. Если отберем из трехзначных чисел те, которые являются точными кубами (а таких чисел всего 5), то методом проб можно найти искомое. Трехзначные числа, являющиеся точными кубами, можно проще всего по- лучить, возведя в куб определенные числа. Какие же и сколько их? 16. Для определения, на каком участке натурального ряда надо искать простые числа, следует найти среднее значение очков домино. Зная среднее значение числа очков в кучках, нетрудно подобрать нужные значения сумм очков. 17. Из задачи 14 известно, что если сумма двух слагаемых есть число нечетное, то слагаемые - различной четности. Это дает возможность в данной задаче определить одно из слагаемых. Зная его, методом перебора последних возможных цифр суммы можно найти, какими цифрами может оканчиваться второе слагаемое. 18. Очевидно, Толя и Коля решали задачу наугад, мы же не будем следовать их примеру. Сначала «отсеем» все те цифры, появление которых в конце трехзначного числа приводит к тому, что данное число составное. (На основании признака делимости это очевидно.) Сколько у вас осталось цифр? Верно,. четыре. Вот теперь нетрудно составить все возможные трехзначные 46
числа и установить, имеет ли задача решение. (Можно воспользоваться таблицей простых чисел.) 19. Эта задача в отличие от предыдущей хотя и имеет решение (и не одно), но и ее не будем решать вслепую, иначе придется очень долго и, возможно, безуспешно бродить по лабиринтам. Для выработки стратегии решения обратим внимание на то, что суммы чисел должны быть одинаковы в каждой из двух окружностей и в каждом из четырех треугольников. Весьма заманчиво знать эти суммы. А это разве трудно сделать? (Ведь сумму всех чисел найти можно.) В кружках внутренней окружности записывается чисел меньше, чем в круж- ках внешней окружности. Поэтому подумаем, где надо вписывать большие числа. (Правда, придется одно из этих чисел потом заменить на меньшее, так как 138 >118, где 138-сумма четырех наибольших из данных чисел, а 118-сумма чисел на внутренней и внешней окружностях.) Имея по одному числу в вершинах треугольников, нетрудно затем подобрать еще по два числа. 20. Для отыскания р следует разбить натуральные числа на группы по какому-нибудь признаку, например по отношению делимости на 3 (см. задачу 26, с. 138). Представив р каждым из трех возможных видов, исследовать числа р, р + 10 и р + 14 (после подстановки значений р}. 21. Используется идея решения задачи 20. А если есть идея, то реализуем ее в конкретной ситуации. 22. Решение задачи сводится к исследованию трех последовательных не- четных чисел. Для этой цели надо эти числа записать в общем виде. (Если, например, одно из них 2и—1, то как запишутся последующие?) Дальше использовать представление натурального числа п возможными тремя видами (см. задачи 20, 21). 23. Так как число 2 условию не удовлетворяет, то искать простые числа следует среди нечетных. Но если сумма и разность двух чисел нечетные, то могут ли оба компонента быть одинаковой четности? Отсюда определяется один компонент-четное простое число, а затем-и второй. 24. А существуют ли такие числа? Если произведение двух чисел делится на некоторое простое число, а один из множителей не делится, то второй множитель обязательно делится на это число. Но тогда при условии делимо- сти суммы этих чисел и одного из слагаемых второе также должно делиться. Как вы думаете, что показывает полученное противоречие? 25. Вы, очевидно, уже догадались, что если числа, например, а, b и с, то надо записать равенство по. условию задачи и исследовать его. (Не забудьте, что числа а, b и с простые и только одно из них может быть четным.) 26. Доказательство значительно упростится, если предположить, что число п 4- 1 составное. Порассуждаем: ведь если оно составное, то его можно представить в виде произведения множителей, каждый из которых меньше, чем и + 1. Значит, эти множители имеются среди множителей записанного произведения. Почему? И что из этого следует? 28. Используется идея решения задачи 27, но возникает затруднение при разложении числа 737 на множители: «не срабатывают» известные признаки делимости. В этом случае простой делитель можно найти методом проб. Поищем его. 47
29. Решение задачи сводится к разложению данного числа на простые множители и исследованию характера их суммы. 30. Чтобы показать ложность утверждения, надо получить разложение на множители данного числа, зная, что в десятичной записи числа могут быть лишь цифры 0, 1, 2, ..., 9. 31. Достаточно число записать в общем виде и разложить его на мно- жители. 32. А не попробовать бы решить сначала более легкую задачу, обратную данной? Если в результате получим 4 простых числа, то затем поищем способ их расстановки в клетках квадратной таблицы. 33. Конечно, не стоит (да и сможем ли?) возводить в степень числа 8 и 9, достаточно исследовать и установить, какими цифрами оканчиваются эти степени. 34. Для определения количества нулей, которыми оканчивается произ- ведение чисел, надо знать, сколько раз в этом произведении встречается множитель 10. Но 10 получается при умножении чисел 2 и 5. Какой же из этих множителей встречается реже в разложении на простые множители данного произведения? Значит, задача сводится к подсчету числа пятерок. 35. Вам уже неоднократно приходилось проводить подобные доказа- тельства, и вы, разумеется, пришли к верному плану действий: надо записать число в общем виде суммой разрядных слагаемых и преобразовать получен- ные выражения с учетом данных условия задачи, выделив слагаемые, кратные числу 7. 36. Для решения задачи надо уметь: 1) записать число, дающее данный остаток при делении на 7; 2) записать выражение, полученное в результате приписывания одного трехзначного числа к другому; 3) преобразовать по- лученное выражение так, чтобы выделился множитель 7. А все это мы уже умеем делать. 37. Не в пример Угадайкину и Растеряйкину постараемся решить задачу безошибочно и найдем возможные суммы делителей. Записать шестизначное число в общем виде-дело уже нам привычное. Остается разложить его на простые множители и записать их сумму. Разложение чисел 1001 и 111 уже встречалось (см. задачу 35, с. 140). Если шестизначное число записано при помощи цифры а, то какие простые делители, отличные от имеющихся, может иметь а? 38. Для решения задачи следует записать число в общем виде (в виде суммы разрядных слагаемых) и преобразовать полученное выражение, стре- мясь выделить множитель 37, используя данные условия о равенстве сумм цифр. 39. Данную задачу можно свести к более простой, так как, составив десятичное число с цифрой 0 в конце его, делимое и делитель можно разделить на 10. Исследуем возможность делимости девятизначного числа на 198, для чего число 198 представим в виде произведения простых чисел. Возможность деления на 2 и на 9 легко устанавливается. Наиболее трудная здесь проблема - найти способ такой расстановки цифр числа, чтобы оно делилось на 11, или показать, что при любой расстановке цифр число на 11 не делится. Порас- суждаем: во-первых, можно ли так расставить цифры, чтобы сумма цифр, стоящих на четных местах, равнялась бы сумме цифр, стоящих на нечетных местах? (Для этого определим сумму всех цифр девятизначного числа.) Если 48
это сделать нельзя, то, во-вторых, можно ли так расставить цифры, чтобы разность между указанными суммами равнялась 11? Чему при этом равняется сумма цифр, стоящих на четных местах? Верно, 17. Вот теперь нетрудно выбрать из имеющихся цифр 4 такие, чтобы сумма их была 17, и расставить остальные цифры. (Не забудьте, какой должна быть последняя цифра.) Если ваш ответ не совпадает с приведенным в конце книги, то вы нашли еще .одно решение из многих возможных. И это вовсе не плохо, а даже наоборот. 40. Если бы знать возраст капитана, то нашли бы произведение шести чисел, подлежащих определению. Но, как вы думаете, какое число, являю- щееся точным кубом, может выражать возраст капитана? (А его можно получить, возвышая в куб однозначные числа.) Разложив полученное число на простые множители, можно выбрать наиболее правдоподобную ситуацию. (При решении практических задач так именно и поступают.) 41. Для отыскания чисел, удовлетворяющих условию задачи, записав двузначное число в общем виде, надо выполнить указанные в условии задачи операции и получить равенство. Его исследование и детальный анализ и приведут к определению цифр двузначных чисел.
НАИБОЛЬШИЙ ОБЩИЙ ДЕЛИТЕЛЬ И НАИМЕНЬШЕЕ ОБЩЕЕ КРАТНОЕ В практике вычислений возникает необходимость не только определять, делится данное число на другое или нет и находить такие числа, но и отбирать из них те, которые являются общими делителями для двух или нескольких чисел. Таких общих делителей для данных чисел может быть несколько, но всегда конечное число. (Почему?) Среди них будет наименьшее число 1 и наи- большее, которое, как вам, очевидно, уже известно, называют наибольшим общим делителем данных чисел. Зная, например, что наибольшим общим делителем чисел 429 и 286 есть 143, частное 429:286 можно записать проще так: 3:2. А для чего это надо? Еще в начальных классах вы получили первые 286 сведения о дробях. Как по-вашему мнению, какая из дробей проще: или 2 -? Наверное, все согласятся, что вторая дробь проще. А ведь они обозначают одно и то же число, поэтому первую из них можно заменить второй. Такое сокращение чисел на их наибольший общий делитель полезно и при решении уравнений и неравенств. В других случаях приходится выполнять и обратную задачу: нахрдить число, которое делится на каждое из данных чисел. Например, поскольку складывать и вычитать можно дроби только с одинаковыми знаменателями (одинаковые доли), то возникает необходимость имеющиеся дроби с разными знаменателями заменять такими, знаменатели которых одинаковы. При этом придется находить число, делящееся на каждое из данных чисел. Такое число называют общим кратным данных чисел. Общих кратных всегда существует сколько угодно, но среди них одно наименьшее. Вот оно-то представляет наибольшую практическую ценность. 50
Вы уже знаете, как находить НОД и НОК разложением на простые множители данных чисел. Однако разложение чисел на множители, как вы, очевидно, уже убедились, решая задачи предыдущего параграфа, не всегда на практике легко осуществимо. Попробуйте, например, разложить на мно- жители число 29233. А ведь это число составное, а не простое. Конечно, методом проб можно найти его делители, но придется изрядно потрудиться. (Правда, работу можно облегчить, призвав на помощь калькулятор.) Как же быть с определением НОД и НОК? Выход из этого положения был указан еще Евклидом. Правило нахождения НОД, обоснованное Евклидом, состоит в следующем: большее из двух данных чисел делят на меньшее, затем меньшее число делят на первый полученный остаток, первый остаток на второй и т.д. (Почему такой процесс деления закончится?) Последний не равный нулю остаток при таком делении и будет наибольшим общим делителем данных чисел. Например, найдем наибольший общий делитель чисел 455 и 312. Последовательно имеем 455:312= 1 (ост. 143), 312:143 = 2 (ост. 26), 143:26 = 5 (ост. 13), 26:13 = 2 (ост. 0). Итак, НОД (455; 312)= 13. Правило это известно под названием алго- рифма, или алгоритма Евклида. А как же найти НОК? Это уже нетрудно, если знать, что произведение НОД и НОК равно произведению данных чисел. Поэтому, разделив произведение данных чисел на их наибольший общий делитель, найденный по алгоритму Евклида, получим наименьшее общее кратное данных чисел. Решив небольшое число задач этого параграфа, вы глубже познакомитесь с применением НОД и НОК в различных ситуациях. ЗАДАЧИ 1. Когда Витя, определяя наибольший общий делитель трех чисел, назвал в ответе число 15, Вера, не зная этих чисел, но, зная, что их произведение равно 3000, заметила, что он ошибся. Права ли она? 2. Чему может равцяться наименьшее общее кратное трех последователь- ных натуральных чисел? 3. Возле лужицы отряд Длинноногих лягушат, Очень стройная колонна: По пять ровно каждый ряд. По два, по три, по четыре Как ни ставил командир, Неизменно оставался Лишним кто-нибудь один. А сейчас в любой пятерке Все довольны, все в восторге! Ну а сколько лягушат не в ряду, а в целом? Свой расчет произведи с правильным прицелом. Примечание. Учтите, что в лягушечьей колонне меньше, чем 10 рядов. SI
4. Работая на уборке фруктов, 5 А класс собрал 560 кг яблок, 5 Б-595 кг и 5 В-735 кг. Все собранные яблоки разложили в ящики, положив в каждый из них одно и то же наибольшее из возможных число килограммов. Сколько таких ящиков потребовалось каждому классу? 5. Через железнодорожную станцию проследовало три воинских эшелона. В первом находилось 462 солдата, во втором-546 и в третьем-630. Сколько вагонов было в каждом эшелоне, если известно, что в каждом вагоне находилось одинаковое число солдат и что это число было наибольшим из всех возможных? 6. Числа 100 и 90 разделили на одно и то же число. В первом случае получили в остатке 4, в другом-18. На какое число делили? 7. В одной школьной библиотеке 24850 книг, а в другой-55 300. Когда эти книги стали расставлять на стеллажи поровну, то в первой библиотеке осталось 154 книги, а в другой-175. По скольку книг ставили на каждый стеллаж? 8. На кольцевой дорожке длиной 660 м проводится эстафета, длина каж- дого этапа которой 150 м. Старт и финиш находятся в одном и том же месте. Какое наименьшее число этапов может быть в этой эстафете? 9. В морской порт теплоход «Счастливый» прибывает один раз в 3 дня, теплоход «Удачный»-один раз в 4 дня и теплоход «Надежный»-один раз в 5 дней. В прошлый понедельник все три теплохода были в этом порту. Через какое наименьшее число дней они все снова прибудут в этот порт и какой это будет день недели? 10. Найдите наибольший общий делитель всех пятизначных чисел, за- писанных при помощи цифр 1, 2, 3, 4, 5 без повторений. 11. В одном классе обучается меньше, чем 50 учащихся. На улице Горького 1 л 1 проживает - учащихся этого класса, на улице Пушкина--, на улице Гайдара— и на улице Корабельной-остальные ученики. Сколько учеников проживает на Корабельной улице? 12. Выполняя приказ царя Гороха, генерал Муштралкин пытался вы- строить всех солдат в ряды сначала по 2, а затем-по 3, 4, 5, 6, 7, 8, 9, 10, но, к его удивлению, каждый раз последний ряд оказывался неполным, так как оставалось соответственно 1, 2, 3, 4, 5, 6, 7, 8, 9 солдат. Какое наименьшее число солдат могло быть? 13. На лесоскладе были бревна длиной по 6 м и 8 м одинаковой толщины. Какие бревна выгоднее брать, чтобы получить больше кусков при меньшем числе распиловок для распиловки их на метровые куски? 14. Какое наибольшее значение может принимать наибольший общий делитель тринадцати натуральных чисел, еелй их сумма равна 1988? S2
Поиски решений 1. Для того чтобы ответить на вопрос задачи, надо знать, какие значения может принимать НОД двух чисел, произведение которых 3000. А это можно узнать, разложив 3000 на простые множители. 2. Ответ не может быть однозначным, так как возможны две ситуации: либо меньшее из данных чисел нечетное, либо оно четное. 3. Решение задачи состоит в отыскании такого числа, которое кратно 5, а при делении на 2, на 3 и на 4 каждый раз дает остаток 1. 4. Для определения числа ящиков, потребовавшихся для яблок, надо найти, сколько килограммов яблок клали в каждый ящик. Подумаем, чем это число является для чисел, данных в условии задачи. 5. Способ решения тот же, что и задачи 4. 6. Зная делимое и остаток от деления, всегда можно найти произведение делителя и частного. Чем же является этот делитель для полученных таким образом чисел? Ответив на этот вопрос, определим неизвестное число. 7. Способ решения тот же, что и задачи 6. 8. Если бы длина дорожки делилась нацело на длину этапа, то можно было бы найти число этапов. Чем же должно быть это число для чисел 660 и 150, чтобы воспользоваться им? 9. Выяснив, чем должно быть искомое число дней для чисел, данных в условии задачи, дадим ответ на первый вопрос задачи, а затем, зная число дней недели, ответим и на второй. 10. Только не будем браться за непосильную работу: буквально находить делители, записывая все числа. (А их ведь 120!) Имеется другая идея. Постараемся подметить общую закономерность для всех таких чисел. (На некоторые выводы наведет, возможно, разложение на простые множители числа 12345.) 11. Число проживающих на Корабельной улице можно было бы найти, если бы знали, сколько всего учащихся в классе. Каким же числам должно быть кратно это число? Чем оно должно быть для этих чисел? 12. Подумаем, чем было бы для всех чисел от 2 до 10 искомое число, если бы его увеличить на 1. Определив это число, найдем затем и искомое. 13. Выгоднее брать те бревна, из которых можно получить больше кусков при наименьшей (или одинаковой) распиловке. Надо сравнить количество полученных кусков из данных бревен при одинаковом числе распиловок. А что представляет это число относительно чисел-распиловок шестиметровых и восьмрметровых бревен? Ответив на этот вопрос, найдем решение. 14. НОД примет наибольшее значение, если все 13 чисел одинаковы. Но делится ли 1988 на 13? Однако оно делится на число, близкое ему. На какое? 53
ДРОБИ И ПРОЦЕНТЫ Решая даже простейшие задачи типа «Разделить 3 яблока между четырьмя детьми поровну», результат нельзя выразить целым числом, придется делить на доли и подсчитывать их число. В этом случае результат будет выражаться при помощи двух натуральных чисел, одно из которых покажет, на сколько равных долей разделили, а другое-сколько таких долей взяли. Если первое - а число Ь9 а второе а, то записывают Выражение такого вида называют b дробью. Дроби появляются не только при делении, а и при измерении величин, так как выбранная единица измерения величины далеко не всегда содержится целое число раз в измеряемом объекте. Значит, введение дробей, как и целых чисел, продиктовано практическими потребностями. Один известный философ прошлого говорил, что действительное изобра- жается в мышлении не в целых числах, а в дробях. Первой дробью, с которой познакомились люди, была половина и дво- _ 1 1 1 1 ичные дроби -, -, —,..., затем к ним присоединилась дробь - и ее двоичные 4 о 16 3 1 1 деления — и т.д. От двоичных дробей египтяне перешли к дробям вида о 12 -, которые называли единичными или основными дробями. Другие дроби они п представляли при помощи единичных, составляя для этой цели специальные таблицы. Вавилоняне пользовались шестидесятеричными дробями, знаменателями которых являются степени числа 60. 54
Римляне пользовались дробями со знаменателем 12, называя унцией. В средние века, как и в древности, учение о дробях считалось самым трудным разделом арифметики. Римский оратор и писатель Цицерон говорил, что без знаний дробей никто не может признаваться знающим арифметику. А у немцев сохранилась такая поговорка «Попасть в дроби», что означает попасть в трудное положение. Трудности при изучении дробей обусловлены тем, что надо было заучивать таблицы и умножения, и сложения дробей зачасТую без понимания и выяснения сущности этих действий. У многих народов дроби называли ломаными числами. Этим названием пользуется и автор первого русского учебника по математике Л.Ф. Маг- ницкий. Интересное и меткое «арифметическое» сравнение делал Л. Н. Толстой. Он говорил, что человек подобен дроби, числитель которой есть то, что человек представляет собой, а знаменатель-то, что он думает о себе. Чем большего человек о себе мнения, тем больше знаменатель, а значит, тем меньше дробь. С введением десятичной позиционной системы счисления появляются и новые дроби-десятичные. Выполнение действий с такими дробями значи- тельно упрощается, так как они основаны на действиях с целыми числами. Заслуга в развитии учения о десятичных дробях принадлежит средне- азиатскому ученому Джемшиду ал-Каши (XV в.). Систему десятичных дробей он описал в книге «Ключ к арифметике» (1427). В Европе первым изложил учение о десятичных дробях голландский математик и инженер Симен Стевин, посвятивший этому вопросу труд под названием «Десятая» (1585). Записывал он десятичные дроби не так, как теперь. Например, число 28,375 представлялось так: 28©3ф 7@ 5(3), где цифры в кружочках показывают место десятичных знаков. Запятую как| знак дробности ввел знаменитый математик, физик и астроном И. Кеплер (1571-1630). В России учение о десятичных дробях впервые изложил Л. Ф. Магницкий в своей «Арифметике». Из школьного курса математики вам известно, что слово «процент» происходит от латинских слов и означает «на сто». Поэтому одну сотую часть стали называть процентом. Идея выражения частей целого в одних и тех же долях родилась еще в древности и вызвана практическими потребностями. И сейчас проценты применяются в различных областях человеческой дея- тельности. Глубже и основательнее познакомиться с дробями и процентами вам помогут задачи настоящего параграфа. Первые задачи построены на ма- териале V класса, остальные-VI. Однако это вовсе не значит, что одни из этих задач нельзя решать пятиклассникам, а другие-шестиклассникам. Решайте их руководствуясь интересами и свойми возможностями. ЗАДАЧИ 1. Найдите дробь, у которой числитель меньше знаменателя и которая не изменяется, если ее запись перевернуть «вверх ногами». 55
X Рыбаки выловили сетью в ставке 80 рыбин, пометили их и снова выпустили в ставок. На второй день они поймали 150 рыбин, среди которых 5 оказалось помеченных. Сколько всего рыбин в ставке? Примечание. Помеченная рыба равномерно распределяется с остальной. 3. Когда спросили у пастуха, сколько овец в отаре, то он ответил: «60 овец пьют воду, а остальные 0,6 всех овец пасутся». Сколько овец в отаре? 4. Масса зерна составляет 0,5 т и еще 0,8 всей массы. Какова масса зерна? 5. Найдите ошибку: 12,5:12,5-2,4:2,4 25 (0,5:0,5) «4(0,6:0,6) 25'1—41 5 -5 - 2*2(7) 6. Кузнечик может прыгать ровно на 0,5 м в любом направлении. Может ли он за несколько прыжков переместиться ровно на 7,3 м? 7. Восстановите недостающие числа в примерах (сумма дробей в примере в)-правильная дробь): 8. Вместо звездочек поставьте такие знаки действий, чтобы равенства были верными: а) 37,3*- = 74-; в) 0,45*^ = ^; 7 ’ 2 5 20 5 33 10 1 б) 40*ТТ = °’75’ г) 0,375*—= 0,4. 9. Половину пути туристы шли пешком, а половину ехали автобусом, затратив на весь путь 5,5 ч. Если бы весь путь они ехали автобусом, то затратили бы 1 ч. Сколько всего времени затратят туристы, если весь путь будут идти пешком? Во сколько раз быстрее ехать автобусом, чем идти пешком? 10. Числа 2,75 и 8 обладают тем свойством, что произведение этих чисел равно сумме составляющих их цифр: 2,75 - 8 = 2 + 7 + 5 + 8 = 22. Найдите еще хотя бы одну такую пару неравных между собою чисел. И. Медведь с базара плюшки нес, Но на лесной опушке Он половину плюшек съел И плюс еще полплюшки. Шел, шел, уселся отдохнуть И под «ку-ку» кукушки Вновь половину плюшек съел И плюс еще полплюшки. 56
Стемнело, он ускорил шаг, Но на крыльце избушки Он снова пол-остатка съел И плюс еще полплюшки. С пустой кошелкою-увы! Он в дом вошел уныло... Хочу, чтоб мне сказали вы, А сколько плюшек было? 12. Ручные часы отстают на 5 мин в час; 5,5 ч назад они были поставлены на точное время. Сейчас на часах, показывающих точное время; 1 ч дня. Через сколько минут ручные часы покажут 1 ч дня? 13. Минуткин обычно заводил часы до отказа два раза в сутки: утром в 8 ч 30 мин и ночью, ложась спать. Утром приходилось делать 9 полных оборотов головки часов, а ночью-И. В котором часу ложился спать Минуткин? 14. Проводя наблюдения на географической площадке, ученики записыва- ли температуру ежедневно с 15 до 19 апреля включительно. Средняя тем- пература за эти дни оказалась равной 17,5°. Какая температура была 19 и 15 апреля, если она ежедневно повышалась на 1,5°? 15. В классе 35 учеников. Всем им вместе 280 лет. Найдется ли в этом классе 25 учеников, которым вместе будет не менее 225 лет? 16. С пришкольного участка прямоугольной формы размером 70 х 35 м собрали 14,7 ц кукурузы. Докажите, что среди квадратов этого участка, каждый из которых имеет размер 0,7 х 0,7 м, непременно найдутся такие, которые дали одинаковый урожай. 17. При делении одного числа на другое получили десятичную дробь, целая часть которой равна делителю, а после запятой, записано делимое. Какие это числа? 18. Группа туристов должна была прибыть на вокзал в 5 ч. К этому времени с турбазы за ними должен был прибыть автобус. Однако, прибыв на вокзал в 3 ч 15 мин, туристы, не ожидая автобуса, пошли пешком на турбазу. Встретив по дороге автобус, они сели в него и прибыли на турбазу на 15 мин раньше предусмотренного времени. С какой скоростью шли туристы до встречи с автобусом, если скорость автобуса 60 км/ч? 19. Тане надо было разложить 80 тетрадей на две стопки так, чтобы число тетрадей в одной из них составляло 60% числа тетрадей в другой. Помогите ей решить эту задачу. 20. В драмкружке число мальчиков составляет 80% от числа девочек. Сколько процентов составляет число девочек в этом драмкружке от числа мальчиков? 21. Под кукурузу отвели участок поля в форме прямоугольника. Через некоторое время длину этого участка увеличили на 35%, а ширину уменьшили на 14%. На сколько процентов изменилась площадь участка? 22. Из числа построенных в этом году домов в одном из районов города более 94% имеют больше пяти этажей. Какое наименьшее число домов возможно в данном случае? 23. В цехе не больше 100 рабочих, треть из них женщины, 8% рабочих имеют сокращенный рабочий день. Сколько в цехе рабочих? Сколько из них женщин и сколько человек имеют сокращенный рабочий день? 57
24. Влажность свежескошенной травы 70%, а влажность сена 16%. Сколь- ко надо скосить травы, чтобы получить 1 т сена? 25. В свежих грибах 90% воды, а когда их подсушили, то они стали легче на 15 кг при влажности 60%. Сколько было свежих грибов? 26. Цену на товар сначала снизили на 10%, а затем-на 15%. Каков общий процент снижения? Был бы он таким, если бы сначала снизили на 15%, а затем-на 10%? 27. Собаки Отгадай и Угадай соревновались в беге. Прыжок Угадая на 30% короче, чем прыжок Отгадая, но зато он успевал за то же время делать на 30% прыжков больше, чем Отгадай. Кто из них победит в соревновании? 28. Определите длину пути, на котором уменьшится на 14% количество зерна в ящике сеялки с шириной захвата 4 м, если в ящик было засыпано 250 кг зерна, а норма высева 175 кг на 1 га. 29. В комиссионном магазине в первый день яблоки продавали по 60 к. за 1 кг. Во второй день после снижения цены продали яблок на 50% больше, чем в первый, а выручка возросла на 12,5%. Какова стала цена яблок после снижения? 30. На одном из полукустарных производств, отливая в формах заготовки 2 и вытачивая из них детали, использовали на каждую деталь лишь 66-% материала заготовки, а остальное шло в стружку. В целях экономии де- фицитного материала стружку переплавили и отлили снова заготовки. Какое наибольшее число деталей можно получить из 20 одинаковых заготовок и каков при этом будет отход? 31. Сравните дроби: ч 373737 37 „ 41 411 а) уптп и 77’ б) бТ и 61Т‘ 32. Какая дробь больше: 200200201 300 300 30 Ц 200200203 ИЛИ 300300304’ 33. Какая из дробей больше: 5555 553 6666664? 5555 557 ИЛИ 6666669’ 34. Сравните дроби: 368972 368975 764797 И 764804’ 35. Говорят, что на вопрос о том, сколько у него учеников, древне- греческий математик Пифагор ответил так: «Половина моих учеников изучает математику, четверть изучает природу, седьмая часть проводит время в мол- чаливом размышлении, остальную часть составляют 3 девы». Сколько уче- ников было у Пифагора? 58
36. своего 37. Вода при замерзании увеличивается на - своего объема. На какую часть объема уменьшается лед при превращении в воду? 35 28 25 Найдите наименьшее число, при делении которого на —, и —- 66 165 231 получаются натуральные числа. 38. Найдите наибольшее число, при делении на которое каждой из дробей 154 385 231 и — получаются натуральные числа. 39. Юра взял у Лены книгу на три дня. В первый день он прочитал полкниги, во второй-треть оставшихся страниц, а в третий день прочитал количество страниц, равное половине страниц, прочитанных за первые два дня. Успел ли Юра прочитать за три дня эту книгу? 40. Миша заплатил в кассу столовой за 3 блюда, а Саша-за 2 (все блюда по одной цене). За столом к ним присоединился Гриша, и они втроем съели 5 блюд. Во время расчета оказалось, что Гриша должен заплатить товарищам 50 к. Сколько из этой суммы он должен дать Мише и сколько Саше, если каждый из них должен внести одинаковую плату? 41. Дорога от дома до школы занимает у Сережи 30 мин. Однажды по дороге он вспомнил, что забыл дома ручку. Сережа знал, что если он продолжит путь в школу с той же скоростью, то придет туда за 9 мин до звонка, а если вернется домой за ручкой, то, идя с той же скоростью, он опоздает к началу урока на 11 мин. Какую часть пути он прошел? 2 42. Как от куска материи в - м отрезать полметра, если под руками ничего нет, чем можно было бы измерить? Не пользуясь никакой мерой длины или другими подсобными средст- как от куска материи м отрезать ровно полметра? I 1 Ткань во время стирки садится Ha*jg часть по длине и на — по ширине. Какой длины надо взять кусок ткани, чтобы после стирки иметь 221 м2, если до стирки ширина ее была 875 мм? Вычислите наиболее рациональным способом: _ 225 + 375-138 Х~ 375-139 - 150’ 43. вами, 44. 45. 46. Вычислите наиболее рациональным способом: 71 573 2 \ 111 111 + 222222 7-37-ЗЛ 333 47. сосуда В одном сосуде имеется а литров воды, а другой пустой. Из первого переливают половину имеющейся в нем воды во второй, затем из 1 второго переливают - имеющейся в нем воды в первый, затем из первого 59
1 „ переливают - имеющейся в нем воды во второй и т.д. Какое количество литров воды будет в первом сосуде после 1987-го переливания? 48. На листе написано несколько ненулевых чисел, каждое из которых равно полусумме остальных. Сколько чисел написано? 49. Сумма нескольких чисел равна 10. Может ли сумма их квадратов быть меньше, чем 0,2? 50. В числителе дроби-число, у которого крайняя слева цифра 1, а за ней записано 1989 шестерок, в знаменателе же-число, у которого крайняя справа цифра 4, а перед ней слева -1989 шестерок. Вася сократил эту дробь, зачеркнув все шестерки в числителе и знаменателе, и получил -. Верен ли результат, полученный Васей? Можно ли только по ответу судить о правильности решения задач? 51. За каждый из двух месяцев-январь и февраль-завод выпустил более - продукции высшего качества. Какая часть продукции выпущена заводом высшего качества за январь и февраль отдельно, если известно, что каждая из этих дробей несократима, не изменяется при одновременном прибавлении к числителю 2 и умножении знаменателя на 2 и если за январь выпущено бодьше, чем за февраль? ' „ „ _ 11 52. Прибавив к числителю и знаменателю дроби — одно и то же число, 41 „ 3 Ира получила после сокращения в результате -, а Оля, вычтя из числителя О 37 3 дроби — некоторое число и прибавив его к знаменателю, получила —. Какие 63 17 числа прибавляли и вычитали девочки? 537 53. Вычтя из числителя дроби —— некоторое число и прибавив его 463 к знаменателю, после сокращения Таня получила Какое число она вычитала из числителя и прибавляла к знаменателю? 54. Продолжая свои исследования с дробями, Таня прибавила к числителю некоторой дроби 4, а к знаменателю 10, после сокращения получила, к своему удивлению, исходную дробь. Она попробовала проделать это же с другой дробью, но результат уже не повторился. Какую же дробь имела перво- начально Таня? 55. Когда Таня познакомила со своей задачей Светлану (см. задачу 54), то 2 Светлана решила найти еще одно интересное свойство дроби -. При этом она обнаружила, что если к числителю этой дроби прибавить 2, а знаменатель умножить на 2, то после сокращения снова получится исходная дробь. Светлана подумала, что, как и в предыдущем случае, такая дробь единствен- ная. Права ли она? 60
56. Обобщая задачу 55, члены математического кружка установили, что к числителю несократимой дроби можно прибавить не только число 2, умножая при этом знаменатель на это же число. Они нашли общий вид такого числа. Найдите и вы его. 57. Сократима ли дробь, дополняющая данную несократимую дробь до 1? Всегда ли будет справедливой подмеченная вами закономерность? Докажите. 58. Найдите натуральные числа а и Ь, такие, чтобы число, обратное их разности, было в три раза больше числа, обратного их произведению. 59. Найдите целые неотрицательные значения и, при которых 30л + 2 12и + 1 является целым числом. 60. В бочке ровно 30 л олифы. Для трех строительных бригад из нее наполнили 3 бидона, каждый из которых вмещает целое число литров, причем 2 3 емкость первого составляет - емкости второго или - емкости третьего. Сколько литров олифы осталось в бочке? 61. Число выданных из библиотеки книг читателям составляет — от числа 16 книг, находящихся на полках. После того как передали из библиотеки в читальный зал 2000 книг, число книг, отсутствующих на полках, ста- 1 ло составлять — числа книг, оставшихся на полках. Сколько книг имеет библиотека? 62. На трех полках стояли книги. На нижней было в 2 раза меньше, чем на остальных двух, на средней-в 3 раза меньше, чем на остальных двух, а на верхней-30 книг. Сколько книг всего на трех полках? 63. Четверо друзей купили вместе футбольный мяч. Первый из них внес 2 р. 30 к., второй-третью часть суммы, вносимой остальными, третий- четверть суммы, вносимой остальными, а четвертый-пятую часть суммы, вносимой остальными. Определите стоимость покупки и величину взноса каждого мальчика. 64. В чашку влили один стакан кофе и затем из стакана, наполненного молоком, в эту чашку отлили четверть стакана молока. Перемешав тщательно содержимое чашки, из нее долили стакан с молоком. Сколько осталось в чашке молока и сколько кофе? 65. В одном стакане имеется вода, в другом-столько же молока. Из второго стакана набирают ложку молока, вливают в первый стакан и разме- шивают. Затем берут из первого стакана ложку смеси и вливают во второй стакан. Чего больше: молока в воде или воды в молоке? 66. После того как туристы прошли 1 км и половину оставшегося пути, им еще осталось пройти треть всего пути и 1 км. Чему равен путь? 67. Из работавших на пришкольном участке учеников треть были девочки, 5 половина которых учится в VI классе. Из работавших мальчиков - не учатся в VI классе. Какое наименьшее число шестиклассников могло быть среди работавших на пришкольном участке? 61
68. Когда Васе Верхоглядкину предложили такую задачу: «Два туриста вышли одновременно из Л в В. Первый половину времени, затраченного на весь путь, шел со скоростью 5 км/ч, а остальную часть времени шел со скоростью 4 км/ч. Второй турист первую половину пути шел со скоростью 5 км/ч, а вторую-со скоростью 4 км/ч. Кто из них раньше прибудет в Б?»-то он заявил: «Ясно, что они придут в В одновременно». А как думаете вы? 69. В магазине имеются конфеты по 2 р. и по 3 р. за 1 кг, каждого сорта на одинаковую сумму. По какой цене надо продавать смесь этих конфет, чтобы стоимость всех конфет осталась прежней? 70. На некоторую сумму куплена клубника по 2 р. 40 к. за 1 кг одного сорта и на такую же сумму другого сорта по 1 р. 60 к. Найдите среднюю цену 1 кг купленной клубники. 71. Автобус прошел три равных по длине участка пути: первый со средней скоростью 50 км/ч, второй-30 км/ч и третий-70 км/ч. Какова средняя ско- рость движения автобуса на всем пути? 72. Найдите произведение: 73. Электронно-вычислительная машина может быстро подсчитывать сумму 2 2 2 2 1-3 + 3-5 + 5-7 + ” + 99-10Г Однако вы здесь можете с ней посоревноваться, если откроете секрет и по-иному запишите слагаемые. Как это сделать? 74. Установите правило, по которому составлена таблица, и исключите «лишнее» число: 1 1 23 3 8 7 ,2 4 1 37 11 3 5 4 0,125 13 П 75. Найдите все дроби с однозначными знаменателями, каждая из которых _ 7 8 больше х,. но меньше -. 9 9 76. Андрей, Борис и Сергей, соревнуясь в беге на стометровке, одно- временно стартовали. Когда Андрей финишировал, Борис был позади его в десяти метрах, а когда финишировал Борис, Сергей был позади его в десяти 62
метрах. На каком расстоянии находился Сергей от Андрея, когда Андрей финишировал? (Предполагается, что каждый бежит с постоянной скоростью.) 77. В таблице переставьте дроби так, чтобы сумма чисел в каждой строке, в каждом столбце, а также по диагоналям была равна 1. 1 5 2 5 3 5 1 15 2 15 4 15 7 15 1 3 8 15 78. «Ну, погоди!»-зарычал волк, заметив в 30 м зайца, бросился за ним, когда тому оставалось до места укрытия 250 м. Догонит ли волк зайца, если он пробегает за минуту 600 м, а заяц-550 м? 79. Трем геологам надо было добраться до станции, находящейся в 60 км от их базы, за 3 ч. Смогут ли они это сделать, если в их распоряжении есть мотоцикл, на котором можно ехать не больше чем двоим со скоростью не больше 50 км/ч, а пешеход идет со скоростью 5 км/ч? 80. Задача И. Ньютона. Трава на всем лугу растет одинаково густо и быстро. Известно, что 70 коров поели бы ее за 24 дня, а 30-за 60 дней. Сколько коров поели бы всю траву за 96 дней? 81. Ровно в 24 ч батарея получила приказ начать артподготовку че- рез наименьшее время, которое пройдет до того момента, когда часовая и минутная стрелки снова совпадут. В котором часу надо начинать артпод- готовку? 82. Андрей и дедушка Гриша отправились за грибами где-то между шестью и семью часами утра, в момент, когда стрелки часов были совмещены. Домой они вернулись между двенадцатью и часом дня, в момент, когда стрелки часов были направлены в прямо противоположные стороны. Сколько времени длилась их грибная «охота»? м „ 111 1 83. При вычислении суммы - + - + - + ... + - при различных п никто из ребят не получил целого числа. Случайно ли это? А как вы думаете, если вычислять на ЭВМ, то при достаточно больших п может все-таки удастся получить целое число? 84. Числа а, b и с удовлетворяют такому условию: Ь + с — а а + c — b а + Ь — с _ „ -------------,------------- и натуральные и равные между собой. Может а----------------------b-с (а + Ь)(Ь + с\(а + с\ ли выражение 1;——-------------- равняться 12? abc 63
85. Какому числу может равняться выражение (а + 6)(6 + с) (д + с) abc если числа а, b и с удовлетворяют условию задачи 84? 86. Расшифруйте числовой ребус СЛОВ, О + СЛОВ, О = ПЕСНЯ. (Одинаковым буквам соответствуют одинаковые цифры, разным-разные.) 87. Любитель математики записал числовой ребус: «Н : Е = О, СТАРЕЮ СТАРЕЮ СТАРЕЮ “» Расшифруйте эту запись, если одинаковые буквы обозначают одинаковые цифры, разные-разные цифры. (После знака равенства записана не буква, а цифра 0.) Поиски решений 1. Поскольку запись дроби не изменяется, если ее перевернуть «вверх ногами», то непричастны ли здесь цифры-«акробатки»? Что это за цифры? 2. Количество рыбы в ставке можно было бы найти, если бы знали, какую часть имеющейся там рыбы составляют выловленные 80 рыбин. Очевидно, для этой цели и вылавливали повторно с меченой рыбой. Порассуждаем над этим. 3. Число овец в отаре можно узнать, если знать, какую часть этого числа составляют 60 овец. 4. Как и в предыдущих задачах, решение сводится к определению, какую часть всей массы составляет масса, данная в условии задачи. 5. Причиной получения неверного равенства из данного верного может быть нарушение математического правила, закона. Переходя от равенства к равенству, надо обнаружить это нарушение. 6. Прыгая по прямой, кузнечик за некоторое число прыжков» может переместиться на 7 м. За какое? Подумаем, как он должен прыгать, чтобы переместиться на оставшееся расстояние. 7. Искать недостающие числа следует, отправляясь от имеющихся резуль- татов вычитания а), б), г) и известных знаменателей и числителей некоторых компонентов. (Используется метод проб, но целенаправленно.) 8. При отыскании знаков действий важно сравнивать полученный резуль- тат с первой компонентой. 64
9. Если бы удалось найти, за какое время туристы прошли пешком полпути, то можно было бы ответить и на вопросы задачи. Найти это нетрудно, так как известно, за какое время они могут проехать весь путь автобусом. 10. Порассуждаем так: во-первых, нельзя ли использовать эти же цифры, которые даны в условии для записи другого примера? Как изменить ком- поненты, чтобы результат остался тот же? Во-вторых, надо попробовать найти примеры, в которых числа записаны иными цифрами. 11 Задачу можно решить с помощью уравнения, но, судя по условию зада*чп, такое решение предвидится не очень удобным. Поищем иное решение. Обратим внимание на последнее количество съеденных плюшек. Какую часть составляет полплюшки от съеденных в третий раз плюшек? Зная число плюшек, съеденных в третий раз, можно найти, сколько плюшек было съедено во второй раз и сколько их было перед этим. Так, рассуждая «с конца», дойдем до исходного момента и определим, сколько было плюшек. 12. Если часы отстают за 1 ч на 5 мин, то за 60 мин они «отсчитают» не 60 мин. А сколько? Узнав это, можем найти, за сколько минут (часов) они «отсчитают» 5,5 ч. 13. Решение задачи сводится к определению времени продолжительности работы часов при одном обороте головки. Число нужных оборотов головки часов за сутки найти можно. 14. Задачу можно решать с помощью уравнения. Поищем иной путь решения. Температуру в первый и последний дни наблюдений можно было бы узнать, если бы знали, в какой день была средняя температура (при нечетном числе дней). В данном случае это сделать можно. 15. Ответ на вопрос задачи будем искать, сравнивая средний возраст учеников различных групп класса. (Среднее нескольких чисел есть частное от деления их суммы на количество слагаемых.) Каков был бы средний возраст ученика из группы в 25 человек, если бы им вместе было не менее 225 лет? А средний возраст ученика из остальной группы (10 человек)? Вывод? 16. Если предположить, что нет квадратов с одинаковым урожаем, то можно определить, какой бы урожай собрали со всего участка. Но для этого надо знать число всех квадратов и в каких единицах массы выражается средний их урожай. Сравнив полученный при таком предположении урожай с действительным, сделаем определенный вывод. 17. Для отыскания чисел, удовлетворяющих условию задачи, обозначим их буквами, запишем равенство согласно условию и исследуем его. Но предва- рительно записанное равенство преобразуем так, чтобы в правой его части было целое число. Для этого обе части равенства надо умножить на некоторое число. На какое же? Из равенства следует 10-£, откуда определяются возможные значения Ь, а затем и а. 18. Скорость, с какой шли туристы, можно было бы найти, если бы знали, какое расстояние они прошли и за какое время. Для этого обратимся к исследованию движения автобуса, осуществляемого по графику-расписанию. Поскольку сэкономлено 15 мин, то это как раз то время, которое автобус должен был затратить на дорогу от места встречи до вокзала и обратно. Зная время движения автобуса и его скорость, можно найти расстояние от вокзала до места встречи. Время, в течение которого шли туристы до встречи t автобусом, можно найти, зная время их выхода и встречи. 3 Зак 253! Д В Клименченко 65
19. Обратим внимание на то, что число тетрадей одной стопки составля- ет некоторое число процентов не от всего числа тетрадей, а от того числа, которое имеется в другой стопке. Поэтому надо постараться найти, сколько процентов составляют все 80 тетрадей. Тогда что же принять за 100%? 20. При решении этой задачи будем осторожно обращаться с процентами. Если девочек а. то подумаем, как выразить число мальчиков. 21. Ясно, что надо сравнить площади участков до изменения их сторон и после изменения. Поэтому надо выражать эти площади через одни и те же величины - длины сторон. 22. Наименьшее число всех домов будет в том случае,, если наибольший возможный процент домов, имеющих не больше пяти этажей, составит наименьшее число этих домов. Каково же наибольшее возможное число целых процентов домов, имеющих не больше пяти этажей? 23. Поскольку женщины составляют треть всех рабочих, то число рабочих кратно числу.... Какому? А какому числу кратно еще это число, если 8% рабочих имеют сокращенный рабочий день? (8%, конечно, мы можем за- писать в виде обыкновенной дроби.) Какое же число, меньшее 100, удов- летворяет этим условиям? 24. Подобные задачи приходится нередко решать специалистам разных профессий. Давайте поможем им в этом. Главное, из чего надо при решении исходить,-это знать, что при изменении процента влажности изменяется только содержание воды, а масса сухого вещества остается постоянной. Зная процентное содержание сухого вещества, можно найти общую массу при любой влажности. Сколько же процентов составляет сухая масса при 16% влажности? А теперь подумаем, сколько же будет сухой массы в 1 т такого сена. 25. Расчеты и в целом решение значительно упрощаются, если массу свежих грибов принять, например, за х кг и составить уравнение. Ведь при влажности 90% легко определить (выразить через х) количество сухой массы, которая при влажности 60% будет составлять... . Сколько же? А теперь постараемся выразить испарившиеся 15 кг через х. 26. При решении не будем уподобляться Васе Верхоглядкину, получивше- му в ответе 25%, а подойдем к решению со знанием дела. Поэтому, приняв первоначальную цену за а р., выполним все расчеты, опираясь на, опыт решения предыдущих задач. 27. Для ответа на вопрос задачи надо сравнить расстояния, которые пробежали собаки за одно и то же время. Хорошо, но откуда взять эти расстояния? А их просто надо выразить в общем виде, приняв за какую-то условную единицу (или обозначив буквой) длину прыжка Отгадая, а за другую-число его прыжков за некоторое время. 28. Длину пути мы могли бы определить, если бы, кроме ширины захвата, знали площадь. Как же ее найти? Известна норма высева на 1 га, значит, для определения площади надо знать количество высеянного зерна. 29. Для решения задачи надо знать, сколько продано яблок во второй день и какова выручка (полученная за них сумма денег). Если нельзя получить численные значения этих величин, то, возможно, их удастся выразить через другие величины. Для этой цели можно обозначить буквой или количество проданных яблок в первый день, или сумму вырученных денег. 66
30. Изготовив из 20 заготовок 20 деталей, какая-то часть пойдет в стружку. Какая же и на сколько деталей хватит этого материала? После изготовления из него деталей снова останется материал. Сколько? На сколько деталей его хватит? Продолжив такой расчет до тех пор, пока оставшегося материала не хватит на одну деталь, определим общее число деталей, которое можно изготовить, и количество оставшегося материала. 31. а) Для сравнения дробей необходимо либо сократить первую (если можно), разложив на множители числитель и знаменатель, либо умножить числитель и знаменатель на некоторое число второй дроби. б) Замечаем, что каждая из дробей близка к единице. Подумаем, нельзя ли использовать этот факт для их сравнения. (Сравним дроби, дополняющие данные до единицы.) 32. Пусть вас не смущает то, что числители и знаменатели данных дробей такие большие числа. Если внимательно присмотреться к этим дробям, то нетрудно заметить их сходство с дробями задачи 31 б), а значит, и метод сравнения их аналогичный. 33. Здесь также придется сравнивать дополнения дробей до единицы, но способ сравнения их иной: сравнить произведения числителя первой на знаменатель второй и знаменателя первой на числитель второй, но не выполняя действий умножения. 34. Начав сравнение дробей с общего приема (если имеем дроби у и то b а сравнивать надо ad и Ьс), в дальнейшем постараемся отыскать рациональные приемы сравнения произведений, не выполняя действий умножения. Здесь поможет нам хорошее «видение» чисел. 35. Число всех учеников можно определить, если знать, какую часть общего числа учеников составляют 3 ученика (девы). 36. Как и при решении задачи 26, не будем соблазняться первыми поверх- ностными впечатлениями, а проведем глубокий анализ и точный расчет, обратив внимание на то, что лед уменьшается на какую-то часть своего объема, а не объема воды. 37. Для решения задачи надо подумать и установить, чем должны быть числитель и знаменатель искомой дроби по отношению к числителям и зна- менятелям данных дробей. 38. «Ключ» для решения здесь такой же, как и задачи 37, но «пользоваться надо обратной стороной». 39. Ответить на вопрос задачи можно, сравнив, какую часть книги оста- лось прочитать после двух дней и какую часть Юра прочитал за третий день согласно условию задачи. 40. Чтобы узнать, сколько из 50 к. надо дать Мише и сколько Саше, надо знать, сколько внесено в кассу каждым из них, а это можно узнать, если знать стоимость 1 блюда. Как же подойти к этому? Не поможет ли здесь знание той суммы, которую внес Гриша? Какую часть она составляет от общей стоимости всех блюд? А дальше, зная стоимость 5 блюд, можно ответить на поставленный нами вопрос, а затем получить ответ на вопрос задачи. 41. Для того чтобы ответить на вопрос задачи, надо знать, какую часть пути Сережа проходит за 1 мин и сколько минут он затратил, чтобы пройти расстояние от дома до того места, где он вспомнил о том, что забыл ручку. з 67
Определение части проходимого в минуту пути особого труда не представ- ляет, а чтобы найти время, за которое пройдено расстояние от дома до места, где Сережа вспомнил о забытой ручке, надо найти время, которое было затрачено на дорогу к дому и обратно. 42. Поскольку задачу надо решать без подручных средств, кроме самой материи, то надо найти способ, как ее складывать. Если нельзя сразу выделить 1 2 1 1 ч 2 - м, то нужно попытаться получить - — - = - (м), имея - м. 43. Аналогично задаче 42 получение м сводить к поискам получения 8 1 1 z ч — — - = — (м) путем складывания материи. 44. Зная ширину материи до стирки, можно найти ее после стирки, а затем и длину после стирки, после чего до цели уже два шага. 45. Поищем способ преобразования знаменателя, а не будем спешить выполнять указанные действия. 46. Раскрыв скобки, упростим полученные дроби и так же, как и в пре- дыдущей задаче, будем выполнять тождественные преобразования и не спе- шить выполнять указанные действия. 47. Составив таблицу переливаний и проанализировав ее, поищем за- кономерность. 48. Искать ответ будем, представив задачу в общем виде, г. е. взяв числа а2, ап. Записав равенства по условию задачи и сложив их почленно, придем к некоторому заключению. 49. На первый взгляд ответ кажется отрицательным, но не будем про- являть поспешность, а, трезво все взвесив, поищем примеры, рассмотрев в качестве слагаемых равные, довольно малые дроби. 50. Для проверки результата будем умножать числитель на 4 (или делить знаменатель на 4). Разумеется, мы не будем записывать буквально число, имеющее 1990 цифр, ибо это на практике вряд ли осуществимо. Будем записывать числа, употребляя троеточие. Например, 66.. .64. Значит, и дейст- вия умножение или деление не могут быть выполнены буквально от начала до конца. Главное-увидеть этот процесс и предсказать, как он будет про- текать. Как видим, ответ иногда может быть получен верный при неверных операциях. Значит, судить о том, верно или неверно решена задача, только по ответу нельзя. 51. Для отыскания дробей, удовлетворяющих условию задачи, следует взять дробь в общем виде и, выполнив преобразования согласно условию задачи, найти возможные значения числителя и знаменателя. 52. Решение задачи сводится к составлению и решению несложных урав- нений согласно условию. 53-56. При решении этих задач используется та же идея, что и при решении задач 51-52. а 57. Взяв дробь, например -, найдем дробь, дополняющую ее до 1. Могут b 68
ли иметь числитель и знаменатель полученной дроби общие делители, отличные от 1? 1 2 58. Числом, обратным, например, числу 5, является -, а числу --число 3 тл -. Используя это, запишем равенство в общем виде и, исследуя его, найдем а и Ь. 59. Поскольку числитель данной дроби больше знаменателя, то можно выделить целое число. Для этого надо преобразовать числитель и почленно разделить на знаменатель. При каком значении п выделенная правильная дробь обратиться в О? 60. Узнать, сколько олифы осталось в бочке, мы смогли бы, если бы знали, сколько отлили. Последнее можно было бы узнать, если бы знали емкости бидонов. Попытаемся отыскать емкости бидонов, выразив емкости второго и третьего бидонов через емкость первого и приняв во внимание, что каждый бидон вмещает целое число литров. 61. В условии задачи известно некоторое число книг (2000). Если бы удалось найти, какую часть это число составляет от общего числа книг библиотеки, то до решения задачи остался бы один шаг. Найти, какую часть составляет 2000 книг от общего числа, можно будет, если найдем, какая часть от общего числа книг отсутствовала на полках вначале и после того, как сняли с полок и передали 2000 книг. Для этого будем рассуждать так: если одно 1 1 6 слагаемое составляет - другого, то сумма их составляет - + 1 = -, и тогда это слагаемое от суммы составляет 7:7 = 7. (Идея есть, надо ее реализовать.) 5 5 6 62. Поскольку в условии задачи известно некоторое число книг, то будем стремиться отыскать, какую часть оно составляет от общего числа книг. Для этого число книг на нижней полке примем за 1. Сколько таких единиц будет составлять количество книг на трех полках? Какую же часть всех книг составляет число книг на нижней полке? а на средней? Если это определим, тогда без особого труда можно определить, какую часть составляют 30 книг от общего числа книг. А это «ключ» к решению. 63. Для поиска решения используем идеи решения задачи 61, определяя, какую часть общей суммы составляет взнос первого-2 р. 30 к. Предвари- тельно узнаем, какую часть составляет взнос второго, третьего и четвертого мальчиков. 64. Если в чашку, в которой имеется стакан кофе, влили четверть стакана молока, то можно определить, какую часть стакана составляет содержимое чашки и сколько при этом частей будет содержаться кофе и сколько молока, х Если из чашки дополним стакан, то в чашке останется стакан кофе с молоком. А изменится ли после этого соотношение кофе и молока в чашке? Теперь не представляет труда дать ответ на вопрос задачи. 65. Предположив, что в каждом стакане по а ложек жидкости, проведем рассуждение в соответствии с условием задачи, определяя, какую часть в смеси составит молоко и какую часть-вода. 66. Для облегчения поисков решения задачи представим условие задачи 69
схематически при помощи отрезков. Если известно, что, после того как прошли 1 км, половина оставшегося пути составляет - всего пути и 1 км, то нетрудно представить весь оставшийся путь. А это значит: открывается возможность найти, какую часть всего пути составляет некоторое число километров. И до ответа на вопрос задачи остается один шаг. 67. Для ответа на вопрос задачи будем пытаться определить, какую часть всех работавших составляют шестиклассницы-девочки и шестиклассники- - о 5 мальчики. Если - работавших на участке мальчиков не учатся в шестом классе, то какую часть составляют шестиклассники? С другой стороны, какую часть среди работавших учащихся составляют мальчики? Если шестиклассницы 1 А 4 - составляют - всех работавших, а шестиклассники-—, то можно наити наименьшее возможное число всех работающих, а затем и ответить на вопрос задачи. 68. Действительно, ответ * Васи кажется правдоподобным. Но лучшей гарантией истины в математике есть доказательство, а не чувственное пред- ставление (интуиция). Вот и давайте последуем этому: примем длину пути за а, выразим время, затраченное каждым туристом и сравним. Поучительно! Не правда ли? 69. Как и при рассмотрении задачи 68, не будем спешить с ответом, руководствуясь только чувственными представлениями. Ведь соблазнительно ответить: 2 р. 50 к. А так ли на самом деле? Приняв, например, стоимость конфет каждого сорта за а р., выразим стоимость всех конфет и их массу и найдем цену 1 кг. 70. Вы уже приобрели некоторый опыт при решении задач 68, 69, поэтому не допустите ошибки при решении этой задачи-не будем искать среднюю цену как полусумму цен первого и второго сортов. (Средняя цена есть частное от деления общей стоимости товара на его количество.) 71. Вы, очевидно, уже заметили, что здесь представляется ситуация, аналогичная той, которая имела место в задаче 70. 72. Находить разности надо, а вот перемножать полученные результаты- дело, пожалуй, безнадежное. А как же быть? Разложить на множители числители дробей (записать два множителя), а знаменатели-квадраты неко- торых чисел. А вот после этого хорошо бы выполнить сокращение дробей. 73. Каждая из дробей-слагаемых-это результат выполнения действий над двумя дробями. Знаменатели этих дробей узнаются в знаменателях имею- щихся дробей-слагаемых. Заменив каждую дробь-слагаемое разностью двух дробей, легко получим результат. 74. Чтобы обнаружить «лишнее» число, надо представить числа в одина- ковой форме. 75. В поисках нужных дробей проведем небольшое исследование. Посколь- ку числители данных дробей отличаются на единицу, то искомых дробей со знаменателем 9 нет. Будем заменять данные дроби, пользуясь основным свойством дроби, равными им дробями, но с большими знаменателями и искать дроби, большие меньшей из данных и меньшие большей. Если после сокращения получим дроби с однозначными знаменателями, то они искомые. 70
76. Для ответа на вопрос задачи надо найти, какое расстояние пробежал Сергей за то время, за которое Андрей пробежал 100 м. Для этого, обозначив скорости ребят, выразим время, за которое они пробегут каждый свое 100 90 100 90 расстояние. Если их скорости соответственно а. b и с, то-= — и —— = —. a b b с 100 Из этого находим, какое число надо делить на Ь9 чтобы получить--. а 77. Чтобы суммировать дроби, надо представить их с одинаковыми знаменателями, а затем использовать один из способов составления «маги- ческих» квадратов. (Если вам неизвестен ни один из них, смотрите решение в конце книги.) 78. Ответить на вопрос задачи можно, сравнивая время, необходимое волку и зайцу, чтобы достичь места укрытия. Расстояния они должны пробежать разные, но и скорости их тоже различные. 79. Если один из геологов отвезет другого на станцию, а затем возвратится за третьим и вместе они доедут на мотоцикле до станции, то сразу видно, что времени на это понадобится больше 3 ч. А может, они успеют при условии, что третий геолог не будет ждать возвращения мотоцикла, а выйдет одно- временно с выездом мотоциклистов и его затем встретит на пути возвратив- шийся мотоциклист и вместе доедут до станции? Проверим такую возмож- ность. Если и в этом случае геологи не успевают, то надо рассмотреть возможность, когда двое на мотоцикле доезжают до такого места, откуда можно вовремя дойти до станции. От этого места один геолог на мотоцикле возвращается и забирает третьего геолога. (Выполните все необходимые расчеты.) 80. Для удобства расчетов целесообразно принять за 1 (единицу) коли- чество травы, поедаемой одной коровой за 1 день. Тогда можно определить количество травы, поедаемой некоторым количеством коров за определенное число дней. Из сравнения количества травы, поедаемой за разное число дней, можно найти количество травы, вырастающей за 1 день. Зная количество травы, съедаемое коровами за 60 дней, и количество травы, вырастающей за 1 день, можно найти количество травы, которой хватит нескольким коровам на 96 дней, а зная, сколько травы съедает одна корова за 1 день, найдем количество коров. 81. По истечении 1 ч минутная стрелка, следуя за часовой, окажется от нее на некоторую часть полного оборота. На какую же? Зная это, можно определить, на какую часть полного оборота больше пройдет за 1 ч минутная стрелка, чем часовая, а затем и за какое время по истечении 1 ч минутная стрелка «догонит» часовую. 82. Пользуясь данными решения задачи 81, можно найти, какую часть полного оборота (окружности) пройдет часовая стрелка от показания 6 ч, пока минутная пройдет от 12 ч до 6 ч, т.е. пол-оборота. Зная, на какую часть оборота «отстанет» минутная стрелка от часовой в 6 ч 30 мин, можно найти, через какое время она ее «догонит», и тогда узнаем, какое время буду! показывать часы. Аналогичные рассуждения проведем, когда часовая стрелка будет между 12 ч и 1 ч, а минутная направлеца противоположно ей. 83. Если возьмем самое меньшее слагаемое-дробь вида то при 71
приведении дробей к общему знаменателю дополнительными множителями для всех дробей, кроме этой дроби, будут числа четные (почему?), а для дроби -нечетное. Каким числом будет числитель суммы всех дробей - четным или нечетным? а знаменатель? Может ли при этом числитель нацело делиться на знаменатель? ол и А (fl + />)(/> + c)(fl + c) 84. Чтобы установить вид числа - значения выражения ---—-——------ аос а + b b + с а + с надо сравнить------,-----и —-—. Для этого путем деления на знаменатель с а b преобразуем данные в условии задачи выражения. Если окажутся сравни- ваемые выражения равными (числовые значения их равны), то их произведение не может равняться 12. Подумаем почему. 85. Используя решение задачи 84, будем пытаться сравнивать числа а, b и с. (Попробуем доказать их равенство.) 86. Обратив внимание на то, что при сложении двух одинаковых деся- тичных дробей получаем целое число, определяем цифру, обозначенную буквой О. Определяется также сразу цифра, обозначенная буквой П, так как в целой части каждого слагаемого по 4 цифры, а в полученном результате 5. Так как Н / 1, то для Н остается одно значение. Какое? Методом проб определяем остальные цифры. 87. Из условия видно, что в результате деления однозначных чисел получается периодическая дробь, в периоде которой записано 6 цифр. Каким же однозначным числом может быть делитель? Определив это число, методом проб находим Н. (Есть идея, ее надо реализовать.)
ПОЛОЖИТЕЛЬНЫЕ И ОТРИЦАТЕЛЬНЫЕ ЧИСЛА Если мы, например, говорим, что в нашем классе 32 ученика или что куртка стоит 25 р., то никаких вопросов не возникает. Но если сказать, что темпера- тура воздуха 25°, то сразу же последует вопрос: «25° тепла или холода?» Поэтому для определенноеги числа в таких случаях называют со знаком « + » или « —». Для нас стало теперь обычным слышать о погоде: «В Крыму 4-15°, в Карелии — 3°» и т.д. Или, чтобы не говорить: «ЗООр. прибыли, 100 р. убытка», тоже используют символику: «4-300, —100 р.». Такой символикой удобно пользоваться при вычислениях: В результате введения отрицательных чисел мы уже не будем говорить, что из числа 15 нельзя вычесть число 25. Теперь уже запишем 15 — 25 = —10. И возможности при решении задач при этом значительно расширились. Когда же и где появились отрицательные числа? Ни египтяне, ни вавилоня- не, ни даже древние греки чисел этих не знали. Впервые с отрицательными числами столкнулись китайские ученые (II в. до н. э.) в связи с решением уравнений. Однако знаков «4-» или « —» тогда не употребляли, а изображали положительные числа красным цветом, а отрицательные-черным, называя их «фу». Индийские математики Брахмагупта (VII в.) и Бхаскара (XII в.) при помощи положительных чисел выражали имущество, а отрицательных-долг. Они составили правила действий для этих чисел. Однако долгое время отрицательные числа считали ненастоящими, фиктивными, абсурдными. Даже Бхаскара, который пользовался этими числами, писал: «Люди не одобряют отрицательных чисел». В Европе к отрицательным числам обращается итальянский математик Леонардо Фибоначчи (XIII в.), но в учении об отрицательных числах значи- тельно далее продвинулся М. Штифель (XVI в.). Отрицательные числа он называл как «меньшие, чем ничто» и говорил, что «нуль находится между 73
истинными и абсурдными числами». И только после работ выдающегося ученого Р. Декарта (XVII в.) и других ученых XVII-XVIII вв. отрицательные числа приобретают «права гражданства». Изучая математику и другие учебные предметы в старших классах, вы непременно будете иметь дело с отрицательными числами. Весьма полезно как можно лучше познакомиться с ними теперь. В этом вам помогут следующие задачи. задачи 1. На горизонтальной координатной прямой точками отмечены несколько целых чисел, сумма которых равна 25. Если каждую точку переместить на 5 единичных отрезков влево', то сумма чисел, соответствующих этим точкам, станет равна — 35. Сколько чисел было отмечено на прямой? 2. Какая из точек А (0; — 7) или В (—4; 0) дальше удалена: а) от начала координат; б) от точки С (—4; —7)? 3. Частное от деления двух чисел равно — 1. Чему равна сумма этих чисел? Чему может равняться их разность? 4. Произведение чисел каждой строки таблицы «1 а2 а3 а4 а5 а6 а1 а8 а9 отрицательно. Какие знаки могут иметь произведения чисел столбцов? 5. Витя и Миша заспорили: первый из них утверждал, что если а = |6|, то всегда верно равенство а + b = 0; второй же говорил, что равенство это всегда неверно. Кто из них прав? 6. Впишите в свободные клетки числа так, чтобы каждое число, начиная с третьего (считая слева направо), равнялось сумме двух предшествующих ему чисел. 2 0 7. Имеется 9 чисел: —6, —4, —2, —1, 1, 2, 3, 4, 6. Известно, что сумма некоторых из этих чисел равна — 8. Запишите эти числа. 8. Прошу подумать в тишине, Учтите, случай редкий: Сидела белка на сосне, На самой средней ветке. Потом вскочила вверх на пять, Потом спустилась на семь. (Вы все должны запоминать, Как на уроке в классе.) 74
Затем проворно белка вновь Вскочила на четыре, Потом еще на девять И уселась на вершине. Сидит и смотрит с высоты На пни, березки и кусты. А сколько веток у сосны, Мы с вами вычислить должны. 9. В одну строку выписали 19 чисел. Сумма любых трех соседних чисел положительна. Может ли при этом сумма всех 19 чисел быть отрицательной? 10. Вместо букв в таблице записать такие числа, чтобы сумма их в каждой строке, столбце и диагонали были равными. Г 8 К -14 2 М -8 —4 -6 -2 В А 4 Б -18 д 11. В числовой пирамиде 12 = 3 12 3 = 4 1 2 3 4 = 5 1 2 3 4 5 = 6 1 2 3 4 5 6 = 7 1234567 = 8 12345678 = 9 расставить знаки «4-» и « —» так, чтобы выполнялись указанные равенства. Между некоторыми соседними цифрами можно не ставить знака, объединяя их в одно число. 75
12. На основании первых трех строк установите правило, по которому получается число с из чисел а и ft, и впишите в пустые клетки недостающие числа: а b с 5 -11 6 -2,7 -2,3 5 32 -18 -14 — 17 5 14 -14 13. Найдите сумму: - 100 - 99 - 98 - ... - 1 + 1 + 2 + ... + 101 + 102. 14. Найдите сумму: — 101 -99 - 97-... 4- 95 4- 97. 15. Найдите сумму: 1 4- 2 — 3 — 4 4- 5 4- 6 — 7 — 8 4- ... 4- 301 4- 302. 16. Сколько слагаемых в правой части равенства 17 = 17+16+15...+%? 17. В квадрате 4x4 клетки расставьте числа, не равные нулю, так, чтобы сумма всех чисел данного квадрата и каждого квадрата 2x2 клетки была равна нулю. 18. Можно ли в таблице 5x5 клеток расставить 25 чисел так, чтобы сумма четырех чисел в каждом квадрате 2x2 клетки была отрицательной, а сумма всех чисел положительной? 19. На числовой прямой были отмечены точки А, В, С, D. Известно, что первым трем точкам соответствуют числа 5, 8 и — 10. Какое число соответст- вует точке D, если при изменении направления числовой прямой на противо- положное сумма четырех чисел, соответствующих этим точкам, не изме- нилась? 20. Переменная а может принимать такие значения: — 32, — 30, — 28,..., 2, 4, 6, 8, а переменная Ь: —17, —15, —13, ..., 11, 13. Сколько различных значений может принимать выражение а + bl Найдите произведение наи- большего и наименьшего значений выражения а + Ь. 76
21. Упростите выражение — 1 — (— 1 — (— 1 — (— 1 —(..., если оно в записи содержит: а) 1989 единиц; б) 1990 единиц. 22. Заданы 5 чисел: аг = — 1, а2 = 0, а3 = 1, а4 = 2, а5 = 3. Шестое равно произведению первого на второе, седьмое-произведению второго на третье, восьмое - произведению третьего на четвертое и т. д. Чему равно в этом ряду последнее неравное нулю число? 23. Заданы 5 чисел: ах = 1, а2 = — 1, а3 = — 1, а4 = 1, а5 = — I. Следующие числа определяются так: ае = at a2, а7 = а2а3, а8 = а3 а4 и т. д. Чему равно #1988? 24. Дано 1989 чисел, каждое из которых 1 или — 1. Вычисляя сумму всех этих чисел, Ира получила в результате 846. Покажите, что она ошиблась. А может ли сумма равняться 845? Сколько при этом среди данных чисел было бы 1 и — 1? 25. Найдите два числа, если сумма, произведение и частное от деления их равны между собой. 26. Найдите такие два целых числа, чтобы их произведение равнялось сумме и разности этих чисел. 27. Найдите такие целые значения а, при которых значения выражения а 4- 9 ------есть целые числа. а 4- 6 28. Один из учеников записал на доске 1 *2* 3*4* 5* 6* 7* 8* 9 = 21, причем вместо звездочек он поставил «4-» или « —». Второй ученик переправил несколько знаков на противоположные и в результате получил 20. Можно ли утверждать, что по крайней мере один из учеников ошибся при вычислении результата? 29. Пусть частные а :Ь и с :d равны, т. е. а: b = с: d. Если делимое а больше делителя Л, то делимое с больше d. Возьмем произвольное положительное число а. Так как а:( —а) = ~ 1 и (—#):#=—1, то а:(— а) = (— а}\а. Но а > — а, поэтому на основании предыдущего вывода из правой части равенства имеем — а > а. Получили противоречие. Найдите ошибку. 30. Установите закономерность в расположении чисел ряда 2, 3, 0, -2, -7, -14, ... и допишите еще два числа. 31. В первой строке записаны три различных целых числа: а, Ь, с. Во второй строке под ними три разности: а — Л, b — с, с — а. Числа в третьей строке образованы по такому же правилу и т. д. Покажите, что ни в одной из строк ниже седьмой не может встретиться число 855. 32. 1, 2, 3, ..., п представлены в некотором произвольном порядке и обозначены аг, а2, ..., ап. а) Чему равна сумма (ах — 1) 4- (а2 — 2) 4-... 4- (ап — и)? б) Каким будет произведение (аг — 1) (а2 — 2)— и)-четным или не- четным, если п нечетное? 77
Поиски решений 1. Определить количество отмеченных точек-это значит определить количество слагаемых, сумма которых известна. Но, чтобы определить коли- чество слагаемых, надо знать, на сколько изменилось каждое слагаемое и их сумма при перемещении точек по координатной прямой. 2. Для облегчения поисков решения можно построить данные точки на координатной плоскости. 3. Поскольку частное от деления двух чисел равно — 1, то подумаем, что общего у этих чисел и чем они отличаются. (При определении разности не забудьте рассмотреть 2 случая.) 4. Если будем перебирать все возможные случаи по каждой строке, то решение значительно усложнится. Не проще ли сначала решить вопрос о знаке произведения всех чисел, а тогда уже и о знаках произведений чисел столбцов? 5. Чтобы разрешить спор мальчиков, надо рассмотреть разные случаи для b (h 0, h < 0). Ведь то, что верно в одном случае, неверным бывает в другом, и наоборот. Рассматривая любую ситуацию, надо «осветить» ее с разных позиций, при разных условиях. Это очень важно во всех видах деятельности. 6. Если начинать вписывать числа с крайней левой клетки (или еще какой-нибудь наугад), то вряд ли можно рассчитывать на успех. Мы уже знаем, к чему приводят «блуждания по лабиринтам вслепую». Поэтому будем осмотрительны, действуя целенаправленно. С чего начинать? Разумеется, начинать надо вписывать числа в клетках, соседних с заполненными, двигаясь влево и вправо. 7. Чтобы действовать целенаправленно в отборе нужных чисел без про- пусков и повторений, целесообразно по порядку брать по одному числу из записи данных чисел, «нанизывая» возможные ему компоненты. Найдите все 7 решений. 8. Чтобы найти число всех веток, нужно сначала, следуя условию задачи, найти число веток от средней до верхней. (Не забудьте только посчитать и саму верхнюю ветку.) 9. Для положительного ответа на вопрос задачи достаточно одного'при- мера. Если взять три числа, одно из которых отрицательное, а два других равные между собой положительные числа, то всегда можно подобрать числа так, чтобы сумма положительных была больше, чем модуль отрицательного. Тогда любые тройки чисел, записанных подряд, будут удовлетворять усло- вию. Чтобы ответить на вопрос задачи, надо брать такие числа, чтобы сумма чисел указанной тройки была как можно меньшей, а крайние числа ряда отрицательны. 78
10. Поищем сначала возможность определения суммы чисел какого- нибудь ряда (строки, столбца, диагонали). Если такая сумма будет найдена, то последовательно определяем неизвестные слагаемые. 11. Примем к сведению, что знаки необязательно ставить между каждыми соседними цифрами, можно, например, оставить 12. (Подобные задачи уже решались.) 12. Поскольку неизвестно число с, то его следует выразить через а и Ь. Подумаем, как, например, получить 6, имея 5 и —11. Й. Очевидно, буквальное суммирование всех чисел (а сколько их?) доста- вит мало удовольствия, задачу-то можно решить «с одного взгляда». Какое слагаемое должно быть перед 101? Чему же равна сумма всех чисел от — 100 до 100? Остается найти сумму двух чисел. 14. Как и в задаче 13, будем искать наиболее рациональный прием решения, обратив внимание на сумму чисел от —97 до 97. 15. На первый взгляд кажется, что для подсчета суммы здесь нельзя указать рационального приема по примеру решения предыдущих задач. Но не будем пессимистами и все-таки попробуем поискать. Нельзя ли выделить группу рядом стоящих чисел, сумма которых равна 0? Сколько всего таких групп в записи? А какие числа вне этих групп? 16. Прежде всего выясним, каким должно быть число х- положительным или отрицательным. Сколько же в правой части слагаемых со знаком « —»? (При подсчете числа слагаемых не забудем и о том числе, которое записано между 1 и — 1.) 17. Числа, очевидно, надо брать равные по модулю, но разные по знакам, и тогда можно найти сколько угодно решений. Найдем хотя бы одно из них. 18. Попробуйте расставить 9 двоек и 16 раз по — 1. Какой будет общая сумма? Расставляя числа 2 и — 1 в квадратах 2x2 клеток, надо добиться того, чтобы сумма их была отрицательной. Сколько раз для этого нужно записать число — 1? 19. Поскольку сумма не изменилась при изменении направления числовой прямой на противоположное, то она представляет собой такое число, которое «не реагирует» на изменение знака. Что же это за число? Зная сумму четырех чисел и трех слагаемых, не представит труда определить четвертое слагаемое. 20. Число всех значений выражения a -I- b можем определить, если будем знать число всех значений переменных а и Ь. Из условия задачи это можно найти. 21. Для решения задачи пронаблюдаем результаты при четном и нечетном числе единиц и установим закономерность. 22. Решение задачи сводится к составлению таблицы и ее анализу. 23. Было бы слишком обременительным и практически невыполнимым продолжение процесса определения всех а, вплоть до я1988. Надо поискать более разумный способ решения. А это возможно, если установим законо- мерность следования чисел в таблице, составленной для нескольких первых чисел. Нельзя ли выделить в последовательности записанных чисел группу чисел, которая повторяется? Подумаем, сколько таких групп будет до а1988. 24. Выявить ощибку в данном случае - значит показать, что сумма не может равнятья 846. А что это значит? В каком случае может быть сумма 846? Верно, если на столько больше слагаемым повторяется 1, чем — 1. Сколько же раз в этом случае повторится слагаемым — 1? Возможно ли это? Аналогично исследуем, если сумма равна 845.
23. 1 м3 = 1000000 см3; одна спичка имеет объем 5 • 0,2 • 0,2 = 0,2 (см3); 1 000000:0,2 = 5000000. Итак, 5 млн. спичек. 24. Во-первых, точки J, В и С должны лежать на одной прямой, так как в противном случае существует точка внутри треугольника АВС. равноудален- ная от его вершин-точек А. В и С (центр окружности, проходящей через точки А, В и Q. Во-вторых, точка В не может лежать между А и С, равно как и точка С между А и В. так как в этом случае найдется сколько угодно точек Л/, таких, что AM > ВМ и AM > СМ. Остается единственное: точки А. В и С лежат на одной прямой и точка А между В и С. 25. Изобразим схематически окна точками и занумеруем их: 4е *3 1 • «2 . а) зажигание света во всех четырех окнах дает один сигнал; б) зажигание света в одном из окон воспринимается как один сигнал, так как в темноте нельзя различить положение окон относительно дома; в) зажиганием света в двух окнах можно подать 4 различных сигнала: один сигнал-зажиганием в окнах 1, 4 или 2, 3; один сигнал-зажиганием в окнах 1, 2 или 4, 3; два сигнала-зажиганием в окнах 1, 3 и 2, 4, т.е. при этом имеем такие конфигурации: 1) • 2) • • 3) • 4) • г) зажиганием света в трех окнах можно подать четыре сигнала: 1, 2, 3; 2, 3, 4; 3, 4, 1 и 4, 1, 2, т.е. имеем_такие конфигурации: • • • • • • 1) 2) 3) 4) Всего имеем различных сигналов: 1 + 1+ 4 + 4=10. 26. Как показано на рисунке 28, общей частью треугольника и четырех- угольника могут быть отрезок, треугольник, четырехугольник, пятиугольник, шестиугольник и семиугольник. 27. Поскольку должна получиться ломаная замкнутая, то, начав выклады- вать палочки от некоторой точки, мы должны сюда же возвратиться. Это значит, сколько выложено палочек в одном направлении, столько же должно быть выложено и в противоположном. Если, например, вниз выложено а палочек, то и вверх будет выложено а. вправо -Ь. то и влево -Ь. Всего будет выложено 2а + 2Ь = 2 (а + Ь). По условию 2 (а + Ь) = 1989. Так как 1989 не делится на 2, то требуемую ломаную выложить нельзя. 80
ВЫРАЖЕНИЯ С ПЕРЕМЕННЫМИ. УРАВНЕНИЯ И НЕРАВЕНСТВА Арифметические способы решения задач на вычисление не всегда доста- точно быстро приводят к желаемому результату, а порой и вовсе оказываются бессильными. И вот тут на помощь приходят уравнения. Разные уравнения рассматриваются в математике, но в V-VI классах вы имеете дело с простейшими, хотя они, конечно, посложнее тех, с которыми вы познакомились в начальных классах. В старших классах и в дальнейшем вы познакомитесь с другими видами уравнений и лучше прочувствуете их «силу». Чтобы решить задачу с помощью уравнения, прежде всего выбирают одну (или несколько) из величин и обозначают буквой (чаще х-от слова «хау»-куча, неизвестное). Затем в соответствии с условием задачи выражают через эту переменную другие величины, после чего сравнивают полученные выражения и записывают равенство (уравнение). Далее по определенным правилам находят значения переменной и проверяют, удовлетворяют ли они условию задачи. Приведем задачу, которую предложил маленькому Алек- сандру Пушкину великий полководец А. В. Суворов, гостивший в доме Ган- нибалов (деда А. С. Пушкина): «Летела стая гусей, а навстречу им гусь. - Здравствуйте, сто гусей! - говорит он им. - Нас не сто,-отвечают они ему.-Вот, если бы нас было столько, сколько есть, да еще раз столько, да полстолько, да четверть, да ты с нами, тогда было бы сто. Сколько было гусей в стае?» Мальчик долго размышлял над задачей и, только когда карета с гостем почти скрылась, он крикнул вдогонку, называя ответ. 81
Если летело х гусей, то согласно условию можно записать такое уравнение: х + х + ^х + ^х + 1 = 100, 2 4 откуда Их = 99-4; х = 36. Проверим: 36 + 36 + 18 + 9 + 1 = 100. Значит, летело 36 гусей. Видимо, так рассуждал маленький Пушкин. Когда же и какие народы начали первыми пользоваться уравнениями? Еще за 3-4 тыс. лет до н. э. египтяне и вавилоняне, пользуясь таблицами и гото- выми выработанными рецептами, умели решать некоторые уравнения. Разу- меется, приемы решения у них были вовсе не такими, как теперь. Греки, унаследовавшие математические знания египтян и вавилонян, пошли дальше. Наибольших успехов в решении уравнений добился греческий ученый Диофант (III в.). О нем потом писали: Посредством уравнений, теорем Он уйму всяких разрешил проблем: И засуху предсказывал, и ливни- Поистине его познанья дивны. Стройное учение об уравнениях разработал среднеазиатский ученый Мухаммед аль-Хорезми (IX в.). В дальнейшем проблема решений уравнений занимала умы всех математиков. Решая задачи предыдущих параграфов, вам приходилось уже обращаться к помощи уравнений и выражений с переменными. В этом параграфе пред- ставлены специальные задачи, решаемые с помощью выражений с пере- менными, уравнений и неравенств. ЗАДАЧИ 1. Одному участнику игры было предложено 30 вопросов. За каждый правильный ответ ему начислялось 7 баллов, а за неправильный (или от- сутствие ответа) снимали 12 баллов. Сколько верных ответов дал участник игры, если он набрал 77 балов? 2. Если из числа учащихся 5 А класса вычесть 23 и полученную разность умножить на 23, то получится столько же, как если бы из этого числа вычли 13 и разность умножили бы на 13. Сколько учеников в 5 А классе? 3. Сумма двух чисел равна 13,5927. Если в большем из этих чисел перенести запятую влево на одну цифру, то получим меньшее число. Найти эти числа. 4. Лев старше дикобраза В два с половиной раза. По сведеньям удода Тому назад три года В семь раз лев старше был, Чем дикобраз. Учтите все и взвесьте: Сколько же им вместе?- Позвольте мне спросить у вас. 82
5. - Я на два года старше льва,- Сказала мудрая сова. - А я в два раза младше вас,- Сове ответил дикобраз. Лев на него взглянул и гордо молвил, Чуть наморща нос: - Я старше на четыре года, Чем вы, почтенный иглонос. А сколько всем им вместе лет? Проверьте дважды свой ответ. 6. Количество деревьев, посаженных учащимися во время субботника, выражается трехзначным числом, в котором десятков на 3 больше, чем сотен, а единиц на 4 меньше, чем десятков, причем полусумма всех цифр равна цифре десятков. Сколько деревьев посадили учащиеся? 7. Позавчера школьники собрали макулатуры на 3 кг больше, чем вчера, а вчера на 40 кг меньше, чем позавчера и сегодня вместе. Сколько кило- граммов макулатуры собрали школьники сегодня? 8. Брат говорит сестре: «Когда Коле было столько лет, сколько сейчас нам с тобой вместе, то тебе было столько лет, сколько мне сейчас. А вот когда Коле было столько лет, сколько тебе сейчас, то тебе тогда было...» Сколько же лет было тогда сестре? 9. У Пети столько же сестер, сколько и братьев, а у его сестры Иры вдвое меньше сестер, чем братьев. Сколько в этой семье мальчиков и сколько девочек? 10. Тетушке Маше на три года меньше, чем Саше вместе с его ровесником Пашей. Сколько лет Саше, когда тетушке Маше было столько, сколько сейчас Паше? 11. Вера купила на 6 тетрадей меньше, чем Миша и Вася вместе, а Вася-на 10 тетрадей меньше, чем Вера и Миша вместе. Сколько тетрадей купил Миша? 12. Если к некоторому трехзначному числу приписать сначала цифру 7 слева, а затем-справа, то первое из полученных четырехзначных чисел будет на 3555 больше второго. Найдите это трехзначное число. 13. Туристы были довольны своим поваром: вкусный он суп приготовил на привале. Раскрывая «секреты» кулинарии, юный повар рассказал, что воды он взял столько, сколько крупы, картофеля, лука и жира вместе; крупы-столько, сколько картофеля, лука и жира; картофеля-столько, сколько лука и жира вместе, а жира-вдвое меньше, чем лука. Общая масса супа 12 кг. Сколько отдельно воды, крупы, картофеля, лука и жира взято для супа? 14. В мастерской проверили пять часов. Оказалось, что часы № 3 отстают от часов № 2 на 3 мин в сутки. Отстают на 1 мин в сутки часы № 1 относительно часов № 2, часы № 4-относительно часов № 3, а часы № 5- относительно часов № 1. Если же сложить показания всех пяти часов и сумму разделить на 5, то получится точное время. Известно также, что одни из этих часов идут точно. Какие именно? 15. Сколько человек в бригаде, если средний возраст всех членов бригады 25 лет, бригадиру 45 лет, а средний возраст членов бригады без бригадира 23 года? 83
16. На одном участке площадью 14 га был посажен несортовой картофель, на дру- гом-площадью 4 га-сортовой. Средняя урожайность с 1 га на этих двух участках на 20 ц выше средней урожайности первого участка. На сколько центнеров урожайность сортового картофеля выше, чем урожай- ность несортового? 17. Оказывая помощь колхозу в уборке урожая, 5 А класс за 5 дней собрал 45 715 кг картофеля. Сколько было всего учащихся и сколько в среднем собирал картофеля за день один ученик, если среднедневная вы- работка одного ученика составляет целое число килограммов? 18. Во время стоянки между рейсами матросу исполнилось 20 лет. По этому поводу в кают-компании собрались все 6 членов экипажа. «Я вдвое старше юнги и на 6 лет старше машиниста»,-сказал рулевой. «А я на столько старше юнги, на сколько моложе машиниста,-заметил боцман.-Кроме того, я на 4 года старше матроса». «Средний возраст команды -28 лет»,-дал справку капитан. Сколько лет капитану? 19. В квадрате 3x3 клетки расставили числа 1, 2, 3.9. Заз ем в каждом кружке (рис. 2) записали среднее арифметическое окружающих его четырех чисел. После этого вычислили среднее арифметическое полученных четырех чисел. Какое наибольшее число может при этом получиться? 20. Фамилия русского писателя состоит из шести букв. Известно, что числа, указывающие места этих букв в русском алфавите, находятся в следу- ющих соотношениях: первое равно третьему; второе равно четвертому; пятое на 9 больше первого; шестое на 2 меньше суммы второго и четвертого; если утроим первое, то получим число на 4 меньше второго; сумма всех этих чисел 83. Узнайте фамилию писателя. 21. Когда у юных рыбаков спросили, сколько рыбин поймал каждый из них, то первый ответил: «Я поймал половину числа рыбин, которые поймал мой товарищ, да еще 10 рыбин». Второй сказал: «А я поймал столько же, сколько мой товарищ, да еще 20 рыбин». Сколько рыбин поймали рыболовы? 22. Из дома в школу Юра вышел на 5 мин позже Лены, но шел в 2 раза быстрее, чем она. Через сколько минут после своего выхода Юра догонит Лену? 23. 25 апельсинов стоят столько же рублей, сколько их можно купить на 1 р. Сколько апельсинов можно купить на 3 р.? 24. Готовя сына в школу, отец купил ему новую форму за 24 р. Как он расплатился за покупку, если у него оказались купюры только по 5 р., а у кассира-только по 3 р.? 25. Сколькими способами из отрезков длиной 7 см и 12 см можно соста- вить отрезок длиной 1 м? 26. В соревновании веселых и находчивых за каждое правильно выполнен- ное задание начисляли 9 баллов, а за невыполненное или неверно выполненное снимали 5 баллов. Известно, что команде было предложено не больше 15 84
заданий и она набрала 57 баллов. Сколько заданий команда выполнила верно? 27. Если бы команда по последнему виду спорта набрала 97 баллов, то среднее количество баллов, набранных ею по одному виду спорта, равнялось бы 90; если бы она по этому последнему виду спорта набрала 73 балла, то среднее количество баллов по одному виду спорта составило бы 87 баллов. По скольким видам спорта выступила команда? 28. На вопрос о его возрасте дедушка ответил: «Число, выражающее мой возраст в годах, двузначное, равное сумме количества его десятков и квадрата единиц». Сколько лет дедушке? 29. На стоянке были легковые автомобили и мотоциклы. Мотоциклов с коляской было в два раза меньше, чем без коляски. Какое могло быть наибольшее число автомобилей, если всего колес у этих автомобилей и мото- циклов было 115? 30. Несколько девочек собирали грибы. Собранные грибы они поделили между собой так: одной из них дали 20 грибов и 0,04 остатка, другой-21 гриб и 0,04 нового остатка, третьей-22 гриба и 0,04 остатка и т. д. Оказалось, что все получили поровну. Сколько было девочек и сколько они собрали грибов? 31. Когда Витя станет вдвое старше, Коля будет на 4 года моложе, чем Вера. В прошлом году Коля был вдвое моложе, чем Вера, и в 3,5 раза моложе, чем Витя. Сколько сейчас лет каждому из них? 32. При стрельбе по мишени Миша несколько раз попал в десятку, столько же раз выбил по 8 очков и несколько раз попал в пятерку. Всего он набрал 99 очков. Сколько выстрелов сделал Миша, если при 6,25% выстрелах он не выбил ни одного очка? 33. В построенном школьном интернате несколько комнат было двух- местных, а остальные-трехместные. В целом интернат рассчитан больше, чем на 30, но меньше, чем на 70 мест. Когда заселили - наличных мест, то количество незаселенных оказалось таким, каким было бы общее число мест при условии, что двухместных комнат было бы столько, сколько в действи- тельности имеется трехместных, а трехместных столько, сколько имеется двухместных. На сколько мест построен интернат? 34. Вова во столько раз старше своей сестры Гали, во сколько раз он моложе бабушки Кати. Сколько лет каждому из них, если Гале нет еще и 6 лет, а Вове вместе с бабушкой уже 112 лет? 35. Хорошей была уха, которую приготовили туристы на берегу моря во время удачной рыбалки, но, не имея кулинарного опыта, положили в нее мало соли, поэтому пришлось подсаливать за столом. В другой раз на такое же количество ухи они положили в 2 раза больше соли, чем в первый раз, но и на этот раз пришлось подсаливать уху, правда, использовав для этого соли в 2 раза меньше, чем в первый раз. Какую долю необходимого количества соли было положено в уху поваром в первый раз? 36. Передние покрышки колес автомобиля стираются через 25000 км, а задние-через 15000 км. Когда целесообразно поменять местами покрышки, чтобы они одинаково износились? (Допустим, что покрышки меняются местами один раз, хотя водители делают на практике это чаще.) 37. Два человека, у которых есть один велосипед, должны попасть из пункта А в пункт В, находящийся на расстоянии 40 км от А. Первый 85
передвигается пешком со скоростью 4 км/ч, а на велосипеде-30 км/ч, второй - пешком со скоростью 6 км/ч, а на велосипеде-20 км/ч. За какое наименьшее время они смогут добраться в пункт В? (Велосипед можно оставлять без присмотра.) 38. Выполняя домашнее задание, Петя спешил на футбол и, как всегда в таких случаях, делал ошибки. Вместо того чтобы число возвысить в квадрат, он его удвоил и в результате получил двузначное число, записанное теми же цифрами, что и искомый квадрат, только в обратном порядке. Какой правильный ответ должен был получить Петя? 39. Сумма четырех последовательных нечетных чисел равна кубу одно- значного числа. Найдите все такие четверки чисел. 40. Сумма трех последовательных нечетных чисел равна четвертой степени однозначного числа. Найдите все тройки таких чисел. 41. На одной из трех карточек было написано число 18, на другой-число 75, на третьей - некоторое двузначное число. Если сложить все различные шестизначные числа, которые можно получить, выкладывая эти карточки в ряд, то получится число 2 606058. Какое число написано на третьей карточке? 42. Десять одинаковых книг стоят не больше 11 р., а 11 таких же книг стоят больше, чем 12 р. Сколько стоит одна книга? 43. Олег моложе своего отца на 32 года, а его отец на столько же моложе своего отца (дедушки Олега). Три года тому назад всем им вместе не было и ста лет. Сколько лет сейчас каждому из них? 44. На математическом вечере были ученики V и VI классов. Вместе их больше 70, но меньше, чем 90. Всего рядов стульев, на которых сидели ученики в зале, было на 3 больше, чем сидело в каждом ряду шестиклассников, а пятиклассники сидели по 3 в каждом ряду. Сколько было на вечере шестиклассников и пятиклассников отдельно? 45. Четырьмя комбайнами марки «К» и тремя комбайнами марки «Н» за 5 дней убирают урожая столько же, сколько за 4 дня тремя комбайнами марки «К» и пятью-марки «Н». Каким комбайном (какой марки) убирают больше за день? 46. Во время каникул для восьми классов школы, в каждом из которых одинаковое число учащихся, была организована экскурсия в музей, а для остальных учащихся школы, которых оказалось на 15% больше,-культпоход в кукольный театр. Сколько учащихся в школе, если известно, что их не больше, чем 520, а в экскурсии всего участвовало больше, чем 230 уче- ников? 47. При подведении итогов соревнования по сбору металлолома оказа- лось, что 6 А класс собрал металлолома больше, чем 6 Б и 6 В вместе, но 6 А и 6 Б собрали вместе столько же, сколько 6 В и 6 Г, Кроме того, 6 Б и 6 Г собрали больше, чем 6 А и 6 В. Как распределились места в соревновании этих классов? 48. Расплачиваясь за книгу, Нина дала кассиру монеты по 3 к., по 2 к., по 5 к. и по 10 к., а всего 27 монет. Монет по 5 к. было в 2 раза больше, чем по 2 к.; по 2 к.-в 2 раза больше, чем по 10 к.; а по 3 к. было больше, чем 7, но меньше, чем 20 монет. Какую сумму денег внесла Нина в кассу? 49. Произведение четырех натуральных чисел меньше их суммы, а сумма трех из них равна 28. Найдите все такие числа. 86
50. Число рабочих обувной фабрики составляет числа рабочих швейной 3 фабрики. На первом из этих предприятий месячный план перевыполнили - числа рабочих, а остальные, менее 1000 рабочих, выполнили план на 100%; на 2 - числа рабочих, а остальные, более 1000 100%. Сколько рабочих на каждой из этих другом-план перевыполнили рабочих, план выполнили на фабрик? _ „ 13 5 7 51. Докажите, что 99 1 2 4 6 8 ’ ’ ’ 100 < 10’ Поиски решений 1. Если вопросов 30, то сколько всего должно быть ответов? Число каких ответов целесообразно принять за х? 2. Для составления уравнения в соответствии с условием задачи состав- ляем сначала два выражения, содержащие неизвестные. 3. Если большее число х, то подумаем, как записать меньшее. Во сколько раз оно меньше большего? Обозначив одно из чисел через х, можно составить уравнение, так как известна сумма чисел. 4. Для составления уравнения необходимо выбрать и обозначить буквой одно из неизвестных. (Что предпочтительнее обозначить?) Затем второе неизвестное нужно выразить через первое. Уравнение составляется по одному из соотношений возрастов дикобраза и льва. 5. Чтобы найти, сколько лет всем вместе, надо найти, сколько лет каждому из них. Поэтому число лет кому-то одному обозначим буквой, а числа лет другим выразим через введенное обозначение. Успех будет во многом за- висеть от выбора первого неизвестного, а затем и соотношения, по которому составляется уравнение. 6. Для записи трехзначного числа надо знать количество единиц каждого разряда. Соотношение между ними, заданное в условии задачи, позволяет составить уравнение для определения числа десятков, сотен и единиц. 7. Если вчера собрано х кг макулатуры, то можно выразить, сколько собрали позавчера и сколько сегодня и позавчера вместе. На основании двух последних утверждений (выражений) можно найти, сколько собрали сегодня. (Можно ввести две переменные и составить уравнение, которое решается без особого труда.) 87
8. Не правда ли, читая текст, с трудом приходится улавливать соотноше- ния между возрастами в разное время? Чтобы облегчить понимание, пере- ведем текст задачи на язык символики, положив брату х лет, а сестре у лет. 9. Если в этой семье девочек х, то как выразить число мальчиков по первой части условия? По второй части условия составляем уравнение. 10. Поскольку Саша и Паша-ровесники, то им одинаковое число лет. Если принять возраст каждого из них за х лет, то, как и в задаче 8, можно словесный текст задачи перевести на язык символики и получить ответ. 11. Решение задачи значительно облегчается, если обозначим число тет- радей каждого, введя три разные буквы. Записав по условию два равенства и сложив их, получим уравнение. 12. Для определения трехзначного числа нет необходимости в данном случае записывать его в виде суммы разрядных слагаемых. Достаточно обозначить его через х и затем записать два числа: одно из них получается путем приписывания цифры 7 К числу х слева. (Как это записать на «языке» арифметических действий?) Другое-путем приписывания цифры 7 справа. (Как это записать?) Если числа записаны, то составление уравнения труда не представляет. 13. Для составления уравнения важно удачно выбрать, что обозначить через х, чтобы затем легче выразить массы* остальных пищевых компонентов. Думается, что для этой цели лучше подходит тот компонент, масса которого наименьшая. Как вы думаете, какой это продукт? (Задачу можно также решить и арифметическим способом.) 14. Чтобы определить, какие часы показывают точное время, надо выразить через одну и ту же переменную показания всех часов, обозначив через эту переменную показание одних из часов. Затем, разделив сумму показаний часов на 5, полученный результат поискать среди показаний часов. Если какое-то показание совпадает с полученным результатом, то эти часы показывают точное время. 15. Если число членов бригады обозначить через х, то можно выразить суммарный возраст (сумму лет) всех членов бригады. Второе выражение для сравнения с первым и составления уравнения получим аналогично, приняв во внимание, что членов бригады на одного меньше, если считать без бригадира. 16. Если обозначить через х среднюю урожайность несортового картофеля с 1 га, а через у-сортового, то достаточно будет найти г — х. Уравнение с двумя переменными составляется одно, но нам не надо ведь определять отдельно каждую переменную. 17. Как и в предыдущей задаче, можно ввести две переменные-для числа учащихся и среднедневной выработки. Получив х\у = 9143, постараемся разложить на множители число 9143, пользуясь методом проб. (Задачу можно было бы решить, не вводя переменные и не составляя уравнения.) 18. Зная возраст матроса, можно определить возраст еще одного члена экипажа. Кого? Для определения возраста капитана по среднему возрасту членов экипажа и их числу надо знать возраст еще трех членов экипажа: машиниста, юнги и рулевого. Для этого составляется уравнение на основании данных из заявления рулевого. 19. Вы уже вспомнили, что среднее арифметическое получим, если разде- лим сумму чисел на их количество. Чтобы получить в результате наибольшее среднее арифметическое, надо добиться получения в кружках наибольших 88
чисел-результатов. Этого можно достичь, если при определении средних арифметических чисел, записанных в клетках, чаще использовать наибольшее число и реже-наименьшее. Принимая это во внимание, подумаем: какое число надо записать в центральной клетке? а в угловых? 20. Для определения чисел, указывающих места букв в алфавите, состав- ляем уравнение, приняв за х первое число. Вы, очевидно, уже заметили, что основная трудность состоит в выражении каждого числа через первое. Но если внимательно «пройдетесь» по условию задачи, то трудность одолеете. 21. Как вы уже обратили внимание, каждый из рыбаков число пойманных им рыбин определяет через число рыбин, пойманных его товарищем. По- этому, приняв число рыбин, пойманных одним из них, за'х, выражаем число рыбин, пойманных вторым, а затем «возвращаемся» к первому и составляем уравнение. 22. Если Лена в 1 мин проходит а м, то сколько метров пройдет Юра? Предположив, что он догонит ее через х мин, можно составить уравнение. В этом уравнении, как вы видите, будет фигурировать два неизвестных, но оно легко сводится к уравнению с одним неизвестный относительно х. 23. Если 25 апельсинов стоят х р., то на 1 р. можно купить х апельсинов. Значит, можно составить уравнение в виде пропорции. Определив, сколько можно купить апельсинов на 1 р., можно ответить на вопрос задачи. 24. Поскольку неизвестными есть как число купюр по 5 р., внесенных в кассу, так и число купюр по 3 р., полученных из кассы в качестве сдачи, то для составления уравнения можно ввести две переменные. Но уравнение-то будет одно, а неизвестных два! Ничего страшного, подобные уравнения успешно решал еще древнегреческий ученый Диофант, и вы с ним также справитесь. Достаточно выразить из полученного уравнения одно неизвестное через другое и найти то наименьшее значение неизвестного, при котором значение выражения-натуральное число. 25. Задача решается аналогично решению задачи 24. 26. Как и в задачах 24 и 25, можно составить уравнение с двумя неизвест- ными и найзи решение в натуральных числах. 27. Будем искать возможность составления уравнения, приняв за х число видов спорта, по которым выступала команда. Но может быть, предвари- тельно составим два уравнения, так как в условии описано две ситуации? При этом придется ввесз и еще одну букву для обозначения числа баллов, набран- ных командой по всем видам спорта, кроме последнего, и тогда вычитанием из одного уравнения другого получим уравнение с одним неизвестным х. 28. Трудность решения состоит в том, что составленное уравнение одно, а неизвестных два. Однако, приняв во внимание, что неизвестные есть цифры, их следует искать среди чисел 1, 2, 3, ..., 9. И, получив а = , без труда можно определить Ь. 29. Если мотоциклов с коляской х, а автомобилей у, то, зная количество всех колес, можно составить уравнение. Полученное уравнение с двумя неизвестными, но мы уже знаем, как его решать (см. задачи 24, 25 и др.). Выразив у через х и приняв во внимание, что ^должен быть наибольшим из всех возможных, найдем х, а затем и у. 30. В задаче требуется найти две величины, однако следует ли вводить две переменные (две буквы)? Верно, не следует. Достаточно обозначить буквой 89
число собранных грибов и для составления уравнения выразить число грибов, собранных отдельно первой и второй девочками. Так как по условию эти числа равны, то составляем уравнение. 31. Как правило, при решении задач с помощью уравнений обозначают буквой то, о чем спрашивается в задаче. Однако далеко не всегда целесообраз- но так поступать. Примером может быть данная задача. Для обозначения здесь удобно выбрать возраст одного из персонажей в такое время, когда известны отношения возрастов других по сравнению с его возрастом. Очевид- но, целесообразно обозначить через х число лет Коле «в прошлом году». Зная это, легко выразить, сколько лет каждому из них «в этом году». Если Вите сейчас (в этом году) 3% + 1 лет, то через сколько лет его возраст удвоится? Тогда к тому времени и возраст Коли и Веры увеличится на столько же. Можно это выразить? Разумеется. На основе утверждения, что Коля будет на 4 года моложе Веры, составляем уравнение и определяем х. 32. Для определения числа всех выстрелов по мишени надо узнать, сколько из них было попаданий. (Число попаданий составляет 93,75% от общего числа выстрелов.) Для определения числа попаданий надо составить уравнение на основании данных о числах выбитых очков. Для этого придется вводить две переменные (например, х и у). Получив уравнение с двумя переменными, находим натуральные его решения. Для этого достаточно установить, на какую цифру может оканчиваться 5у, и тогда в зависимости от этого определяем, на какую цифру должно оканчиваться 18х. 33. Так как неизвестно число всех комнат, а также двухместных и трех- местных, то придется, как и в предыдущих задачах, вводить две переменные. Введя эти переменные, выразим число всех наличных мест, а затем-за- селенных и незаселенных. Зная, что число незаселенных мест равнялось бы числу всех наличных мест при некоторых условиях, составляем уравнение. Принимая во внимание, что неизвестные могут быть только натуральными числами, получаем пары чисел. Из них отбираем те, которые удовлетворяют условию задачи (число всех мест больше, чем 30, но меньше, чем 70). 34. Подумайте, на основании чего можно составить уравнение. В этом отношении привлекает внимание сумма возрастов Вовы и бабушки. Значит, если удастся выразить их возрасты через одну и ту же переменную, то, сложив эти выражения и приравняв полученную сумму 112, получим уравнение. Так как Вова во столько раз старше Гали, во сколько раз бабушка старше Вовы, то это число, показывающее отношение возрастов, примем за х. Сколько же лет согласно условию может быть Гале? Перебрав 5 возможных значений, выявляем то уравнение, которое имеет решение. «Ключом» при этом является то, что в левой части имеем х(х + 1), т. е. произведение двух последовательных чисел. 35. Поскольку требуется определить, какую долю необходимого коли- чества соли положено в уху, то следует ввести две переменные (для обозна- чения необходимого количества соли и того, которое фактически положили) и одну из них выразить через другую. Для этого составляется уравнение с двумя переменными, предварительно обозначив нужное количество соли через а г, а положенное количество соли в первый раз-через х г. 36. Если покрышки поменять через х км пробега, то подумаем, сколько километров останется пройти передним колесам до износа и сколько-задним. Но если передние поставить вместо задних, то они могут пройти только часть 90
пути, который прошли бы, будучи на своем месте. Какую же часть? Ана- логичные рассуждения проведем и относительно задних колес. Так как после замены колеса должны пройти до износа одинаковое расстояние, то состав- ляем уравнение. 37. Для определения наименьшего времени, затраченного каждым челове- ком на дорогу, будем исходить из того, что каждый из них был в пути одинаковое время. А это значит, что если первый ехал х ч, то второй столько часов шел, а если первый шел у ч, то второй столько часов ехал. Составив уравнение на основании равенства пути, который каждый из них преодолел, выразим одну переменную через другую. Затем, зная весь путь, составим второе уравнение относительно времени и определим последнее. 38. Так как при возведении в квадрат некоторого числа должны получить двузначное число, то подумаем, каким числом должно быть число, воз- водимое в квадрат. Если это число х, то какое равенство (уравнение) можно записать на основании выполненного Петей действия? Записав это равенство, проанализировав его и учитывая, что х-число однозначное, можем опреде- лить, среди каких чисел надо искать двузначное число, равное удвоенному х. Число, обращенное одному из этих чисел, являющееся точным квадратом, и будет искомым. 39. Прежде всего подумаем над тем, каким числом будет сумма четырех нечетных чисел-четным или нечетным. А дальше рассуждаем так: если куб числа есть число четное, то каким числом будет возводимое в куб? В результа- те приходим к необходимости составления и рассмотрения нескольких урав- нений. Однако левая часть каждого из них будет одна и та же. Есть идея, но поищем лучший путь обозначения четырех последовательных нечетных чисел. 40. Идея решения задачи такая же, как и задачи 39. Будем внимательны при отборе возможных однозначных чисел и решении уравнения. 41. Понятно, что для того, чтобы складывать шестизначные числа, надо их иметь, т.е. надо их записать. Подумаем^над тем, как их записать, если на третьей карточке было записано число ab. Одно из возможных шестизначных чисел запишется, например, так: 1875aZ>. Записав остальные (а сколько их всего можно записать?), сложим все эти числа и преобразуем слагаемые_так, чтобы выделилось неизвестное. Например, 1875aZ> = 180000 -I- 7500 -I- ab. Зная из условия задачи сумму, мож- но составить уравнение. 42. В отличие от предыдущих задач для решения этой придется составлять неравенства, положив стоимость книги равной х к. Чтобы не ошибиться в записи неравенства, уясним смысл выражения «не больше 11 р.». 43. Для составления неравенства рассуждаем так: обозначим возраст одного из персонажей, а возраст других выразим с помощью введенной переменной; после этого выразим возраст каждого из них, который был три года тому назад. (Не ошибиться бы-Олегу больше, чем 3 года.) 44. Есть идея составления неравенства, но для ее реализации надо найти лучший путь выбора неизвестного для обозначения и выражения с его помощью других неизвестных. Главное здесь-подумать над тем, какова зависимость между числом рядов в зале и числом учащихся, сидящих в одном ряду. 45. Поскольку в условии задачи сказано, что «убирают урожая столько же», то придется думать о составлении уравнения, из которого делать потом 91
заключение, какая величина больше. Для этого положим, что один комбайн за день убирает х га, а другой у га. 46. Как видим из условия задачи, число всех учащихся школы не больше, чем 520, а участвующих в экскурсии в музей больше, чем 230. Значит, если бы выразить каждую из этих величин с помощью одной и той же переменной, то можно было бы составить два неравенства относительно этой переменной. Подумаем, какую же величину целесообразно принять за х. (От этого зависит успех решения задачи.) 47. Если нам удастся перевести условие задачи на «язык» символики, то получим неравенства и равенство. В целях удобства для обозначения коли- чества металлолома, собранного каждым классом, можно ввести те же буквы, которыми обозначены классы, т.е. а, б, в, г. На основании записанных двух неравенств и одного равенства устанавливаем соотношения между а, б, в и г. 48. На основании данных условия задачи рассуждаем так: число монет по 2 к., по 5 к. и по 10 к. можно выразить с помощью одной и той же переменной; тогда общее число этих монет будет меньше, чем 27, так как были еще монеты по 3 к. Принимая во внимание, что монет по 3 к. было больше, чем 7, и меньше, чем 20, преобразуем неравенство. Из полученных неравенств 1х < 20 и 7х > 7 делаем вывод о значении х. (Между числами 7 и 20 имеется только одно число, кратное 7.) 49. Приняв те же обозначения, что и в задаче 47, запишем неравенство и равенство по условию задачи. Подумаем, какими должны быть числа, чтобы их произведение было меньше суммы. Для этого проверим неравенство для наименьших, но различных натуральных чисел. Затем, заменяя большие из этих чисел на меньшие, будем пробовать получить нужное неравенство. На основании испытаний и наблюдений устанавливаем, какими должны быть три числа из четырех. 50. Нетрудно заметить, что число рабочих каждой фабрики можно вы- разить с помощью одной и той же переменной. Тогда можно выразить и число рабочих, выполнивших план на 100%, и составить неравенства, которые покажут, в каких пределах надо искать значение переменной. Поскольку оно должно быть натуральным числом, кратным некоторым числам (каким?), то методом проб его можно найти. 51. При решении задач нужна и догадка. Но догадаться может только ищущий и думающий. Нельзя ли для решения данной задачи использовать очевидное равенство 12 3 4 98 99 1 9 2’з’4 5 ’ ’*99 100 “100* , 2 14 3 Если заменять некоторые дроби на меньшие, т.е. - на - на - и т.д., то как изменится произведение в левой части? Запишите левую часть и упростите полученное неравенство. Вот и все доказательство.
ЗАДАЧИ С ГЕОМЕТРИЧЕСКИМ СОДЕРЖАНИЕМ Предметы окружающего нас мира обладают определенными свойствами, изучением которых занимаются ' различные науки. Математика исследует числовые отношения и пространственные формы. Геометрия, как вы уже знаете,-это раздел математики. Она рассматривает различные фигуры и их свойства. Первоначальные геометрические представления человек получил на заре развития человеческого общества. Складывались они постепенно, на про- тяжении тысячелетий, под воздействием практической деятельности. Еще первобытный человек, создавая орнаменты и расписывая глиняную посуду, использовал простые геометрические формы. С геометрическими формами имели дело и первые ремесленники, и земледельцы: кожевнику надо было правильно разметить и разрезать кожу, портному - полотно на куски, кузнецу выковать металлическое изделие определенной формы и размера, земледельцу разделить землю на участки и измерить их, а также изготовить для меры и хранения собранного урожая посуду разных форм и объемов, построить хранилища. Древние строители искусно решали нелегкие задачи геометрии. Особо интенсивное развитие строительное и землемерное дело получили в Древнем Египте, где возводили величественные пирамиды-усыпальницы для фараонов-и где надо было после каждого разлива реки Нил восста- навливать границы размеренных участков ценной плодородной земли. Унаследовав знания египтян о землеизмерении, греки назвали эту науку геометрией (от слов «гео»-земля и «метрио»-измеряю). Но, получив эти разрозненные сведения от египтян и других народов, греки не остановились на этом, а пошли дальше, заложив основы построения стройной научной системы о фигурах и их свойствах. 93
У входа в Академию, основанную философом древности Платоном, была такая надпись: «Не знающий геометрии да не войдет в Академию». Наибольший вклад в развитие геометрии внесли древнегреческие ученые Фалес, Пифагор (VII-VI вв. до н. э.) и особенно уже упоминавшиеся нами Евклид (III в. до н. э.) и Архимед (287-212 гг. до н. э.). В одном из древних описаний рассказывается о том, что царь Птолемей однажды спросил Евклида, нет ли в геометрии более краткого и легкого пути, чем его книги; на это тот ответил, что в геометрии нет царских дорог. Геометрия в своем развитии далеко шагнула за пределы землемерия, став важным разделом математики. Задачи настоящего параграфа окажут вам некоторую помощь в овладении первоначальными знаниями о геометрических фигурах и их свойствах. Как правило, они нестандартные и требуют поисков способов решений. ЗАДАЧИ 1. Имеется 6 палочек длиной по 1 см, 3 палочки - по 2 см, 6 палочек-по 3 см, 5 палочек-по 4 см. Можно ли из этого набора палочек составить квадрат, используя все палочки, не ломая их и не накладывая одна на другую? 2. Найдите длину стороны квадрата, площадь которого численно равна его периметру. 3. Существует ли такой 1989-угольник, чтобы можно было пересечь пря- мой все его стороны в точках, отличных от вершин? 4. На одной из боковых сторон треугольника взято 60 точек, а на другой-50. (Эти точки отличны от вершин треугольника.) Каждая из вершин при основании соединена отрезками прямых с точками, взятыми на противо- положной стороне. Сколько точек пересечения этих отрезков при этом образовалось? 5. Проведите 4 прямые так, чтобы тетрадный лист бумаги (или классная доска) разделился на наибольшее число частей. Сколько получено частей? 6. Часто знает и дошкольник, Что такое треугольник, ^^—^**^1 А уж вам-то как не знать, ___/ 1 Но совсем другое дело- / I Быстро, точно и умело X.I Треугольники считать. X. /_ -4 Например, в фигуре этой X^/V /\ Сколько разных? Рассмотри! \ / \ Все внимательно исследуй / X. \ / I И по краю, и внутри! (Рис. 3.) / \\ / I 7. Сколько всего треугольников изоб- И X/ I ражено на рисунке 4? Рис. 3 8. Из фигуры, составленной из 12 одинаковых палочек, как показано на рисунке 5, исключите 2 палочки так, чтобы получилось два квадрата. 9. Известно, что треугольник разрезали на 2 части и из них составили прямоугольник. Какого вида мог быть треугольник? 94
Рис. 4 Рис. 5 10. Помогите огороднику найти в огороде, имеющем форму четырех- угольника, такую точку, чтобы сумма расстояний от этой точки до вершин четырехугольника была наименьшей. 11. Пришкольный участок прямоугольной формы имеет периметр 160 м. Как изменится его площадь, если длину каждой стороны увеличить на Юм? 12. Деревянный куб, длина ребра которого 5 см, окрашен, а затем разрезан на кубы, ребра которых равны 1 см. Сколько получите^ кубов с тремя, двумя, одной окрашенной гранью? 13. Куб, длина ребра которого 0,5 м, разрезан на кубы, ребро каждого из которых равно 2 мм. Полученные кубы выложили в один сплошной ряд. Чему равна длина этого ряда? 14. Куб, ребро которого равно п, разрезали на кубы, ребро каждого из которых равно 1. Общий объем полученных кубов, очевидно, останется таким же. а вот поверхность, без сомнения, увеличится. Во сколько же раз? 15. Сколько существует треугольников, вершинами которых могут быть только вершины и точка пересечения диагоналей данного прямоугольника, не являющегося квадратом, причем все эти треугольники имеют общую вершину в заданной вершине прямоугольника (эта вершина фиксируется)? Сколько из этих треугольников прямоугольных? Как изменится задача, если вместо пересечения диагоналей взять любую внутреннюю точку прямоугольника? 16. Сложите квадрат из четырех квадратных плиток размером 1x1, восьми-размером 2x2, двенадцати-размером 3x3 и шестнадцати-раз- мером 4x4. 17. Фигуру, изображенную на рисунке 6, разрежьте на две части и составь- те из них квадрат. 18. Участок квадратной формы расши- рили так, что получили новый квадрат, _________________ сторона которого на 5 м больше стороны “ первоначального, а площадь при этом уве-____________________ дичилась на 225 м2. Какова площадь перво- начального участка? --------------------—-г— 19. Найдите все треугольники, длины сторон которых целые числа сантиметров---------------------------- и длина каждой из них не превышает 2 см. 20. Длина одной стороны треугольника 6,31 м, а другой-0,82 м. Чему может рав- I I I I I I I I няться длина третьей стороны, если она выражается целым числом метров? Р"0- 6 95
Рис. 7 21. Площадь фигуры, составленной из 14 одинаковых палочек, в 3 раза больше площади фигуры, составленной из 6 таких же палочек (рис. 7). Не трогая 12 палочек, переложите остальные так, чтобы получилось две фигуры, составленные из 7 и 13 палочек с таким же отношением площадей. 22. Деревянный куб, длина ребра которого 1 дм, надо оклеить цветной бумагой. Хватит ли для этой цели квадратного листа бумаги со стороной 2,5 дм? Как надо разрезать для этой цели данный лист? 23. Сколько всего спичек может быть получено из деревянного куба, ребро которого 1 м, если каждая спичка должна иметь длину 5 см, ширину 2 мм, высоту 2 мм? (Считать, что материал в отход на распиловку не идет.) 24. Какое взаимное расположение точек А, В. С на плоскости, если известно, что для любой точки М этой плоскости расстояние AM меньше хотя бы одного из расстояний ВМ и СМ? 25. Разведчик находится в доме с четырьмя окнами, расположенными в форме прямоугольника. Он должен сигнализировать ночью в море, зажигая свет в окне или в нескольких окнах. Сколько различных сигналов он может подать? 26. Сколько сторон может иметь фигура, являющаяся общей частью треугольника и выпуклого четырехугольника? 27. Можно ли 1989 палочек одинаковой длины, не накладывая одна на другую, выложить так, чтобы получить замкнутую ломаную линию, если палочки располагать только в горизонтальном и вертикальном положениях? 28. Можно выложить 15 шариков в виде треугольника, но нельзя их 96
о о о ООО о о о о о о о о о о о о о о о о о о о о о ООО Рис. 8 выложить в виде квадрата (одного шарика не хватает) (рис. 8). Из какого количества шариков, не превышающего 50, можно выложить как треугольник, так и квадрат? 29. Постройте ломаную линию, которая пересекает каждое свое звено 2 раза. (Все пересечения считать только во внутренних точках звеньев ломаной, а не в вершинах.) 30. Какое наибольшее число точек самопересечения может иметь замкну- тая ломаная линия, состоящая из 7 звеньев? (Общие концы звеньев ломаной не считаются.) 31. Площадка для детских игр прямоугольной формы должна удовлетво- рять таким требованиям: длины сторон должны выражаться целыми числами метров, а площадь численно равняться периметру. Какой длины и ширины может быть такая площадка? 32. Цистерна, имеющая форму прямоугольного параллелепипеда, облада- ет тем свойством, что если поставить ее на любую боковую стенку, то площадь полученного дна не уменьшится по сравнению с данной. Каков наибольший возможный объем этой цистерны, если ее высота 1,5 м? 33. Кусок мыла, лежащий на умывальнике, имеет форму прямоугольного параллелепипеда. Мыло расходуют равномерно каждый день одно и то же количество. Спустя 7 дней размеры мыла уменьшились вдвое. На сколько дней хватит этого мыла, если им будут пользоваться так же интенсивно? 34. За какое время конец минутной стрелки часов опишет дугу, равную длине этой стрелки? 35. В данную окружность, как это показано на рисунке 9, вписаны внешне касающиеся между собой окружности, центры которых расположены на одном диаметре данной окружности. Покажите, что сумма длин вписанных окружностей не зависит от их количества. 36. В большем круге расположены 4 меньших с диаметром вдвое мень- шим, чем диаметр круга (рис. 10). Сравните площадь большего круга и сумму площадей четырех малых и сделайте вывод. 37. На сторонах квадрата как на диаметрах построены полукруги (рис. 11). Определите площадь заштрихованной фигуры (четырех лепестков), если сто- рона квадрата равна а. 38. На квадратном листе бумаги, длина стороны которого равна 3 дм, нарисованы окружности, длина диаметра каждой из которых 5 мм. Известно, 4 Зак 2531 Д В Клименченко 97
Рис. 10 Рис. 9 Рис. II что нельзя соединить никакую точку одной из сторон квадрата ни с какой точкой противоположной стороны отрезком прямой так, чтобы этот отрезок не пересекал хотя бы одной из окружностей. Покажите, что окружностей нарисовано не меньше, чем 60. 39. Покажите, что в любом выпуклом двенадцатиугольнике найдется две диагонали, угол между которыми не больше, чем 3°. 40. Длины ребер прямоугольного параллелепипеда выражаются натураль- ными числами, а его объем - простым числом, большим, чем 2. Покажите, что сумма длин ребер, исходящих из одной вершины,-число нечетное. 41. Разрежьте квадрат на 3 части так, чтобы из них можно было сложить тупоугольный треугольник. 42. Из бумаги склеили куб. Ясно, что его поверхность можно разрезать на 6 равных квадратов. А можно ли ее разрезать на 12 равных квадратов?
1. Мы уже-имеем опыт и знаем, что решение задачи наугад к успеху не приводит, поэтому не будем пытаться составлять квадрат из имеющихся палочек, а выясним прежде всего, выражается ли в данном случае длина стороны квадрата целым числом сантиметров. 2. Задачу будем решать в общем виде, предположив, что такой квадрат имеется и длина его стороны а. Записав равенство, определим а. 3. Вы, очевидно, уже пришли к убеждению, что решить задачу методом проб и испытаний, пытаясь выполнить чертеж 1989-угольника, не удастся, поэтому проведем исследование в общем виде. Подумаем над тем, как распределяются и располагаются вершины многоугольника относительно прямой, пересекающей все стороны этого многоугольника. А отсюда: удовле- творяет ли этому требованию данное число вершин? 4. Вряд ли кто из вас станет выполнять чертеж по условию задачи и непосредственно подсчитывать число точек. Чтобы открыть закономерность и установить правило подсчета точек, можно рассмотреть чертеж, но, взяв, например, 3 и 5 точек на сторонах. 5. Говорить о тетрадном листе или классной доске-это говорить об известной вам фигуре. Вот и начертим ее и подумаем, как провести прямые, чтобы фигура разделилась на наибольшее число частей. 6- 7. При подсчете треугольников надо учитывать как те, которые не состоят из других треугольников, так и те, которые составлены из других. Чтобы не ошибиться в подсчете, пронумеруем отдельные (маленькие) тре- угольники, а составленные из других треугольников (частей) можно, на- пример, записывать так: (1, 3, 4) или (1 + 3 + 4). 8. Если кто думает, что надо получить обязательно квадраты одинаковые, не имеющие общей части, то он неправомерно усиливает требование задачи. Вы обратили внимание, как подсчитывались треугольники при решении задач 6, 7? Вот так и квадраты будем считать. 9. Рассуждаем так: 1) углы прямоугольника прямые, поэтому попро- буем испытать для получения прямоугольника прямоугольный треугольник; 2) проведя к основанию треугольника из его вершины отрезок, можем получить два прямых угла. Подумаем, каким должен быть треугольник, чтобы из полученных частей можно было составить прямоугольник. 10. Из всех внутренних точек четырехугольника одна занимает «особое положение». Это точка, полученная при пересечении отрезков, соединяющих противоположные вершины четырехугольника (точка пересечения диагона- лей). Ее-то и можно выделить из всех точек и проверить, не она ли искомая. 4* 99
11. Нет нужды находить ^первоначальную площадь и площадь после увеличения участка, ведь требуется найти только, па сколько увеличился площадь. Для облегчения поисков решения надо «увидеть» это увеличение площади участка. В этих целях выполним рисунок участка и выделим те фигуры, площадь которых надо найти. (А возможно и не площади каждой из этих фигур, а сумму их площадей.) Обозначив длину одной стороны первоначального участка, можно выра- зить длину другой стороны, а значит, можно выразить и размеры тех частей, площади которых надо найти. 12. Если затрудняетесь мысленно представить куб и ею разрезание на части, то можно призвать на помощь рисунок и при помощи его намел и гь стратегию и тактику решения, выявив, какие кубы имеют по 3 окрашенные грани, какие-по 2 и по 1. 13. Не производя вычислений, оцените примерно длину получаемого ряда, а затем, определив число полученных кубов, найдите длину ряда и сравните полученный ответ с предсказанным вами. Ну как, расхождение большое? 14. Чтобы ответить на вопрос задачи, надо знать поверхность данного куба и сумму поверхностей полученных кубов. Для определения же поверх- ности куба необходимо найти площадь одной его грани (квадрата), число же всех граней известно. Для определения числа полученных кубов в результате разрезания данного куба воспользуемся решением задачи 13. 15. Для подсчета числа треугольников воспользуемся рисунком. Если вместо точки пересечения диагоналей взять иную внутреннюю точку прямо- угольника, то надо рассмотреть 3 случая: 1) точка принадлежи г диаг опали, не проходящей через общую вершину треугольников; 2) точка принадлежит диагонали, проходящей через общую вершину треугольников; 3) точка не принадлежит ни одной диагонали. 16. Как начинать «укладывать» плитки? Конечно же, не наугад. Надо знать длину стороны квадрата, который надо получить, но это можно узнать, зная его площадь. Площадь же можно найти, так как знаем, из каких плиток надо сложить квадрат. 17. Для отыскания способа решения надо четко видеть цель: какой квадрат должны получить? (Из скольких квадратиков он состоит?) Зная, сколько квадратиков должно быть расположено по стороне искомого квадрата, поищем ломаную, по которой надо разрезать данную фигуру. 18. Как видим, надо переложить одну палочку из фигуры, состоящей из 14 палочек, в фигуру, состоящую из 6 палочек. Значит, вместо прямоугольника, составленного из 6 палочек, получим прямоугольник с «надстроенным» треугольником. Вторую фигуру надо преобразовать так, чтобы она состояла из трех таких фигур, как меньшая. (В целях поисков решений задачи надо конструировать фигуры из палочек.) 19. Поскольку задача вычислительного характера, то решим ее или ариф- метическим способом, или с помощью уравнения. Есть идея, будем ее реализовывать. Найти надо площадь, однако для обозначения выберем другую величину. Какую? Выразив через нее величину, на которую увеличи- лась площадь, составим уравнение. (В этом поможет решение задачи 11.) 20. Из условия задачи видим, что длины сторон треугольников могут выражаться лишь числами 1 см и 2 см. Составляем возможные комбинации юо
троек чисел при условии, что сумма двух из них больше третьего. 21. Третья сторона треугольника определяется из условия: длина ее мень- ше суммы длин двух других сторон, но больше их разности. Если искомая сторона а, то, составив неравенства, найдем целые значения а. 22. Поиски ответа на первый вопрос затруднений не вызывают: достаточно сравнить площади листа бумаги и поверхности куба. Особо придется поду- мать в поисках разметки листа бумаги и выбрать лучший вариант. 23. Число спичек можем найти, если будем знать объем куба, лз которого получают спички, и объем одной спички. При этом мы будем считать, что весь материал на 100% идет в дело. На практике дело обстоит немного иначе, так как часть материала идет на отходы при распиловке. 24. Во-первых, исследуем, могут ли точки А, В и С не лежать на одной прямой. Найдется ли в этом случае хотя бы одна точка, одинаково удаленная от точек Л, В и С? Во-вторых, какая из трех данных точек должна быть расположена между двумя другими? 25. Для поисков решений получения и подсчета всевозможных конфи- гураций изобразим схематически окна точками и пронумеруем их. Рассмот- рим и исследуем случаи, когда имеем одну точку, две, три и четыре точки. 26. Для исследования и выявления всех возможных случаев надо обра- титься к рисункам-иллюстрациям. Путем проб поищем все решения. 27. В поисках решения неразумно было бы пытаться выкладывать 1989 палочек, даже заменяя палочки отрезками, вычерчивая их на бумаге. А вот обратиться к примерам с использованием значительно меньшего числа па- лочек и рассмотреть случаи, когда число их нечетное и четное, вполне возможно. Это наведет на определенные мысли. 28. Подумаем, как решаются такие задачи: 1) Какие числа надо сумми- ровать, чтобы определить число шариков, из которых составлен треугольник? 2) Как определить число шариков, из которых составлен квадрат? Затем из чисел, не превышающих 50 и являющихся точными квадратами, отобрать методом проб нужное, т. е. такое, которое является суммой последовательных чисел натурального ряда начиная с единицы. 29. Начнем поиски решений с рассмотрения ломаной, состоящей из двух отрезков. Если провести третий отрезок, пересекающий первые два, то на нем окажется две точки пересечения, а на первых двух по одной. Но эти три отрезка ломаную не образуют. Проведем четвертый отрезок так, чтобы вместе с данными тремя он образовал ломаную. Продолжая этот процесс, получим искомую ломаную. 30. Используя идеи решения задачи 29, начав построение ломаной с двух звеньев, на каждом следующем шагу будем присоединять по одному звену (отрезку) ломаной так, чтобы каждый раз получить наибольшее число точек пересечения. Аккуратно выполненный рисунок поможет получить решение задачи. Для удобства получаемые точки пересечения будем нумеровать. 31. Подобную задачу для равностороннего прямоугольника (квадрата) вы уже рассмотрели (задача 2). Взяв размеры прямоугольника, например а и Ь, и составив равенство, исследуем, при каких целых числах а и b оно будет верным. (Идея имеется, надо поискать лучшие пути исследования равенства.) 32. Цистерна будет иметь наибольший объем, если площадь ее дна наибольшая из всех возможных. Площадь же прямоугольника наибольшей будет при условии, если длина и ширина его принимают наибольшие возмож- 101
ные значения. Для отыскания этого условия обозначаем размеры дна, состав- ляем и решаем неравенства. 33. Доверившись интуиции, как правило, отвечают: «На 7 дней». Не искушает ли и вас такой ответ? На ряде примеров вы уже убедились, что для выяснения истины необходимо математическое доказательство. Не случайно ведь К. Маркс говорил, что наука только тогда достигает совершенства, когда ей удается пользоваться математикой. Вот и давайте пользоваться математи- кой при решении пусть даже такой простой задачи. Если размеры мыла были а, Ь, с, то можем выразить его объем. Выразив объем оставшейся части куска при условии, что размеры уменьшились вдвое, можно определить, сколько (какой объем) смылено за 7 дней, а затем и за 1 день, и получаем ответ, который на первый взгляд кажется неправдо- подобным. Но это только так кажется. 34. Зная, какую дугу опишет конец стрелки за 1 ч (полную окружность), нетрудно найти, за какое время она опишет дугу длиною R (R- длина стрелки). 35. Нетрудно видеть, что для доказательства достаточно сравнить длину охватывающей окружности, выраженную через диаметр, и сумму длин вписан- ных окружностей, также выраженных через их диаметры. 36. Для сравнения указанных площадей достаточно их выразить через одну и ту же величину. Например, через радиус R большей окружности. 37. Площадь заштрихованной фигуры будем искать из сравнения площади квадрата и суммы площадей четырех полукругов. (Исследуем, как полукруги «накрывают» квадрат.) 38. Поищем такой вариант расположения окружностей, чтобы при наи- меньшем их количестве выполнялось условие задачи. Как при этом рас- положены центры окружностей? Зная длину стороны квадрата и диаметр окружности, не представляет труда найти число окружностей. 39. Если найдется две диагонали параллельные, то угол между ними 0° < 3°. Исследуем случай, когда нет ни одной пары параллельных диагоналей. Для этого, очевидно, надо определить число всех диагоналей. Сколько же диагоналей можно провести из каждой вершины? А сколько всего диагоналей? Подумаем, как «свести» все углы между диагоналями к общей вершине. Для этого используем то свойство, что, перенося диагонали параллельно себе, получим угол между ними тот же, что и был. Если провести через какую- нибудь точку прямые, параллельные диагоналям, то получим все углы между диагоналями. На сколько углов при этом разобьется плоскость? Могут ли все углы быть большими по величине, чем 3°? 40. Поскольку произведение трех натуральных чисел (объем параллелепи- педа) есть число простое, то подумаем, во-первых, может ли это число быть четным. Во-вторых, какими числами должны быть два множителя из трех? Может ли быть четным третий множитель? Выяснив все эти вопросы, устанавливаем вид числа-суммы длин ребер, исходящих из одной вершины. 41. Если первый разрез произведен по диагонали квадрата, то подумаем, как выполнить второй разрез. 42. В поисках решения задачи поможет такая задача: разрезать квадрат на 4 части так, чтобы из полученных частей можно было составить 2 одинаковых квадрата. Если такая задача решена, то, проведя диагонали граней куба, увидим, как надо разрезать эти грани, чтобы, получить 12 квадратов.
ЛОГИЧЕСКИЕ ЗАДАЧИ Вам, очевидно, приходилось слышать такие выражения: «В его рассужде- нии нет логики», «Он не умеет логически мыслить». Что бы это значило? А это означает, что человек не владеет правилами науки о законах мышления, называемой логикой. Другими словами, он не умеет мыслить последователь- но, связно, доказательно, т.е. мыслить логически. Воспитанию логического мышления в значительной степени способствуют занятия математикой. Это имел в виду М. В. Ломоносов (1711-1765), говоря: «А математику уже затем учить следует, что она ум в порядок приводит». Прочитав название этого параграфа, не подумайте, что во всех предыду- щих параграфах были задачи нелогические и вот только теперь появились логические. Вообще нелогических задач нет, так как каждой задаче присущи последо- вательность, взаимосвязь фактов, аргументированность, и поэтому при ре- шении ее последовательно переходят от одного суждения к другому. Мы же к логическим задачам отнесли те, при решении которых главное, определяющее-это отыскание связей между фактами (часто скрытых), со- поставление их, установление для достижения поставленной цели цепочки суждений, а вот вычисления, построения играют здесь как бы вспомога- тельную роль (задачи предыдущих параграфов как бы «обслуживают» тот или иной вопрос математической теории). Немало задач вообще без числовых данных. Часто в условии логической задачи имеется такое обилие фактов, что удержать их все в памяти нелегко. Тогда прибегают к составлению схем, таблиц, выполнению рисунков и чертежей. В копилке народной мудрости немало логических задач, передававшихся из поколения в поколение. Вам известна, вероятно, задача о перевозе через реку юз
капусты, волка и козы. Немало подобных задач (ситуаций затруднения) отражено в сказках и легендах. Работая над задачами настоящего параграфа, вы будете развивать и со- вершенствовать свое логическое мышление. ЗАДАЧИ 1. Шел Кондрат в Ленинград, А навстречу-двенадцать ребят, У каждого-по три лукошка, В каждом лукошке-кошка, У каждой кошки -12 котят, У каждого котенка в зубах по 4 мышонка. И задумался старый Кондрат: Сколько мышат и котят Ребята несут в Ленинград? Помогите Кондрату найти ответ. 2. Встретились два друга детства, не видевшиеся со школьных лет и ничего не знавшие друг о друге. Между ними состоялся разговор: - Сколько лет я тебя не видел и не получал никаких вестей! - А у меня уже дочь. - Как же ее зовут? - Да так же, как и ее мать. - А сколько лет твоей Ниночке? Как один из собеседников узнал имя дочери другого? 3. В состав космической экспедиции на должность командира корабля, бортинженера и радиста назначены Гришин, Романов и Павлов. При перво- начальном обсуждении этих кандидатур высказывались гакие предположения: командиром корабля будет Гришин, Романов командиром быть не может, Павлов не будет бортинженером. При окончательном решении выяснилось, что только одно из этих предположений оказалось верным. Распределите указанных членов экипажа по должности. 4. В трех коробках имеются цветные карандаши: в одной-только красные, в другой-только зеленые, в третьей-красные и зеленые. На каждой коробке нарисованы карандаши: на одной-красный, на другой - зеленый, на третьей- красный и зеленый. Известно, что цвет карандашей, .находящихся в каждой коробке, не соответствует рисунку на ней. Как, взяв только один карандаш из одной коробки, поменять на них рисунки в соответствии с цветом карандашей, находящихся в них? 5. Кросс осенний вспоминая, Спорят белки два часа: - Победил в забеге заяц, А второй была лиса! - Нет,-твердит другая белка,- Ты мне эти шутки брось, Заяц был вторым, конечно, Первым был, я помню,-лось! 104
- Я,-промолвил филин важный,- В спор чужой не стану лезть, Но у вас в словах у каждой По одной ошибке есть! Белки фыркнули сердито, Неприятно стало им, Вы же, взвесив все, найдите, Кто был первым, кто вторым. 6. В очереди за мороженым стоят Юра, Ира, Оля, Саша и Коля. Юра стоит раньше Иры, но после Коли. Оля и Коля не стоят рядом, а Саша не находится рядом йи с Колей, ни с Юрой, ни с Олей. В каком порядке стоят, ребята? 7. Трем неутомимым путешественникам надо было переправиться на лодке, выдерживающей массу не более 100 кг, с одного берега реки на противоположный. Андрей знал результат своего недавнего взвешивания - 54 кг и своего друга Олега-46 кг. Зато дядя Миша имел массу 98 кг. Как им надо действовать наиболее рациональным образом, чтобы переправиться через реку? 8. Машина идет со скоростью 60 км/ч. На сколько надо увеличить скорость, чтобы выиграть на каждом километре по одной минуте? 9. Времянкин и Путейкин одновременно вышли из Утреннего в Вечернее. Первый из них половину времени, затраченного на переход, шел со скоростью 5 км/ч, а затем-со скоростью 4 км/ч. Второй же первую половину пути проходил в час по 4 км/ч, а затем шел со скоростью 5 км/ч. Кто из них прибыл раньше в Вечернее? 10. Работая в колхозном саду на уборке фруктов, школьники собрали 22 ящика, в одних из которых-яблоки, в других-груши и в третьих-сливы. Можно ли утверждать, что имеется по крайней мере 8 ящиков, содержимое которых-один из указанных видов фруктов? 11. В школе 33 .класса, 1150 учеников. Найдется ли в этой школе такой класс, в котором не менее 35 учеников? 12. Когда Светлана заявила, что у нее имеется 25 медных монет, то старшая ее сестра Татьяна сказала, что среди этих монет не меньше 7 одина- кового достоинства. Права ли Татьяна? 13. В темной кладовой лежат ботинки одного размера: 12 пар черных и столько же коричневых. Какое наименьшее число ботинок надо взять, чтобы среди них оказалась хотя бы одна пара (левый и правый ботинок) одного цвета, если в темноте нельзя отличить не только цвет ботинка, но и левый от правого? 14. В 7 ящиках имеется всего 100 деталей. В любых двух из этих ящиков находится различное число деталей. Обязательно ли найдутся среди них 3 ящика, в которых не меньше 50 деталей? 15. В каждом из 376 ящиков лежит не более 125 деталей. Покажите, что по крайней мере в 4 ящиках лежит по одинаковому числу деталей. 16. В коробке лежит 120 цветных карандашей: 35 красных, 23 зеленых, 14 желтых, 26 синих, 11 коричневых и 11 черных. Какое наименьшее число карандашей надо взять из. коробки в темноте (не видя карандашей), чтобы среди них определенно оказалось не менее 18 карандашей одного цвета? 105
17. В фотолаборатории лежит 130 фотографии, из которых 35-четвертого класса, 30-пятого, 25-шестого, 20-седьмого, а остальные-1-Ш классов. Фотографии перемешаны и на ощупь их различить нельзя. Какое наименьшее число фотографий надо взять, не видя их, чтобы среди них было не меньше 15 одного класса из состава IV—VII классов? 18. Автоинспектор заметил, что из 20 автомашин, которые прошли по дороге в аэропорт, 14 были «Жигули», 15-темного цвета, за рулем 17 машин были мужчины и в 18 машинах, кроме водителя, были пассажиры. Для какого наименьшего числа автомашин могли иметь место все 4 указанных признака? 19. На столе лежит 4 карточки: А Б 4 5 Известно, что на каждой карточке с одной стороны-буква, а с другой-нату- ральное число. Какие карточки достаточно перевернуть, чтобы выяснить, истинно или ложно такое предложение: «Если на одной стороне карточки гласная буква, то на другой - четное число»? 20. Школьники помогали колхозникам убирать овощи. Четвертый класс собрал 18 ц капусты и столько моркови, сколько лука собрал шестой класс. Пятый класс работал только на уборке огурцов и собрал их целое число центнеров, что меньше, чем собрано было огурцов шестым классом, но больше, чем собрал моркови четвертый класс. Шестой класс собрал 7 ц лука, а огурцов-в два раза меньше, чем собрано капусты четвертым классом. Сколько всего центнеров овощей собрали школьники? 21. В семье 4 детей, им 5, 8, 13 и 15 лет, а зовут их Таня, Юра, Света и Лена. Сколько лет каждому из них, если одна девочка ходит в детский сад, Таня старше, чем Юра, а сумма лет Тани и Светы делится на 3? 22. Распалась цепь на пять частей, Но я надеюсь, что легко ты Соединишь их поскорей, Проделав минимум работы. Разрезать и спаять звено - На это две минуты надо... Учти, задание дано Для очень вдумчивого взгляда. Примечание. В каждом звене три кольца. 23. Подойдя к реке, путешественники попросили владельца лодки пере- править их на противоположный берег. Так как к чужеземным деньгам лодочник питал недоверие, то путешественники предложили ему в качестве платы имевшуюся у них золотую цепочку, состоящую из 6 звеньев. Лодочник согласился, но с условием, что перевозить он будет всех путешественников по одному, так как лодка выдерживает только 2 человека, и плату надо производить за каждый рейс по одному звену цепочки, причем распилить можно не более одного звена. Путешественники, немного подумав, выполнили такое условие. Как они при этом поступили? 24. Жители города А говорят только правду, жители города Б-только ложь, жители города В-попеременно правду и ложь (т.е. из двух утвержде- на
ний, высказанных ими, одно истинно, а другое ложно). Дежурному по пожарной части по телефону сообщили: «У нас пожар, приезжайте скорее!» «Где?»-спросил дежурный. «В городе В»,-ответили ему. Куда должна выехать пожарная машина? 25. В одной книге было написано 100 следующих утверждений: 1) «В этой книге ровно одно неверное утверждение». 2) «В этой книге ровно два неверных утверждения ...» 100) «В этой книге ровно сто неверных утверждений». Какое из этих утверждений верное? 26. В бочке 28 л бензина. Имеется два ведра емкостью по 7 л, в которые нужно налить по 6 л бензина. Кроме того, есть черпак емкостью 4 л. Как можно осуществить разлив? 27. В бочке хранится несколько ведер бензина. Как из нее отлить 6 л бензина в другую бочку с помощью девятилитрового и пятилитрового бидонов? 28. Задача, которую решил в юности Пуассон (1781-1840). Некто имеет 12 пинт (мера емкости) меда и хочет отлить из этого количества половину, но у него нет сосуда вместимостью в 6 пинт. У него 2 сосуда: один-вмести- мостью в 8 пинт, а другой-вместимостью в 5 пинт. Каким образом налить 6 пинт меда в сосуд на 8 пинт? Какое наименьшее число переливаний необходимо при этом сделать? (Эта задача и определила жизненный путь Пуассона: математике он посвятил всю свою жизнь.) 29. Как при помощи чашечных весов и гири 200 г разделить 9 кг сахара на 2 пакета-2 кг и 7 кг, если разрешается взвешивать не больше трех раз? 30. Три пятницы некоторого месяца пришлись на четные даты. Какой день недели был 18 числа этого месяца? 31. В одном месяце три среды пришлись на четные числа. Какого числа в этом месяце будет второе воскресенье? 32. В каких случаях месяц имеет наибольшее число суббот? Чему равно это число? 33. Учащиеся нашего класса первое воскресенье месяца провели в Карпа- тах, а в первое воскресенье после первой субботы этого месяца были на экскурсии в Киеве. В следующем месяце в первое воскресенье они совершили экскурсию в лес, а в первое воскресенье после первой субботы посетили музей. Какого числа и какого месяца учащиеся были в Карпатах, Киеве, в лесу и в музее? 34. Числа записаны в таблице 6 7 4 6 3 31 28 или 29 31 30 31 в определенной закономерности. Установите ее и впишите в свободные клетки нужные числа. 107
35. Продолжите такой ряд: О. д. т. Ч, П, Ш. ... 36. Акробат и собачонка Весят два пустых бочонка. Шустрый пес без акробата Весит два мотка шпагата. А с одним мотком ягненок Весит - видите - бочонок. Сколько весит акробат В пересчете на ягнят? 37. Футбольные команды проводили турнир в один круг. Один из поклон- ников этой игры, вернувшись из отпуска, застал такую картину: общее число очков, набранных всеми командами, 44; одна команда, набравшая наименьшее число очков, получила одно очко; две команды, занимающие первые два места, набрали одинаковое количество очков. Сколько команд участвует в турнире и сколько игр им еще предстоит провести? 38. В турнире, где каждые 2 команды встречались между собой по 2 раза, участвовало 4 команды. За победу в каждой встрече давалось два очка, за ничью-одно, за поражение-0. Команда, занявшая последнее место, набрала 5 очков. Сколько очков набрала команда, занявшая первое место? 39. В шахматном турнире каждый из 8 участников играет с каждым по одной партии. Все участники набрали разное, количество очков (целое число), причем второй призер набрал столько же очков, сколько все вместе шахма- тисты, занявшие с пятого по восьмое место. Как сыграли между собой шахматисты, занявшие третье и пятое места? 40. В футбольном турнире команда «Торпедо», занявшая первое место, набрала 5 очков, отношение забитых и пропущенных мячей у нее 7:0. На втором месте команда «Азовец», забившая 2 мяча и пропустившая 3. Она набрала 4 очка. На третьем-«Энергия», у которой 2 очка, забито 2 и про- пущено 6 мячей. Замыкал таблицу «Строитель», имевший одно очко, за- бивший 1 и пропустивший 3 мяча. Все команды сыграли между собою по одному матчу. Определите результаты отдельных встреч, помня, что за победу команда получает 2 очка, за ничью-1, за поражение-0. 41. В таблице футбольного турнира четырех команд сохранились лишь некоторые записи: Восстановите таблицу: Команда «Старт» «Вымпел» «Авангард» -«Звезда» Очки Общий счет Место «Старт» X 1:1 :3 «Вымпел» X 1 :4 «Авангард» X 3:1 1 «Звезда» :5 1: X 3 :7 108
Примечание. За выигрыш команда получает 2 очка, при ничьей-1 очко, при поражении-0. При одинаковой сумме очков лучшее место определяет лучшее соотношение мячей. 42. Четыре футбольные команды «Старт», «Комета», «Ракета» и «Вымпел» провели каждая с каждой по одной игре. На изготовленной таблице после дождя остались только некоторые записи. Но судья помнил, что остальные матчи окончились со счетом 2:0, 1:1, 2:2, 3:1 и 5:3. Помогите судье восстановить таблицу. (В графе «Мячи» слева записывается общее количество забитых, справа - пропущенных мячей. За победу - 2 очка, за ничью-1, за поражение-0.) Команда «Старт» «Комета» «Ракета» «Вымпел» Победа Ничья Пораже- ние Мячи Очки «Старт» X 6 «Комета» X 2: «Ракета» X :8 «Вымпел» 0:1 X 43. Имеется 4 арбуза различной массы. Как, пользуясь чашечными весами без гирь, путем не более пяти взвешиваний расположить их по возрастанию массы? 44. Из четырех внешне одинаковых деталей одна отличается по массе от трех остальных, однако неизвестно, больше ее масса или меньше. Как выявить эту деталь двумя взвешиваниями на чашечных весах без гирь? 45. Можно ли ходом шахматного коня попасть из левой нижней клетки шахматной доски в правую верхнюю, побывав при этом на каждой клетке один, и только один раз? 46. Переложите кубики, сложенные пирамидой, так, чтобы форма пирами- ды осталась прежней, но каждый кубик соприкасался только с новыми кубиками. 109
' 47. Барсук позвал к себе гостей: Медведя, рысь и белку, И подарили барсуку Подсвечник и тарелку. Когда же он позвал к себе Рысь, белку, мышку, волка, То он в подарок получил Подсвечник и иголку. Им были вновь приглашены Волк, мышка и овечка, И получил в подарок он Иголку и колечко. Он снова пригласил овцу, Медведя, волка, белку, И подарили барсуку Колечко и тарелку. Нам срочно нужен ваш совет (На миг дела отбросьте): Хотим понять, какой предмет Каким дарился гостем. И кто из шестерых гостей Явился без подарка? Не можем мы сообразить, Сидим... Мудрим... Запарка! 48. При возвращении из Дворца спорта между Ниной и ее тренером состоялся такой диалог: - У тебя есть брат? - У меня три брата,-ответила Нина. - А какого они возраста? - Произведение чисел, выражающих возраст каждого из них, равно 36, а сумма-номеру вон того автобуса, что стоит на остановке. Посмотрев на номер автобуса, тренер сказал, что этих данных ему недостаточно. - А старший из них тоже любит спорт,-добавила Нина. - Теперь я знаю, сколько лет каждому из них,-сказал тренер и точно назвал возраст каждого мальчика. Назовите и вы. Поиски решений 1. Встречаются и такие случаи, когда при решении подобных задач ищут способы вычисления 12 • 3 • 12 • 4, не прочитав внимательно первые две строки текста условия задачи. Оцените ситуацию, кто куда шел. 2. При решении задачи надо преодолеть инертность мышления: будто и второй собеседник обязательно тоже мужчина. 3. Если сразу не удалось решить задачу, анализируя высказывания, то можно использовать метод проб. 4. Надо найти правильный первый ход. Если взять карандаш из той коробки, для которой нет гарантии, что там карандаши одного цвета, то на основании цвета взятого карандаша нет основания утверждать, что в ней карандаши только этого цвета. Отсюда ясно, из какой коробки надо взять карандаш. ПО
5. Если допустить, что заяц был первым или вторым, то оба высказывания второй или первой белки ложные. Это дает «ключ» к решению задачи. 6. В поисках решения целесообразно воспользоваться схемой записи за- главных букв имен ребят в соответствии с условием задачи. 7. Поскольку нельзя сразу переправиться троим, то надо подумать о том, как переправляться по одному или по два так, чтобы можно было доставить лодку с противоположного берега. 8. Чтобы видеть, на сколько возможна экономия во времени на одном километре, надо узнать, за сколько минут машина проезжает 1 км. Можно ли выиграть минуту? 9. Для решения задачи достаточно сравнить, кто из них шел большую часть пути с большей скоростью-5 км/ч. Ясно, что Путейкин с такой скоростью шел половину пути. А Времянкин? 10. Задача на применение так называемого принципа Дирихле. (Если предметы раскладывать в ящики, которых меньше, чем предметов, то найдет- ся хотя бы один ящик, в котором больше, чем один предмет.) Надо рассматривать самый «неблагоприятный» случай с точки зрения рассматри- ваемой задачи, т.е. пусть имеется по 7 ящиков каждого вида. 11. Для решения используется та же идея, что и в задаче 10. Если предположить, что в каждом классе 34 ученика, то сколько всего было бы учеников? А в действительности? 12. Установив число всех возможных различных видов медных монет и применяя идеи решения задач 10 и 11, получаем ответ на вопрос задачи. 13. Какой случай может оказаться самым «неблагоприятным» в данной ситуации? Какое наибольшее число ботинок может оказаться на одну ногу? (К такому же результату приводят рассуждения о числе ботинок, если они оказались парными-левым и правым, но разноцветными.) 14. Будем исследовать самый «неблагоприятный» случай, т. е. такой, когда имеется наибольшая вероятность того, что любые три из семи слагаемых в сумме дают результат, меньший чем 50. А это будет, если слагаемые «близко группируются» возле среднего значения. (Среднее значение 100:7.) 15. Как и в предыдущих задачах, выявим и исследуем самый «неблаго- приятный» случай, т.е. тот, когда в одном ящике одна деталь, в другом-две и т. д. Сколько при этом будет ящиков? По скольку же ящиков с одинаковым числом деталей? Но еще остался ящик, в который надо положить или 1, или 2, ...,'или 125 деталей. 16. При самом «неблагоприятном» случае среди взятых карандашей может оказаться по 17 карандашей некоторых цветов. Каких? В других случаях могут оказаться все имеющиеся карандаши одного цвета. Исследуем этот случай. 17. Во-первых, надо выяснить, сколько фотографий было 1-1П классов, во-вторых, сколько фотографий каждого класса из состава IV-VII классов могут оказаться при самом «неблагоприятном» случае. Исследуем этот случай. 18. Наименьшее число автомашин, обладающих всеми четырьмя указан- ными признаками, будет в том случае, если будет наибольшее возможное число машин, не обладающих хотя бы одним из признаков. На этом пути и будем искать решение. 19. Будем рассуждать так: если на обратной стороне карточки с буквой А будет нечетное число, то получим сразу ответ, если же число окажется in
четным, то надо перевернуть карточку с числом 4; при этом может оказаться снова один из двух случаев, от чего будет зависеть дальнейшее действие. 20. Как видим, в поисках решения этой задачи основную трудность представляет не вычислительная ее сторона, а логическая. Количества собран- ных тремя классами овощей взаимосвязаны, но надо найти начало и конец той «ниточки», за которую и следует «потянуть», чтобы последовательно нахо- дить искомые величины. К каким же классам надо сначала обращаться? 21. Как и в задаче 20, в решении этой задачи вычисления выполняют вспомогательную роль. Поиски решения начинаем с. отбора чисел, которые выражают возраст Тани, а в этой связи и Светы. 22. Если соединять последовательно 5 звеньев, то надо распаять 4 кольца и выполнить 4 соединения, на что будет затрачено 8 мин. Поищем более экономный способ. (Можно затратить всего 6 мин. Но как надо при этом поступить?) 23. Подумаем, какое звено надо распилить, чтобы затем можно было расплачиваться с получением сдачи. 24. Для выяснения, куда надо ехать (т.е. в каком городе пожар), устано- вим, из какого города могли звонить. Для этого последовательно будем соотносить два высказывания к жителям городов А, В и Б. Если бы звонили из А, то каким был бы ответ на второй вопрос? А могли ли звонить из В? (То есть может ли одно из двух данных высказываний быть истинным, а другое- ложным?) 25. Будем рассуждать, предполагая, что верно первое утверждение, затем- второе и т.д. включительно до 98-го утверждения. К какому заключению будем приходить каждый раз? Дальше выясним, будет ли верным утверждение 99-е, а затем и 1и0-е? 26. Чтобы в семилитровом ведре осталось 6 л, поищем способ, как из него отлить 1 л. Для этого воспользуемся черпаком. Если бы в нем было 3 л бензина, тогда, долив черпак из ведра, мы получили бы в ведре 6 л. Получить же в черпаке 3 л бензина, воспользовавшись ведром, нетрудно. 27. Задача сводится к тому, чтобы, выполнив действия над числами 9 и 5, получить в результате 6. Будем действовать экономно и из возможных вариантов выберем тот, где придется наименьшее число раз вливать бидоном по 9 л и отбирать по 5 л. 28. В целях облегчения поисков решений составим таблицу-схему пере- ливаний: Сосуд на 12 пинт на 8 пинт на 5 пинт 12 — — 4 8 — 112
29. Если бы после двух взвешиваний удалось получить 2 кг 200 г (или 1 кг 800 г), то третьим-получили бы 2 кг. Но 2 кг 200 г можно было бы получить, еслй бы имели 4 кг 400 г. Подумаем, нельзя ли получить 4 кг 400 г, воспользовавшись гирей 200 г и имеющимся сахаром 9 кг. 30. Проверим прежде всего, на какую дату (четную или нечетную) по- падает следующая пятница, если предыдущая приходилась на четную дачу. Установив это, выясняем, сколько пятниц в данном месяце и на какую дату (какое число месяца) приходится первая пятница. После этого уже не пред- ставляет труда установить день недели 18 числа. 31. Используя идеи решения задачи 30, устанавливаем сначала, каким днем недели начинается этот месяц и затем какого числа будет второе воскресенье. 32. Проанализируем возможные ситуации в зависимости от того, сколько дней имеет месяц. В этом отношении особо следует рассмотреть февраль в високосном и невисокосном годах. Выясним также, какими днями должны начинаться апрель, июнь, сентябрь и ноябрь, чтобы число суббот было наибольшим. Это же касается остальных месяцев. 33. Подумаем над тем, нельзя ли из условия задачи установить, о каких месяцах идет речь. Обратим внимание на то, каким днем недели начинается каждый из данных двух месяцев. Ведь первое воскресенье месяца не совпадает с первым воскресеньем после первой субботы этого месяца. Но если один месяц начинается с некоторого дня недели и с этого же дня недели начинается следующий за ним месяц, то может ли первый из этих месяцев иметь не целое число недель? Какой же это месяц? (Такой месяц только один.) Установив месяц и первый его день, получаем ответы на вопросы задачи. 34. Нижняя строка таблицы наводит на мысль, что записаны очевидно, числа дней месяцев. Подумаем, что же обозначают числа верхней строки. 35. Записаны начальные буквы каких-то слов, часто потребляемых. 36. Для отыскания решения задачи достаточно записать схематически ее условие в виде равенств и, проанализировав, преобразовать их. (Можно, например, записать: акробат 4- собачонка = 2 бочонка.) 37. Число команд-участниц турнира - можем определить, посчитав число слагаемых, записанных в соответствии с условием задачи, дающих в сумме 44. Зная число команд, можно определить число всех очков, которые они должны набрать, а тогда уже и число игр, которые осталось провести. 38. Для определения числа очков, набранных командой-победительницей, надо знать общее число очков. (Оно определяется на основании числа всех игр.) Поскольку команд, занявших последнее место, одна, то в записи выражения для подсчета суммы очков команд будет только одно слагаемое 5, другие три слагаемых будут больше, чем 5, причем одно из них больше каждого из двух других. Зная сумму и наименьшее слагаемое, можно подоб- рать остальные три. 39. Зная число участников турнира, не представляет труда определить число всех набранных очков, а зная общее число набранных очков, определим число очков, набранных участником турнира, занявшим второе место, а зна- чит, и участниками, занявшими с пятого по восьмое место. (Запишем число всех набранных очков в виде суммы восьми различных слагаемых.) Подумаем над тем, могли ли участники, занявшие с пятого по восьмое место, выиграть у тех, кто занял с первого по четвертое место. А если они набрали очки только в играх между собой, то можно определить, кто победил - третий или пятый? 113
40. Для решения задачи будем составлять таблицу розыгрыша. Ее состав- ляют по форме: Команда «Торпедо» «Азовец» «Энергия» «Строитель» Мячи Очки «Торпедо» «Азовец» «Энергия» «Строитель» Если, например, «Торпедо» выиграло у «Азовца» со счетом 3:0, то в строке «Торпедо» и в колонке «Азовец» пишем 3:0, а в строке «Азовец» и колонке «Торпедо»-0:3. Заполнение таблицы начинаем с анализа результатов игр «Торпедо» и «Строителя». Поскольку «Торпедо» набрало 5 очков, не пропустив ни одного мяча, то у него две победы и одна нулевая ничья. «Строитель» имеет одну ничью и три поражения. (Подключая дальше «Азовец» и «Энергию» для анализа, заполняем остальные строки и столбцы.) 41. Так как «Вымпел» имеет одно очко, то у него одна ничья, а остальные проигрыши. Учитывая, что «Авангард» пропустил только один мяч, можно заполнить его строку. Дальнейший анализ в сопоставлении с другими коман- дами дает возможность заполнить таблицу. 42. Имея уже опыт составления таблиц розыгрыша по футболу, успешно справитесь с этим заданием. Обратим сразу внимание на то, что «Старт» не потерял ни одного очка, у «Кометы» только два забитых мяча и один матч со счетом 1:0 она выиграла у «Вымпела». Значит, имеющийся счет 1:1 подходит для «Кометы», и тогда третий ее счет-проигрыш с «сухим» счетом. 43. Обозначив массы арбузов, например, а1г а2, а3 и а4, будем выбирать пары и сравнивать массы арбузов этих пар. И тогда, если, например, имеем а1 > а2 и а3 > а4 (два взвешивания), продолжаем исследовать, рассматривая другие пары. 44. Аналогично тому, как в задаче 43, но применительно к условию данной задачи введем обозначения массы деталей. Взяв, например, для сравнения а, и а2, можем получить два случая: 1) а3 = а2, 2) а3 > а2 или а3 <а2. Дальше рассматриваем каждый из этих случаев, используя для сравнения одну из оставшихся деталей. 45. Будем рассуждать так: если конь находится на белом поле (клетке), то, сделав один ход, на каком поле он окажется? Проверим цвет клетки, на которую он попадет после второго хода. Сравним результаты после четного числа ходов и после нечетного. Число всех клеток шахматной доски известно поэтому будет известно, сколько надо сделать ходов, чтобы обойти все клетки. Для этой цели полезно взять шахматную доску и практически выполнить конем несколько ходов. Кто незнаком с шахматами и правилами 114
этой интересной игры, обязательно познакомьтесь, обратившись за помощью к старшим или сверстникам, кто умеет играть в шахматы. 46. Замечаем, что кубики с числами 5 и 1 занимают «особые» положения. Попробуем их поменять местами. Тогда надо поменять местами, например, кубики с числами 2 и 3 с кубиками 7 и 10. Затем расставим кубики с оставшимися числами. 47. Удержать в памяти все варианты, представленные в условии, и со- поставить их, как вы, очевидно, в этом убедились, трудновато. Поэтому все четыре случая, о которых идет речь в задаче, изобразим схематически, записав множества зверей и соответствующие им множества подаренных предметов. Сопоставив полученные четыре схемы, получим решение. 48. Решение задачи сводится к выделению тройки чисел из возможных троек, произведение чисел каждой из которых равно '36. Запишем все такие тройки и суммы чисел каждой из них. Подумаем: почему, взглянув на номер автобуса, тренер попросил дополнительную информацию? Рассматривая сум- мы чисел троек, можно узнать, какой номер автобуса увидел тренер.
РЕШЕНИЯ И ОТВЕТЫ Нумерация чисел 1. а) 100-100-100= 1000000; б) 10-10-10-10-10-10= 1 000000. 2. 2000000, 1 100000,r 1 010000, 1001 000, 1 000 100, 1 000010, 1 000001. Всего 7 чисел. 3. Повернув лист бумаги на половину полного оборота, получим число 989, которое больше чем 686 на 303 (989-686 = 303). 4. Света увидела число, которое получается в результате вращения лисга бумаги на половину полного оборота, поэтому в записи числа могли быть только цифры 0, 1, 8, 6 и 9. Условию удовлетворяет лишь число 91. Света увидела 16, тогда 91 — 16 = 75. 5. Девяносто записано в римской нумерации ХС. Если исключить X, то и будет С (сто). 6. ХШ - VII = VI; VI = V + I; XI - V = VI. 7. а) Цифрой 3 начинается 10 чисел (от 30 до 39 включительно), оканчива- ется этой цифрой всего 9 чисел, но число 33 уже учтено, поэтому остаемся таких чисел 8. Значит, искомых чисел будет 10 + 8 = 18. б) От 10 до 20 таких чисел 8 (12, 13,..., 19), от 21 до 30 7, от 31 до 40 6 и т. д., от 81 до 90-одно число 89. Значит, всего таких чисел 8 + 7 + 6 + ...+ + 2+ 1 = 9-4 = 36. 8. Среди двузначных таких чисел 36 (см. задачу 7). Подсчитаем, сколько таких чисел среди трехзначных. От 123 до 129-7 чисел, от 134 до 139 6 и т.д.; наконец, одно число 189. Значит, среди трехзначных чисел, у которых цифра сотен 1, таких чисел 7 + 6 + ... + 2+1 =28. Среди трехзначных чисел, начи- нающихся цифрой 2, таких чисел 6 + 5 + ... + 2+ 1 = 21; начинающихся цифрой 3, таких чисел 5 + 4 + 3 + 2+1 = 15; начинающихся цифрой 4, таких чисел 4 + 3 + 2+1 = 10; начинающихся цифрой 5, таких чисел 3 4-2+1=6: с цифрой сотен 6 таких чисел '2 + 1 = 3; с цифрой сотен 7 только одно число 789. Таким образом, всего таких трехзначных чисел 28 + 21 + 15 + 10 + 6 + + 3 + 1 = 84, а вместе с двузначными их будет 36 + 84 = 120. 9. 555 + 55 + 55 + 55 + 55 + 55 + 55 + 55 + 55 + 5 = 1000. 10. 1023456789. 11. 1023467895. 12. Две средние цифры искомого числа .образуют число, кратное 3 и 5, т. е. 15. Принимая во внимание, что при делении на 5 и на 3 этого двузначного числа должны получаться однозначные числа (цифры тысяч и единиц), приходим к выводу, что две средние цифры 1 и 5, и, значит, искомое число 3155. 116
13. a) 89406; б) 13406. 14. а) В записи чисел от 1 до 40 включительно использована 71 цифра. Для того чтобы оставшиеся 11 цифр выражали наименьшее число, надо вычерк- нуть 6Q больших цифр, а в оставшемся числе слева были наименьшие цифры. Таковы: во-первых, три нуля (оставшиеся от чисел 10, 20, 30), а затем 1. 2, 3, 3, 3, 3, 3, и 0. 'Значит, наименьшим числом есть 00012 333330= 12333 330. б) Наибольшее число должно начинаться с наибольшего числа девяток (их может быть не больше трех): это девятки из первых трех десятков, четвертая цифра должна быть наибольшей из тех, справа от которых еще остается 7 цифр. Такой является цифра 6. Значит, наибольшим числом есть 99 967 383 940. 15. Последняя страница выпавшего куска может иметь номер, обозначен- ный четным числом, так как выпасть может целое число листов. Таким числом при данном условии может быть только 314. Числа же 341, 431 и 413 нечетные. Тогда количество страниц выпавшего куска есть 314 — 142 = 172. 16. Таких чисел, которые начинаются с 1, будет 2, а именно 1230 и 1032; таких, которые начинаются с 2, будет 4, а именно 2310, 2130, 2301, 2103; таких, которые начинаются с 3, будет 2, а именно 3210 и 3012. Значит, всего таких чисел 8. 17. Подсчитаем сначала количество таких трехзначных чисел, в записи каждого из которых есть по крайней мере одна цифра 7. Во-первых, замечаем, что от 700 до 800 таких чисел 100; во-вторых, в пределах каждой иной сотни таких чисел 19. (Например, от 100 до 200 чисел с цифрой десятков 7 будет 10. Это числа 170, 171, ..., 179. Чисел с цифрой единиц 7 также 10. Это числа 107, 117, ..., 197. Всего 10+10 = 20. Но число 177 учтено дважды, поэтому 20 — 1 = 19.) Всего трехзначных чисел с цифрой 7 будет 19-8 + 100 = 252. Принимая во внимание, что всех трехзначных чисел 900, найдем количество трехзначных чисел, в записи которых нет ни одной цифры 7: 900 — 252 = 648. 18. Предположим, что существует такое число Юл + у, тогда 10.v + у = = х + у, 9х = 0, откуда х = 0, что противоречит нашему предположению. Значит, такого числа не существует. 19. Пусть шифр aaabbbb, тогда по условию За + 4Ь = 10а + Ь, откуда 7а — ЗЬ. Так как а и b-цифры, то равенство возможно лишь при а — 3 и b = 7. Значит, шифр 3 337 777. (Запись aaabbbb означает семизначное число, в кото- ром b единиц первого разряда, b единиц второго разряда,..., а единиц пятого разряда, ..., а единиц седьмого разряда.) 20. Если искомое число 10а + Ь, то 10а + b = 176, откуда 5а = 86. Равенст- во возможно лишь при а = 8 и b = 5. Ответ. 85. 21. Пусть расстояние выражается числом 10а + Ь. Если вставим нуль между цифрами десятков и единиц, то получим 100а + Ь. Согласно условию имеем 100а + b = 9 (10а + Ь), значит, 10а = 86, откуда 5а = 46, а = 4, 6 = 5. Ответ. 45 км. 22. Пусть двузначное число аб, тогда полученное четырехзначное abba. Так как abba — 1000а + 1006 + 106 + а = 1001а + 1106 и 1001 • 11, а также 110• И, то abba\ 11. Примечание. Запись 1001-11 означает, что 1001 кратно 11. 23. Пусть искомое число 10а + 6. Тогда а = (10а + 6) - (106 + а), а = 9а - 96, 96 = 8а, откуда а = 9, 6 = 8. Ответ. 98 км. 117
24. Очевидно, искомое число трехзначное. Пусть оно 100а + 106 + с, тогда 328 — 100а — 106 — с = а + 6 + с, 328 = 101а + 116 + 2с, откуда а = 3. Значит, 116 + 2с = 328 — 303 = 25, откуда 6 = 1 и с = 7. Итак, искомое число 317. 25. Допустим, что есть такое число 100а + 106 + с, которое удовлетворяет условию задачи, тогда 100а + 106 + с = 3(1006 + 10с + а), 97а — 2906 — 29с = 0, 97а откуда с = —— 106. Так кай а цифра, отличная от 0, то 1 а 9. При этом 97а «ело не монет быть целым, а значит, и с не может быть целым. В «шло же 100а + 106 + с цифра с обозначает целое число. А это опровергает сделанное нами предположение. 26. Пусть первое трехзначное число 900 + 10а + 6, тогда второе 100а + 106 + 9. Большим будет первое число, поэтому (900 + 10а + 6) - (100а + 106 + 9) = 216, 90а + 96 = 675, 10а + 6 = 75, 900 + 10а + 6 = 900 + 75 = 975. Значит, один класс собрал 975 кг, а дру- гой-759 кг. 27. Пусть натуральное число а, а приписываемая цифра х. Тогда согласно условию а-10 + х = а + х + ах, откуда 9а = ах, а значит, х = 9. 28. Пусть цифры числа а, а + 1, а + 2 и а + 3, тогда одно число 1000а + + 100(а + 1) + 10(а + 2) + (а + 3), а число, записанное теми же цифрами, но в обратном порядке, 1000(а + 3) + 100(а + 2) + 10(а + 1) + а. Найдем раз- ность этих чисел: 1000а + 3000 + 100а + 200 + 10а + 10 + а — 1000а — 100а — - 100 - 10а - 20 - а - 3 = 3000 + 200 + 10 - 100 - 20 - 3 = 3087. Как видно, разность при указанном условии задачи всегда равна 3087. 29. Нечетные числа 1, 3, 5, 7, ..., 99; четные числа 2, 4, 6, 8, ..., 100. Сравним суммы однозначных чисел этих групп: 1+3 + 5 + 7 + 9 = 25, 2 + + 4 + 6 + 8 = 20, 25 — 20 = 5. Рассмотрим числа с цифрой десятков 1. Сумма цифр будет больше у той группы, у которой больше сумма цифр единиц. Как было показано выше, такая сумма больше в группе нечетных чисел на 5. Аналогично для чисел с цифрой десятков 2, 3, ..., 9. Таким образом, сумма всех цифр нечетных чисел от 1 до 99 больше суммы цифр четных чисел от 2 до 98 на 5 • 10 = 50. Но к группе четных чисел принадлежит число 100, сумма цифр которого 1, поэтому 50 — 1 = 49. Значит, сумма всех цифр, использованных для написания нечетных чисел от 1 до 100, на 49 больше, чем сумма цифр, использованных для написания четных чисел от 1 до 100. 30. Пусть искомое число 100а + 106 + с, тогда можно составить такие двузначные числа: 10а + 6, 106 + а, 10а + с, 10с + а, 106 + с, Юс + 6. По условию имеем (10а + 6) + (106 + а) + (10а + с) + (Юс + а) + (106 + с) + (Юс + 6) = = 2(Ю0а + 106 + с), 22а + 226 + 22с = 2(Ю0а + 106 + с), Юс + 6 = 89а. Число Юс + 6 двузначное, поэтому и 89-а двузначное, что возможно при а = 1. Тогда с = 3 и 6 = 9. Итак, в V-VI классах школы учатся 198 учащихся. 31. Поскольку сумма трехзначного и двузначного чисел 144, то цифра сотен трехзначного числа 1. Сумма этой цифры и цифры десятков двузначного числа 3, так как сумма чисел, записанных в обратном порядке, 603; поэтому цифра десятков двузначного числа 2 (3 — 1 = 2). Цифра единиц искомого трехзначного числа 5 (6 — 1=5), так как сумма обращенных чисел 603, а значит, цифра единиц двузначного числа 9 (144 — 1*5 = 29). Значит, поса- дили 115 тополей и 29 лип. 118
32. При умножении на 37 второго числа, оканчивающегося цифрой 1, получим число, оканчивающееся цифрой 7. J3hh4ht, первоначально были записаны числа, оканчивающиеся цифрой 7, т. е. al. По условию 100а7 = а7Г 37, а71-37 = (100а 4- 71)-37 = 3700а + 2627. Легко заметить, что а = 2. Действи- тельно, 3700-2 = 7400, 7400 + 2627 = 10027. Значит, на доске было написано 27, 27. 33. Всех различных шестизначных чисел 900000. (Как это определить?) Из них оканчивается нечетной цифрой половина, т.е. 450000. Из этих чисел имеют вторую справа нечетную цифру 450000:2 = 225000, из них имеют третью справа нечетную цифру 225000:2 = 112500, из них имеют четвертую нечетную цифру справа 56250, из них имеют пятую нечетную цифру справа 28 125 28125, из них шестую нечетную цифру справа имеют —-—-5 = 15 625 (шестой цифрой справа,‘т.е. первой слева, не может быть 0). Значит, всего 15625 шестизначных чисел, у которых все цифры нечетные. 34. Чтобы сумма была наибольшей, цифрами сотен должны быть 9, 8 и 7, цифрами десятков 6, 5, 4 и цифрами единиц 3, 2 и 1. Большее число имеет 9 сотен, а меньшее 7. Чтобы разность их была наименьшей, большее из этих чисел должно иметь как можно меньше десятков и единиц, а меньшее, наоборот, как можно больше. Значит, большее число 941, а меньшее 763. Тогда третье число 852. Имеем 941 + 852 + 763 = 2556, 941 — 763 = 178. 35. Запишем числа при помощи букв: aba, be и b. Тогда V) а + b + а = Ъс и 2) b + с = Ь. Из последнего равенства имеем с = 0, а из первого 2d + b = 10/?, 2а = 9Ь, откуда а = 9 в b = 2. В результате имеем 929, 20, 2. 36. Сопоставляя числа А □ и □, получим, что А = 1, так как только при умножении числа □ на 1 получим это же число □. Сопоставляя 1ОО и 1Q, получим, что 0=4, так как только при умножении единицы на 4 и еще раз на 4 получим число с цифрой десятков 1. Имеем 144; 1 - 4-4 = 16; 1 - 6 = 6. В V-VI классах 144 ученика. 37. Первый раз было вычеркнуто 50 нечетных цифр и осталось 50 четных. Второй раз вычеркнули 25 цифр, из которых 5 двоек, 5 четверок, 5 шестерок, 5 восьмерок и 5 нулей. Осталось 25 в такой последовательности: 4, 8, 2, 6, 0, 4, '8, 2, 6, 0,..., 2, 6, 0. (Группа цифр 4, 8, 2, 6, 0 записывается подряд 5 раз.) При третьем вычеркивании будет вычеркнуто 13 цифр, а останется 12 таких цифр: 8, 6,4, 2,0, 8, 6,4,2, 0, 8, 6. После четвертого вычеркивания останутся 6, 2, 8, 4, 0, 6, после пятого-2, 4, 6 и после шестого-4. Значит, последней (седьмой) раз будет вычеркнута цифра 4. 38. Пусть искомое число х^х,... х„. Известно, что х;х£ + t • 23. Для Х,Х( + ! возможны такие значения: 00, 23, 46, 69, 92. Учитывая эти 5 возможных вариантов, получим 46 923. Это число будет наибольшим из возможных таких чисел, так как никакое из пяти чисел, кратных 23, не оканчивается на 4 и не начинается на 3, и поэтому слева и справа нельзя приписать никакую цифру. 39. Пусть х = 100а + 106 + с. Тогда (100а + 106 + с) — (100с + 106 + а) = = 99 (а —с). Разность цифр (а — с) может принимать значения от 2 до 8. Придавая такие значения (а — с), получим 198, 297, 396, 495, 594, 693, 792. Рассмотрим все возможные случаи. 1) Искомое число может состоять из цифр 1, 9 и 8. Тогда имеем 891 - 198 = 693, 981 - 189 = 792, 918 - 819 = 99. Каждое из этих чисел не удовлетворяет условию. 119
2) Пусть искомое число состоит из цифр 2, 9 и 7. Тогда имеем 792 - 297 = 495, 972 - 279 = 693, 927 - 729 = 198. Каждое из этих чисел не удовлетворяет условию. 3) Пусть искомое число состоит из цифр 3, 9 и 6. Тогда имеем 693 - 396 = 297, 963 - 369 = 594; 936 - 639 = 297. Каждое из этих чисел не удовлетворяет условию. 4) Пусть искомое число состоит из цифр 4, 9 и 5. Тогда имеем 594 - 495 = 99, 954 - 459 = 495, 945 - 549 = 396. Условию удовлетворяют числа 954 и 459. 40. «Тысяча девятьсот восемьдесят» имеет 26 букв, «двадцать шесть» - 13 букв, «тринадцать»-10 букв, «десять»-6 букв, «шесть»-5 букв, «пять»-4 буквы, «четыре»-6 букв, «шесть»-5 букв, «пять»-4 буквы. Имеем: 1980, 26, 13, 10, 6, 5, 4, 6, 5, 4, 6, 5, 4. а) Начиная с пятого числа, т. е. с числа 6, группа чисел 6, 5, 4 повторяется. б) Каким бы ни было первое число ряда,, количество букв в словах, выражающих его название, будет определенным числом. Для чисел двузнач- ных и большей значности каждое последующее число ряда меньше предыду- щего. Значит, на некотором месте в ряду появится однозначное число. Наименьшее число, которое может появиться, будет 3. так как наименьшее количество букв в названии чисел равно трем. (Например, число 100.) Но если появилось число 3, то все последующие числа ряда будут гройки. Значит, повторяется одно число ряда. Если первым однозначным числом появится число 4 (четыре), то следующим за ним будет 6 (шесть), дальше 5 (пят ь), за ним 4, т. е. числа 4, 6, 5 составляют повторяющийся цикл. Если появится первым однозначным числом 5 (пять), то за ним будет 4, дальше 6, зачем снова 5 и т.д. Значит, повторяется цикл 5, 4, 6. Если первым появится 6, то за ним будет 5, затем 4, 6 и т.д., т.е. повторяется 6, 5, 4. Если первым появится 7 (семь), то за ним будет 4 и, как видно из предыдущего, появится цикл 4, 6, 5. Если первым появится 8 (восемь), то будет цикл 6, 5, 4. Наконец, если появится 9 (девять), то снова будет цикл 6, 5, 4. Таким образом, либо будет повторяться число 3, либо одна из троек (4, 6, 5), (5. 4, 6), (6, 5. 4). Арифметические действия над натуральными числами и нулем и их свойства 1. Если сложить четыре числа 84, 80, 76, 72, то получим утроенную стоимость всех книг, поэтому стоимость книг равна: (84 + 80 -Ь 76 -+- 72): 3 = = 312:3 = 104 (к.). Значит, первая книга стоит 104 — 84 = 20 (к.), вторая 104 — 80 = 24 (к.), третья 104 — 76 = 28 (к.), а четвертая 104 — 72 = 32 (к.). 2. Коля, Вася, Юра и Саша купили вместе 86 тетрадей (52 + 34 = 86). Значит, Сережа купил 14 тетрадей (100 — 86 =14). Тогда Саша купил 16 120
(30-14=16), Юра 18 (34-16=18), Вася 25 (43 -18 = 25) и Коля 27 (52 - 25 = 27). 3. Поскольку в V—VIII классах на 8 учеников больше, чем в 1—IV классах (7 — 5 + 10 — 4 = 8), то в V—VIII классах всего 138 учеников (130 + 8 = 138), а всего в I—VIII классах 268 учеников (130 + 138 = 268). 4. Если число килограммов макулатуры, собранной двумя мальчиками, утроить, то получили бы 81 (27 -3 = 81). Тогда 111 — 81 = 30 (кг) есть удвоен- ное число килограммов макулатуры, собранной Витей, а значит, Витя собрал 15 кг (30:2 = 15), тогда Вова -12 кг (27 - 15 = 12). 5. Сравнивая проданное за первый и третий дни (1-1 большая банка и 4 средние; III - 1 большая банка, 3 средние и 3 литровые), получаем, что средняя банка имеет емкость 3 л. Значит, за: 1 день-1 большая банка и 3-4= 12 (л); II день-2 большие банки и 6 л. Отсюда емкость большой банки 6 л. Итак, за один день продали 6 + 12= 18 (л), а за три дня - 54 л. 6. Используем такую схематическую запись. Так как уменьшаемое -|- вычитаемое + разность = 1024 и уменьшае- мое = вычитаемое + разность, то уменьшаемое • 2 = 1024, откуда умень- шаемое 512. Если разность х, то вычитаемое х + 88, и тогда х + х + 88 = = 512, откуда х = 212. Значит, разность 212, вычитаемое 300 и уменьшаемое 512. 7. Если бы были все индюки, то ног было бы 2 • 11 = 22. Всего же ног было 30, что на 8 (30 — 22 = 8) больше, так как были не только индюки, но и жеребята. Жеребенок имеет на две ноги больше, чем индюк, значит, жеребят было 8:2 = 4. 8. Чтобы произведение чисел делилось на 10, среди множителей должны быть числа 2 и 5. Принимая это во внимание, возможны такие произведения, удовлетворяющие условию задачи: 1)2-5; 3)2-5-7; 5)2-5-3-7; 7) 2 • 5 • 7 • 9; 2)2-5-3; 4)2-5-9; 6)2-5-3-9; 8)2-5-3-7-9. Всего 8 произведений. 9. Замечаем, что 185 = 37 - 5 и 16 + 21 =37. Поэтому магазин мог получить 5 ящиков по 21 кг и 5 ящиков по 16 кг. Так как разложение числа 185 выполнено на простые множители, то задача имеет единственное решение. 10. У Андрея 100 г конфет одного вида и 50 г-другого, у Юры по 75 г конфет каждого вида. Конфеты Андрея дороже конфет Юры на столько, на сколько 25 г одного вида дороже, чем 25 г другого. Поскольку 1 кг дороже на 80 к., то 25 г дороже на 2 к. (80:40 = 2). Значит, Андрей заплатил за конфеты на 2 к. больше, чем Юра. 11. (1 +2 + 3 + ...+ 12) 2 + 24= 13-6-2 + 24= 180. 12. 2000 = 94-21 + 26. Значит, срубив 94 раза по 21 голове, останется 26 голов. Замечаем, что если отрубить 17 голов, то вырастет 14, т.е. количество голов уменьшится на 3. Тогда проделаем это три раза, и останется 26 — 3 • 3 = = 17, которые можно отрубить последним ударом. 13. Так как на одном этаже в одном подъезде 6 квартир (60 — 54 = 6), то на восьми этажах одного подъезда имеется 48 квартир. Поскольку 211 = 48 • 4 + + 19, то квартира 211-в пятом подъезде, а так как 19 = 6-3+ 1, то эта квартира-на пятом этаже. 121
14. В первом слагаемом цифры десятков и единиц одинаковые. Возьмем наименьшее из таких чисел-число 11, тогда второе слагаемое должно быть 989 или 999 (чтобы получить в сумме четырехзначное число), но 11 + 989 = = 1000 и 11 + 999 = 1010 условию не удовлетворяют. Пусть первое слагаемое 22, тогда 22 4- 999 = 1021. Чтобы сумма удовлетворяла условию задачи, надо уменьшить ее на 20, уменьшив на 20 и второе слагаемое. В результате получим 22 + 979= 1001. 15. Обозначим недостающие числа так: 8 *i *2 *3 *4 х5 X* 5 Х% х9 Тогда из условия следует х3 = х6 = х8 = 8. Так как 5 + хя + х9 = 20, то х9 = 7 (20 — 5 — 8 = 7). И тогда х7 = х4 = Xj = 7. Так как 8 + х2 + х2 = 20, то х2 — 5 и х5 = 5. Окончательно имеем 8 7 5 8 7 5 8 7 5 8 7 16. Так как при увеличении второго множителя на 5 произведение увели- чится на 1250, то первый множитель равен 1250:5 = 250, т.е. в первой партии было 250 тетрадей. При увеличении первого множителя на 7 произведение увеличивается на 3150, поэтому второй множитель равен: 3150:7 = 450, т.е. во ,второй партии было 450 тетрадей. Значит, всего магазин отпустил 250 + + 450 = 700 (тетрадей). 17. Число будет наибольшим, если при делении его на 31 получается наибольший из возможных остатков. Таким остатком будет 30. Значит, искомое число равно: 30-31 + 30 = 32-30 = 960. 18. В данном случае имеется делимое, оканчивающееся цифрой 7, дели- тель-цифрой 5, частное-цифрой 3 и остаток-цифрой 1. Известно, что делимое равно произведению делителя и частного, сложенному с остатком. Но произведение делителя и частного оканчивается цифрой 5 (5-3 = 15), а сумма этого произведения и остатка оканчивается цифрой 6 (5 + 1 = 6), в то время как делимое оканчивается цифрой 7. Полученное противоречие говорит об ошибочности записи. 19. Так как 1 ч имеет 3600 с, то, умножив это число на целое число часов, получим число,, оканчивающееся двумя нулями. Значит, предпоследняя цифра полученного числа секунд 0. Тогда число 234 * 200:3600 должно быть целым. Имеем 234*2 / 36 216 65 18* 180 * 2 122
Отсюда число * 2 должно делиться на 36. Оно равно 72. Итак, искомые цифры 7 и 0. Дима в секундах выразил 652 ч. 20. Номера страниц идут от 1 до 64. Страницы на двойных листах расположены так: 1-й-1, 2, 63, 64; 2-й-З, 4, 61, 62; 16-Й-31, 32, 33, 34. Так как в последовательности суммы чисел, равноот- стоящих от концов, одинаковы, т. е. 1 + 64 = 2 + 63 = ... = 32 + 33, то суммы чисел, обозначающих номера страниц двойных листов, одинаковы, а именно 65 + 65 = 130. 21. Три участника-призера набрали 57 очков (20 + 19 + 18 = 57), тогда два других набрали 37 очков (94 — 57 = 37). Так как 37 = 19 + 18, то один из них набрал 19, а другой-18 очков. Значит, первое место занял один участник, второе-два участника и третье-два участника. 22. Так как длина всей проволоки-число четное (102 см) и сумма длин отрезков по 12 см четная, то и сумма длин отрезков по 15 см тоже четная. Значит, число кусков по 15 см должно быть четным. Рассмотрим все возможные случаи: 1) Если по 15 см 2 куска, то 15-2 = 30 и 102 — 30 = 72, а 72:12 = 6. Значит, по 12 см будет 6 кусков. 2) Если по 15 см 4 куска, то 15 • 4 = 60 и 102 — 60 = 42, но 42 не делится без остатка на 12. 3) Если по 15 см 6 кусков, то 15-6 = 90 и 102 — 90 = 12. Таким образом, возможны только 2 случая: а) 2 куска по 15 см и 6 кусков по 12 см; б) 6 кусков по 15 см и 1 кусок 12 см. 23. Так как 231 = 3-7-11, но 3 + 7 + 11 = 21, то это число можно предста- вить в виде суммы слагаемых, произведение которых равно 231, так: 231 = 3 + 7 + 11+ 1 + 1 + ... + 1. 210 раз 24. Два младших брата получили 2400 р., а каждый-1200 р. (2400:2 = = 1200). Значит, стоимость всего наследства, т.е. трех домов, 6000 р. (1200 х х 5 = 6000). Тогда один дом стоит 2000 р. (6000:3 = 2000). 25. а) Числа возрастают на 1, 2, 3, 4, ... и т. д., поэтому следующие будут: 29 + 5 = 34, 34 + 6 = 40. б) Числа возрастают на 3. 6, 12, 24, ... и т.д., поэтому следующие будут: 50 + 48 = 98, 98 + 96 = 194. в) Каждое последующее число на 15 меньше предыдущего, поэтому следующие два числа будут: 193 — 15 = 178, 178 — 15 = 163. г) Записаны вперемешку числа двух рядов: 12, 16, 20, 24, ... и 11, 16, 21, 26,... В первом из них каждое последующее число на 4 больше предыдущего, во втором ряду-на 5. Поэтому следующие два числа будут: 24 + 4 = 28, 26 + 5 = 31. 123
д) Числа удваиваются, а затем вычитаются 1, 2, 3, ... Следовательно, последующие два числа будут: 214-2 — 5 = 423, 423-2 -6 = 840. 26. (5 + 5 + 5): 5 = 3 (5-5 - 5): 5 = 4 (5 — 5)-5 + 5 = 5 (5-5 + 5):5 = 6 (5 + 5): 5 + 5 = 7 5-5 + 5:5 = 26 (5:5 + 5)5 = 30 5-5 + 5-5 = 50 (5 + 5)-5 + 5 = 55 5-5-5 - 5 = 120 5-5-5 + 5 = 130 5-5-5-5 = 625 27. 7-7+ 7:7= 1 7:7 + 7:7 = 2 (7 + 7 + 7):7 = 3 77:7-7 = 4 7 -(7 + 7):7 = 5 (7-7— 7):7 = 6 (7 — 7)-7 + 7 = 7 (7-7 + 7):7 = 8 (7 + 7): 7 + 7 = 9 (77-7):7= 10 28. Для печатания однозначных чисел потребуется 9 знаков; двузнач- ных-180 (2 -90 = 180); трехзначных - 2700 (900 -3 = 2700); четырехзначных - 36000 (4 -9000 = 36000); пятизначных - 450 000 (5-90000 = 450000); шестизнач- ных — 5400000 (6 -900000 = 5400000) и одного семизначного числа (1000000)- 7 знаков, а всего 9 + 180 + 2700 + 36000 + 450000 + 5400000 + 7 = 5888896. Интервалы между числами займут: 2 • 999 999 = 1 999 998 знаков. Значит, всего надо места для 7887894 знаков (5 888896+ 1999998 = 7887894). На одном листе (странице) вмещается 1800 знаков (60-30= 1800). Так как 7 887 894:1800 = 4382 (ост. 294), то всего требуется 4383 листа бумаги. 29. а) Неправомерно применен распределительный закон для частного. б) Выполнено деление на 0 (143 -7 = 1001, 1001 — 1001 = 0). 30. Пусть задумали число х. Тогда последовательно получаем: I) 2х; 2) 2.у + 3; 3) Юх + 15 = 10(х + 1) + 5; 4) 10(л + 1) + 12; 5) 2; 6) 2 + 18 = 20; 7) 20:5 = 4. 31. Пусть делимое а, а делитель Ь, тогда по условию частное равно а:2 или 66, т.е. а:2 = 66, откуда а = 12b и а:Ь = 12. Значит, частное а:b равно 12. 32. Если детей было а, то всех тетрадей 15а. При делении на 13 числа 15а получим в частном айв остатке 8. Значит, 15а = 13а+ 8, откуда 2а = 8 и а = 4. Итак, тетрадей было 60 (15-4 = 60). 33. Пусть Сережа делил число а, а частные, которые он получил, равны соответственно b и с. Тогда а = 156 + 8 и а = 20с + 17, откуда 156 + 8 = 20с + + 17, или 156 = 20с + 9. Левая часть полученного равенства делится на 5, а правая нет, так как 9 не делится на 5. Значит, равенство ни при каких целых 6 и с не может быть верным, т.е. Сережа допустил ошибку. 34. Поскольку один множитель оканчивается цифрой 6 (436), а произведе- ние-цифрой 8 (...728), то второй множитель должен оканчиваться цифрой 8 или 3. Если он оканчивается цифрой 8. то 436-8 = 3488. Имеем ,3 48 8 + ...х ...7 2 8 Поскольку (8 + .v) оканчивается цифрой 2, то х = 4. Значит, 6- у должно оканчиваться цифрой 4, поэтому у = 4 или у = 9. Если у = 4, то количество 124
проданной материи 48 м или 148 м, но каждое из этих чисел не удовлетворяет условию, так как получаем 208 р. 28 к. и 1081 р. 28 к. Остается положить у = 9. Тогда 4 р. 36 к. • 98 = 427 р. 28 к. Если предположить, что второй множитель (число метров) оканчивается цифрой 3, то аналогичный анализ показывает, что ни одно из возможных чисел 23, 123, 223, 73, 173 не удовлетворяют условию задачи. Итак, продано 98 м по 4 р. 36 к. за 1 м на сумму 427 р. 28 к. 35. Сумма чисел каждой строки или стдлбца равна ПО. Поэтому если, например, задумано число 30, то сумма чисел строки без него: 14 + 32 + 8 + 4- 26 = 80. Если вам сообщили эту сумму, то вы находите 110 — 80 = 30. 36. Пусть размер вашей обуви ab (двузначное-число). Выполняем последо- вательно: 1) ab-2; 2) 2//6 + 39; _ 3) (2а6+^39) -50= 100 • ab + J950; 4) 100 -ab + 1950 + 40 = 100а6 + 1990. __ Если год рождения 1975, то 1990— 1975 = 15. Получим число ЮОаб + 15, А это и есть четырехзначное число, у которого две первые цифры образуют число ab (номер обуви), а последние — 15 (возраст). 37. Пусть задумано число 100а + 106 + с. Тогда, выполнив последователь- но указанные операции, получим ((2а + 3)-5 + 6)-10 + с, или (100а + 106 + с) + 150, откуда видно, что для получения задуманного числа достаточно из получен- ного результата вычесть 150. Если, например, получили 790, то было задумано 790- 150 = 640. 38. Пусть, например, были записаны числа 58431 и 61 388. Отгадывающий дописал такое третье слагаемое, которое в сумме со вторым числом, записан- ным ребятами, даст число, в записи которого только девятки. Например, 61388 + 38611 =99999. Сумму трех слагаемых легко получить, пользуясь числом 58431, поставив крайней слева цифру 1, уменьшив на 1 цифру единиц, а все другие цифры оставив без изменения. В конверт отгадывающий вложил число 158430. 39. Для решения задачи составим таблицу: Класс Число баллов за составление задачи Число баллов за составление задачи Всего баллов Место, занятое классом 5А 51=5 5-4 = 20 25 « Il II TI- т1- 25 + 16 = 41 п 5Б t-л LO N) U) II II bh О 1У1 4-4= 16 4-4= 16 32 25 + 32 = 57 I 5В 5-4 = 20 51 =5 25 II II ГЗ 00 25+ 12 = 37 III 125
40. Если число команд п, то всего игр---, а количество набранных ими очков равно (п — 1)и. В нашем примере имеем 16+14+10+10 + 8 + + 6 + 5 + 3 — 72. Значит, (л — 1)-и = 72, или (п — 1)-и = 8-9, поэтому п = 9, т. е. было 9 команд. Максимальное количество очков, которое может набрать команда, равно: 2-8 = 16. Значит, первая команда не потеряла ни одного очка, вторая — 2, третья и четвертая-по 6 очков, а всего ими потеряно 14 очков (2 + 6 -2 =14). (Одна команда не набрала ни одного очка.) (1 | 67) 67 41. Найдем общую (суммарную) массу всех гирь: 1 + 2+ ... + 67 =---= 68-67 = —-— = 34 -67 (см. решения задач 11,20). Так как 34 и 67 не делятся на 3 без остатка, то разложить на равные по массе группы эти гири нельзя. 42. Если бы значения на косточках изменялись от 0 до 12, то были бы такие косточки: 0 0 1 1 2 2 II II 12 12 01 1223 II 12 0 2 1 3 2 4 ... 0 3 : : I 12 2 12 0 12 Значит, косточек с нулем было бы 13, с единицей (но без нуля) -12, с 2 (но без нуля и единицы)-11 и т.д. Тогда всего косточек было бы: 13 + 12+ 11 + ... + 3 + 2+ 1 =(13 + 1)-6 + 7 = 91. 43. Сумма цифр числа 1986 равна 24, а так как 1986 = 82-24+ 18, то достаточно вычеркнуть крайние слева 2 цифры (8 и 6), оставив 1, 9 и 8 (1 + 9 + + 8 = 18) и 82 идущих за ними групп цифр 6, 1, 9, 8 (6 + 1 + 9 +8 = 24), а затем вычеркнуть все оставшиеся 67 цифр (400 — 5 — 4 • 82 = 67). 44. Запишем сумму в порядке возрастания слагаемых: 111 + 113 + ...+ + 119 + 131 + ... + 311 +313 + ... + 319 + ... + 911 +913 + ... + 993 + 995 + + 997 + 999. Чисел-слагаемых, которые начинаются с цифр 1, 3, 5, 7, 9, будет по 25, поэтому всего слагаемых 25-5 = 125. Сумма двух слагаемых, равноуда- ленных от концов, равна 1110. Таких’сумм 62, да среднее слагаемое 555, поэтому искомая сумма будет 1110-62 + 555 = 69375. 45. Пусть числа, расставленные по окружности, at, а2, а3, ..., а9. Тогда суммы чисел по три, стоящих подряд, + а2 + а3, а2 + а3 + а4,..., аа + а9 + + а2, а9 + а2 + а2. Всего таких сумм 9. При этом общая сумма этих сумм содержит слагаемым трижды каждое из чисел от 1 до 9. Имеем (1 + 2 + ... + 9)-3 = 45-3 = 135. Так как каждая сумма трех чисел, стоящих подряд, больше 12 и делится на 3, то она может равняться только 15. Значит, + а2 + а3 = а2 + а3 + п4, откуда tij = а4, что противоречит условию. Значит, задача не имеет решения. 46. Поскольку при сложении четырех неотрицательных чисел в сумме надо получить число 1, то из четырех слагаемых три должны быть нули. Значит, первую тройку чисел можно взять 1, 2, 3, а в каждой из остальных троек иметь по нулю. Чтобы получить в сумме число 4, надо в одной из троек, кроме 126
первой, иметь число 3. Пусть оно будет во второй тройке вторым числом. Чтобы в сумме получить число 7, возьмем во второй тройке третьим числом 6. Получим суммы, равные числам от 1 до 9. Чтобы получить в сумме 10, возьмем вторым числом в третьей тройке число 9. Продолжая, получим: I 2 3 0 3 6 0 9 18 0 27 54 47. Если в первый год награждено х колхозников, то в последний Зх, а за эти два года 4х. Значения х = 1 и х = 2 условию не удовлетворяют, так как за второй, третий и четвертый годы будет соответственно награждено 23(27 — 4 = 23) и 19 (27 — 8 = 19), что не отвечает условию «в каждом последующем году было награждено больше, чем в предыдущем». Испытаем х = 3. При этом в первом году было награждено 3 колхозника, а в пятом-9. В оставшиеся три года награждено 15 колхозников. Так как 15 = 4 + 5 + 6, то в третий год было награждено 5 колхозников (3 < 4 < 5 < 6 < 9). 48. Тремя выстрелами можно выбить не меньше 9 очков (2 + 3 + 4 = 9) и не больше 28 (10 + 9 + 9 = 28). Поскольку 9-3 = 27, то, очевидно, Коля тремя последними выстрелами выбил 10, 9 и 8 очков, а Петя-4, 3, 2. Остальные очки 9, 8, 5 и 4 выбили первыми двумя выстрелами. Если Коля выбил 5 и 4 (5 + 4 = 9), а Петя 9 и 8 (9 + 8 = 17), то третьим выстрелом Коля выбил 10 (9 + 10 = 19), а Петя-2 (17 + 2 = 19). Другие варианты условию не удовлетворяют. Итак, третьим выстрелом Коля выбил 10 очков, а Петя-2. 49. Поскольку при умножении пятизначного числа на 9 получено число пятизначное, то в исходном числе крайняя слева цифра 1. Так как число, полученное после умножения данного числа на 9, оканчива- ется цифрой 1, то исходное число оканчивается цифрой 9 (9-9 = 81) 1***9 х 9^ 9***1 Так как вторая цифра после 1 может быть только 0, то 10**9 х 9. 9**01 Так как вторая цифра в исходном числе 8, то 10*89 х 9. 9 8*01 Так как третья цифра в исходном числе 9, то 1 0 9 8 9 х 9. 9 8 9 0 1 127
50. 1 9 8 5 * * | |02 ~ 9 1 8 4 7*4-9 “ 4 0 8 7 1* <-4 7 1 4 0 51. Из условия следует, что при делении на 5 исходного шестизначного числа, начинающегося цифрой 7, получаем шестизначное число, оканчиваю- щееся цифрой 7. Значит, в полученном числе крайняя слева будет цифра 1, а в исходном-крайняя справа цифра будет 5 (данное число кратно 5), Имеем I * * * * 7 • 5 — 7****5. Вторая справа цифра в числе 1 ♦ * * * 7 будет 5, так как это крайняя справа цифра исходного числа. Тогда 1***57-5 = 7***85. Получаем третью справа цифру 8, и тогда 1**857-5 = 7**285. Получаем четвертую справа цифру 2, и тогда 1*2857-5 = 7 *4285. Наконец, получаем между 1 и 2 цифру 4: 142857-5 = 714285. Итак, первоначальное число 714285. 52. Число взять слагаемым 99999 раз-это значит умножить его на 99999 или на (100 000 — 1), т. е. приписать к данному числу 5 нулей и вычесть из полу- ченного результата данное число. В данном случае имеем ПОТОП ООООО - ПОТОП = ... 285. Отсюда получаем П = 5, 0=1, Т = 7 (10 — 5 = 5, 9 — 8=1, 9 — 2 = 7). Значит, искомое число 51715. 53. Сумма трех А оканчивается на А, поэтому А = 0 или А = 5. Но, если А = 5, тогда (К + К + К + 1) не может оканчиваться на К. Следовательно, А = 0, К = 5. Так как (Ш + Ш + Ш + 1) оканчивается на А = 0, то Ш = 3. Так как К + К + К = 15, тоС=1. Имеем 5 * 3 5 0 5 6 3 5 0 5 7 3 5 0 4- 5 ♦ 3 5 0 5 6 3 5 0 5 7 3 5 0 5 ♦ 3 5 0 5 6 3 5 0 ИЛИ 5 7 3 5 0 1 * * 0 5 0 169050 172050 54. (ЕЖИК + ЕЖИК + ВЕЛОСИПЕД + 1) оканчивается цифрой 0. Зна- чит, (ЕЖИК + ЕЖИК + ВЕЛОСИПЕД) = 9 (или 19). Равенство ЕЖИК + + ЕЖИК + ВЕЛОСИПЕД = 19 невозможно. Значит, возможна сумма 9, тогда из случаев 1 + 1+ 7 = 9, 2 + 2 + 5 = 9, 3 + 3 + 3 = 9 и 4 + 4+1=9 подходит только 2 + 2 + 5 = 9. В результате ЕЖИК = 2, ЗВЕЗДА = 3, ВЕЛОСИПЕД = 5: 5 2 7 + 324 6 5 2 15 0 3 128
55. I) Л = 0 или Л = 5. При испытании Л = 5 не подходит. Значит, Л = 0; 2) У = 5; 3) Н = 2 или Н = 7. При испытании Н = 2 не подходит. Значит, Н = 7; 4) К 4. При испытании П = 8, тогда К = 3; 5) Е = 2, тогда И = 4; 6) Ш = 6. Р= 1. Имеем 628750 628750 +628750 628750 628750 3143750 56. Поскольку при умножении трехзначного числа на трехзначное в проме- жуточных вычислениях получаем два слагаемых ОЛЛО и ЧОЛ (а не три), то на основании того, как подписаны эти слагаемые, заключаем, что В = 0. При умножении ДВА на Д получаем трехзначное число ЧОЛ, поэтому Д будет 3 или 2. При умножении А на А получаем число, оканчивающееся на 0. Учитывая, что ДВА • А = ОЛЛО и что произведение числа самого на себя не может оканчиваться на 2, О = 1. Тогда А = 9 и Д = 2. В результате имеем 2 0 9 х 209 18 8 1 4 1 8 4 3 6 8 1 57. Буквой Д зашифрована цифра 0, если нет передачи единицы из разряда десятков, так как Д + Д = Д;И = 4 или И = 2, так как О + О = И, а также И < 5, так как при складывании цифр десятков нет передачи разрядной единицы в сотни. Рассмотрим случай, когда И = 4. При этом 0 = 2, а В равно 8 или 9. Если В = 9, то Н > 5. При Н = 5 имеем А = 0, но 0 = Д; при Н = 6 имеем А = 2, но О = 2; при Н = 7 имеем А = 4, но И = 4; при Н = 8 имеем А = 6. Тогда 2 0 4 8 2 0 4 8 4 0 9 6 Если В = 8, то Н < 5, и возможно лишь Н = 3. Тогда 2 0 4 3 2 0 4 3 4 0 8 6 Рассмотрим случай, когда И = 2. При этом 0=1. Если В = 4, то Н = 3 и А = 6. При этом имеем 10 2 3 10 2 3 2 0 4 6 5 Зак. 2531 Д. В. Клименченко 129
Если В = 5, то Н будет 7, 8 или 9. Тогда 1027 1028 1029 1 0 2 7 или 1 0 2 8 или 10 2 9 2054 2056 2058 Если же имеется передача единицы из разряда десятков, то Д = 9. При этом, возможны случаи: Н = 5 или Н = 7. В результате имеем 2953 2954 2957 2953 2954 2957 5906 5908 5914 2 9 5 8 2 9 5 8 5 9 16 3 9 7 1 3 9 7 1 7 9 4 2 3 9 7 6 3 9 7 6 7 9 5 2 3 9 7 8 3 9 7 8 7 9 5 6 Задача имеет 13 решений. 58. Поскольку А • А оканчивается на Е, то А 1, А 5, А / 6. Так как А • В оканчивается на В, то один из этих множителей 9, а другой 5. Возможно только А = 9, тогда В = 5. Тогда А • А = 9-9 = 81, и поэтому Е = 1. Из Д59 • Д = 1*** следует, что Д > 2, но Д < 5, поэтому Д = 3 или Д = 4. Итак х 3 5 9 х 4 5 9 359 459 3231 4131 + 1 795 ИЛИ + 2 295 1077 1836 128881 210681 Но в первом случае 4 = 1, что невозможно, так как Е = 1. Значит, Д = 4 и условию удовлетворяет лишь второе решение. Примечание. Для решения достаточно задать в полученном произведе- нии лишь крайние буквы Ч и Е, т. е. Ч ♦ ♦ ♦ * Е. В этом случае задача имела бы два решения. 59. Чтобы получить наибольший выигрыш во времени и прибыть одновре- менно, Петя и Вася должны поступить так: один из них выезжает на велосипеде, а другой выходит пешком. Проехав полпути, первый оставляет велосипед и идет дальше пешком. Второй, дойдя до велосипеда, берет его и едет на нем до конца пути. При этом каждый из них полпути шел пешком и полпути ехал на велосипеде, поэтому они прибывают в конечный пункт одновременно. (При других условиях прибыть одновременно не могут.) Если Петя и Вася будут всю дорогу идти пешком, то затратят 1 ч. Но так как при этом они опоздают на 10 мин, то с момента их выхода матч начинается через 50 мин (60 — 10 = 50). Если же полпути идти пешком, а полпути ехать на велосипеде, то будет затрачено 40 мин (60:2 + (60:2): 3 = 30 + 10 = 40). Зна- чит, мальчики прибудут за 10 мин до начала матча (50 — 40 = 10). Примечание. Велосипед можно оставлять через любые равные отрезки пути. 130
60. Изобразим схематически: 4 мин _ ч Пятачок /______________________________\ г < *'----------------=------------------------------?• Винни-Пух ' После встречи Винни-Пух прошел расстояние СВ, т. е. то, которое прошел до встречи Пятачок, в 4 раза быстрее, чем Пятачок после встречи расстояние С А. Значит, скорость передвижения Винни-Пуха больше, чем Пятачка. Расстояние АС больше, чем ВС, во столько раз, во сколько раз больше скорость Винни-Пуха, чем Пятачка. Значит, Винни-Пух прошел быстрее расстояние ВС, чем Пятачок-Л С, во столько раз, во сколько его скорость больше, чем скорость Пятачка. °ТСЮДа АС : ВС = 2 и гВвввв.Пуха •' = 2; (2 + 2 = 4) Винни-Пух был в пути 4:2 + 1 = 3 (мин), а Пятачок-3-2 = 6 (мин). 61. Когда выйдут с четными номерами, останется 50 учеников с нечетны- ми-1, 3, 5, 99. Дальше выходят с номерами 3, 7, 11, ..., после чего останется 25 учеников. Последний оставшийся номер при этом 97, так как 1 + 4-(25 — 1) = 1 + 96 = 97, т.е. остаются номера 1, 5, 9, 13, ..., 97. Дальше выходят номера 5, 13,.... При этом выйдут 12 учеников, последний из которых имеет номер 93, так как 5 + 8-(12 — 1) = 93, а останется 13. Их номера такие: 1, 9, 17, 25, 33, 41, 49, 57, 65, 73, 81, 89, 97. Поскольку последний вышел с номером 93, то остается номер 97, а выходят дальше 1, 17, 33, 49, 65, 81, 97. После этого выходят 25, 57, 89, затем-номер 41 и, наконец, 9. Остается номер 73. 62. Сумма чисел в каждом ряду равна 38 (3 + 17 + 18 = 38), поэтому между числами 3 и 16 должно быть 19, между 16 и 10-число 12 и т. д., пока не получим Найдем числа а, Ь, с. Имеем и + b + 1 + 18 + 10 = 38, откуда а + b = 9 и b + с + 16 + 2 + 9 = 38, откуда b + с = 11. Из последних равенств получаем с — а = 2. Поскольку числа 1, 2 и 3 уже вписаны, то а равно 4 или 5. Если а = 4, то7> = 5ис = 6;6 — 4 = 2. Если а = 5, то b = 4 и с = 7, но число 7 уже вписано. Значит, а = 4, b = 5, с = 6. После этого нетрудно заполнить оставшиеся круги, и в результате получим решение. 5» 131
63. Наблюдая запись цифр в ряду 24610112358134711235813471123..., за- мечаем, что начиная с шестой цифры группа цифр 1123581347 (10 цифр) повторяется. Назовем ее периодом. До первого периода имеется 5 цифр-2, 4, 6, 1, 0. Как среди этих цифр, так и среди цифр повторяющихся отсутствует цифра 9, поэтому, как бы далеко ни продолжали этот ряд, в его записи будет отсутствовать цифра 9. Определим цифру, стоящую на сотом месте. Так как до первого периода 5 цифр, то в записи первого и последующих периодов до искомой цифры будет 95 цифр (100 — 5 = 95). Но в периоде 10 цифр, поэтому полных периодов будет 9 да еще 5 цифр десятого периода. Пятой цифрой периода есть 5. Эта цифра и будет на сотом месте ряда. 64. В результате анализа записи цифр ряда 236188642483261224832612248... замечаем, что начиная с десятой цифры группа цифр 48326122 повторяется (как и в предыдущей задаче назовем ее периодом). До первого периода 9 цифр, а в периоде 8 цифр. Среди этих цифр нет 5, 7, 9л и 0. Значит, эти цифры будут отсутствовать в записи данного ряда, сколько бы его ни продолжали. Найдем цифру, стоящую на тысячном месте. Так как до периода 9 цифр, то в записи первого и последующих периодов до искомой цифры будет 991 цифра (1000 — 9 = 991). Но в периоде 8 цифр, поэтому полных периодов будет 123 да еще 7 цифр 124-го периода (991 = 8-123 + 7). Седьмой цифрой периода есть 2. Значит, на тысячном месте в ряду стоит цифра 2. 65. Подсчитаем сумму 1 + 2 + 3 + ... + и, для чего запишем ее иным способом, а именно п + (п — 1)4-. ..4-34-2+ 1. Складывая соответствующие слагаемые этих сумм (первое с первым, второе со вторым и т.д.), получим п + 1, 2 + п — 1 = п + 1 и т. д. Значит, удвоенная сумма 1 + 2 + 3 + ... + п равна (п + 1) + (п + 1) + ... + (п + 1) = (п + 1)и, а значит < ~ (л + 1)л 1 + 2 + 3 + ...+л = -— (п + 1)л --- ----------- По условию --------= ааа. Так как ааа = 111 • а и 111 =37 -3, то и (и + 1) и должно делиться на 37. Это значит или п = 37, или п + 1 = 37. Если п = 37, то 38-37 п + 1 = 38, и тогда —= 19 -37 = 703, что не удовлетворяет условию. Если 37-36 п + 1 = 37, то п = 36, и тогда —-— = 37• 18 = 666; 1 + 2 + 3 + ... + 36 = 666. Итак, п = 36, т. е. слагаемых 36. 132
Квадрат и куб числа 1. Квадраты натуральных чисел оканчиваются цифрами 0, 1,4, 9, 6, 5. Из них только 6 — 4 = 2, поэтому уменьшаемое оканчивается цифрой 6, а вычи- таемое цифрой 4. 2. Имеем , ,, 1 = Г з + 5 = & = 23 7 + 9 + 11 = 27 = З3 13+15 + 17+ 19 = 64 = 43 21 + 23 + 25 + 27 + 29 = 125 = 53 Можно заметить такую закономерность: если возвести в куб число элементов группы, то получим сумму чисел (элементов) этой группы. 3. Так как 92 = 81, 162 = 256, 63 = 216, то числа средней колонки есть етепени, левой-их основания, а числа правой-показатели. Значит, в правой колонке недостает числа 1, а в средней 125 (53 = 125). 4. Степени числа 4 могут оканчиваться либо на 6, либо на 4, поэтому три степени не могут иметь три различные цифры единиц. 5. Сумма в рублях, которую уплатил каждый турист за набор сувениров, выражается числом вида а2, где а-количество сувениров или цена одного сувенира в рублях. Поскольку квадраты натуральных чисел могут оканчиваться только цифрами 1, 4, 5, 6, 9, то возможно только 5 различных сдач, а именно 9 р., 6 р., 5 р., 4 р. и 1 р. Наибольшее возможное число туристов 5. 6. Искомое число содержит множители 23 = 8 и З2 = 9. Таким наимень- шим числом есть число 8-9 = 72. Тогда 72-2 = 144 = 122 и 72-3 = 216 = 63. Искомое число 72. 7. Если данное число делится на 3, то 425 102348 541 = 3-л, но п на 3 не делится, так как данное числб на 9 не делится. Число же 3 не является точным квадратом, поэтому и данное число не является точным квадратом. 8. Данное число делится на 3, так как сумма его цифр 123 делится на 3, но оно не делится на 9, поэтому число это не может быть точным квадратом. 9. Двузначное число, возводимое в куб, не должно превышать 22, так как 223 = 10648-число пятизначное. Полученный куб оканчивается на 9, поэтому и число, возводимое в куб, оканчивается на 9. В результате имеем 193 = 6859. 10. Из условия видно, что при возвышении двузначного числа в степень получим трехзначное число. Если принять во внимание, что даже 103 = 1000, то показатель может быть только 2, т. е. А = 2. Поскольку основание степени оканчивается той же цифрой, что и степень, то □ может быть 0, 1, 5 или 6. Условию удовлетворяет лишь □ = 6. В результате имеем 262 = 676. 133
11. И3 = 1331. 12. Так как при возведении двузначного числа в некоторую степень получили трехзначное число, то показатель может быть только 2. Значит, М = 2, тогда А = 1; Р может быть 5 или 6, но 152 = 225 не удовлетворяет условию, поэтому Р = 6. Имеем 162 = 256. 13. Так как общий урожай равен произведению урожайности на площадь участка, то четырехзначное число, выражающее общий урожай, является квадратом некоторого двузначного числа. Этот квадрат не может оканчиваться цифрами 0, 2 или 3. Значит, цифра единиц этого четырехзначного числа может быть только 5. Цифра десятков возможна только 2, тогда цифра сотен О, а тысяч-3. Таким образом, общий урожай равен 3025 ц, а урожайность с 1 га-55 ц, так как 552 = 3025. 14. Поскольку по условию кубы являются четырехзначными числами, то их основания - числа двузначные. Если куб оканчивается цифрой 8, то основа- ние оканчивается цифрой 2 (23 = 8). Основанием в этом случае может быть только 12, так как 223 = 10 648 - число пятизначное. Значит, возможно только 123 = 1728. Если куб оканчивается цифрой 9, то и основание оканчивается этой цифрой. Возможно только 193 = 6859. 15. Так как 43 = 64 и 103 = 1000, то сумма цифр возможна 5, 6, 7, 8 или 9. Из этих пяти чисел отберем путем проб те, которые удовлетворяют условию задачи: 53 = 125, 63 = 216, 73 = 343, 83 = 512, 93 = 729. Условию удовлетворяет 512 = (5 4- 1 + 2)3. 16. Поскольку квадрат числа может оканчиваться цифрами 0, 1, 4, 9, 5 или 6, то в искомом числе две последние цифры могут быть 00, 11, 44, 99, 55 или 66. Это четырехзначное число является квадратом двузначного числа, цифры которого одинаковы. Таким двузначным числом не может быть 11 или 22, так как их квадраты - трехзначные числа. Из семи других чисел нетрудно найти, что указанным требованиям удовлетворяет 88 (882 = 7744). 17. Последние три цифры искомого числа выражают число, являющееся кубом четного числа. Возможны такие два случая: 83 = 512 и 63 = 216. Но так как нет квадрата натурального числа, оканчивающегося на 2, то второй случай отпадает. Наибольшее трехзначное число, являющееся точным квадратом и оканчивающееся на 5, есть 625. Значит, искомое число 62 512. 18. Всего трехзначных чисел, являющихся точными квадратами и в записи которых цифры не повторяются, 13: 169, 196, 256, 289, 324, 361, 529, 576, 625, 729, 784, 841, 961. Из них условию удовлетворяют 729, 256, 961: 7 2 9 2 5 6 9 6 1 20. Пусть имеем двузначное число 10а 4- Ь, выражающее возраст в годах. Сложив его с обращенным числом, получим 11а 4- life = 11 (а 4- fe). Получен- ное число будет точным квадратом лишь при условии, если а 4- b = 11, т. е. если сумма цифр двузначного числа равна 11. Этому условию удовлетворяют числа 134
29, 38, 47, 56, 65, 74, 83, 92. Значит, подобное может случиться (будем оптимистами и пожелаем долголетия) в будущем еще 6 раз. 21. Так как квадрат натуральных чисел может оканчиваться цифрой 0, 1,4, 9, 6 или 5, то из данных цифр цифрой единиц четырехзначного числа может быть 5, 6 или 9 (9 получим, перевернув цифру 6). Но если квадрат оканчивается цифрой 5 или 6, то и основание оканчивается этой же цифрой, а в наборе таких цифр по одной. Значит, четырехзначное число оканчивается цифрой 9, тогда двузначное оканчивается цифрой 3 или 7. Исцытания дают число 73 (732 = 5329). 22. Среди трехзначных чисел, цифра сотен которых 1, искомых чисел нет, так как сумма цифр десятков и единиц будет числом однозначным или двузначным, а возведение этого числа в первую степень не изменит его. Среди трехзначных чисел с цифрой сотен 2 точных квадратов три: 225, 256, 289. Из них условию удовлетворяет лишь 289. Среди трехзначных чисел с цифрой сотен 3 точным кубом является одно число 343 = 73, 343 = (4 + З)3. 23. Так как квадрат вообще может оканчиваться цифрой 0, 1, 4, 5, 6 или 9, а каждая последующая цифра шестизначного числа больше предыдущей, то шестизначное число, являющееся квадратом трехзначного, может оканчивать- ся либо цифрой 6, либо 9. Если оно оканчивается на 6, то запишется 123 456. Но это число не является точным квадратом. Значит, оно оканчивается цифрой 9, а это значит, что искомое трехзначное число оканчивается цифрой 3 или 7. Крайней левой цифрой шестизначного числа не может быть 4, так как число 456789 не является точным квадратом. Значит; она меньше, чем 4. Пусть имеем 1 * * * * 9, тогда трехзначное число либо 4*7, либо 3*7. Но 407, 417, 427,437, 447 условию не удовлетворяют, а 4572 = 208 849. (Получим число с крайней левой цифрой не 1, а 2.) Будем искать среди чисел вида 3*7. Проверка показывает, что таким числом есть 367 (3672 = 134689). 24. Задачу можно решить просто, пользуясь таблицей квадратов чисел. А если нет ее под руками? Тогда поступим так. Наименьшим четырехзначным числом, являющимся точным квадратом, есть 1024 (322 = 1024), а наи- большим-9801 (992 = 9801). Значит, если длина стороны участка хм, то 32 < х < 99. Из этих двузначных чисел отберем те, квадраты которых оканчи- ваются четной цифрой (отличной от 0). Так как квадраты могут оканчиваться цифрами 1, 4, 9, 6 или 5, то искомое четырехзначное число должно оканчиваться либо цифрой 4, либо 6. Двузначные числа (длину стороны квадрата) надо искать среди чисел 32, 34, ... , 96, 98. Таких чисел всего 28. Исключим те из них, квадраты которых имеют нечетную цифру тысяч. Так как 442 = 1936, то исключим 6 чисел от 32 до 44. Так как 552 = 3025 и 632 = 3969, то исключим три числа: 56, 58, 62. Так как 702 = 4900 и 772 = 5929, то исключим числа 72, 135
74, 76. Так как 842 = 7056 и 882 = 7744, то исключим 84, 86, 88. Так как 942 = 8836 и 952 = 9025, то исключим 94, 96, 98. Остаются для испытания такие числа: 46, 48, 52, 54, 64, 66, 68, 78, 82 и 92. Из этих чисел только 78 удовлетворяет требованию задачи: 782 = 6084. Значит, длина стороны квадрата 78 м, а его площадь 6084 м2. Делимость натуральных чисел 1. Так как натуральные числа могут быть либо четными, либо нечетными, то среди трех чисел будет по крайней мере два числа одинаковой четности (четные или нечетные). Сумма двух чисел одинаковой четности всегда число четное. 2. Всего чисел до 100, делящихся на 3, 33 (100:3 = 33). Среди этих чисел имеют в своей записи цифру 3 такие числа: 3, 30, 33, 36, 39, 63, 93, т. е. 7 чисел. Значит, чисел, делящихся на 3, но не имеющих в своей записи цифру 3, 26 (33 - 7 = 26). 3. 100а 4- 106 + с — (а + 6 + с) = 99а + 9Ь = 9(11а + 6). Так как 9(11а + 6) делится на 9 при любых а и Ь, то утверждение истинное. 4. I способ. ХА ХА ХА = 105Х + 104А + 103Х + 102А + ЮХ + А = = 10Х(104 + 102 + 1) + А(104 + 102 + 1) = (104 + 102 + 1) (10Х + А) = = 10 101 (ЮХ + А). Так как 10 101 • 7, то и число ХА ХА ХА делится на 7 при любых X и А. II способ. ХАХАХА = ХА-10 101. Так как число 10101 делится на 7(10101:7 = 1443), то и ХА ХА ХА делится на 7. 5. Задуманное число делится на 7, 8, 9. Наименьшим числом, делящимся на 7, 8 и 9, есть число 7 • 8 • 9 = 504. Это число и отвечает требованию задачи. 6. Для двузначных чисел, кроме 99, справедливо утверждение: «Если число делится на 9, то сумма его цифр равна 9, и наоборот». До 99 включительно чисел, делящихся на 9, 11 (99:9 =11). Однако среди них два числа (9 и 99) не рассматриваются в данной задаче, поэтому чисел, удовлетво- ряющих условию задачи, 9 (11 —2 = 9). 7. Искомое число должно делится на 3 -7 -13 = 273, а наименьшее шести- значное число 100000 = 366-273 + 82. Если прибавить к нему 191, то получим 100 191 = 367 -273. Это и есть искомое число. 8. Пусть имеем пятизначное число а. После того как к нему приписали такое же число, получили 100001а. Так как число 100001 делится на 11 (100001 :11 = 9091), то и 100001 а делится на 11. 9. I способ. Пусть имеем число abc, тогда по условию получим abccba abc cba = 105а + Ю46 + 103с + 102с + 106 + а = а(105 + 1) 4- 106(103 + 1) + 136
4- 102с(10 4- 1) = 100001а 4- 106-1001 + 102с* 11. Так как каждое слагаемое делится на 11, то и сумма разделится на 11, а значит, и abc cba • 11. II способ. Если данное число 100а 4-106 4-с, то полученное (100а 4- 4- 106 4- с) -1000 4- 100с 4- 1.06 4- а = 100001а 4- 100106 4- 1100с. Так как 100001 делится на 11, 10010 делится на 11 и 1100 делится на 11, то полученное число разделится на 11. 10. Каждое из искомых чисел должно оканчиваться четной цифрой (так как делится на 6), поэтому цифра 7 не может быть крайней справа. Чтобы число было наибольшим (наименьшим), цифра сотен должна быть наибольшей (наименьшей) из возможных. Принимая во внимание условие делимости на 3, получаем искомые числа 978 и 174. 11. ‘Имеем число 100а 4- 106 4- а = 10(а 4- 6) 4- 91а. Так как (а 4- 6) по усло- вию делится на 7 и 91 делится на 7( 91:7 = 13), то и число 100а 4- 106 4- а делится на 7. 12. При делении на 7 всех возможных различных остатков будет 7, а именно 0, 1,2, 3, 4, 5, 6. Так как по условию имеется 8 чисел, то найдутся по крайней мере два числа, дающие одинаковые остатки при делении на 7. Поэтому их разность будет делиться на 7. 13. Известно, что за 9 книг заплатили 113 хк. Поскольку это число должно делиться на 9, то 1 4- 1 4- 3 4- х = 9, откуда х = 4. Значит, одна книга стоит 126 к. = 1 р. 26 к., а 17 книг- 126-17 = 2142 к. = 21 р. 42 к. 14. а) Четных чисел от 1 до 99 всего 49 (98:2 = 49). Но среди них есть такие, которые делятся на 3 (такие числа делятся на 6). Их всего 96:6 = 16. Значит, чисел, делящихся на 2, но не делящихся на 3, 49 — 16 = 33. б) Чисел, делящихся на 2, всего 49, а делящихся на 3, всего 33 (99:3 = 33), чисел, делящихся на 6, всего 16. Значит, чисел, делящихся на 2 или на 3, 49 4- 33 - 16 = 66. в) Чисел, не делящихся ни на 2, ни на 3, 99 — 66 = 33. 15. Искомое число делится на 5, поэтому оно оканчивается цифрой 5 или цифрой 0. Оно также делится на 9, поэтому сумма его цифр делится на 9. Такими числами есть 6390 и 1395. Так как 6390 > 5000, а 1395 < 5000, то искомое расстояние 1395 км. 16. Если не хватало трех тарелок до полного числа десятков, то это значит, что, как и при счете дюжинами, оставалось 7 тарелок. Значит, число тарелок без семи делится без остатка на 10 и на 12, т. е. на 60. Среди чисел, меньших 600 и больших 500, только одно число 540 делится на 60, значит, тарелок было 540 4- 7 = 547. 17. Если число делится на 72, то оно делится на 9 и на 8. Поскольку 4 4- 2 + 4 = 10, то должно а + 6 = 8 или а 4- 6 = 17. Если а 4- 6 = 17, то или а = 9 и 6 = 8, или а = 8 и 6 = 9. Но в обоих случаях получаемые числа не делятся на 8. Значит, а 4- 6 = 8. Исходя из делимости на 8 возможно лишь а = 8 и 6 = 0, или а = 0 и 6 = 8. В самом деле, 42 840: 72 = 595, 42 048: 72 = 584. Имеем решения: 1) а = 8, 6 = 0; 2) а = 0, 6 = 8. 18. Искомое число делится на 45, а значит, делится на 9 и на 5. Цифра единиц может быть 0 и 5. Если 72 * 30, то цифра сотен 6, так как 74-24-34-0=12 и 18 делится на 9. Если 72 ♦ 35, то 74-24-34-5= 17, поэтому цифра сотен 1. Итак, 72 630 или 72 135. 19. Крайние справа две цифры есть нули, так как среди множителей имеется число 10 да еще 2 • 5 = 10. Произведение чисел от 1 до 11 делится на 9, 137
поэтому сумма цифр результата должна делиться на 9. Сумма известных цифр 3 + 9 + 9 + 6 + 8 = 35. Число, большее 35 и ближайшее ему делящееся на 9, есть 36. Значит, неизвестная цифра 1. 20. Искомое число должно делиться на 72, а значит, на 9 и на 8; 1 + 2 + ... + 9 = 45, которое делится на 9. Для того чтобы число делилось на 8, оно должно оканчиваться такими тремя цифрами, которые образуют трехзначное число, делящееся на 8. Учитывая все это, цифры необходимо расставить так, чтобы наименьшие стояли левее. Таким числом есть 123457968. 21. Так как искомое число делится на 36, то оно должно делиться на 4 и на 9. В этом числе две последние цифры должны выражать наибольшее двузначное число, делящееся на 4. Таким числом есть 96. Остальные цифры необходимо записать от цифры 9 влево в порядке убывания, но так как крайней слева цифрой не может быть 0, то крайняя слева цифра 1. Так как сумма всех однозначных чисел 45, то искомое число делится на 9. Искомое число 1023457 896. 22. Так как 31 число нечетное, то полученные суммы по 1 р., по 3 р. и по 5 р. должны быть или все нечетные, или 2 четные и 1 нечетная. Значит, числа купюр по 1 р., 3 р., 5 р. или все нечетные, или только одно нечетное. В обоих этих случаях общее число всех купюр должно быть числом нечетным, а 10-число четное. 23. Если коробка карандашей стоит х к., а линейка-у к., то карандаши и линейки стоят (9х + 15>’) к. С другой стороны, они стоят 22 р. 85 к. - -(6 + 12) р. = 4 р. 85 к. Значит, 9х + 15+ = 485. Полученное равенство не может быть истинным ни при каких натуральных значениях х и у, так как левая часть его делится на 3, а правая не делится (4 + 8 + 5=17). 24. Так как обезьяны собрали орехов поровну и поровну бросили, то принесли они поровну. Число 35 делится на 5, поэтому имеем 35 = 5-7. Может представиться 2 случая: 1) Обезьян было 5, принесли по 7 орехов, бросили по 4 ореха, а значит, каждая собрала 7 + 4=11. 2) Обезьян было 7, принесли по 5 орехов, бросили по 6 орехов, а значит, каждая собрала 5 + 6 = 11. 25. Стоимость книги-число копеек-делится на 3 и на 5, а значит, на 15. На каждые три монеты Иры Марина должна платить пятью монетами. Так как 3 + 5 = 8 < 10, а 8 • 2 = 16 > 10, но 16 < 20, то они внесли в кассу 16 монет: Ира-6 монет, а Марина-10. Значит, книга стоит 30 к. (5-6 = 3-10 = 30). 26. Любое число либо делится на 3, тогда остаток 0, либо не делится на 3, тогда остаток 1 или 2. Поэтому оно может быть представлено одним из трех видов: 3/7, 3/7 + 1, Зп + 2. Если имеем число вида 3/7, то оно представляется целым числом троек. Если 3/7 + 1 > 8, то и > 3, и тогда Зп + 1 можно записать в виде 3 (п — 3) + 10, а это 2 пятерки и несколько (или ни одной) тройки. Если имеем число вида Зп + 2 > 8, то Зп + 2 = 3(п — 1) + 5, и, значит, оно пред- ставляется пятеркой и несколькими тройками. Итак, любую сумму денег, большую чем 8 к., можно разменять по 5 к. или 3 к. 27. Если каждое из двух трехзначных чисел не делится на 37 (в их записи не все цифры одинаковы), а сумма их делится на 37, то, приписав одно из них к другому, получим шестизначное число, делящееся на 37. Докажем это. Пусть а и h-трехзначные числа, удовлетворяющие условию, т.е. а не делится на 37 и b не делится на 37, но а + b делится на 37. Рассмотрим шестизначное число 1000я + Ь. Представим его так: 999а + (а + Ь); 999 делится 138
на 37 (999:37 = 27) и (а + b) делится на 37 (по условию), поэтому и число 999а + (а + Ь) делится на 37, а это значит число 1000а + b делится на 37. Аналогично можно убедиться, что и число 10006 + а также делится на 37. 28. Пусть из 9 порученных после первого деления участков разделили п, тогда получится всего п • 9 + (9 — п) = 8п + 9 участков. И так после каждого деления. Пусть после некоторого деления получилось 1986 участков. Тогда 8л + 9 = 1986, откуда 8л = 1977. Так как число 1977 не делится на 8, то ни при каком натуральном п это равенство не может быть верным, а это значит, что не может получиться 1986. 29. Всего чисел 333, из них на 3 делится 111 чисел (333:3 = 111); среди этих чисел 15 таких*, которые делятся на 7 (333 :(3-7) = 333:21 = 15 (ост. 18)). Значит, делящихся на 3, но не делящихся на 7 всего 96 чисел (111 — 15 = 96). Делящихся на 7 всего 47 чисел (333:7 = 47 (ост. 4)). Среди этих 47 чисел 15 делится на 3, поэтому делится на 7, но не делится на 3 всего 32 числа (47 — 15 = 32). Итак, надо вычеркнуть (исключить) 96 + 32 = 128 чисел, тогда останется 333 — 128 = 205 чисел. Получив 215 чисел, Таня допустила ошибку. 30. Наименьшее число, отличное от 0, делящееся на 17, есть 17, следующее за ним 34. Сумма цифр одного из искомых чисел должна равняться 17, тогда другого-34. Чтобы число было наименьшим, оно должно быть возможно меньшей значности, а значит, цифры в его записи наибольшими из возмож- ных. Если возьмем, например, 7999, то 9 • 3 + 7 = 34, но следующее за ним число 8000 имеет сумму цифр 8. Рассмотрим число 8899, большее чем 7999. Сумма его цифр 8 • 2 + 9 • 2 = 34. Следующее за ним число 8900 имеет сумму цифр 8 + 9 = 17, что удовлетворяет условию. 31. Подсчитаем количество чисел, делящихся на 2, или на 3, или на 5. На 2 делится 250 чисел (500:2 = 250). На 3 делится 166 чисел (500:3 = 166 (ост. 2)). При этом дважды учтены числа, делящиеся на 2 и на 3, т. е. на 6. Таких чисел 83 (500:6 = 83 (ост. 2)). Значит, чисел, делящихся на 2 или на 3, 250 + 166 — 83 = = 333. На 5 делится 100 чисел (500:5 = 100). Однако при этом учтены числа, делящиеся на 5 и на 2, т.е. на 10. Таких чисел 50(300:10 = 50). Так как они вошли в число чисел, делящихся на 2, то здесь их надо исключить: 100 — 50 = 50. Среди этих чисел учтены те, которые делятся на 5 и на 3, т. е. на 15. Таких чисел 33. Они учтены в числе тех, которые делятся на 3, здесь их надо исключить: 50 — 33 = 17. Однако, исключая числа, делящиеся на 2 и на 5, а также на 3 и на 5, мы дважды исключили числа, делящиеся на 2, на 3 и на 5, т.е. на 30. Таких чисел 16. Таким образом, всего чисел до 500, делящихся на 2, или на 3, или на 5, будет 333 + 17 + 16 = 366, а значит, чисел, не делящихся ни на 2, ни на 3, ни на 5, будет 500 — 366 = 134. 32. Так как данное число не делится на 3, то либо остатки, получаемые при делении на 3 его последней цифры и числа, изображенного остальными цифрами (назовем его «укороченным» числом), одинаковы, либо «укороченное» число делится на 3 без остатка. Если «укороченное» число делится на 3, а последняя цифра данного числа согласно условию при делении на 3 дает остатки 1 или 2, то достаточно эту цифру дважды приписать к данному числу, чтобы получить число, делящееся на 3, так как сумма остатков в этом случае будет 1 + 1 + 1 = 3 или 2 + 2 + 2 = 6. Если же «укороченное» число при делении на 3 дает остаток 1 или 2, то и последняя цифра при делении на 3 дает такие же соответственно остатки. В этом случае к данному числу достаточно приписать один раз его последнюю цифру, чтобы получить число, делящееся 139
на 3, так как в этом случае сумма остатков будет 14-1 + 1=3 или 2 + 2 + 2 = 6. Таким образом, к данному числу достаточно приписать послед- нюю цифру один или два раза, чтобы получить число, делящееся на 3. 33. Поскольку остаток 8 и делитель число однозначное, то он равен 9. Остаток при делении числа аа5 на 9 такой же, как при делении на 9 суммы цифр этого числа, поэтому 2а = 3 или 2а = 12, откуда а = 6. Значит, делимое 665, делитель 9 и частное 73 (665 = 73-9 + 8). 34. Для нумерации страниц, пронумерованных однозначными числами, потребуется 9 цифр, двузначными-2-90 = 180 цифр, а всего 189 цифр. Остав- шиеся 301 — 189 = 112 цифр должны использоваться для нумерации страниц с трехзначными номерами, но 112 на 3 не делится. 35. Так как 135135 = 135-1001 и 1001 =7-11-13, а 135 = 5-3-9, то имеем 3-5-7-9-11 • 13 = 135135. 36. Числа п и (и + 2) одинаковой четности, но так как их произведение четное (оканчивается цифрой 4), то эти числа четные. Значит, произведение п (п + 2) делится на 4. Но на 4 делятся числа, у которых две последние цифры обозначают число, делящееся на 4. Такими числами есть 24, 44, 64, 84. Значит, предпоследняя цифра может быть 2, 4, 6 или 8. 37. В данной задаче делимое восьмизначное число. Известно, что на 7 делятся числа вида abc abc, поэтому последние или первые две цифры восьмизначного числа должны составить число 21 (число, составленное из двоек и единиц и кратное семи). Из данных цифр можно составить числа: 12212221, 21221221, 22122121, 21 122122. 21212212. Имеем 12212221:7 = 1 744603, 21 221 221:7 = 3031603, 22122121:7 = 3 160303, 21 122122:7 = 3017446, 21 212212:7 = 3030316. 38. Наименьшее натуральное число, сумма цифр которого 63, это число, записанное семью девятками (9 • 7 = 63). Но число 9 999 999 на 7 не делится, так как на 7 делится 9999990, а 9 на 7 не делится. Заменим 0 числом 63. Но число 99999963 не наименьшее. Чтобы получить искомое число, переставим 63 в начало числа и получим 63999999. Простые и составные числа. Разложение чисел на множители 1. Простым испытанием находим, что условию удовлетворяет число 5. Действительно, 5 + 2 = 7, 5 + 6=11, 5 + 8=13, 5 + 12=17, 5 + 14 = 19. 2. Так как произведение простых чисел оканчивается на 0, то среди множителей имеются 2 и 5. Поскольку множители простые последовательные числа, то имеем 2 • 3 • 5 • 7 = 210. 140
3. Так как 17:4 = 4 (ост. 1), то простые числа надо искать среди чисел, близких четырем. Такими являются 2, 3, 5 и.7. Именно или удовлетворяют условию 2 + 3 + 5 + 7=17, и тогда искомое произведение 2-3-5-7 = 210. 4. Простые неоднозначные числа нечетные и не оканчиваются на 5, т.е. они могут оканчиваться цифрами 1,3,7 или 9. Произведение двух таких чисел оканчивается одной из этих четырех цифр, так как 1 • 1 = 1, 3 • 3 = 9, 7 • 7 = 49, 9-9 = 81, 3 1 = 3, 7-1 = 7, 9-1 = 9, 3-7 = 21, 3-9 = 27, 7-9 = 63. 5. Число 19861986 оканчивается цифрой 6, а число 1990*"° оканчивается цифрой 0, поэтому сумма их оканчивается цифрой 6, а такое число четное, отличное от 2, значит, оно составное. 6. 19911991 оканчивается цифрой 1, поэтому 19911991 + 1 оканчивается цифрой 2, а число 19911991 — 1 цифрой 0. Каждое из таких чисел составное. 7. Олег получил число 123456789, сумма цифр которого (1 + 9) + (2 + 8) + (3 + 7) + (4 + 6) + 5 = 45, но 45 • 9, поэтому данное число также кратно 9, и, значит, оно составное. Как бы ни изменяли порядок цифр в записанном числе, сумма его цифр не изменяется, поэтому все получаемые числа составные. 8. Если между цифрами ставить только знаки « + », то после выполнения действий получим 45. Простых двузначных чисел, меньших чем 45, будет всего 10. Это числа 11, 13, 17, 19, 23, 29, 31, 37, 41, 43. Их можно получить так: 1) 1+2 + 3 + 4 + 5 + 6-8-9=11; 2) 1+2 + 3 + 4 + 5 + 6- 7 + 8- 9=13; 3) 1+2 + 3 + 4- 5 + 6 + 7 + 8- 9=17; 4) 1 + 2 + 3- 4 + 5 + 6 + 7 + 8- 9=19; 5)1-2 + 3 + 4 + 5 + 6 + 7 + 8- 9 = 23; 6) 1+2 + 3 + 4 + 5 + 6 + 7-8 + 9 = 29; 7) 1+2 + 3 + 4 + 5 + 6- 7 + 8 + 9 = 31; 8) 1+2 + 3- 4 + 5 + 6 + 7 + 8 + 9 = 37; 9) 1-2 + 3 + 4 + 5 + 6 + 7 + 8 + 9 = 41; 10) 2-1+3 + 4 + 5 + 6 + 7 + 8 + 9 = 43. 9. 2-3-5-7 = 210 2-3-5-11 =330 2-3-5-13 = 390 2-3-517 = 510 2-3-5-19 = 570 2-3-5-23 = 690 2-3-5-29 = 870 2-3-5-31 =930 2-3-7-11 =462 2-3-7-13 = 546 2-3-7-17 = 714 2-3-7-19 = 798 2-3-7-23 = 966 2-5-7-11 = 770 2-5-7-13 = 910 2-3-1113 = 858 10494 141
10. Записав указанные простые числа, получим '2357111317192329. Поскольку это число имеет 16 знаков, то следует оставить 8. а) Чтобы получить наименьшее число, надо оставить слева наименьшие однозначные числа. Такими есть единицы. Значит, надо убрать 2, 3, 5, 7, 3, 7,9, а также восьмую цифру 3. В результате получим 11 111 229. б) Чтобы получить наибольшее число, надо оставить слева наибольшие из возможных однозначные числа. Поскольку до цифры 9 имеется 11 цифр, то крайней левой цифрой не может быть из оставшихся цифра 9. Поэтому исключаем цифры 2, 3, 5, 1, 1. I. 3. 1, а остается число 77192329. 11. Будем получать указанные суммы: 1 +3 = 4 = 2-2 1+3 + 5 = 9 = 3-3 1 +3 + 5 + 7= 16 = 4-4 1 +3 + 5 + 7 + 9 = 25 = 5-5 Итак, получить простое число нельзя. 12. Числа, найденные Леной, не могут содержать четных цифр и цифры 5. Они могут быть записаны лишь при помощи цифр 1, 3, 7, 9. Имеем числа: 1) 31 и 13, но 31-13 = 18 не есть квадрат; 2) 71 и 17, но 71-17 = 54 не есть квадрат; 3) 97 и 79, но 97-79 = 18 не есть квадрат; 4) 73 и 37, и 73-37 = 36 = 62. Значит, Лена нашла числа 73 и 37. 13. Так как обращенное число четное, то первая цифра номера 2. (Это единственное четное простое число.) Из точных квадратов 16, 25, 36, 49, 64, 81 только одно число 16 дает обращенное простое число 61. Значит, номер телефона руководителя 2-61-16. 14. Поскольку сумма двух слагаемых есть число нечётное (оканчивается цифрой 9), то одно слагаемое четное, а другое нечетное. Так как четности квадрата и его основания одинаковы, то одно из двух искомых простых чисел четное, а другое-нечетное. Но четное простое число только одно-2, поэтому его квадрат 4, а значит, квадрат второго простого числа оканчивается циф- рой 5. Но если квадрат оканчивается цифрой 5, то и его основание тоже оканчивается цифрой 5. Среди простых чисел, оканчивающихся цифрой 5, нет чисел, отличных от самого числа 5. Итак, задача имеет одно решение: 2 и 5. 15. Из трехзначных чисел только 5 являются точными кубами: 53 = 125, 63 = 216, 73 = 343, 83 = 512, 93 = 729. Условию удовлетворяет лишь число 125, так как только 521-простое. 16. Найдем сумму всех очков домино. Кости домино выглядят так: 00 11 22 33 44 55 66 01 12 23 34 45 56 12 02 13 24 35 46 21 03 14 25 36 27 04 15 26 30 05 16 30 06 27 21 142
Сумма всех очков: 21 + 27 + 30 + 30 + 27 + 21 + 12 = 168. Найдем среднее число очков в кучках: 168:4 = 42. Тогда искомые простые числа 37, 41, 43, 47. Действительно, 37 + 41 + 43 + 47 = 168. Разложить кости домино в такие кучки можно разными способами. 17. Если сумма двух чисел оканчивается нечетной цифрой, то одно слагае- мое четное, а другое-нечетное. Так как среди четных чисел простым является только число 2, то одним слагаемым есть 2. Если сумма оканчивается цифрой 1, то второе слагаемое должно оканчиваться цифрой 9. Такой случай возмо- жен, например 19 + 2 = 21. Если сумма оканчивается цифрой 3, то второе слагаемое должно оканчиваться цифрой 1. Такой случай тоже возможен, например 2 + 11 = 13. Если сумма оканчивается цифрой 5, то второе слагае- мое должно оканчиваться цифрой 3. Такой случай возможен, например 2 + 13 = 15. Если сумма оканчивается цифрой 7, то второе слагаемое должно оканчиваться цифрой 5. Среди чисел неоднозначных, оканчивающихся цифрой 5, простых нет, поэтому сумма не может оканчиваться цифрой 7. Если сумма оканчивается цифрой 9, то второе слагаемое должно оканчиваться цифрой 7. Такой случай возможен, например 2+17=19. Итак, сумма может оканчи- ваться цифрой 1, 3, 5 или 9. 18. Среди цифр не должно быть ни одной четной или пятерки. Значит, для написания трехзначных чисел можно использовать 1, 3, 7, 9. а) Из чисел 137, 173, 317, 371, 713, и 731 три последних числа составные. Так, 371 = 53-7. б) Из чисел 139, 193, 319, ... число 319 составное, так как 319 = 29 -11. в) Из чисел 179, 197, 791, ... число 791 составное, так как 791 = 113-7. г) Из чисел 379, 397, 739, 793, ... число 793 составное, так как 793 = 61-13. Иных трехзначных чисел, пользуясь данными цифрами, составить нельзя. Значит, нет таких трех цифр, чтобы все трехзначные числа, составленные из них, были простыми, т.е. задача не имеет решения. 19. Сумма всех простых чисел от 3 до 41 включительно равна 236. Значит, сумма чисел на внутренней (и внешней) окружности равна: 236:2 = 118, а сумма чисел в вершинах каждого треугольника равна: 236:4 = 59. В кружках внутренней окружности, очевидно, следует записать большие из данных чисел. Но 41 +37 + 31 +29= 138 > 118. Заменим 31 на 11,тогда 41 + 37 +11 +29 = = 118. Если в одном из кружков внутренней окружности запишем 41, то два другие числа у вершин треугольника на внешней окружности 5 и 13; если 11, то два другие-31 и 17; если 29, то-23 и 7; наконец, 37, 3 и 19. Имеем одно из возможных решений, представленное на рисунке 12. 20. Все натуральные числа можно разделить на 3 группы: 1) делящиеся на 3, т.е. вида 3/с; 2) при делении на 3 дающие остаток 1, т.е. вида Зк + 1; 3) при делении на 3 дающие остаток 2, т. е. вида Зк + 2. Если р = Зк, то оно простое только при к = 1; тогда р + 10 = 13 и р + 14 = 17. Еслир = Зк + 1, тор + 14 = Зк + 15 = 3(к + 5)-составное. Еслир = Зк + 2, то р + 10 = Зк + 12 = 3(к + 4)-составное. Имеем решение: р = 3. 21. Число р можно представить одним из трех видов: Зк, Зк + 1 или Зк + 2. При р = Зк + 1 имеем: 2р + 1 = 2(ЗЛ + 1) + 1 = 6к + 3 = 3(2к + 1)-число составное. При р = Зк + 2 имеем: 4р + 1 = 12Л + 9 = 3(4/с + 3)-составное. 143
31 Рис. 12 Если р = 3/с, то только при к = 1 число р будет простым. Это число р = 3. Тогда 2р+1=7и4р+1 = 13 тоже числа простые. Значит, при р = 3 все три числа простые. 22. Допустим, что такие числа имеются. Если меньшее из них 2п — 1 (л > 2), то следующее за ним 2п + 1, а наибольшее 2п + 3. Возможны такие три слу- чая: а) п = Зк, тогда 2и + 3 = 6/с + 3 = 3(2/с + 1) не является простым числом; б) п = Зк + 1, тогда 2и + 1 = 6Л + 3, т. е. составное; в) л = 3/с + 2, тогда 2л — 1 = 6/с + 3, т. е. составное. Значит, допущение о том, что имеются еще три последовательных нечетных простых числа, отличных от 3, 5 и 7, не оправдалось. 23. Так как искомое число р должно быть суммой двух простых чисел, то р > 2. Все такие простые числа нечетные. Чтобы получить при сложении и вычитании двух чисел нечетные числа, один из компонентов должен быть числом нечетным, а другой-четным. Так как единственным четным простым числом есть 2, то р = pt + 2 и р = р2 — 2. Значит, простые числа р, рг и р2 последовательные нечетные, а такими есть только 3, 5, 7; р = 5; 5 = 3 + 2 и 5 = 7 - 2. 24. Надо выяснить, существуют ли такие числа. Если а-b делится на простое число р, то по крайней мере одно из чисел а или Ь должно делиться на р. Но если, например, а делится на р, то из делимости суммы вытекает, что и b делится на р. Аналогично, если делится Ь, то делится и а. Значит, если делится на простое число произведение и сумма двух чисел,1 то каждое из них делится на это простое число. Разумеется, сколько бы ни пытался Миша найти нужные ему числа, он их не найдет, так как таких чисел нет. 25. Пусть искомые числа а, b и с, тогда abc = 3(а + b + с), откуда а + b + с = abc:3. Равенство возможно при условии, если одно из чисел равно 3. Пусть а = 3, тогда 3 + b + с = Ьс. Из двух простых чисел b и с по крайней 144
мере одно нечетное. Пусть b — 2п + 1, тогда 3 + 2п + 1 + с = 2сп + с, откуда и + 2 = сп и и = 2: (с — 1). Последнее равенство справедливо при с = 2, при этом b = 5. Значит, искомые числа 2, 3, 5, и тогда 2 • 3 • 5 = 3 • (2 + 3 + 5). 26. Допустим, что п + 1 число составное, тогда п+\—а-b и а<п+1, b <п + 1. Поскольку 1-2-3 ... п есть произведение всех чисел, меньших-и + 1, то среди множителей найдутся числа, равные а и Ь. Значит, это произведение разделится на и + 1. По условию же оно не делится на л + 1, значит, л + 1 не может быть числом составным. 27. Так как 123 = 41 • 3 и числа 41 и 3 простые, то наиболее правдоподобно, что мальчиков было 41 и каждый из них изготовил 3 стульчика. 28. Поскольку 737 = 67-11 и числа 67 и 11 простые, то вагонов, очевидно, было 11, и в каждом вагоне 67 туристов. 29. ЗООЗО = 2-3-5-7-11 • 13. 2 + 3 + 5 + 7 + 11 +13 = 41. Число 41 простое. 30. Число 6552 можно представить таким разложением: 6552 = 23 • З2 • 7 • 13, или 6552 = 8-9-7-13. Так как в десятичной системе счисления нет цифры 13, то утверждение Мюнхгаузена ошибочно. 31. Пусть имеем число ааа. Его можно записать ааа = 100а + 10а + а = = 111-а = 37-3-а. 32. Так как 1989 = 3-3-13-17, т. е. представляется в виде произведения четырех простых чисел, то задача имеет решение: 3 3 13 17 3 3 17 13 13 17 3 3 17 13 3 3 33. Так как 91 = 9, 92 = 81, 93 = 729, то, очевидно, нечетная степень девяти оканчивается цифрой 9, а четная-цифрой 1. Рассмотрим степени числа 8: 81 = 8; 82 = 64-оканчивается цифрой 4; 83 оканчивается цифрой 2; 84-цифрой 6; 85-цифрой 8 и т.д. Значит, имеем повторяющуюся группу цифр 8, 4, 2 и 6, на которые оканчиваются степени числа 8. Так как 1980 делится на 4, то 81980 оканчивается цифрой 6, а значит, данная разность оканчивается цифрой 5. Следовательно, она делится на 5, но не делится на 10. 34. Среди множителей числа 1 • 2 • 3 • ... • 50 на 5 делится 10 чисел (50:5= 10), из них на 52 = 25 делится 2 числа. Значит, произведение чисел от 1 до 50 в разложении на простые множители будет содержать 12 пятерок, двоек же будет, безусловно, больше. Так как 5-2 = 16, то (5-2)12 = 1012, а'это значит, что данное произведение оканчивается двенадцатью нулями. 35. Пусть имеем abb — Ю0а + 116. Преобразуем эту сумму 100а + 116 = = (98а + 76) + (2а + 46 J = 7(14а + Ь) + 2(а + 26). По условию а + 26 = 7к (так как делится на 7). В результате получим abb = 7-(14а + 6 + 2к). 145
36. Пусть одно число (7а + к), а другое (1Ь + к). Тогда шестизначное число будет 1000 (7а + к) + (lb + к) или 1000(76 + к) + (7а + к). Рассмотрим одно из них: 1000(7а + к) + (76 + к) = 7(1000а + 6) + 1001k. Так как 1001 = 7-11 • 13, то шестизначное число представляется так: 7 (1000а + 6 + 11 • 13k). Аналогично можно показать это же и для другого шестизначного числа. 37. Так как aaaaaa = 1001 • 11 la — 7 • 11 • 13 • 3 • 37a, то искомая сумма равна: 7 + 11 + 13 + 3 + 37 + т = 71 + т, где т-сумма различных простых делите- лей однозначного числа а, отличных от 3 и 7. При а = 1 имеем т = 0, так как число 1 простых делителей не имеет. При а = 3иа = 9не появится делителей, отличных от уже учтенных. При а = 6 простыми делителями будут 2 и 3, а при а = 8 простым делителем будет 2. Значит, необходимо рассмотреть только a = 2 и a = 5 при т1 — 2 и т2 = 5. Таким образом, возможные суммы различных делителей 71, 71 + 2 = 73 и 71 + 5 = 76. 38. Пусть имеем a1a2a3a4a5a6 Запишем это число в виде суммы разрядных слагаемых и преобразуем: 100000a! + 10000a2 + 1000a3 + 100a4 + 10a5 + a6 = = (100a! 4- 100a4) + (10a2 + 10a5) + (a3 + a6) + (99900aj + 9990a2 + 999a3) = = 100(at + a4) + 10(a2 + a5) + (a3 + a6) + 999(100a, + 10a2 + a3). Так как 999 • 37 и при at + a4 = a2 + a5 = a3 + a6 = a имеем (100a + 10a + a) • 37, то данное шестизначное число в разложении на простые множители имеет множитель 37. (На этом основан «фокус» приписывания к трехзначному числу некоторого трехзначного числа так, чтобы полученное шестизначное делилось на 37.) 39. Искомое число должно оканчиваться цифрой 0, тогда при делении его на 1980 производим сокращение на 10. Полученное девятизначное число должно делиться на 198. Так как 198 = 2-9-11, то это девятизначное число должно делиться на 2, 9 и 11. Значит, оно должно оканчиваться четной цифрой, иметь сумму цифр, делящуюся на 9, и знакопеременную сумму цифр, делящуюся на 11. Так как 1+2 + 3 + 4 + 5 + 6 + 7 + 8 + 9 = 45 и 45;9, то действительно такое девятизначное число делится на 9. Цифры в нем рас- ставить нельзя так, чтобы их знакопеременная сумма равнялась нулю, так как 45 не делится на 2. Поэтому цифры надо расставить так, чтобы их знако- переменная сумма равнялась И, 45 — 11 = 34, 34:2= 17. Таким числом, например, является число 238471596. Тогда число 2384715960 делится без остатка на 1980. 40. Так как 23 = 8, З3 = 27, 43 = 64, 53 = 125, то, очевидно, капитану 27 лет или 64 года. Если капитану 64 года, то из числа 4752862 следует вычесть 4 и затем полученное число разложить на 6 множителей (4752862 — 4 = = 4752858; 4752858 = 2-3-11-23-31 • 101). Из этого следует, что наиболее правдоподобным может быть такой ответ: пароход имеет 2 винта, 3 трубы, на его борту 101 человек, он отправился в рейс 23 ноября 1931 г. Проверка показывает, что возраст капитана 27 лет условию не удовлетворяет. 41. Пусть искомое число аб, тогда трехзначное аоб. По условию ab-n = аоб 146
или (10а 4- Ь)-п = 100а 4- Ь, откуда 10а(10 — и) = b(n — 1). Значит, 1 < п < 10. Левая часть равенства кратна пяти, значит, должна быть кратной пяти и правая. Если (п — 1) • 5, то п = 6 и тогда 2а • 4 = Ь. Последнее равенство возможно только при а = 1 и b = 8. Имеем 18-6 = 108, что удовлетворяет условию. Если b • 5, то Ь = 5 и тогда 2а(10 — «) = « — 1. Очевидно, п — 1 > 10 — и, откуда п > 5. Так как левая часть равенства кратна двум, то (и — 1) • 2, поэтому п нечетное. Остается, что п = 1 или п = 9. При п — 7 имеем 2а • 3 = 6, а = 1. Получаем 15-7 = 105, что удовлетворяет условию. При и = 9 имеем 2а = 8, а = 4. Получаем 45 • 9 = 405, что удовлетворяет условию. Таким образом, имеется 3 числа: 15, 18 и 45, которые обладают указанным свойством. Наибольший общий делитель и наименьшее общее кратное 1. Так как 3000 = 23 • 3 53, то наибольший общий делитель таких трех чисел может принимать лишь такие значения: 1, 2, 5 или 10. Значит, Вера права. 2. Пусть имеем 3 числа: п, п 4- 1, п 4- 2.. Если п 4- 1 четное, то НОК этих чисел равно их произведению. Например, НОК (3,4,5) = 60. Если «4-1 нечетное, то « и п + 2 четные, и тогда НОК таких чисел равен их полупроиз- и(п 4- 1)(л 4- 2) ведению, т.е.------------. 4-5-6 Например, НОК (4, 5, 6) = = 60. 3. Искомое число при делении на 2, 3 и 4 каждый раз. дает остаток 1, но кратно 5. НОК (2, 3, 4) = 3-4 = 12, но число 12 4- 1 = 13 не кратно 5. А вот число 12-2 + 1 = 25 и есть наименьшее из чисел, удовлетворяющих условию. Так как по условию рядов меньше, чем 10, то лягушат было 25 (5 рядов). 4. Число килограммов яблок в каждом ящике является наибольшим общим делителем чисел 560, 595 и 735. Так как 560 = 24-5-7, 595 = 5-7-17 и 735 = 3 • 5 • 72, то НОД (560, 595, 735) = 35. Значит, в каждый ящик укла- дывали 35 кг яблок. Тогда первому отряду потребовалось 16 ящиков (560:35 = 16), второму-17(595:35 = 17) и третьему-21 (735:35 = 21). 5. Число солдат, ехавших в каждом вагоне, является наибольшим общим делителем чисел 462, 546 и 630. НОД (462, 546, 630) = 42. Значит, в первом эшелоне было 11 вагонов (462:42 = 11), во втором 13 (546:42 = 13), в третьем -15 (630:42 = 15). 6. На искомый делитель делятся числа 96(100 — 4 = 96) и 72(90 — 18 = 72). Этот делитель будет общим для чисел 96 и 72. Так как остатки при делении 147
равны 4 и 18, то делитель больше, чем 18. Таким числом является 24, так как НОД (96, 72) = 24. 7. Искомое число есть общий делитель чисел 24 696 (24 850 — 154 = 24 696) и 55125(55300 - 175 = 55125). Так как 24696 = 23-32-73 и 55 125 = 32-53-72, то общие делители есть 3, 7, 9, 21, 49, 63, 147, 441. Так как делитель должен быть больше остатка, то условию задачи удовлетворяет лишь 441. Итак, на каждый стеллаж ставили по 441 книге. 8. Поскольку 660 не делится без остатка на 150, то найдем наименьшее общее кратное этих чисел: 660 = 22 • 3 • 5 • 11, 150 = 2 • 3 • 52, НОК (660, 150) = = 660 • 5 = 3300. Значит,, наименьшее число этапов в этой эстафете 3300: 150 = 22. 9. Все три теплохода будут в порту через число дней, являющееся наименьшим общим кратным чисел 3, 4 и 5, т.е. через 3-4-5 = 60 дней. Это будет в четвертый день недели, считая от вторника, т.е. в пятницу, так как 60:7 = 8 (ост. 4). 10. Каждое из таких пятизначных чисел делится на 3, так как 1 + 2 + + 3 + 4 + 5= 15, а 15-3. Общих делителей, больших, чем 3, эти числа не имеют, так как, например, 12 345 = 3 • 5 • 823, но уже число 12 354 не делится без остатка ни на 5, ни на 823. Значит, наибольший общий делитель данных чисел равен 3. 11. Число учащихся класса должно быть кратным числам 7,’ 3 и 2. Наименьшим общим кратным этих чисел есть 42 (7-3-2 = 42). Оно меньше, чем 50, поэтому удовлетворяет условию. Тогда 42-^ = 6; 42-^ = 14; 42-^ = 21; 42 — (6 + 21 + 14) = 1. Итак, на Корабельной улице проживает 1 ученик данного класса. 12. Если к искомому числу прибавить 1, то получится число, кратное каждому из чисел от 2 до 10 включительно. Значит, искомым числом есть наименьшее общее кратное чисел от 2 до 10, уменьшенное на 1, т.е. 5-7-8-9 — 1 = 2520 - 1 = 2519. 13. Выгоднее брать те бревна, из которых можно получить больше кусков при меньшем или одинаковом количестве распиловок. Из шестиметрового бревна получается 6 кусков при помощи 5 распиловок, а из восьмиметро- вого-8 при 7 распиловках. Сравним количество полученных кусков из этих бревен при одинаковом числе распиловок. Для чисел 5 и 7 наименьшее общее кратное есть 35. Значит, взяв 7 шестиметровых бревен и сделав 35 распиловок, получим 6 • 7 = 42 куска, а из пяти восьмиметровых бревен при таком же числе распиловок получим 8 • 5 = 40 кусков. Значит, выгоднее брать шестиметровые бревна. 14. Наибольший общий делитель тринадцати чисел примет наибольшее значение, если эти числа будут равны между собой. В данном случае эти числа не могут быть все равными между собой, так как 1988 не делится без остатка на 13. Но 1988 делится на 14(1988:14 = 142). Поэтому все слагаемые, кроме одного, равны по 142, а одно 284. В этом случае их наибольший общий делитель примет наибольшее возможное значение 142.
Дроби и проценты 2. Очевидно, намеченная рыба перемешалась со всей рыбой, имеющейся в ставке, равномерно. Среди выловленной рыбы намеченная составляет 5:150 = —. Значат, всего в ставке 80-30 = 2400 (рыбин). 3. 60 овец составляют 0,4 всех овец (1 — 0,6 = 0,4). Значит, всего овец в отаре 60:0,4 = 150. 4. 0,5 т составляет 0,2 всего зерна (1 — 0,8 = 0,2). Значит, масса зерна 0,5:0,2 = 2,5 (т). 5. Неправомерно применен распределительный закон относительно де- ления. 6. За Ц прыжков кузнечик может переместиться на 7 м (0,5-14 = 7). На- 0,3 м (7,3 — 7 = 0,3) он может переместиться двумя прыжками, «описав» боковые стороны треугольника длиной по 0,5 м с основанием 0,3 м. 9_10 _17 5_1_3 б) 7 21 “ 21 ’ Г) 8 4 “ 8' 8. а) Знак деления; б) знак умножения; в) знак вычитания; г) знак сложения. 9. Полпути туристы проехали автобусом за 0,5 ч (1:2 = 0,5), тогда пешком полпути они прошли за 5 ч (5,5 — 0,5). Значит, весь путь туристы пройдут за 10ч (5-2 = 10). Автобусом ехать в 10 раз быстрее, чем идти пешком. 10. 1) 0,275-80 = 22; 2 + 7 + 5 + 8 + 0 = 22; 2) 2,6-5 = 13; 2 + 6 +5 = 13; 3) 26-0,5 = 13; 2 + 6 + 0 + 5 = 13; 4) 6,5-2= 13; 6 +5+ 2= 13. (Можно привести сколько угодно решений.) 11. На крыльце избушки медведь съел все оставшиеся плюшки-пол- остатка и полплюшки. Отсюда полплюшки составляют пол-остатка, т.е. в третий раз он съел одну плюшку. Значит, когда он съел во второй раз половину плюшек и еще полплюшки, то у него осталась одна плюшка. Тогда 1 + 0,5 = 1,5 составляет половину имевшихся в этом случае плюшек, а всего 149
при этом было 1,5 • 2 = 3 плюшки. В первый раз он съел половину плюшек и еще полплюшки, поэтому 3,5 составляет половину имевшихся плюшек. Значит, было плюшек вначале 3,5 • 2 = 7. 12, За 60 мин ручные часы «отсчитывают» 55 мин. Им надо «отсчитать» /5,5-60 \ 5,5 ч. Это они сделают за 6 ч I ——— = 61. Значит, 1 ч дня они покажут через 0,5 ч или 30 мин (6 — 5,5 = 0,5). 13. За ночь пружина часов раскручивалась на 9 оборотов головки, за день-на 11, а за 24 ч-всего на 20 оборотов (9+11 =20). Один оборот головки часов дает возможность работать пружине 1,2 ч (24:20 = 1,2). Таким образом, с того времени, когда Минуткин ложился спать, до 8 ч 30 мин утра проходит 10 ч 48 мин (1,2-9=10,8). Значит, Минуткин ложился спать в 9 ч 42 мин (10 ч 48 мин-8 ч 30 мин = 2 ч 18 мин, 12 ч-2 ч 18 мин = = 9 ч 42 мин). 14. Если температура ежедневно, 15, 16, 17, 18 и 19 повышалась на 1,5°, то средняя температура была 17 числа. Если 17 апреля температура была 17,5°, то 19 апреля 17,5° + 3° = 20,5°, а 15 апреля 17,5° - 3° = 14,5°. 15. Средний возраст ученика этого класса 8 лет (280:35 = 8). Если бы удалось найти в классе 25 учеников, которым вместе не меньше 225 лет, то средний возраст ученика этой группы не меньше 9 лет (225:25 = 9). Тогда остальным 10 ученикам было бы вместе не больше 55 лет (280 — 225 = 55), а значит, средний возраст ученика этой группы не больше 5,5 лет (55:10 = 5,5), что противоречит действительности. / 70-35 16. Всего на данном участке 5000 квадратов указанного размера I ? = >000 ). В среднем каждый квадрат дает урожай 294 г (14,7 ц: 5000 = 294 г). Если предположить, что нет квадратов с одинаковым урожаем, то со всего участка собрали бы не меньше чем (0 + 1 + 2 + ... + 4999) г = (0 + 4999) г х х 2500 = 124,975 ц, что превышает действительный урожай. Для уменьшения записанной суммы надо заменить большие слагаемые меньшими. Но такие уже есть, поэтому некоторые из них повторяются. Значит, найдутся квадраты с одинаковым урожаем. (Такие квадраты найдутся даже больших размеров.) 17. Пусть искомые числа а и Ь, тогда а: b = Ь, а или после умножения на 10 получим 10a:Z> = 10Z> + а. Так как а не делится нацело на Ь, то должно делиться на b число 10, а это значит b = 2 или b = 5 (Ь 10, иначе получим а = 100 + а, что невозможно). Если b = 5, то 2а = 50 + а, откуда а = 50, и тогда 50:5= 10, что условию не удовлетворяет. Если b = 2, то 5а = 20 + а, откуда а = 5, и тогда имеем 5:2 = 2,5. Итак, а = 5 и b = 2. 18. Так как на турбазу автобус прибыл на 15 мин раньше, то от места встречи его с туристами до вокзала он должен был ехать ч (15 мин:2 = О = 7,5 мин = ч). За это время он проедет 7,5 км (60:8 = 7,5), а это значит, что О туристы шли пешком 7,5 км. Такое расстояние они прошли за 1,5 ч (5 — 3,25 — = 1,75; 1,75 — 0,25 = 1,5). Значит, их скорость 5 км/ч (7,5:1,5 = 5). 150
19. Примем число тетрадей в одной стопке за 100%, тогда в другой будет 60%, а вместе-160%. Значит, в одной стопке должно быть 80:1,6 = 50 (160% = 1,6), тогда в другой-30. 20. Пусть число девочек а, тогда мальчиков 0,8 а. Число девочек составляет а от числа мальчиков —— • 100% = 125%. 0,8 а 21. Пусть длина участка а, а ширина Ь, тогда его площадь ab. После увеличения длины на 35% она стала равна 1,35 а, а после уменьшения ширина стала равна 0,866. Площадь участка стала равна: 1,35а х 0,866= 1,161 ab. Значит, площадь увеличилась на 1,161 ab — ab = 0,161 ab, что составляет 5^-юо% = 16,1%. ab 22. Наименьшее число домов будет в том случае, если наибольший возможный процент домов, имеющих не больше пяти этажей, составляет наименьшее число домов. Это возможно, если 5%, т.е. часть, составляет один дом. Тогда всех домов будет 1 • 20 = 20. 2 23. Так как 8% = —, то число рабочих в цехе должно быть кратным 25. Кроме того, так как треть рабочих-женщины, то число рабочих кратно 3. Таким числом, не превышающим 100, есть 75. Значит, число рабочих 75. Женщин в цехе 75:3 — 25, а с сокращенным рабочим днем 6 человек (75:25 = = 3, 3-2 = 6, или 75 0,08 = 6). 24. В 1 т сена сухой массы будет 840 кг (100%-16% = 84%, 1000-0,84 = 840). В .свежескошенной траве сухая масса составляет 30% (100%-70% = 30%). Значит, травы должно быть 840:0,3 — 2800 (кг). Итак, чтобы получить 1 т сена, надо скосить 2,8 т травы. 25. Если свежих грибов было х кг, то сухой массы в них 0,1 х кг. После подсушивания сухой массы стало 40%, а воды-60%. Значит, при влажности АПО/ А 0,1x100% 1 3 60% грибов стало-----------= - х (кг), а испарилось - х кг, что составляет 3 15-4 15 кг. Значит, - х = 15, откуда х = - = 20 (кг). 26. Пусть первоначальная цена а р. После снижения на 10% она стала 0,9 а, а после снижения на 15% она стала 0,765 (0,9 а -0,85 = 0,765 а), что на 23,5% ниже первоначальной (100% — 76,5% = 23,5%). Если бы сначала снизили на 15%, то она стала бы 0,85 а; если бы затем снизили на 10%, то она стала бы 0,85а-0,9 = 0,765а, т.е. на 23,5% ниже. Значит, общий процент снижения в обоих случаях одинаков. 27. Пусть длина прыжка Отгадая равна а, тогда Угадая 0,7 а. За некоторое время Отгадай делает п прыжков, тогда за это же время Угадай сделает 1,3 п прыжков. За некоторое время Отгадай пробежит расстояние ап, а Угадай 0,7 а- 1,3 л = 0,91 ап. Так как ал > 0,91 ал, то победит в соревновании Отгадай. 28. Если количество зерна в ящике сеялки уменьшится на 14%, то будет 151
35 1 высеяно 35 кг зерна (250 0,14 = 35). Это составляет нормы высева на 1 га, а это значит, что засеяна площадь в | га, или 2000 м2 (10000:5 = 2000). Зная ширину захвата, найдем длину пути 2000:4 = 500 (м). 29. Пусть в первый день продали а кг яблок, тогда выручка в этот день составила 60 а к. После снижения цены во второй день выручка составила 60 а-1,125 к. и продали 1,5 а кг яблок. Цена 1кг яблок составила 60 а х х 1,125:1,5 а = 45 (к.). 30. Получив 20 деталей из 20 заготовок, в стружке окажется - всего 2 ( 2\ материала, т.е. на 6- заготовок (20:3 = 6- I. Из этого материала получают 2 6 заготовок да еще остается материала на - заготовки. Изготовив 6 деталей, получают в стружке материал на 6:3 = 2 заготовки. Получив из этого материала 2 заготовки и выточив из них 2 детали, в стружке остается материала на 2:3 = | заготовки. Таким образом, всего имеется материала на 2 2 4 - + - = - заготовки. Используя этот материал, получают еще одну заготовку, из которой вытачивают одну деталь. В результате получают 20 + 6 + 2 + 1 = 29 1 1 2 деталей и в отходе материала на - + - = - заготовки. 373 737 37-10101 37 31. а) Так как = 77.10101 = 77’ то данные дРоби равны. б) Найдем дроби, дополняющие данные до 1, и сравним их. 41_20. 411 200 20 200 -бТ“бТ; -бТТ~6Н’ 61-610‘ 200 200 20 200 41 411 Так как — > —, то — > —, и поэтому — < —. 610 611 61 611 61 611 200200201 2 300300301 _ 3 2’ 200200203 “ “ 200200203 ’ 300300304 “ “ 300 300 304 ’ 2 6 3 6 200 200 203 “ 600 600 609 ’ 300 300 304 “ 600 600 608 ’ 6 6 200200201 300300301 ТаК КаК 600600609 < 600600608' ™ 200200203 > 300300 304’ 33. Первому числу недостает до единицы - , а второму---------. 5 555 557 6666669 152
4 5 Так как 6666669-4 < 5 555 557-5, то - —— —— < , а значит, 5 555 557 6666669 5555553 6666664 5 555 557 > 6666669’ 34. Сравним 368972-764804 и 368975-764797 или 368972-764797 + + 368972-7 и 368972-764797 + 764797-3. Остается сравнить 368972-7 и 764797-3. Так как даже 36-7 >76-3, то 368792• 7 > 764797• 3, и поэтому 368972 368975 764797 > 764804’ 1 1 1 25 25 3 3 35' ’> 2 + 4 + 7 = 28; 2)1 -й = 28;” 3:2i = a 1 10 36. Пусть объем воды 1, тогда объем льда будет 1 + - = —. 9 9 „ 1 10 1 При превращении в воду лед уменьшится на -: — = — своего объема. 37. Числитель дроби искомого числа должен быть наименьшим общим кратным чисел 35, 28 и 25, а знаменатель-наибольшим общим делителем чисел 66, 165 и 231. Первым из этих чисел есть 4-52-7 = 700, а вторым 700 3-11 =33. Тогда искомое число 38. Числитель дроби искомого числа должен быть наибольшим общим делителем числителей данных дробей, а знаменатель-наименьшим общим кратным для знаменателей этих дробей. Так как 154 = 2-7 -11, 385 = 5-7-11 и 231 = 3-7-11, то искомый числитель 77. Так как 195 = 3-5-13, 156 = 22-3-13 и 130 = 2-5-13, то искомый знаменатель 22-3-5-13 = 780. Тогда искомое 'll ЧИСЛ° 780’ 39. Во второй день Юра прочитал книги, тогда за 2 дня 1 1 = т • 2 3 6 2 6 3 „ ~ 2^1 За третий день он прочитал -: 2 = -. Столько же и надо было ему прочитать, , 2 1 так как 1 — - = - 3 3 40. Гриша заплатил стоимости всех блюд, что составляет 50 к. Значит, стоимость всех блюд 50-3 = 150 (к.), а одно блюдо стоит 30 к. (150:5 = 30). Миша заплатил 30-3 = 90 (к.), а Саша-60 к. Значит, Гриша должен дать Мише 40 к. (90 - 50 = 40), а Саше 10 к. (60 - 50 = 10). а 41. Пусть расстояние от дома до школы а, тогда скорость —. Чтобы возвратиться домой и вернуться к тому месту, откуда возвращался, Сережа 153
должен был затратить 20 мин (9+11= 20). За это время он прошел бы а 2 расстояние — -20 = - а. Значит, Сережа от дома до того места, откуда должен 2 а 1 был возвратиться, прошел = р т-е- 3 ПУТИ- _ 2 11 2,1 42. Так; как - — - = - и = то КУСОК слеДУет сложить вчетверо и отрезать четвертую часть. Оставшаяся часть и будет полметра. 4 43. Сложить кусок пополам (получим — м), полученный еще раз сложить z 2 ч z 1 X пополам (получим — м), затем еще раз пополам (получим — м) и, наконец, полученный кусок сложить еще раз пополам (получим м). Если отрезать эту 8 1 1 /8 1 15 1\ часть от куска, т.е. — - —, то получим - м I — - — = — = - I. 13 7 13 13 44. Ширина материала после стирки 0,875 • — = - • — = — (м), 13 „„„ z ч 17 „ длина - 221 : — = 272 (м), что составляет — первоначальной. Значит, надо 16 18 17 взять 272: — = 288 (м). 1 о 45. Так как 375-139 - 150 = 375-138 + 375 - 150 = 375-138 + 225, то х = 1. „ 333-71 333-573 2-333 3-71 3-573 2-3 3-71 - 3-286 111-1001 222-1001 7-111 1001 2-1001 7 1001 3-573 3-573 3-215 3 / \ 3-143* 3 2-1001 2-1001 1001 2-1001\ J 2-7-1113 14 47. После переливания из второго сосуда в первый в нем станет 1114 2 1 1 1 2 1 „ -а + ---а = -а = -а, а во втором станет -а — ---а = -а = -а. Рассуждая 23263 23263 таким образом, получим таблицу Переливание 1 2 3 4 5 1-й сосуд 1 2а 2 за 1 2Я 3 5° 1 2а 2-й сосуд 1 2а 1 за 1 2а (N | 1 2в 154
Таблицу эту можно продолжить как угодно далеко. При этом замечаем, что после каждого нечетного числа переливаний в сосудах воды оказывается поровну—а л. Поэтому после 1987-го переливания в первом сосуде будет 1 —а л воды. (Это можно доказать.) 48. Пусть написано п чисел, тогда а2 4-а3 + ... ап ai =--------------- 2 «1 +а3 + ... +ап «2 =-------------- 2 Д1 + °2 + ••• + °в-1 . 2 (а1 + а2 + ... + а„) = (и — 1) • (ах + а2 4- ... + а„), откуда 2 = п — 1; п = 3. Значит, бьщо написано 3 числа. 49. Мож!ет. Если, например, 0,01 взять слагаемым 1000 раз, то получим 10, сумма квадратов этих чисел равна 0,1, т.е. 0,012 + 0,012 4- ... 4- 0,012 = 0,0001 • 1000 = 0,1 < 0,2. Ч----------V-----------' 1000 раз 50. Если числитель 166 ... 6 умножить на 4, то получим 66 ... 64. Значит, 166... 6 1 г —----— = -. При таком наборе цифр ответ действительно получился верный, 66 ... 64 4 но это вовсе не означает, что так можно сокращать. Возьмите, например, вместо шестерок пятерки или иные какие-нибудь цифры, и ответ будет неверный. а а 4- 2 а 51. Пусть искомая дробь -, тогда —-— = откуда ab 4- 2Ь = 2 а и а = 2. b 2а b 1 2 2 Из несократимых дробей, больших -, с числителем 2 есть две дроби: - и -. Так 2 2 2 как - > -, то за январь выпущено продукции высшего качества -, а за х 2 февраль-- продукции. 52. Пусь Ира прибавляла число х, тогда Х = |, откуда 88 4- 8х = 11+7 18 3 = 123 4- 3 х, 5х = 35 и х = 7. Дейсвительно ? = — = -. Пусть Оля при- 155
37 - у .3 бавляла и вычитала число у, тогда = —, откуда 629 — 17^= 189 4-3^; г. - 37 - 22 15 3 у = 22. Действительно = — = 63 + 22 о5 1 / м 537 -х 1 “ 9’ °ТКуДа * “ 437' a a 4- 4 a 54. Пусть первоначальная дробь -, тогда -—— = -, откуда ab + 4b = b b 4- 10 b a 2 2 4-4 6 2 = ab + 10a, 2b = 5a и - = Действительно ----------— = —= 7. Для этого b 5 5 4" 10 15 5 случая такая дробь единственная. 55. Нет, не права, таких дробей сколько угодно. Установим их вид. Пусть а а 4- 2 а исходная дробь -, тогда = - откуда а = 2. При числителе, равном 2, b 2b b и любом нечетном знаменателе дробь будет несократимой. Значит, искомые 2 2 2 2 2 дроби имеют вид ------. Такие дроби: -, -, -, - и т. д. 2 w I 1 3 5 / 9 п а + х а а 56. Пусть искомое число х, тогда —-= -, откуда а + х = ах и х =--. b-x b а — 1 Это число будет целым только при а = 2, и тогда имеем случай, рассмотрен- • ный в задаче 55. Если же, например, имеем а = 5, то х = , и тогда 5 + 4 25 5 L 5 5-Ь Ь' Ь4 а 57. Пусть дробь - несократима. Дробь, дополняющая данную до 1, есть b . а Ь — а „ 1 — - = —-—. Так как по условию а и о не имеют общих делителей, кроме 1, b b то числитель Ь — а полученной дроби не делится ни на один делитель числа Ь, _ Ь — а а это значит дробь —-— несократима. 58. Пусть а > Ь. По условию--- = 3- —, откуда ab = 3а — ЗЬиа = -—-. а — b ab 3 — b Число а будет натуральным при условии, что Ь = 2. При этом а = 6. Значит, а = 6, b = 2. 30п + 2 (24и + 2) + 6 л _ 6 л 6л 59. = ----------------= 2 Ч-------; ------- 12л + 1 12л+1 12и+1 12и + 1 156
будет целым только при п = 0, при иных значениях «-правильная дробь. Значит, только при п = 0 является целым числом (2). 12и + 1 60. Выразим емкости второго и третьего бидонов через емкость первого 3 5 и схематически запишем так: II = - I; III = -I. Так как в каждом бидоне целое число литров и наименьшим числом, кратным 2 и 3, есть 6, то емкость первого бидона 6 л, второго-9 л и третьего-10 л. Значит, в бочке осталось 5 л (30-(6+ 9+ 10) = 5). 61. До передачи книг в читальный зал наличных книг в библиотеке было в 16 раз больше, чем выданных, а это значит, что число выданных книг (отсутствующих в библиотеке) составляет — от общего числа книг библиотеки. После того как выдали 2000 книг (передали в читальный зал), число отсут- ствующих книг стало составлять от общего числа книг. Значит, 2000 книг 16 составляют от общего числа книг. Значит, библиотека имеет 2000 -272 = 544000 книг. 62. Число книг на нижней полке примем за 1, тогда число книг на двух остальных полках составит 2 единицы, а на трех полках-3 единицы. Значит, 1 число книг на нижнеи полке составляет - всех книг. Аналогично число книг на , х 1 составит 1 — I - + - \3 4, средней полке составляет i всех книг. Тогда число книг на верхней полке 5 5 —, или 30 книг. Значит, всего книг 30: — = 72. 12 12 „ „ 1 1 63. Взнос второго составляет - стоимости покупки, третьего -- и четверто- 1 о 23 го——. Значит, взнос первого составляет — стоимости покупки / 1 1 1_23\ V 4 5 6"6ОЛ 23 а отсюда стоимость покупки 2,3: — = 6 р. Взнос второго 1 р. 50 к., взнос 60 третьего 1 р. 20 к. и-взнос четвертого 1 р. 64. Когда к имеющемуся в чашке стакану кофе влили четверть стакана 5 молока, то в чашке стало - стакана кофе с молоком. При этом молоко составит 1 долю, а кофе-4 таких долей. Чтобы долить стакан с молоком, надо из чашки отлить четверть стакана содержимого чашки. В чашке после этого 157
останется стакан кофе с молоком. Соотношение кофе и молока в чашке не . 4 1 изменилось, поэтому кофе в нем осталось - стакана, а молока--. 65. Пусть в каждом стакане по а ложек жидкости. Влив в стакан с водой ложку молока, получим а + 1 ложек смеси, в которой молоко составляет ——- а + 1 а часть. Взяв ложку смеси, в которой вода составляет -------------------------------------- часть, влили ее а + 1 в стакан с молоком. В нем станет всего а ложек жидкости, а вода составит а 1 с 1 -----: а = часть. Значит, в молоке воды будет -- часть и в воде а + 1 а + 1---------------------------------------------а + 1 1 молока также-------- часть. а + 1 66. После того, как туристы прошли 1 км, половина оставшегося пути составляет треть всего пути и 1 км, а значит, весь оставшийся путь составляет 2 2 - всего пути и 2 км. Таким образом, весь путь составляет - всего пути и 2 + 1 = 3 (км), а это значит, что 3 км составляет - всего пути 11 — - = - I. Тогда весь .путь равен 9 км (3 • 3 = 9). 67. Пусть всего работало х учеников, тогда девочек было ^-х, из них 1 „ „ 2 2 2 шестиклассниц--х. Мальчиков было -х, из них шестиклассников--х- 6 3 3 7 4 „ 1 4 = 2|’х Так как -х и —х-числа натуральные, то х должен быть кратным числам 2, 3 и 7. Таким наименьшим числом есть 42. Значит, наименьшее число. 1 4 шестиклассников могло быть 42-- 4- 42- — = 15. 6 21 68. Если длина пути а км и первый турист прошел его за х ч, то х х 2а а а 5 • - + 4 • - = а, откуда х = —. Второй турист этот путь прошел за —— + —— = Хг Хг 7 X* J Ал 9а _ 2а 9а 2а 18а 9а 18а _ 18а 18а = -ух. Сравним — и —, — = —-; — = ——. Так как —— < то первый 40 9 40 9 81 40 80 81 80 F турист затратил на весь путь времени меньше, чем второй, а это значит, что он прибудет в В раньше, чем второй. 69. Пусть стоимость каждого сорта конфет а р. Тогда по цене 2 р. за 1 кг а а а а 5а было а: 2 = - (кг), а по 3 р. -- кг. Всего было - + - = — (кг), а значит, средняя 2 3 2*36 158
кг 5а 12 цена за 1 кг смеси будет 2 а: — = — = 2,4, т.е. 2 р. 40к. (но не 2 р. 50 к.). 6 5 70. Пусть куплено каждого сорта клубники на сумму а р. Тогда первого а а ( а а сорта куплено а:2,4 = — (кг), а второго-— кг. Всего куплено I + — на сумму 2 а р. Средняя цена за 1 кг составляет л ( а а 2 а * I —— + — \ 2,4 1,6. 2-2,41,6 _ —-----— = 1,92, т.е. 1 р. 92 к. 2,4 +1,6 71. Пусть длина одного участка а км, тогда на прохождение всего пути длиной 3 а км автобус затратил а а а \ — + — + — I ч. Значит, средняя скорость движения равна /а а а\ 3-1050 3°Ч5О + 30 + 70Г 21+35+15 « 44 (КМ/,)' 72. 1 - 3 8 15 224 _ 1-3 2-4 3-5 14-16 4 9 16225 ““F’I5" 152 ‘ Выполнив последовательно сокращения, получим первый множитель 1 „ 16 -, последний —, а все промежуточные-единицы. Следовательно, искомое 1 16 8 произведение равно 2’15 = 15 ’ ~ „ 2,12.11 2 11 73. Так как ——=1— -; —- = - — -; ... = — — ——, то будем 1-3 3 3-5 3 5 99-101 99 101 J , 11111 1 1 , 1 100 иметь - - + - - - + - - - + ... + — - — - - — - —. 74. Представив числа таблицы в ином виде, получим 1 1 23 3 8 7 23 4 1 7 11 3 1 5 4 8 13 11 159
.. „ - „5 Из этой таблицы видно, что числа, записанные в ней, кроме —, попарно „ 5 равны. Значит, следует исключить число —. 75. Заменим данные дроби равными им, умножая числитель и знаменатель на 2, 3, ... , 8. В результате получим: ,ч 14 16 г 15 1) — и —; между этими дробями заключена одна дробь —, после 18 18 18 5 сокращения которой получим -. 6 21 24 22 23 2) — и —. Имеем — и — . Не удовлетворяет условию. 28 32 29 30 31 3) — и —. Имеем —; —; —. Удовлетворяет условию только одна дробь 36 36 36 36 36 30 5 — = -, но такая уже есть. 36 6 3 35 40 „ 36 37 38 39 „ 4) — и —. Имеем —; —; —; —. Удовлетворяет условию только одна 45 45 45 45 45 45 _ 36 4 дробь- = 5. 5) — и —. Из пяти дробей только одна — = - удовлетворяет условию, но такая уже есть. ал 49 56 тл . - 54 6 6) — и — .Из шести дробей только одна удовлетворяет условию — = -. 63 63 63 7 _ 56 64 г „ 60 5 63 7 7) — и —. Из семи дробей условию удовлетворяют две: — = -и — = -,но о . 5 4 6 7 новая только одна. Значит, искомые дроби -и -. 6 5 7 8 76. Пусть Андрей пробегает за одну секунду а м, а Борис b м и Сергей с м. 100 Дистанцию в 100 м Андрей пробежал за -----секунд, а Борис за это время а 90 пробежал 90 м, т.е. он затратил —секунд. Когда Борис пробежал 100 м за b 100 г 100 90 о —— секунд, то Сергеи за это время пробежал 90 м, поэтому —- = —. Значит, О ОС 100 90 100 90 90 81 100 81 „ имеем --= — и—— = —, откуда — = —, поэтому -= —. Последнее а о о с ос ас 160
означает, что если Андрей финишировал, то Сергей был позади его в 19 м (100 - 81 = 19). 77. Так как - = 3 2_ 15’ 5 “ 6 3 9 1 5 — ,- = — и - = —, т< 15 5 15 3 15 ) имеем 3 15 6 15 9 15 1 15 2 15 4 15 7 15 5 15 8 15 Задача сводится к составлению «магического» квадрата. Применим такой способ. Начертим фигуру, как показано ниже. Впи- шем в квадраты в направлении, показанном стрелкой, последовательно числа в порядке их возрастания, причем только в те квадраты, где их в ряду 3. Затем числа, записанные крайними в горизонтальном и вертикальном рядах, в которых по 5 квадратов, переносим в свободные квадраты, но не в соседние с этими числами, а в дальние от них. Например: В результате имеем 2 15 7 15 6 15 2 15 7 15 2 5 9 5 1 3 1 1 — — — или 15 15 15 5 3 15 4 3 8 4 1 8 — 15 15 15 15 5 15 78. Заяц пробежит 250 4 м за у Y мин f 250:550 = j-j, , а волку до места 6 Зак. 2531 Д. В Клименченко 161
280:600 = 7 укрытия надо пробежать 280 м (250 + 30 = 280), на что потребуется — мин 7\ 5 7 5 75 7 77 77 75 — I. Сравним — и —. Так как — = —; 77 = 777 и — > —, то 15/ 11 15 11 165 15 165 165 165 волк зайца не догонит. 79. Двое садятся на мотоцикл и едут 1 ч, а третий идет этот час пешком за ними. Проехав за 1 ч 50 км, один из геологов идет остальные 10 км пешком и своевременно прибывает на станцию (10:5 = 2,1 +2 = 3). Второй едет на мотоцикле навстречу третьему в то время, когда их разделяет 45 км (50 — 5 = 9 ( 45 9 \ = 45), и встречает его через — ч I 45: (50 + 5) = — = — I. За это время 450 /9 450\ „ мотоцикл проедет уу- км I 50- — = уу- I. Значит, от места встречи до станции 560 / 450 ,Л 560\ ~ им осталось проехать уу- км I уу- + Ю = уу- I. Это расстояние они проедут 56 за — ч 55 . Таким образом, на все расстояние от базы до станции , т.е. меньше, чем 3 ч. 46 / 56 45 эти два геолога потратят 2—ч11+ — + — •J \ 80. Количество травы, поедаемое одной коровой за 1 день, примем за 1. Тогда 70 коров за 24 дня съедят 70-24= 1680 единиц, а 30 коров за 60 дней-30-60 = 1800 таких единиц. Значит, 30 коров за 60 дней съедят больше, чем 70 за 24 дня на 1800 — 1680 = 120 единиц, так как трава росла 36 дней (60 — 24 = 36). За 1 день вырастало 120:36 = — единицы. Пусть за 96 дней траву съедят х коров. Тогда, зная, что за 96 — 60 = 36 дней вырастает 10 — 36 -- 120 единиц травы, а за 60 дней коровы съедят 30-60 единиц, получим уравнение 96 х = 30-60 + 120; 96 х = 1920; х = 20. Значит, за 96 дней траву съедят 20 коров. 81. Через 1 ч минутная стрелка делает полный оборот и будет показывать 12, а часовая пройдет окружности и будет показывать 1. Значит, по истечении 1 ч минутная стрелка, следуя за часовой, окажется от нее на расстоянии, равном окружности. Так как за 1 ч минутная стрелка пройдет больше часовой на 1 — — = — окружности, то она «догонит» часовую по , 1 ( 1 11 1 \ о истечении 1 ч еще через —ч I —: — = — I. Значит, часовая и минутная 162
стрелки снова совпадут, начиная движение от 12, через 1 -^j-ч, т.е. в 1 ч 5 мин 27 с. В это время (с точностью до секунды) и надо начать артподготовку. 82. За 1 ч минутная стрелка делает полный оборот (ее конец описывает ч - 1 окружность), а часовая пройдет — окружности. Значит, за час минутная стрелка пройдет больше часовой на окружности. Пока минутная стрелка пройдет от 12 ч до 6 ч, т. е. окружности, часовая пройдет окружности, т. е. в 6 ч 30 мин, минутная стрелка отстанет от часовой на окружности. Она ее 1 । 1 11 1 1 и « догонит через — ч I —: — = — |. Часы будут 22 \24 12 22/ показывать бтГ /л 1 ,6i * = ^налогичные рассуждения приводят часовая стрелка будет между 12 ч и 1ч, а минутная 6 противоположно ей, то часы будут показывать 12— ч. Значит, грибная к тому, что направлена когда прямо «охота» длилась 6 ч 83. Рассмотрим самое меньшее слагаемое-дробь вида Ее знаменатель содержит наибольшее число двоек по сравнению с любой другой дробью, поэтому дополнительные множители при приведении к общему знаменателю для каждой из дробей-слагаемых, отличные от , будут четными, а дополни- тельный множитель для дроби нечетным. Значит, в числителе будет сумма, 2К все слагаемые которой четные, а одно-нечетное. Такая сумма-число нечетное, знаменатель же-число четное. Таким образом, ни при каком натуральном п рассматриваемая сумма не может быть целым числом. 84. Из условия путем почленного деления получим b с * а с 4 а b b+с а+с а + Ь -+--1=-+--1=-+--1 или---------= —— =----. a a b b с с а b с гг а + ь (Ь + с) (а + Ь) (а + с) 3 3 , 1Э 1О Пусть --= п, тогда ------------= и , но и*5 / 12, так как 12 не с abc является кубом натурального числа. Значит, ответ не может быть поло- жительный. 6* 163
85. Из решения предыдущей задачи имеем а + b = сп; b + с = ап\ а + с = Ьп. Пееде вычитания получим а — с = п (с — а). Равенство возможно при а — с = О, т. е. а = с. Аналогично и а = Ь, поэтому а — b = с, а отсюда а + b b + с а + с (а + Ь) (Ь + с) (а + с) о cab abc 86. Запишем сложение в столбик СЛОВ,О СЛОВ.О ПЕСНЯ Так как в результате получим целое число, то 0 = 5. Буква П может обозначать только цифру 1, тогда Н = 0. Так как С > 5, то методом проб находим С = 9, Л = 4 и т.д. Получаем 9453,5 + 9453,5 = 18 907. 87. Из условия видно, что в результате деления однозначных чисел получена периодическая дробь с длиной периода 6. Значит, делитель есть число 7, т.е. Е = 7. Так как в частном целых 0, то Н < Е, т.е. Н < 7. Если Н = 1, то - = 0,14 ... , что не удовлетворяет условию, так как цифра десятых С, а не Н. Если Н = 2, то | = 0,2 ... и также не удовлетворяет условию. Если 3 Н = 3, то - = 0,(428 571), что условию удовлетворяет; Н = 4, Н = 5, Н = 6 условию не удовлетворяет. Значит, 3:7 = 0,(428 571). Положительные и отрицательные числа 1. Сумма чисел уменьшилась на 25 — ( — 35) = 60. Значит, было отмечено 60:5= 12 чисел. 2. а) От начала координат да'льше удалена точка А, чем В; б) от точки С дальше удалена точка В. 3. Данные числа отличаются только знаком, поэтому их сумма равна 0. Разность этих чисел равна удвоенному числу, взятому со знаком « + » или « —». 4. Поскольку произведения чисел каждой строки отрицательны, то про- изведение всех чисел таблицы также отрицательно. Значит, произведение всех 164
чисел может быть отрицательным или только в одном столбце, или во всех трех; положительными произведения чисел могут быть только в двух столбцах. 5. Если b 0, то а + b = |Z>| + b = 0; если же b > 0, то а + b = 2Ь # 0. Значит, ни один из мальчиков не прав. 6. Двигаясь от нуля вправо и от двух влево, заполняем последовательно клеточки, в результате чего получим —42 26 -16 10 -6 4 -2 2 0 2 2 7. Эти числа могут быть: 1) -6, — 2; 2) -6, -2, -1, 1; 3) -6, -2, -4, 4; 4) -6, -2, -1, 1, 4, —4; 5) -6, -4, 2; 6) -6, -4, 2, -1, 1; 7) -6, -4, -2, -1, 2, 3. 8. От средней ветки до верхней будет +5 — 7 + 4 + 9= 11 веток. Значит, всего веток 11-2+ 1 =23. 9. Рассмотрим, например, ряд чисел — 7, 4, 4, ..., —7, 4, 4, —7 . Сумма 19 чисел любых 3 соседних чисел этого ряда равна 1, т. е. положительна, сумма же всех 19 чисел равна — 1, т. е. отрицательна. Ответ. Может. Найдите еще подобный пример. 10. Имея числа 4, —2, —8, —14 одной диагонали, находим сумму 4 — 2 — 8— 14 = —20. Зная эту сумму, последовательно находим: Г = -20 - (4 - 6 + 2) = -20; М = -20 - (2 - 8 - 4) = -10; К = -20-(-20+ 8 -14) = 6; В= -20-(6-8- 18) = 0; А= -20-(-6-2 + 0)= -12; Б= -20-(8- 10-2)= -16; Д = -20-(4- 16- 18)= 10. 11. 1+2 = 3, -1+2 + 3 = 4; 12-3-4 = 5; 12 + 3-4-5 = 6; 1—2 + 3 + 4 — 5 + 6 = 7 или 1+2 — 3 — 4 + 5 + 6 = 7; 1+2 + 3- 4 + 5- -6 + 7 = 8; 12-3 + 4 + 5 + 6- 7- 8 = 9 или 12 + 3 + 4- 5- 6- 7 + 8 = 9. 12. Очевидно, с = — (а + Ь), поэтому недостающие числа —(—17 +5) =12; -(14 - 14) = 0. 13. Так как -100 - 99 - 98 - ... - 1 + 1 + 2 + ... + 99 + 100 = 0, а 101 + + 102 = 203, то искомая сумма 203. 14. Так как —97 — 95 — ... + 95 + 97 = 0, а —101 — 99 = —200, то искомая сумма равна —200. 15. Запишем сумму так: 1 +(2 — 3 — 4 + 5) +(6 — 7 — 8 + 9) + ... + (298 — — 299 — 300 + 301) + 302. В каждой скобке получаем 0, поэтому конечный результат 1 + 302 = 303. 16. В данном случае х < 0. Имеем 16 слагаемых от 16 до 1, столько же слагаемых от —1 до —16 да еще два слагаемых 0 и 17. Значит, всего 34 слагаемых. 165
17. Числа, очевидно, одинаковы по модулю, но разные по знакам. Задача имеет сколько угодно решений. Вот одно из них: 1 -1 1 — 1 1 -1 1 — 1 -1 1 — 1 1 -1 1 -1 1 18. Расставим девять двоек и 16 раз по — 1 так: 2 -1 2 -1 2 -1 -1 — 1 -1 — 1 2 -1 2 — 1 2 — 1 — 1 — 1 — 1 -1 2 -1 2 -1 2 19. Пусть точке D соответствует число х. Сумма чисел до изменения направления прямой 5 + 8 — 10 + х, а после изменения направления — 5 — 8 + 10 — х. По условию 5 + 8— 10 + х = 5 — 8+10 — х, откуда 2х = = — 6 и л = - 3. Задачу можно решить проще, не обращаясь к уравнению. Поскольку сумма не изменилась при изменении знаков на противоположные, то она равна нулю. И если имеем слагаемые 8, 5 и —10, то четвертое слагаемое — 3, так как при этом 8 + 5 — 10 — 3 = 0. Итак, точке D соответствует число —3. 20. Поскольку первая переменная может принимать 21 значение, а вторая-16, то выражение а + b может принимать всего 21-16 = 336 раз- личных значений. Наибольшее значение выражения а+ b равно 21 (8 + 13 = 21), а наименьшее равно —49 (—32 + (—17) = —49). Их произве- дение (—49)-21 = -1029. 21. Наблюдая — 1 —(—!)= -1 + 1 = 0; — 1 _(_ 1 _ (—1 _(_!))) = 0; _(_!))))=-1, замечаем, что при четном количестве единиц в результате получаем 0, а при нечетном количестве — 1. Значит, в случае а) имеем в результате — 1, а в случае б) 0. 166
22. Составим таблицу: ^2 «3 «5 «а «7 «8 «9 «10 а11 «12 «13 «14 «15 «16 «17 — 1 0 1 2 3 0 0 2 6 0 0 0 12 0 0 0 0 Начиная с а14, все числа будут нули, поэтому последнее не равное нулю число будет Д1з = 12. 23. Составим таблицу: «1 «2 «3 «4 «5 «6 «7 «8 «9 «10 «11 «12 1 — 1 -1 1 -1 -1 1 — 1 -1 1 -1 -1 Замечаем, что в полученном ряду группа чисел 1, — 1, — 1 периодически повторяется. Если чисел 1988, то полных периодов будет 662(1988:3 = 2\ = 662 + -!. Следующий 663-й период имеет два числа, они равны 1 и — 1. Значит, на 1988 месте будет —1. 24. Если в сумме получено 846, то это значит, что на столько было больше чисел 1, чем — 1, так как каждый раз 1 + ( — 1) = 0. Значит, среди данных чисел — 1 должно встретиться (1989 — 846): 2 раз. Но это число не является целым, поэтому сумма всех чисел не может равняться 846. Если сумма 845, то число — 1 встречается (1989 — 845): 2 = 572 раза, тогда число 1 встретится 1989 — 572 = 1417 раз, что вполне возможно. 25. 0,5 + (-1) = 0,5•( — 1) = 0,5:( —1) = -0,5. 26. Пусть искомые числа а и Ь. Тогда по условию а + b = ab, откуда а — b:(b — 1). Это число целое только при b = а = 2 или а = b = 0. По условию также ab = а — Ь, откуда b = а:(а + 1), что дает целое число при а = b = 0 или Ь = 2 и а=— 2. Но при втором значении а + b = 0, поэтому оно не удовлетворяет условию. Значит, ab = a + b = a — b при а = b — 0. а + 9 (а + 6) + 3 3 3 27. ----- =------— =1-1------Значение выражения ------- не будет а + 6 а + 6 а + 6 а + 6 целым числом ни при каком целом неотрицательном а. Значит, значение а + 9 выражения------ может быть целым только при отрицательных а. Таковыми а + 6 являются —3, —5, — 7, —9. 28. Сумма всех чисел от 1 до 9 равна 45. Поскольку 45 — 21 =24, то, чтобы получить 21, знаки «—» надо поставить перед числами, которые в сумме дают 12. Такими числами, например, есть 2, 4 и 6. Тогда 1—2 + 3—4 + 5 — — 6 + 7 + 8 + 9 = 21. Изменение знака на противоположный изменяет сумму не меньше, чем на 2. Так как 21 — 20 = 1 < 2, то один из учеников ошибся Примечание. Можно было сразу сказать, что ошибся второй ученик. 167
29. Правило, справедливое для натуральных чисел а, Ь, с и d, неправомерно распространено на отрицательные числа. Если, например, 6:3 = (— 6): (— 3) = 2, то из 6 > 3 вовсе не следует, что — 6 > — 3. 30. Каждое число, начиная с третьего, равно сумме двух предыдущих без пяти. Так 2 + 3 — 5 = 0, 0 — 2 — 5 = —7 и т.д. Значит, за числом —14 будет следовать число —7 — 14 — 5 = —26, а за ним------14 — 26 — 5 = —45. 31. 1) а; Ь; с; 2) а — b; Ь — с; с — а; 3) а — 2b + с; Ь — 2с + а; с — 2а + Ь; 4) Зс - ЗЬ; За - Зс; ЗЬ - За; 5) 6с — ЗЬ — За; 6а — Зс — ЗЬ; 6Ь — За — Зс; 6) 9с — 9а; 9а — 9b; 9Ь — 9с; 7) 9с - 18а + 9Ь; 9а - 186 + 9с; 9Ь - 18с + 9а; 8) 21b - 21а; 21с - 21Ь; 21а - 27с. Числа восьмой и всех последующих строк делятся на 27, а число 855 на 27 не делится. Значит, при любых а, Ь, с ни в одной строке ниже седьмой число 855 не может встретиться. 32. a) (at - 1) + (а2 - 2) + ... + (а„ - л) = (aj + a2... + a„) - (1 + 2 + ... + п) = 0; б) произведение будет четным, если четный хотя бы один из множителей. Из данных п чисел при п нечетном нечетных чисел будет на одно больше, чем четных. Значит, если составлять разности ак — к нечетного и четного чисел, то из чисел ак и к останется по одному нечетному числу, их разность будет четным числом. В этом самом «неблагоприятном» случае получим один четный множитель. В других случаях их будет больше. Значит, при п нечетном указанное произведение четное. Выражения с переменными. Уравнения и неравенства 1. Пусть верных ответов было х, тогда неверных 30 — х. Имеем lx - 12 - (30 - х) = 77; 19х = 437; х = 23. 2. Пусть в 5А классе х учеников, тогда (х — 23)-23 = (х — 13) -13, откуда 1 Ох = 360 и х = 36. 3. Одно из искомых чисел больше другого в 10 раз. Пусть меньшее число х, тогда большее 10х. Имеем 10х + х = 13,5927, значит, 11х= 13,5927, откуда х = 13,5927:11 = 1,2357. Значит, меньшее число 1,2357, а большее 12,357. 4. Пусть дикобразу х лет, тогда льву 2,5х. Три года назад дикобразу было х — 3, а льву 2,5х — 3. По условию задачи 7(х - 3) = 2,5х - 3, откуда х = 4. Значит, дикобразу и льву вместе 14 лет. 168
5. Пусть дикобразу х лет, тогда сове 2х, а льву (2х — 2). Имеем уравнение (2х — 2) — х = 4, откуда х = 6. Значит, дикобразу 6 лет, сове-12, льву-10, а вместе им 28. 6. Пусть десятков х, тогда сотен х-3, а единиц х-4. По условию задачи х 4- (х — 3) 4- (х — 4) 2 ~Х' откуда х = 7. Значит, искомое число 473. 7. Если вчера школьники собрали х кг макулатуры, то позавчера (х 4- 3) кг. Пусть сегодня они собрали у кг, тогда (х + 3) + у — х = 40, откуда у = 37. 8. Пусть сейчас брату х лет, а сестре у. Когда Коле было (х + у) лет, го сестре в это время было х лет. Коле было у лет х лет тому назад, но х лет тому назад сестре было х — х = 0 лет, т. е. она родилась в этот год. 9. Если девочек х, то мальчиков х 4- 1. Без Иры девочек будет х — 1, тогда из условия имеем 2(х-.1) = х+ 1, откуда х = 3. Значит, девочек в семье 3, а мальчиков 4. 10. Пусть Саше и Паше по а лет, тогда тетушке Маше (2а — 3) лет. Тетушке Маше было а лет (а — 3) года тому назад (2а — 3 — а = а — 3). Но столько лет тому назад Саше было 3 года (а — (а — 3) = 3). 11. Пусть Вера купила а тетрадей, Вася-Л, а Миша-с. Тогда по условию а 4-6 = 6 4-с и 6 4-10 = a 4-с. Сложив почленно эти равенства; получим а + b + 16 = а 4- b + 2с, откуда 2с = 16, а значит, с = 8. 12. Пусть искомое трехзначное число х. Если припишем слева цифру 7, то получим число 7000 4- х; если припишем цифру 7 справа, то получим число 10х 4- 7. По условию (7000 + х) - (10х + 7) = 3555, откуда 9х = 3438 и х = 382. Задачу можно также решить подбором цифр. 13. Пусть жира было х кг, тогда лука-2х кг, картофеля-Зх кг (х + 2х = = Зх), крупы-6х кг (х 4- 2х 4- Зх — 6х), воды - 12х кг (6х 4- Зх 4- 2х 4- х = 12х). Имеем 12х 4- 6х 4- Зх 4- 2х 4- х = 12; 24х = 12; х = 12:24 = 0,5. Итак, жира 0,5 кг, лука-1 кг, картофеля -1,5 кг, крупы-3 кг, воды-6 кг. 14. *Пусть часы № 1 показывают х мин, тогда часы № 2-(х4-.1), часы № 3-(х — 2), часы № 4-(х — 3) и часы № 5-(х — 1). Сумма показаний всех часов х 4- (х 4- 1) 4- (х — 2) 4- (х — 3) 4- (х — 1) = 5х — 5. Так как (5х — 5): 5 = = х — 1, а это показание часов № 5, то эти часы показывают точное время. 15. Пусть в бригаде х человек, тогда имеем 25х = 23 (х — 1) 4- 45, откуда 2х = 22 и х = 11. 16. Пусть средняя урожайность несортового картофеля х ц с 1 га, а сорто- вого-у ц с 1 га. Тогда средняя урожайность двух участков (х 4- 20) ц с 1 га. Имеем (х 4- 20)18 = х-14 4-J-4, откуда 4у — 4х = 360; у — х = 90, а это значит, что урожайность сортового картофеля выше, чем несортового, на 90 ц. 17. Пусть было х учеников, а среднедневная выработка ученика у кг. Тогда у-х-5 = 45715, откуда ху = 9143. Так как 9143 = 41-223 и в классе не может быть 223 учеников, то учеников было 41, и каждый из них собирал ежедневно в среднем 223 кг картофеля. 169
Рис. 13 18. Поскольку боцман старше матроса на 4 года, то ему 20 + 4 = 24 года. Пусть боцман моложе машиниста на х лет, тогда машинисту (24 + х), а юнге (24 — х) лет. Согласно заявлению рулевого имеем (24- х)-2 = (24 + х)+6, откуда х = 6. Значит, машинисту 24 + 6 = = 30, юнге 24 — 6 = 18, а рулевому 36 лет. Если капитану у лет, то (20 + 24 + 30 + 18 + 36 + у):6 = 28, откуда у = 40. 19. Наибольшее число получим в том случае, если в кружках получены наиболь- шие значения средних арифметических. При определении этих средних арифметических числа, стоящие в угловых клетках, берутся по одному разу, число, стоящее в централь- ной клетке,-4 раза, а числа, стоящие в сред- них клетках, прилегающих к сторонам квад- рата, берутся по два раза. Исходя из этого, наибольшее число 9 запишем в центральной клетке, а наименьшие-1, 2, 3 и 4 запишем в угловых клетках г ~ 21 27 26 (рис. 13). В результате получаем xt = —; х2 = 6; х3 = —; х4 = — и 98 (Х| + х2 + х3 + х4): 4 = (21 + 24 + 27 + 26): 16 = —— = 6,125. 16 20. Пусть первое число х, тогда второе (Зх + 4), третье х, четвертое (Зх + 4), пятое (х + 9), шестое (6х + 6). По условию х + (Зх + 4) + х + (Зх + 4) + (х + 9) + (6х + 6) = 83, откуда х = 4. Четвертая буква алфавита Г; (Зх + 4 = 16) шестнадцатая буква О; (х + 9 = 13) тринадцатая буква Л; (6х + 6 = 30) тридцатая - мягкий знак. В результате получили ГОГОЛЬ. 21. Пусть второй поймал х рыбин, тогда первый -х + 10 рыбин. По утверждению второго рыбака он поймал Qx + 10^ + 20 рыбин. Имеем |х + 10^ + 20, откуда х = 60, т.е. второй поймал 60 рыбин. Тогда первый поймал ^-60 + 10 = 40 рыбин, а вместе -100 рыбин. 22. Если Лена проходит за 1 мин а м, то Юра 2а м. Пусть он догонит ее через х мин после своего выхода. Тогда имеем а • 5 + а • х = 2а • х, откуда х = 5. 23. Пусть 25 апельсинов стоят х р., тогда на 1 р. можно купить х апельси- нов. Один апельсин стоит 1 :х = х:25, откуда х = 5. Значит, на 3 р. можно купить 15 апельсинов. 24. Если покупатель дал кассиру х купюр, а получил сдачу у купюр, то , , 24 + Зу имеем уравнение 5х — Зу = 24, откуда х = —-—. Так как х должно быть уравнение х = 170
целым, то 24 + Зу должно делиться на 5. Это возможно при наименьшем значении у = 2. Значит, покупатель дал 6 купюр (30 р.) и получил сдачу 6 р. (2 купюры по 3 р.). 25. Пусть отрезков по 7 см-х, а по 12 см-у. Тогда 7х + 12у = 100, откуда 100 - 12у _ , ы х = -—- будет натуральным только при у = 6. Имеем единственное решение х = 4, у = 6, а это значит, что из данных отрезков можно единствен- ным способом составить отрезок длиной 1 м. 26. Пусть выполнено верно х заданий, а неверно или не выполнено „ _ пс<-7 57 + 5у у заданий. Тогда имеем 9х — 5у = 57, откуда х = —-—. Так как х натураль- ное, то (57 + 5у) должно делиться на 9. Наименьшее значение у, при котором выполняется это условие, есть 3. При этом х = 8. Так как 8 + 3 = 11и11<15, то найденное значение х удовлетворяет условию. 27. Пусть команда выступала по х видам спорта и по всем видам, кроме последнего, она набрала п баллов. Тогда имеем (97 + п):х = 90; (73 + п):х = 87, откуда 97 + п = 90х и 73 + п = 87х, а значит, х = 8. 28. Пусть дедушке (10а + Ь) лет. Тогда 10а + b = а + Ь2, откуда 9а = Ь2 — Ь, или а = ——. 9 Так как а и b однозначные числа, то равенство выполняется только при b = 9. При этом а = 8. Значит, дедушке 89 лет. 29. Если мотоциклов с коляской х, то без коляски 2х. Пусть автомобилей у, тогда всего колес Зх + 2х • 2 + 4у = 7х + 4у. По условию 7х + 4у = 115, откуда 115 - 7х У = ~^- Значение у наибольшее при х наименьшем. Пусть х = 1, тогда у = 27. Значит, наибольшее число автомобилей 27. 30. Пусть девочки собрали х грибов. Первая получила 20 + 0,04 (х — 20), вторая 21 + 0,04(х — 21 — 20 — 0,04(х — 20)) грибов. По условию 20 + 0,04х — - 0,8 = 21 + 0,04(х - 21 - 20 - 0,04(х - 20)), откуда 0,0016х = 0,192 и х = 120. Значит, собрано 120 грибов, каждая девочка получила 24 гриба, девочек было 5. 31. Пусть в прошлом году Коле было х лет, тогда Вере 2х и Вите 3,5х. Сейчас Коле (х + 1) лет, Вере (2х + 1) лет, Вите (3,5х + 1) лет. Витя станет вдвое старше через (3,5х +1) лет, тогда Коле будет (4,5х + 2) лет ((х + 1) + + (3,5х + 1) = 4,5х + 2), а Вере (5,5х + 2) лет ((2х + 1) + (3,5х + 1) = 5,5х + 2). По условию имеем z_ . , 3 (5,5х + 2) - (4,5х + 2) = 4, откуда х = 4. Значит, Коле сейчас 5 лет, Вере-9 лет и Вите-15 лет. 32. Пусть в десятку и восьмерку Миша попал по х раз, а в пятерку у раз. Тогда 10х + 8х + 5у = 99, или 18х + 5у =99. Слагаемое 5у может оканчивать- ся на 5 или на 0, тогда 18х должно оканчиваться на 4 или на 9. Но на 9 18х оканчиваться не может, поэтому 18х оканчивается на 4. Так как 18х < 99, то х = 3. Имеем 18-3 + 5у = 99, откуда у = 9. Значит, всего попаданий было 171
15100 .«Л 93,75 16Л 3 + 3 + 9=15, что составляет 93,75% от общего числа выстрелов, отсюда выстрелов сделано 16 33. Пусть в интернате было х комнат двухместных и у трехместных. Тогда всего мест в интернате 2х + Зу. Если заселили -, то осталось незаселенных 4 4 4 5 -, т.е. -(2х + Зу). По условию -(2х + Зу) = Зх + 2у, откуда 2у = 7х. Прини- мая во внимание, что х и у натуральные числа, будем иметь xt = 2; yt = 7; х2 = 4; у2 = 14 и т. д. Если х = 2 и у = 7, то всего мест 25; если х = 4, у = 14, то мест 50; если х = 6; у = 21, то мест 75. Поскольку условию удовлетворяет только 50 (30 < 50 < 70), то мест в интернате было 50. 34. Пусть Вова старше Гали в х раз. Если Гале 5 лет, то Вове 5х, а бабушке Кате 5х-х = 5х2. Тогда по условию должно быть 5х + 5х2 = 112. Но это невозможно, так как 112 не делится на 5. Не может быть Гале также и 3 года, так как 112 не делится на 3. Ей не может быть и 4 года, так как 4х + 4х2 = 112, откуда х(х + 1) = 38, а 38 не является произведением двух последовательных чисел. Если Гале 2 года, то 2х + 2х2 = 112, откуда х(х + 1) = 56. Равенство выполняется при х = 7. Значит, Гале 2 года, Вове 14 лет (2-7 = 14), а бабушке Кате 98 лет (14-7 = 98). (Пожелаем бабушке здоровья!) 35. Пусть в первый раз надо было положить а г соли, а положили х г соли, недостача составила (а — х) г. В другой раз положили 2х г соли, недостача составила а — 2х. Так как в другой раз соли добавили в 2 раза меньше, чем а в первый, то а — х = (а — 2х) • 2, откуда х = -. Значит, в первый раз было 1 3 положено в уху - необходимого количества соли. 36. Покрышки целесообразно поменять так, чтобы износ их наступил при одинаковом пробеге всех колес. Пусть их следует поменять при х км пробега, тогда передним колесам остается пройти до износа (25000 — х) км. Но если их 15000 3 поставить вместо задних, то они могут проити только = - того пути, 25 000 5 который бы они еще прошли, будучи на своем прежнем месте, т. е. 3 -(25 000 — х). Задние до замены пройдут х км, а если бы они остались на своем месте, то еще прошли бы (15000 — х) км. На месте же передних колес они 5 пройдут -(15000 —х) км. После замены колеса должны пройти до износа 3 5 поэтому -(25 000 — х) = - (15 000 — х), откуда х ч, а шел у ч, тогда второй ехал у ч, а шел х ч. „ 2 _ одинаковое расстояние, х = 9375 км. 37. Пусть первый ехал Имеем уравнение ЗОх + 4у = 20у + 6х, откуда х = ^у. Значит, первый шел у ч, 2 2 3 а ехал -у ч, а второй шел -у ч, а ехал у ч. Зная весь путь, можно составить 172
2 5 „ г 5 2 5 уравнение 4у + -у • 30 = 40, откуда у = - ч. Первый всего был в пути - + - • - 7 1 2 2- (ч), второй столько же был в пути, так как он шел 1- ч и ехал 1- ч. Значит, наименьшее время, за которое они могут добраться до пункта В, 2 ч 46 мин 40 с. 38. Пусть Пете надо было возвести в квадрат число х. Он должен был получить х2 = 10а + Ь, но на самом деле получил 2х = 106 + а. Значит, 106 +а Л • 2 ’ . х = —-—. Отсюда следует, что число х однозначное (так как х двузначное), а число 106 + а четное двузначное. Число 106 + а следует искать среди чисел 12, 14, 16, 18. Условию задачи удовлетворяет только число 18. Значит, х = 18:2 = = 9 и х2 = 81. Петя должен был получить 81, а получил 18. 39. Сумма четырех последовательных нечетных чисел есть число четное, поэтому однозначные числа следует искать среди чисел 2, 4, 6, 8. 1) 23 — 8; (2х — 3) + (2х — 1) + (2х + 1) + (2х + 3) = 8, откуда 8х = 8; х = 1 - не удовлетворяет условию, так как 2х — 3 < 0. 2) 43 = 64; 8х = 64, откуда х = 8; 13 + 15 + 17 + 19 = 43; 3) 8х = 63 = 216; х = 27. Имеем 51 + 53 + 55 + 57 = 63; 4) 8х = 83, откуда х = 64. Имеем 125 + 127 + 129 + 131 = 512 = 83. Итак, задача имеет три решения: а) 13,15,17,19; б) 51, 53, 55, 57; в) 125, 127, 129, 131. 40. Сумма трех последовательных нечетных чисел есть число нечетное, а если четвертая степень есть число нечетное, то и основание нечетное. Значит, однозначные числа надо искать среди чисел 3, 5, 7, 9. 1) (2х — 1) + (2х + 1) + (2х + 3) = З4, 6х + 3 = З4, откуда 2х + 1 = З3 и х = = 13; 2х - 1 = 25; 2х + 1 = 27; 2х + 3 = 29; 25 + 27 + 29 = 81 = З4; 2) 6х + 3 = 54 (целых решений нет); 3) 6х + 3 = 74 = 2401 (целых решений нет); 4) 6х + 3 = 94; 2х + 1 = 2187; 2х - 1 = 2185; 2х + 3 = 2189. Итак, задача имеет два решения: а) 25, 27, 29; б) 2185, 2187, 2189. 41. Пусть на третьей карточке написано число, имеющее а десятков и 6 единиц, т. е. об. Тогда, выкладывая карточки в ряд, можно получить такие числа: 1875а6, 18а675, 7518а6, 75а618, а67518, а61875. Представим каждое из этих чисел в виде суммы, например, так: 1875а6 = = 180000 + 7500 + аб. Тогда при определении суммы этих шести шестизнач- ных чисел получим 181 818 + 181 818 + 757575 + 757575 + ababab + ababab или 1 878 786 + 2ababab = 2 606 058, откуда ababab = 363 636, а значит, ab = 36. Значит, на третьей карточке было написано число 36. 173
42. Пусть одна книга стоит х к. Тогда имеем х-10 1100 и х • 11 > 1200 иЛи х ПО и х > 109—. Значит, х = ПО к., т.е. книга стоит 1 р. 10 к. 43. Пусть сейчас Олегу х лет, тогда его отцу (х + 32) года, а дедушке (х + 64) года. Три года назад им соответственно было лет: (х — 3), (х + 29) и (х + 61). По условию (х — 3) + (х + 29) + (х + 61) < 100, откуда х < 4-. Принимая во внимание, что х > 3, так как х — 3 > 0, получим х = 4. Значит, Олегу 4 года, его отцу 36 лет и дедушке 68 лет. 44. Если рядов было х, то в каждом из них сидело (х — 3) шестиклассников. Поскольку'в каждом ряду сидело 3 пятиклассника, то в каждом ряду сидело х учеников. Тогда всего учеников было на вечере х-х = х2. По условию х2 > 70 и х2 < 90. Этому условию удовлетворяет только х = 9; 92 = 81. Значит, рядов было 9, шестиклассников было 54 (6 • 9 = 54), а пятиклассников было 27 (3 • 9 = = 27). 45. Пусть комбайном марки «К» за один день убирают х га, а комбайном марки «H»-j га. Тогда запишем уравнение 5(4х + Зу) = 4(3х + 5у), откуда 8х = 5у. Из последнего видно, что у > х. Ответ. Комбайном марки «Н» убирают за день больше, чем комбайном марки «К». 46. Пусть в одном из восьми классов, участвовавших и экскурсии, было х учеников, тогда музей посетило 8х учеников. По условию 8х > 230, откуда 3 х > 28-. КукЬльный театр посетило 8х + х-0,15 = 9,2х. Значит, всего учеников в школе 8х + 9,2х = 17,2х. По условию 17,2х 520, откуда х < 30^. Значит, х = 29 или х = 30. Принимая во внимание, что только 17,2 • 30 является целым числом, условию удовлетворяет только х = 30. Значит, всего учеников в школе 17,2-30 = 516. 47. Пусть 6 А собрал а кг, а 6 Б-б кг, 6 В-в кг и 6 Г-г кг. Тогда по условию 1) а > б + в; 2) б + г > а + в; 3) а + б = в + г. Вычтя из левой части второго неравенства левую часть третьего равенства, а из правой-правую, получим г — а > а — г, откуда г > а. Но а > б и а > в, аиза + б = в + гиа<г выходит, что б > в. Значит, места распределились следующим образом: 6 Г, 6 А, 6 Б, 6 В. 48. Пусть по 10 к. было х монет, тогда по 2 к.-2х монет, а по 5 к.-4х монет. Имеем х + 2х + 4х < 27, или 7х < 27. Поскольку монет по 3 к. было больше, чем 7, но меньше, чем 20, то 7х < 20 и 7х > 7. Так как среди чисел от 8 до 19 имеется только одно число 14, кратное семи, то 7х = 14, откуда х = 2. По 3 к. было 27 — 14 = 13 монет. Значит, Нина внесла в кассу 10-2 + 2-4 + + 5-8 + 3-13 = 107 (к.), т.е. 1 р. 7 к. 49. Пусть искомые числа а, б, в, г, тогда по условию а-б-в-г<а + б + + в + гиа + б + в = 28, (или сумма других трех). Если даже возьмем четыре наименьших, но различных натуральных числа, т.е. 1, 2, 3 и 4, то и тогда 1 • 2 • 3 - 4 > 1 + 2 + 3 + 4. Значит, среди множителей, т. е. искомых чисел не 174
одно число будет 1, а несколько. Если имеем три числа 1, 1 и 2, то четвертое будет 25 или 26, так как сумма трех должна равняться 28. Но Г 1 • 2 • 25 > 1 + + 1 + 2 + 25 и 1 • 1 • 2 • 26 > 1 + 1 + 2 + 26. Поэтому возможно а = б = г = 1, тогда в = 26. При этом 1 • Г 1 • 26 < 1 + 1 + 1 + 26 и 1 + 1 + 26 = 28. Значит, числа таковы: 1, 1, 1 и 26. 50. Пусть число рабочих швейной фабрики х, тогда число рабочих обувной фабрики —х. Количество рабочих обувной фабрики, выполнивших план на 4 15 3 4 15 100%, составляет а швейной фабрики--х. По условию х с 1000 3 2 2 4 15 3 и -х > 1000, откуда х < 2216- и х > 1666-. Так как -• — х и -х-целые числа, то х должно быть кратным для чисел 7, 19 и 5. Между числами (в натуральном ряду) 1667 и 2216 надо найти число, кратное 7 -19 - 5 = 665. Таким числом есть 15 1995. Значит, на швейной фабрике 1995 рабочих, а на обувной -1995- — = 1575. 1 2 3 51. Возьмем очевидное равенство 4 5” 4 3 2 1 4 ->- и т.д., то, заменив - на -; - 5 4 3 2 5 98 99 1 2 1 — — = TZ7- Так как - > 99 100 100 3 2 3 1.1.3 3 5 5 99 99 1 2 2 4 4 6 6 "' 100 100 < ТОО' Последнее запишем так: 4 3 5 99 У 2’46 ”100/ < 1 V 13 5 — , а значит, 10/ 246 99 1 100 < 10’ Задачи с геометрическим содержанием 1. Периметр квадрата должен быть равным 1 см-6 + 2см-3 + 3см-6 + + 4 см • 5 = 50 см, а значит, сторона должна быть 50 см: 4. Как видно, длина стороны не выражается целым числом, поэтому квадрат составить нельзя. 2. Пусть длина стороны квадрата а, тогда площадь S = а-а, а периметр Р = 4а. По условию а • а = 4а, откуда а = 4. 3. Если бы такие многоугольники и прямая существовали, то по обе 175
стороны от прямой лежало бы одинаковое число вершин, что невозможно, так как 1989 не делится нацело на 2. 4. Каждый из 60 отрезков, проведенных из одной вершины, пересекают каждый из 50 отрезков, проведенных из другой вершины, поэтому получим 3000 таких точек (50 • 60 = = 3000). 5. На 11 частей (рис. 14). 6. 18 треугольников. 7. 28 треугольников. 8. Решение представлено на рисунке 15. 9. Треугольник мог быть: а) прямоу- гольным, и тогда его разрезали по линии DE или по линии DM так, как показано на рисунке 16, Е и М -середины сторон А В и АС', б) равнобедренным, и тогда его разрезали по прямой, проведенной через вершину и середину основания (рис. 17). 10. Пусть на рисунке 18 представлен огород. Проведем диагонали этого четырехугольника, и пусть точка их пересечения М. Возьмем внутри четырех- угольника любую точку К, отличную от точки М, и соединим ее с вершинами. Так как АС < АК + СК и BD < ВК + DK, то АС + BD < АК + + DK + СК + ВК или AM + СМ + ВМ + DM < АК + СК + ВК + + DK. Значит, точка М искомая. 11. Пусть длига участка х м, тогда ширина ее равна (80 — х)м (рис. 19) (полупериметр 160:2 = 80). Если AD = х, то CD = 80 — х; СР = 10 м и ЕК = = 10 м. Площадь участка увеличится на величину, равную сумме площадей прямоугольника ADEM и СЕКР, т.е. на х-10 + (80 — х + 10)-10 = 10х + + 900 — 10х = 900. Итак, площадь участка увеличится на 900 м . 12. С тремя окрашенными гранями получим 8 кубов (по количеству вершин данного куба), с двумя-3-12 = 36, с одной-9-6 = 54 и ни од- ной- 3-3-3 = 27. Рис. 15 Рис. 16 Рис. 17 176
Рис. 18 Рис. 19 13. 0,5 м = 500 мм. Объем данного куба равен 500-500-500 мм3, объем каждого полученного куба после разрезания-2-2-2 мм3. В результате разре- зания получено кубов (500 • 500 • 500): (2 • 2 • 2) = 15 625 000. Значит, длина ряда будет 2 мм-15625000 = 31 км 250 м. 14. В результате разрезания получим п-п-п кубов. Поверхность данного куба равна би • и, поверхность каждого полученного куба равна 6, а всех таких кубов би-и-и. Значит, поверхность увеличится в и раз. 15. Рассмотрим рисунок 20. С вершиной в точке А будет 5 треугольников, из которых 3 прямоугольных. Если вместо точки пересечения диагоналей имеем иную внутреннюю точку прямоугольника, а фиксированной остается та же вершина А, то могут представиться такие случаи: а) Эта точка принадлежит диагонали BD (рис. 21). В этом случае всего треугольников будет 6, из них прямоугольных 3 или 5. б) Точка принадлежит диагонали А С (рис. 22). В этом случае имеем всего 5 треугольников, из которых прямоугольных может быть 3 или 4. в) Точка не принадлежит ни одной из диагоналей (рис. 23). Искомых треугольников всего будет 6, из них прямоугольных 3 или 4. 16. Площадь всех плиток, а значит, и искомого квадрата равна 1 • 4 + 4 х х 8 + 9-12 + 16-16 = 400 (кв. ед.). Значит, надо сложить квадрат со стороной 20 единиц. Имеющиеся плитки можно сложить так, как показано на рис. 24. Рис. 20 Рис. 21 177
Рис. 22 Рис. 23 Рис. 24 17. Искомый квадрат, как и данная фигура, должен состоять из 36 квадратов. Значит, в полоске вдоль стороны квадрата должно быть 6 квадра- тиков. Отсюда: разрез надо осуществить по линии А В, как показано на рисунке 25, и фигуру ABCD переместить на одну клеточку вниз и на 2 влево. 178
в с А D Рис. 25 18. Рисунок 26. 19. Если сторона данного квадрата х м, то после его расширения площадь увеличится на величину (5х + 5х + 25) м2, что по условию равно 225. Поэтому имеем 10х + 25 = 225, откуда х = 20. Значит, площадь первоначального участ- ка 400 м2. 20. Возможны треугольники с такими сторонами: а) 2 см, 2 см, 1 см; б) 2 см, 2 см, 2 см; в) 1 см, 1 см, 1 см. Итак, всего три треугольника. 21. Если длина третьей стороны а м, то 6,31 — 0,82 < а < 6, 31 +0,82, откуда 5,49 < а < 7,13. Значит, длина третьей стороны 6 м или 7 м. 22. Площадь одной грани куба 1 дм2, а всех шести граней 6 дм2, а площадь имеющегося листа 6,25 дм2 (2,5 • 2,5 = 6,25). Так как 6,25 > 6, то бумаги хватит. Лист надо разрезать так, как показано на рисунке 27. Рис. 26 АЛЛ 179
23. I м3 = 1000000 см3; одна спичка имеет объем 5 • 0,2 • 0,2 = 0,2 (см3); 1 000 000:0,2 = 5 000 000. Итак, 5 млн. спичек. 24. Во-первых, точки А, В и С должны лежать на одной прямой, так как в противном случае существует точка внутри треугольника АВС, равноудален- ная от его вершин-точек А, В и С (центр окружности, проходящей через точки А, Ви С). Во-вторых, точка В не может лежать между А и С, равно как и точка С между А и В, так как в этом случае найдется сколько угодно точек М, таких, что AM > ВМ и AM > СМ. Остается единственное: точки А, В и С лежат на одной прямой и точка А между В и С. 25. Изобразим схематически окна точками и занумеруем их: 4* *3 1 • *2 . а) зажигание света во всех четырех окнах дает один сигнал; б) зажигание света в одном из окон воспринимается как один сигнал, так как в темноте нельзя различить положение окон относительно дома; в) зажиганием света в двух окнах можно подать 4 различных сигнала: один сигнал-зажиганием в окнах 1, 4 или 2, 3; один сигнал-зажиганием в окнах 1, 2 или 4, 3; два сигнала - зажиганием в окнах 1, 3 и 2, 4, т.е. при этом имеем такие конфигурации: 1) • 2) • • 3) • 4) • г) зажиганием света в трех окнах можно подать четыре сигнала: 1, 2, 3; 2, 3, 4; 3, 4, 1 и 4, 1, 2, т.е. имеем такие конфигурации: • ее ее • 1) 2) 3) 4) Всего имеем различных сигналов: 1 + 1+4 + 4=10. 26. Как показано на рисунке 28, общей частью треугольника и четырех- угольника могут быть отрезок, треугольник, четырехугольник, пятиугольник, шестиугольник и семиугольник. 27. Поскольку должна получиться ломаная замкнутая, то, начав выклады- вать палочки от некоторой точки, мы должны сюда же возвратиться. Это значит, сколько выложено палочек в одном направлении, столько же должно быть выложено и в противоположном. Если, например, вниз выложено а Палочек, то и вверх будет выложено а, вправо-/?, то и влево-/?. Всего будет выложено 2а + 2Ь = 2 (а + /?). По условию 2{а + Ь) = 1989. Так как 1989 не делится на 2, то требуемую ломаную выложить нельзя. 180
Рис. 28 28. Из шариков можно выложить квадрат только в том случае, если число всех шариков есть точный квадрат. Например, 9, 16, 25 и т.д. Если же шарики можно выложить в виде треугольника, то их количество равно сумме последовательных натуральных чисел, начиная с 1. Значит, должно быть: 1 + 2 + 3 + ... + а = п . Из чисел, не превосходящих 50, указанному требова- нию удовлетворяет лишь 36: 1+2 + 3 + 4 + 5 + 6 + 7 + 8 = 36 = 62. 29. Можно, например, построить так, как показано на рисунке 29. 30. Возьмите ломаную из двух звеньев. Присоединим третье звено. Оно может пересекать только одно из двух первоначальных, в результате получим одну точку (1) (рис. 30). Возьмем четвертое звено, оно может пересечь максимально два из трех имеющихся звеньев, в результате чего получим еще две точки (2, 3). Пятое звено может пересечь максимально 3 звена, в результа- те чего получим еще 3 точки (4, 5, 6). Шестое звено максимально может Рис. 29 Рис. 30 181
пересечь 4 звена, в результате чего получим 4 точки (7, 8, 9, 10). Седьмое звено максимально может пересечь 4 звена, в результате чего получим еще 4 точки (11, 12, 13, 14). Таким образом, наибольшее возможное число точек самопере- сечения 14. 31. Если ширина а м, а длина b м, то по условию имеем 2а + 2b = ab, а — 2 откуда 2 = Ь-----. Равенство возможно при а > 2 и при условии: 1) а = b = 4; 2) а = 3; Ь — 6. Значит, возможны площадки таких размеров: 1) 4 х 4 (м2); 2) 6 х 3 (м2). 32. Если размеры дна а м и b м, то площади боковых граней 1,5а м2 и 1,56 м2. По условию 1,5а > ab и 1,56 > аб, откуда 6 < 1,5 и а < 1,5. Значит, возможный наибольший объем цистерны 3,375 м3 (1,5-1,5-1,5 = 3,375). 33. Пусть первоначальные размеры мыла (параллелепипеда) а, 6 и с, тогда • , „ „ 1 1, 1 его объем равен а-о-с. По истечении семи дней размеры стали -а, -6 и -с, _ 11,1 1 , „ а объем станет равным -а--6--с = „abc. Значит, за один день расходуется 2 2 2 о 7 1 -abc\l = -abc. Таким образом, оставшегося мыла хватит на один день. О о 34. За 1 ч конец минутной стрелки опишет дугу длиной 2лЛ, где R- длина стрелки. Тогда дугу длиной R конец минутной стрелки опишет за 60 мин „ 30 мин - -— R =--------« 9,6 мин. 2л7? я 35. Пусть длины вписанных окружностей С, = ndt, С2 = nd2... С„ = л</п. Тогда Cj + С2 + ... + С„ = я + d2 + . . • + <4) = itD, где D- диаметр данной окружности. Значит, сумма длин вписанных окруж- ностей равна длине данной окружности. И она не изменяется. 36. Диаметр малого круга равен радиусу большего. Пусть радиус больше- R го круга R, тогда малого будет Тогда площадь малого круга будет л/?2 _ , а четырех малых-л/r, а это есть площадь большего круга. Вывод: сумма площадей четырех малых кругов равна площади большего круга. 37. Площадь ла2 полукругов —. ж ж - * па полукруга равна а сумма площадей четырех таких О Площадь квадрата равна а2. Тогда искомая площадь будет равна: ла2 2 2 —---а № а 2 4 11 Л 4 2 — - 1 I = -л . к 7 /7 7 п ~ 11 182
Рис. 31 Рис- 32 Рис. 33
38. 3 дм = 300 мм; 300:5 = 60. 39. Если найдется две параллельные диагонали, то угол между ними 0°, т.е. меньше, чем 3°. Если никакие две диагонали непараллельны, то, взяв точку на плоскости и проведя через нее прямые, параллельные соответственно каждой из диагоналей, получим все углы между диагоналями. Всего диагона- лей будет 60, так как из каждой вершины можно провести 10 диагоналей, а вершин 12, то имеем 12-10, но при этом каждая диагональ учтена дважды, поэтому 12-10:2 = 60. В результате проведения указанных прямых плоскость разобьется на 120 углов. Если углы окажутся равными, то каждый из них будет иметь величину 3° (360°: 120 = 3°). Если величина хотя бы одного угла больше 3°, то найдется угол, величина которого меньше, чем 3°. 40. Поскольку объем параллелепипеда-число простое и больше, чем 2, то оно нечетное, т. е. вида (2л + 1). Пусть длины ребер а, b и с, тогда а • Ь • с = 2л + + 1. Так как ata-число простое, то а = b = 1 и с = 2л + 1, но при этом (2л+1)+1 + 1= 2(л+1)+1 - число нечетное. (Возможно, а = с = 1 или b = = с = 1, тогда длина Ь или а равна (2л + 1).) 41. Решение представлено на рисунке 31. 42. Да, можно. Квадрат можно разрезать на 4 части, как показано на рисунке 32, и сложить 2 квадрата. Тогда грани куба разрезаем так, как показано на рисунке 33. Разрезав грани по диагоналям (а не по ребрам куба), получим, например, квадрат, составленный из треугольников 1 и 2. Также получим квадрат, состоящий из треугольников 3 и 4 и т.д. Логические задачи 1. У К. И. Чуковского имеется такой ответ: Глупый, глупый, Кондрат! Он один и шагал в Ленинград, А ребята с лукошками, С мышатами и кошками Шли навстречу ему- В Кострому. 2. Второй собеседник была женщина, которую звали Нина. 3. Гришин - бортинженер, Павлов-радист, Романов-командир корабля. 4. В коробке, на которой нарисованы красный и зеленый карандаши, находятся карандаши одного цвета. Взяв из нее один карандаш, определяем цвет находящихся в ней карандашей. Если, например, там зеленые карандаши, то в коробке с рисунком красного карандаша находятся карандаши разного 184
цвета, а в коробке с рисунком зеленого карандаша находятся красные карандаши. 5. Если бы заяц был первым или вторым, то тогда оба высказывания одной из белок были бы ложными. Значит, верно, что второй была лиса, а первым-лось. 6. Первым стоит Коля, за ним-Юра, потом-Оля, Ира, Саша. 7. Сначала на лодке переправляется Андрей с Олегом, затем один из них, например Андрей, возвращает лодку назад, в которой переправляется дядя Миша. После этого лодку возвращает назад Олег и вместе с Андреем переправляется к дяде Мише. 8. Если машина идет со скоростью 60 км/ч, то 1 км она проходит за 1 мин. При таком условии на каждом километре нельзя выиграть минуту при любом увеличении скорости. 9. Времянкин половину времени шел со скоростью 5 км/ч, поэтому, идя с такой скоростью, он прошел больше половины пути, а Путейкин, идя с такой скоростью, прошел только половину пути, поэтому Времянкин пришел в ко- нечный путь раньше, чем Путейкин. 10. Рассмотрим самый «неблагоприятный» случай. Пусть имеется по 7 ящиков с яблоками, грушами и сливами. Это составит 21 ящик. Если 22-й ящик будет с яблоками, то яблок 8 ящиков; если этот ящик с грушами, то груш 8 ящиков; если же этот ящик со сливами, то слив 8 ящиков. В иных случаях будет больше 8 ящиков, содержимое которых составляет один из трех видов фруктов. 11. Если предположить, что в каждом классе меньше, чем 35 учеников, то всех учеников было бы не более 34-33 = 1122. В действительности же в школе 1150 учеников. Значит, сделанное допущение неверно: есть класс, в котором не меньше 35 учеников. 12. Медные монеты бывают достоинством в 1 к., 2 к., 3 к., 5 к., т. е. всего четырех видов. Если взять по 6 монет каждого вида, то всего будет 24 монеты. Взяв еще одну (двадцать пятую) монету, получим 7 монет одного достоинства. В иных случаях монет одного достоинства будет больше, чем 7. Значит, Татьяна права. 13. Рассмотрим самый «неблагоприятный» случай. Если взять 24 ботинка, то все они могут оказаться на одну ногу. Взяв еще один ботинок, он обязательно окажется парным одному из ранее взятых. Возможно, что взяли сначала по 12 ботинок на разные ноги, но разных цветов, то и в этом случае результат будет тот же. Значит, надо взять 25 ботинок. 14. Пусть имеем 100 =18+174-15-1- 14 4-13-1-12-1-11. В этом случае 18 + 17 + 15 = 50. При замене любого из последних четырех слагаемых числом меньшим необходимо сумму первых трех слагаемых увеличить, в результате она станет больше 50. Если же заменить хотя бы одно из четырех последних слагаемых числом большим (а таким числом может быть 16,19 или больше), то вместо одного из первых трех слагаемых надо взять меньшее число, и тогда числа из первой тройки и последней четверки лишь поменяются ролями. Значит, всегда можно выбрать 3 числа, сумма которых будет не меньше 50, что и дает ответ на вопрос задачи. 15. Рассмотрим самый «неблагоприятный» случай: в одном ящике лежит одна деталь, в другом-две, в третьем-три и т.д. Таких ящиков с различным содержанием деталей будет 125. Но 125 в числе 376 содержится 3 раза 185
с остатком 1 (376 = 125-3 + 1). Значит, при этом будет по 3 ящика с одинако- вым числом деталей. Какое бы теперь число деталей из возможных (от 1 до 125)' ни положили в оставшийся ящик, этот ящик вместе с тремя имеющимися составит 4 ящика, в которых по одинаковому числу деталей. В иных случаях таких ящиков с одинаковым числом деталей будет больше, чем 4. 16. Рассмотрим самый «неблагоприятный» случай: среди взятых каранда- шей может оказаться 14 желтых, 11 коричневых, 11 черных, 17 красных, 17 зеленых и 17 синих. Таким образом, если возьмем 87 карандашей 14 + 11 + + 11-1-17-1-17-1-17 = 87, то может оказаться, что среди них не будет 18 карандашей одного цвета. Взяв еще один карандаш, будем иметь или 18 красных, или 18 зеленых, или 18 синих. Значит, надо взять 88 карандашей. В иных случаях среди 88 взятых карандашей окажется одного цвета больше 18 карандашей. 17. Рассмотрим самый «неблагоприятный» случай: берем все фотографии I—III классов (130 — (35 + 30 + 25 + 20) = 20), дальше может оказаться, что каждого из IV-VII классов взяли по 14 фотографий, а всего 14-4 + 20 = 76 фотографий. Теперь стоит взять еще одну, и тогда какого-то одного из IV-VII классов будет 15. Итак, надо взять 77 фотографий. 18. Из общего числа автомашин не обладали первым признаком (не «Жигули»)-6, вторым-5, третьим-3 и четвертым-2. Значит, наибольшее число автомашин, не обладающих хотя бы одним признаком, равно 16 (6 + 5 + 3 + 2=16). Остальные 4 (20 — 16 = 4) обладают всеми четырьмя признаками. 19. Надо перевернуть карточку с буквой А. Если на обороте окажется нечетное число, то предложение ложное; если-четное, то надо перевернуть карточку с числом 4; если окажется гласная буква, то надо перевернуть карточку с числом 5. 20. Шестой класс собрал огурцов 9ц (18:2 = 9) и лука 7ц, значит, четвертый класс собрал моркови 7 ц. Пятый класс собрал огурцов меньше, чем 9 ц, но больше, чем 7 ц, а значит, 8 ц. Таким образом, всего собрали школьники 18 + 7 + 8 + 9 + 7 = 49 (ц). 21. В детский сад ходит та девочка, которой меньше всех лет, т. е. которой 5 лет. Тане не может быть 5 лет, так как она старше, чем Юра. Ей не может быть и 15 лет, так как 15 в сумме с любым из данных чисел не делится на 3. Не может быть ей также и 8 лет, так как 5 лет не Юре (не мальчику). Значит, Тане-13 лет, тогда Юре-8 лет. Тогда Свете-5 лет, а Лене-15. 22. Распилить 3 кольца одного звена и полученными тремя кольцами соединить (спаять) оставшиеся 4 звена. На это потребуется 6 мин. 23. Путешественники распилили третье звено, разделив цепочку на 3 части. За первый рейс они заплатили одним звеном (распиленным); за второй дали лодочнику 2 звена, получив от него «сдачу» одно звено; за третий дали 3 звена, получив «сдачу» 2 звена; за четвертый заплатили распиленным звеном; за пятый-два звена, получив «сдачу» звено, и, наконец, за шестой-одно звено. 24. Если бы звонили из А, то на вопрос: «Где?»-ответили бы: «В городе А». Из В также не могли звонить, так как из двух утверждений: «У нас пожар» и «В городе В»-не может быть одно истинное, а другое ложное. Значит, звонили из города Б. Но так как они всегда говорят неправду, то пожар в городе А и надо ехать туда. 186
25. Если допустить, что верное утверждение первое, то это будет означать, что в книге ровно одно неверное утверждение, а все остальные верные, т.е. верное второе, третье и т. д., что противоречит первому, так как, например, во втором утверждается, что неверных ровно два, в третьем-ровно три и т.д. Проведя такие рассуждения до 98 утверждения включительно, придем к тако- му же выводу. Если же верное 99-е утверждение, то это значит, что неверных ровно 99, т.е. все, кроме 99-го утверждения. Значит, неверно, что неверных ровно одно, два, ..., 98 и 100. Не может быть верным утверждение сотое, так как в этом случае все утверждения (по утверждению сотого) неверны. Итак, верное утверждение 99. 26. 1) Из бочки наполнить одно ведро, затем черпаком из ведра отобрать 4 л и вылить в бочку. В результате в ведре останется 3 л. 2) Из этого, ведра вылить в черпак 3 л, а из бочки наполнить ведро. Из этого ведра дополнить черпак, т.е. отлить 1 л. В результате в ведре останется 6 л. 3) Повторив такую же процедуру, наливают 6 л бензина во второе ведро. 27. Влить в другую бочку 4 девятилитровых бидона (9 • 4 = 36), а затем из этой бочки отобрать 6 пятилитровых бидонов (5 • 6 = 30; 36 — 30 = 6). 28. Представим такой схемой: Сосуд на 12 пинт на 8 пинт на 5 пинт 12 — — 4 8 — 4 3 5 9 3 — 9 — 3 1 8 3 1 6 5 6 6 — 29. Положить на одну чашку гирю и разложить по чашкам имеющийся сахар так, чтобы весы были в равновесии. Тогда на одной чашке будет 4 кг 600 г сахара, а на другой (там, где гиря)-4 кг 400 г сахара. Второй раз разложим по чашкам 4 кг 600 г сахара. На одной из них (без гири) будет 2 кг 187
400 г сахара, а на другой-2 кг 200 г. Последний (третий) раз от 2 кг 200 г сахара отвешиваем при помощи имеющейся гири 200 г, а остается 2 кг. (При втором взвешивании можно гирю не использовать, а разложить поровну на чашки 2 кг 400 г.) 30. В этом месяце 5 пятниц, так как в одном месяце пятницы приходятся поочередно на четные и нечетные даты. Первая пятница могла быть только 2-го числа (в противном случае 3 пятницы не могут попасть на четные даты), а значит, 18-е число-воскресенье. 1 31. Если в одном месяце три среды пришлись на четные числа, то это возможно при условии, что месяц не февраль и начинается со вторника. Тогда среда будет 2-го, 16-го и 30-го, а второе воскресенье будет 13-го числа. 32. Наибольшее число суббот в одном месяце 5, так как 7-4 = 28; 28 < 30 и 28 < 31. В феврале может быть 5 суббот, если год високосный и этот месяц начинается с субботы. Апрель, июнь, сентябрь и ноябрь будут иметь по 5 суббот, если они начинаются с субботы или пятницы. Остальные месяцы могут иметь по 5 суббот, если они начинаются с субботы, пятницы или четверга. 33. По условию задачи первое воскресенье месяца и первое воскресенье после первой субботы не совпадают. Это возможно в том случае, если первое воскресенье месяца приходится на первое число, а первое воскресенье после первой субботы-на 8 число. Значит, оба месяца начинались с воскресенья. Поскольку интервал между двумя воскресеньями, с которых начинались месяцы, есть целое число недель, то число дней в первом из этих месяцев делится на 7. Этот месяц может быть только февраль. Значит, в Карпатах школьники были 1 февраля, в Киеве-8 февраля, на экскурсии в лесу -1 марта и в музее-8 марта. 34. В верхнем ряду таблицы записаны числа, выражающие количество букв в словах-названиях месяцев, в нижнем-количество дней соответствующих месяцев. Поэтому дальше в верхнем ряду следует записать 4 (июнь), 4 (июль), 6 (август), в нижнем соответственно: 30, 31, 31. 35. Здесь записаны первые буквы названий последовательных натуральных чисел, начиная с 1, т. е. слов «один», «два», «три» ,«четыре», «пять», «шесть». Поэтому дальше будут буквы С (семь), В (восемь) и т.д. 36. Запишем схематически условие задачи: АКРОБАТ + СОБАЧОНКА = 2 БОЧОНКА (1) СОБАЧОНКА = 2 МОТКА (2) ЯГНЕНОК + МОТОК = БОЧОНОК (3) Учитывая равенства (2) и (3), первое равенство получит вид: АКРОБАТ + 2 МОТКА = 2 (ЯГНЕНОК 4- МОТОК). Или: АКРОБАТ + 2 МОТКА = 2 ЯГНЕНКА + 2 МОТКА. Сопоставляя левую и правую части равенства, получаем: АКРОБАТ = 2 ЯГНЕНКА. 37. Так как 1 + 24-3+4 + 54-6 + 7 + 8-1-8 = 44, то в турнире участвует 9 команд. Всего они должны набрать 72 очка (8 • 9 = 72). Значит, осталось еще набрать 28 очков (72 — 44 = 28), что можно сделать за 14 игр (28:2 = 14). 38. Всего было 12 игр (3-4=12). В каждой встрече-2 очка, а всего очков-24. Так как 5 + 6 + 7 + 8 = 26 > 24, то такой случай невозможен. 188
Поэтому 5 + 64-6 + 7 = 24. Значит, команда-победительница набрала 7 очков. 39. Участник турнира, занявший второе место, из общего количества 28 очков, набранных всеми участниками вместе, набрал не более шести очков, так как в противном случае он занял бы или первое место, или разделил бы с другим первое и второе места, что противоречит условию. Возможна только такая ситуация: 28 = 7 + 6 + 5 + 4 + 3 + 2+1+0. С другой стороны, если участники, занявшие с пятого по восьмое место включительно, набрали 6 очков, то эти очки они набрали, играя между собой, проиграв все партии шахматистам, занявшим первые четыре места. Значит, участник турнира, занявший третье место, выиграл у того, кто занял пятое. 40. «Строитель» имеет одну ничью, а остальные поражения. «Торпедо» может иметь 2 победы и одну ничью. Больше ничьих не может быть, так как «Азовец» и «Энергия» имеют по четному числу очков. Ничья между «Торпе- до» и «Строителем» может быть только нулевая, т. е. 0:0, так как «Торпедо» имеет 7:0. Тогда легко заполнить таблицу для «Строителя» и «Торпедо», и она принимает такой вид: Команда «Торпедо» «Азовец» «Энергия» «Строитель» Мячи Очки «Торпедо» X 3:0 4:0 0:0 7:0 5 «Азовец» 0:3 X 1:0 1:0 2:3 4 «Энергия» 0:4 0:1 X 2:1 2:6 2 «Строитель» 0:0 0:1 1:2 X 1:3 1 41. «Вымпел» имеет одну ничью, остальные проиграл, так как получил одно очко. «Авангард» пропустил только один мяч, поэтому в колонках со «Стартом» и «Вымпелом» вторые числа нули, а первые, очевидно, единицы; тогда в колонке со «Звездой» тоже 1, так как сумма забитых мячей «Авангар- дом» 3. Значит, «Старт» и «Вымпел» имеют в колонках с «Авангардом» 0:1 и 0:1, а «Старт» - «Звезда» будет 5:1, так как общий счет: 3. «Звезда»-«Вым- пел» счет 2:1, так как 7 = 5 + 1 + 1, а «Вымпел» имеет проигрыши. В резуль- тате получим такую таблицу: Команда «Старт» «Вымпел» «Авангард» «Звезда» Очки Счет Место «Старт» X 1:1 0:1 5:1 3 6:3 2 «Вымпел» 1:1 X 0:1 1:2 1 2:4 4 «Авангард» 1:0 1:0 X 1:1 5 3:1 1 «Звезда» 1:5 2:1 1:1 X 3 4:7 3 189
42. «Старт» выиграл все встречи, так как набрал 6 очков. У «Кометы» всего 2 забитых мяча. Один матч она выиграла со счетом 1:0 (у «Вымпела»), значит, еще один матч свела вничью 1:1, а третий проиграла с «сухим» счетом. Проиграть она могла только «Старту» со счетом 0:2, поэтому ничья у нее с «Ракетой». «Ракета» проиграла «Старту» и, очевидно, со счетом 3:5, так как у нее пропущено 8 мячей. Тогда с «Вымпелом» она свела вничью 2:2. Остается, что «Старт» выиграл у «Вымпела» со счетом 3:1. Сведя эти данные в таблицу, получим Команда «Старт» «Комета» «Ракета» «Вымпел» Победы Ничья Поражение Мячи Очки «Старт» X 2:0 5:3 3:1 3 0 0 10:4 6 «Комета» 0:2 X 1:1 1:0 1 1 1 2:3 3 «Ракета» 3:5 1:1 X 2:2 0 2 1 6:8 2 «Вымпел» 1:3 0:1 2:2 X 0 1 2 3:6 1 43. Пусть масса арбузов а2, а3, а4. Одним взвешиванием сравним, например, аг и а2, а вторым-а3 и а4, Пусть а± > а2 и а3 > а4. Третьим взвешиванием сравним ах и а3. Если а± > а3, то аг > а3> а4, если аг < а3, то а3 > ai >а2- Четвертым взвешиванием сравним либо а3 и а2, либо аг и а4 (в зависимости от конкретного случая). Если, например, а2<а3, то пятым взвешиванием сравниваем а2 и а4. Если а2 > а4, то аг > а3 > а2 > а4, если я2 < а4.> то аг > а3 > а4> а2. Если а2 > а3, то аг > а2> а3> а4, и в этом случае достаточно четырех взвешиваний. 44. Обозначим массы этих деталей а2, а3, а4. Возьмем произвольно две детали. Пусть, например, массы этих деталей а± и а2. Сравним их. При этом может представиться два случая: аг а2 или ах = а2. 1) Пусть аг / а2. Если ах > а2, то, положив на чашку весов вместо а2, например, а3, получим или аг — а3, или аг > а3. Если ах — а3, то деталь а2 имеет меньшую массу, чем другие; если аг > а3, то деталь ах имеет большую массу, чем другие. Если ах < а2, то поступаем так же, как и в предыдущем случае. 2) аг = а2. Заменив деталь а2 деталью, например, а3, можем получить или аг = я3, или а± а3. Если а{ = а3, то масса а4 не такая, как другие. Вторым взвешиванием определяем больше или меньше а4, чем другие. Если аг > а3 или ах < а3, то а3 или меньше, или больше по сравнению с другими. 45. Если конь находится на черном поле (клетке), то, сделав один ход, он попадает на белое поле (клетку); третья клетка, на которую он попадает, будет черной, т.е. такой, как первая; четвертая - белой и т.д. Значит, клетки, имеющие четные номера, иного цвета, чем первая. Конь, обойдя все клетки, должен попасть на 64, которая должна быть не такого цвета, как первая, а на самом деле она такого же цвета, как первая. Значит, ходом коня нельзя попасть из левой нижней клетки шахматной доски на правую верхнюю, побывав по одному разу в каждой клетке. 190
46. Кубики можно расположить так: 47. Представим схематически условие задачи так: 1) Медведь Подсвечник 3) Волк Иголка Рысь Тарелка Мышка Колечко Белка Овечка 2) Рысь Подсвечник 4) Овечка Колечко Белка Иголка Медведь Тарелка Мышка Волк Волк Белка Сопоставляя схемы 1) и 2), получаем, что медведь подарил тарелку, а подсвеч- ник подарила рысь или белка. Из схемы 4) заключаем, что подсвечник подарила рысь. Из схемы 2) видно, что иголку подарила мышка или волк, но из 4) заключаем, что волк иголку не дарил, значит, иголку подарила мышка. Из схемы 3) получаем, что колечко подарила овечка, в результате имеем: медведь подарил тарелку, рысь-подсвечник, мышка-иголку, овечка-колеч- ко, а без подарка явились волк и белка. 48. Всего троек чисел, произведение которых равно 36, будет: 1) 1, 4, 9 (тогда 1 + 4 + 9 = 14); 2) 1, 6, 6 (сумма 13); 3) 1, 3, 12 (сумма 16); 4) 1, 2, 18 (сумма 21); 5) 2,2, 9 (сумма 13); 6) 3, 3,4 (сумма 10); 7) 2, 3,6 (сумма 11); 8) 1,1, 36 (сумма 38). Если бы тренер увидел номер автобуса, отличный от 13, то он без дополнительных данных назвал бы возраст. Значит, он увидел 13. Утвержде- ние Нины, что старший из братьев любит спорт, равносильно тому, что близнецы не старшие мальчики, поэтому случай 1+6 + 6=13 отпадает, а остается 2 + 2 + 9 = 13. Значит, старшему брату 9 лет, а меньшим-близне- цам-по 2 года.
ОГЛАВЛЕНИЕ Нумерация чисел ................................................................ 5 Арифметические действия над натуральными числами и нулем и их свойства .... 13 Квадрат и куб числа............................................................ 27 Делимость натуральных чисел ................................................... 33 Простые и составные числа. Разложение чисел на множители....................... 41 Наибольший общий делитель и наименьшее общее кратное........................... 50 Дроби и проценты .............................................................. 54 Положительные и отрицательные числа............................................ 73 Выражения с переменными. Уравнения и неравенства .............................. 81 Задачи с геометрическим содержанием............................................ 93 Логические задачи ............................................................*103 Решения и ответы Нумерация чисел .............................................................. 116 Арифметические действия над натуральными числами и нулем и их свойства .... 120 Квадрат и куб числа........................................................... 133 Делимость натуральных чисел ...................................................136 Простые и составные числа. Разложение чисел на множители...................... 140 Наибольший общий делитель и наименьшее общее кратное.........................147 Дроби и проценты ..............................................................149 Положительные и отрицательные числа............................................164 Выражения с переменными. Уравнения и неравенства ..............................168 Задачи с геометрическим содержанием............................................175 Логические задачи .............................................................184 Учебное издание Клименчеико Дмитрий Васильевич ЗАДАЧИ ПО МАТЕМАТИКЕ ДЛЯ ЛЮБОЗНАТЕЛЬНЫХ Зав. редакцией Т. А. Бурмистрова. Редактор Л. В. Туркестанская. Младшие редакторы Л. И. Заседателева, Е. В. Казакова. Художники В. П. Кабанов, Е. В. Викторов, Е. М. Молчанов. Художественный редактор Ю. В. Пахомов. Технический редактор С. В. Китаева. Корректоры И. А. Корогодина, Г. И. Мосякина. ИБ 12646 Сдано в набор 12.03.91. Подписано к печати 11.02.92. Формат 70х90|/|6. Бум. офс. № 2. Гарнит. Таймс. Печать офсетная. Усл. печ. л. 14,044-0,2 форз. Уотт, кр.-отт. 29,6. Уч.-изд. л. 13,36 + 0,41 форз. Ъ|раж 88 000 экз. Заказ № 2531. Ордена Трудового Красного Знамени издательство «Просвещение» Министерства печати и информации Российской Федерации. 127521, Москва, 3-й проезд Марьиной рощи, 41. Отпечатано с диапозитивов, изготовленных на Можайском полиграфкомбинате Министерства печати и информации Российской Федерации. 143200, г. Можайск, ул. Мира. 93, на Смоленском полиграфкомбинате Министерства печати и информации Российской Федерации. 214020, Смо- ленск, ул. Смольянинова, I.